ΠΠΎΡΠ΅ΠΌΡ Π²ΡΡ Π²ΠΎΠΊΡΡΠ³ ΡΠ°ΠΊΠΎΠ΅, ΠΊΠ°ΠΊΠΎΠ΅ ΠΎΠ½ΠΎ Π΅ΡΡΡ?
ΠΠΎΡΠ΅ΠΌΡ Π²ΡΡ Π²ΠΎΠΊΡΡΠ³ ΡΠ°ΠΊΠΎΠ΅, ΠΊΠ°ΠΊΠΎΠ΅ ΠΎΠ½ΠΎ Π΅ΡΡΡ?ΠΠ΅Π»Π»ΡΡ Π‘ΠΈΠ²Π°ΡΠ°ΠΌΠΈΡ Π§Π°Π½Π΄ΡΠ°ΡΠ΅ΠΊΠ°Ρ
ΠΡΠΎΠ΄ΠΎΠ»ΠΆΠ΅Π½ΠΈΠ΅. Π‘ΠΌ. β 2, 4, 6/05
Β 3. ΠΠ»Π΅ΠΊΡΡΠΈΡΠ΅ΡΠΊΠΈΠ΅ ΠΈ ΠΌΠ°Π³Π½ΠΈΡΠ½ΡΠ΅ ΠΏΠΎΠ»Ρ (ΠΏΡΠΎΠ΄ΠΎΠ»ΠΆΠ΅Π½ΠΈΠ΅)
ΠΡΠ΅Π΄ΡΡΠ°Π²ΠΈΠΌ ΡΠ΅ΠΏΠ΅ΡΡ Π½Π΅ΠΊΠΎΡΠΎΡΡΠΉ ΠΎΠ±ΡΡΠΌ ΠΏΡΠΎΡΡΡΠ°Π½ΡΡΠ²Π°, Π² ΠΊΠΎΡΠΎΡΠΎΠΌ Π½Π΅Ρ Π²Π΅ΡΠ΅ΡΡΠ²Π°, Π½ΠΎ Π² ΠΊΠ°ΠΆΠ΄ΠΎΠΉ ΡΠΎΡΠΊΠ΅ Π·Π°Π΄Π°Π½Ρ ΡΠ»Π΅ΠΊΡΡΠΈΡΠ΅ΡΠΊΠΎΠ΅ ΠΈ ΠΌΠ°Π³Π½ΠΈΡΠ½ΠΎΠ΅ ΠΏΠΎΠ»Ρ. ΠΠ»Ρ Π½Π°ΡΠ°Π»Π° ΠΏΡΠ΅Π΄ΠΏΠΎΠ»ΠΎΠΆΠΈΠΌ, ΡΡΠΎ ΡΡΠΈ ΠΏΠΎΠ»Ρ Π½Π΅ Π·Π°Π²ΠΈΡΡΡ ΠΎΡ Π²ΡΠ΅ΠΌΠ΅Π½ΠΈ, Ρ ΠΎΡΡ ΠΌΠΎΠ³ΡΡ ΠΏΡΠΈΠ½ΠΈΠΌΠ°ΡΡ ΡΠ°Π·Π½ΡΠ΅ Π·Π½Π°ΡΠ΅Π½ΠΈΡ Π² ΡΠ°Π·Π½ΡΡ ΡΠΎΡΠΊΠ°Ρ ΠΏΡΠΎΡΡΡΠ°Π½ΡΡΠ²Π° Π²Π½ΡΡΡΠΈ Π²ΡΠ΄Π΅Π»Π΅Π½Π½ΠΎΠ³ΠΎ ΠΎΠ±ΡΡΠΌΠ°. ΠΡΠΈ ΠΏΠΎΠ»Ρ ΠΏΠΎΡΠΎΠΆΠ΄Π΅Π½Ρ Π½Π΅ΠΊΠΎΡΠΎΡΡΠΌ ΡΠ°ΡΠΏΡΠ΅Π΄Π΅Π»Π΅Π½ΠΈΠ΅ΠΌ Π·Π°ΡΡΠ΄ΠΎΠ², ΡΠΎΠΊΠΎΠ² ΠΈ ΠΏΠΎΡΡΠΎΡΠ½Π½ΡΡ ΠΌΠ°Π³Π½ΠΈΡΠΎΠ², ΠΊΠΎΡΠΎΡΡΠ΅ Π½Π°Ρ ΠΎΠ΄ΡΡΡΡ Π²Π½Π΅ Π²ΡΠ΄Π΅Π»Π΅Π½Π½ΠΎΠ³ΠΎ ΠΎΠ±ΡΡΠΌΠ°. ΠΠ°ΠΌ Π½Π΅ Π½ΡΠΆΠ½ΠΎ Π½ΠΈΡΠ΅Π³ΠΎ Π·Π½Π°ΡΡ ΠΎΠ± ΡΡΠΈΡ ΡΠ°ΡΠΏΡΠ΅Π΄Π΅Π»Π΅Π½ΠΈΡΡ , Π²Π°ΠΆΠ½ΠΎ ΡΠΎΠ»ΡΠΊΠΎ, ΡΡΠΎ ΠΎΠ½ΠΈ ΡΡΡΠ΅ΡΡΠ²ΡΡΡ.
Π Π°ΡΡΠΌΠΎΡΡΠΈΠΌ ΠΏΠΎΠ»Ρ Π²Π½ΡΡΡΠΈ ΠΎΠ±ΡΡΠΌΠ°. ΠΡΠ΅Π΄ΠΏΠΎΠ»ΠΎΠΆΠΈΠΌ, ΡΡΠΎ Ρ Π·Π°ΡΡΠ΄Π°ΠΌΠΈ, ΡΠΎΠ·Π΄Π°ΡΡΠΈΠΌΠΈ ΡΠ»Π΅ΠΊΡΡΠΈΡΠ΅ΡΠΊΠΎΠ΅ ΠΏΠΎΠ»Π΅, ΡΡΠΎ-ΡΠΎ ΡΠ»ΡΡΠΈΠ»ΠΎΡΡ ΠΈ ΠΎΠ½ΠΈ Π½Π°ΡΠ°Π»ΠΈ Π΄Π²ΠΈΠ³Π°ΡΡΡΡ, ΡΠ°ΠΊ ΡΡΠΎ ΡΠ»Π΅ΠΊΡΡΠΈΡΠ΅ΡΠΊΠΎΠ΅ ΠΏΠΎΠ»Π΅ Π½Π°ΡΠ°Π»ΠΎ ΠΈΠ·ΠΌΠ΅Π½ΡΡΡΡΡ ΡΠΎ Π²ΡΠ΅ΠΌΠ΅Π½Π΅ΠΌ.
ΠΡΠΎΡΡΠΎΠΉ ΡΠΏΠΎΡΠΎΠ± ΠΏΠΎΡΠΎΠ΄ΠΈΡΡ ΠΏΠ΅ΡΠ΅ΠΌΠ΅Π½Π½ΠΎΠ΅ Π²ΠΎ
Π²ΡΠ΅ΠΌΠ΅Π½ΠΈ ΠΌΠ°Π³Π½ΠΈΡΠ½ΠΎΠ΅ ΠΏΠΎΠ»Π΅ β ΡΡΠΎ Π·Π°ΡΡΠ°Π²ΠΈΡΡ ΡΠΎΠΊ Π²
ΠΏΡΠΎΠ²ΠΎΠ΄Π΅ Π½Π΅ΠΏΡΠ΅ΡΡΠ²Π½ΠΎ ΠΌΠ΅Π½ΡΡΡ ΡΠ²ΠΎΠ΅ Π½Π°ΠΏΡΠ°Π²Π»Π΅Π½ΠΈΠ΅,
ΠΌΠ΅Π½ΡΡΡΡΡ Ρ ΡΠ°ΡΡΠΎΡΠΎΠΉΒ fΒ ΡΠ°Π· Π² ΡΠ΅ΠΊΡΠ½Π΄Ρ.
ΠΡΠΎ ΠΎΠ·Π½Π°ΡΠ°Π΅Ρ, ΡΡΠΎ ΡΠΎΠΊ Π΄ΠΎΡΡΠΈΠ³Π°Π΅Ρ ΠΌΠ°ΠΊΡΠΈΠΌΡΠΌΠ°,
ΡΠΌΠ΅Π½ΡΡΠ°Π΅ΡΡΡ Π΄ΠΎ Π½ΡΠ»Ρ, ΠΌΠ΅Π½ΡΠ΅Ρ Π½Π°ΠΏΡΠ°Π²Π»Π΅Π½ΠΈΠ΅, Π²Π½ΠΎΠ²Ρ
Π΄ΠΎΡΡΠΈΠ³Π°Π΅Ρ ΠΌΠ°ΠΊΡΠΈΠΌΡΠΌΠ° ΠΈ ΡΠΌΠ΅Π½ΡΡΠ°Π΅ΡΡΡ Π΄ΠΎ Π½ΡΠ»Ρ, ΠΈ ΡΠ°ΠΊ
ΠΠ° ΡΠΈΡ. 9-5 ΠΏΠΎΠΊΠ°Π·Π°Π½ ΠΌΠ³Π½ΠΎΠ²Π΅Π½Π½ΡΠΉ ΡΠ½ΠΈΠΌΠΎΠΊ
ΡΡΠ°ΡΡΠΊΠ° ΡΠ»Π΅ΠΊΡΡΠΎΠΌΠ°Π³Π½ΠΈΡΠ½ΠΎΠΉ Π²ΠΎΠ»Π½Ρ. Π‘ ΡΡΠΎΠΉ Π²ΠΎΠ»Π½ΠΎΠΉ
ΡΠ²ΡΠ·Π°Π½Ρ ΡΡΠΈ Π²Π°ΠΆΠ½ΡΡ
Π½Π°ΠΏΡΠ°Π²Π»Π΅Π½ΠΈΡ: Π½Π°ΠΏΡΠ°Π²Π»Π΅Π½ΠΈΠ΅
ΡΠ°ΡΠΏΡΠΎΡΡΡΠ°Π½Π΅Π½ΠΈΡ Π²ΠΎΠ»Π½Ρ (ΠΏΠΎΠΊΠ°Π·Π°Π½Π½ΠΎΠ΅ Π²Π΅ΠΊΡΠΎΡΠΎΠΌ
Π ΠΈΡ. 9-5. ΠΠ»Π΅ΠΊΡΡΠΎΠΌΠ°Π³Π½ΠΈΡΠ½Π°Ρ Π²ΠΎΠ»Π½Π° ΡΠΈΠ½Π΅Π³ΠΎ ΡΠ²Π΅ΡΠ°, ΡΠ°ΡΠΏΡΠΎΡΡΡΠ°Π½ΡΡΡΠ°ΡΡΡ Π² Π½Π°ΠΏΡΠ°Π²Π»Π΅Π½ΠΈΠΈ Π²ΠΎΠ»Π½ΠΎΠ²ΠΎΠ³ΠΎ Π²Π΅ΠΊΡΠΎΡΠ° k. ΠΠΎ Π²ΡΠ΅Π·ΠΊΠ΅ ΠΏΠΎΠΊΠ°Π·Π°Π½Ρ Π²Π·Π°ΠΈΠΌΠ½ΠΎ ΠΏΠ΅ΡΠΏΠ΅Π½Π΄ΠΈΠΊΡΠ»ΡΡΠ½ΡΠ΅ Π½Π°ΠΏΡΠ°Π²Π»Π΅Π½ΠΈΡ ΡΡΡΡ Π²Π΅ΠΊΡΠΎΡΠΎΠ² β k, E, B
Π‘ ΡΠ΅ΡΠ΅Π½ΠΈΠ΅ΠΌ Π²ΡΠ΅ΠΌΠ΅Π½ΠΈ Π²ΠΎΠ»Π½Π° ΡΠ°ΡΠΏΡΠΎΡΡΡΠ°Π½ΡΠ΅ΡΡΡ Π² Π½Π°ΠΏΡΠ°Π²Π»Π΅Π½ΠΈΠΈ Π²Π΅ΠΊΡΠΎΡΠ° k ΡΠΎ ΡΠΊΠΎΡΠΎΡΡΡΡ ΡΠ²Π΅ΡΠ°. Π Π°ΡΡΠΌΠΎΡΡΠΈΠΌ ΡΠΎΡΠΊΡ Π , ΠΌΠΈΠΌΠΎ ΠΊΠΎΡΠΎΡΠΎΠΉ ΠΎΠ½Π° ΠΏΡΠΎΡ ΠΎΠ΄ΠΈΡ. ΠΡΠ° Π²ΠΎΠ»Π½Π° ΡΠΎΡΡΠΎΠΈΡ ΠΈΠ· ΠΊΠΎΠ»Π΅Π±Π»ΡΡΠΈΡ ΡΡ ΡΠ»Π΅ΠΊΡΡΠΈΡΠ΅ΡΠΊΠΎΠ³ΠΎ ΠΈ ΠΌΠ°Π³Π½ΠΈΡΠ½ΠΎΠ³ΠΎ ΠΏΠΎΠ»Π΅ΠΉ, ΠΈ, ΡΡΠΎΠ±Ρ ΠΏΠΎΡΡΠ²ΡΡΠ²ΠΎΠ²Π°ΡΡ ΠΈΡ , Π½ΡΠΆΠ½ΠΎ ΠΏΠΎΠΌΠ΅ΡΡΠΈΡΡ Π² ΡΠΎΡΠΊΡ Π ΠΏΠΎΠ΄Ρ ΠΎΠ΄ΡΡΠ΅Π΅ ΠΏΡΠΎΠ±Π½ΠΎΠ΅ ΡΠ΅Π»ΠΎ. Π§ΡΠΎΠ±Ρ ΡΠ²ΠΈΠ΄Π΅ΡΡ, ΡΡΠΎ ΠΏΡΠΎΠΈΡΡ ΠΎΠ΄ΠΈΡ Ρ ΡΠ»Π΅ΠΊΡΡΠΈΡΠ΅ΡΠΊΠΈΠΌ ΠΏΠΎΠ»Π΅ΠΌ, ΠΏΡΠ΅Π΄ΡΡΠ°Π²ΠΈΠΌ, ΡΡΠΎ Π² ΡΠΎΡΠΊΡ Π ΠΏΠΎΠΌΠ΅ΡΡΠ½ ΡΠ»Π΅ΠΊΡΡΠΎΠ½. ΠΠ½ ΡΡΠ²ΡΡΠ²ΡΠ΅Ρ ΠΊΠΎΠ»Π΅Π±Π»ΡΡΠ΅Π΅ΡΡ ΡΠ»Π΅ΠΊΡΡΠΈΡΠ΅ΡΠΊΠΎΠ΅ ΠΏΠΎΠ»Π΅ ΠΏΡΠΎΠ»Π΅ΡΠ°ΡΡΠ΅ΠΉ ΠΌΠΈΠΌΠΎ Π²ΠΎΠ»Π½Ρ. ΠΠ° Π½Π΅Π³ΠΎ Π΄Π΅ΠΉΡΡΠ²ΡΠ΅Ρ ΡΠΈΠ»Π°, ΠΊΠΎΡΠΎΡΠ°Ρ Π΄ΠΎΡΡΠΈΠ³Π°Π΅Ρ ΠΌΠ°ΠΊΡΠΈΠΌΡΠΌΠ° Π² ΡΠΎΡ ΠΌΠΎΠΌΠ΅Π½Ρ, ΠΊΠΎΠ³Π΄Π° ΡΠΎΡΠΊΡ Π ΠΏΡΠΎΡ ΠΎΠ΄ΠΈΡ Π³ΠΎΡΠ± Π²ΠΎΠ»Π½Ρ (ΡΠΎΡΠΊΠ° Π‘ Π½Π° ΡΠΈΡΡΠ½ΠΊΠ΅).
ΠΠ· ΡΠΈΡ. 9-5 Π²ΠΈΠ΄Π½ΠΎ, ΡΡΠΎ ΠΎΡΡΠΈΠ»Π»ΠΈΡΡΡΡΠ΅Π΅ ΡΠ»Π΅ΠΊΡΡΠΈΡΠ΅ΡΠΊΠΎΠ΅ ΠΏΠΎΠ»Π΅ Π²ΡΡ Π²ΡΠ΅ΠΌΡ Π½Π°Ρ ΠΎΠ΄ΠΈΡΡΡ Π² ΡΠΈΠΊΡΠΈΡΠΎΠ²Π°Π½Π½ΠΎΠΉ ΠΏΠ»ΠΎΡΠΊΠΎΡΡΠΈ. ΠΡΠΎ ΠΆΠ΅ Π²Π΅ΡΠ½ΠΎ ΠΈ Π΄Π»Ρ ΠΌΠ°Π³Π½ΠΈΡΠ½ΠΎΠ³ΠΎ ΠΏΠΎΠ»Ρ Ρ ΡΠΎΠΉ ΡΠ°Π·Π½ΠΈΡΠ΅ΠΉ, ΡΡΠΎ ΠΏΠ»ΠΎΡΠΊΠΎΡΡΡ, Π² ΠΊΠΎΡΠΎΡΠΎΠΉ ΠΊΠΎΠ»Π΅Π±Π»Π΅ΡΡΡ ΠΌΠ°Π³Π½ΠΈΡΠ½ΠΎΠ΅ ΠΏΠΎΠ»Π΅, ΠΏΠ΅ΡΠΏΠ΅Π½Π΄ΠΈΠΊΡΠ»ΡΡΠ½Π° ΠΏΠ»ΠΎΡΠΊΠΎΡΡΠΈ, Π² ΠΊΠΎΡΠΎΡΠΎΠΉ ΠΊΠΎΠ»Π΅Π±Π»Π΅ΡΡΡ ΡΠ»Π΅ΠΊΡΡΠΈΡΠ΅ΡΠΊΠΎΠ΅ ΠΏΠΎΠ»Π΅. ΠΡΠΎ Π²Π΅ΡΠ½ΠΎ Π΄Π»Ρ Π²ΡΠ΅Ρ ΡΠ»Π΅ΠΊΡΡΠΎΠΌΠ°Π³Π½ΠΈΡΠ½ΡΡ Π²ΠΎΠ»Π½. ΠΡΠ»ΠΈ ΠΏΠΎΡΠΌΠΎΡΡΠ΅ΡΡ Π½Π° ΠΏΡΡΠΎΠΊ Π²ΠΎΠ»Π½, ΠΎΠ±ΡΠ°Π·ΡΡΡΠΈΡ ΡΠ²Π΅ΡΠΎΠ²ΠΎΠΉ Π»ΡΡ ΠΎΡ Π‘ΠΎΠ»Π½ΡΠ° ΠΈΠ»ΠΈ ΠΎΡ Π»Π°ΠΌΠΏΡ, ΡΠΎ ΠΎΠ±Π½Π°ΡΡΠΆΠΈΠ²Π°Π΅ΡΡΡ, ΡΡΠΎ ΡΡΠΈ Π΄Π²Π΅ Π²Π·Π°ΠΈΠΌΠ½ΠΎ ΠΏΠ΅ΡΠΏΠ΅Π½Π΄ΠΈΠΊΡΠ»ΡΡΠ½ΡΠ΅ ΠΏΠ»ΠΎΡΠΊΠΎΡΡΠΈ ΠΎΡΠΈΠ΅Π½ΡΠΈΡΠΎΠ²Π°Π½Ρ Π² ΡΠ°Π·Π½ΡΡ Π²ΠΎΠ»Π½Π°Ρ ΠΏΠΎ-ΡΠ°Π·Π½ΠΎΠΌΡ.
ΠΡΠ΅Π΄ΡΡΠ°Π²ΠΈΠ² ΡΠ»Π΅ΠΊΡΡΠΎΠΌΠ°Π³Π½ΠΈΡΠ½ΡΠ΅ Π²ΠΎΠ»Π½Ρ ΠΊΠ°ΠΊ ΡΠΎΠ²ΠΎΠΊΡΠΏΠ½ΠΎΡΡΡ Π²Π·Π°ΠΈΠΌΠΎΡΠ²ΡΠ·Π°Π½Π½ΡΡ ΠΊΠΎΠ»Π΅Π±Π»ΡΡΠΈΡ ΡΡ ΡΠ»Π΅ΠΊΡΡΠΈΡΠ΅ΡΠΊΠΎΠ³ΠΎ ΠΈ ΠΌΠ°Π³Π½ΠΈΡΠ½ΠΎΠ³ΠΎ ΠΏΠΎΠ»Π΅ΠΉ, ΠΌΡ Π΄ΠΎΡΡΠΈΠ³Π»ΠΈ Π·Π½Π°ΡΠΈΡΠ΅Π»ΡΠ½ΠΎΠ³ΠΎ ΡΠΏΡΠΎΡΠ΅Π½ΠΈΡ. ΠΠΎΠ½Π΅ΡΠ½ΠΎ, ΡΡΠΎΠ±Ρ Π²ΠΎΠ»Π½Π° Π²ΠΎΠ·Π½ΠΈΠΊΠ»Π°, Π½ΡΠΆΠ΅Π½ ΠΎΠΏΡΠ΅Π΄Π΅Π»ΡΠ½Π½ΡΠΉ ΠΈΡΡΠΎΡΠ½ΠΈΠΊ β Π»Π°ΠΌΠΏΠΎΡΠΊΠ° Π΄Π»Ρ Π²ΠΈΠ΄ΠΈΠΌΠΎΠ³ΠΎ ΡΠ²Π΅ΡΠ°, ΡΠ°Π΄ΠΈΠΎΠ°ΠΊΡΠΈΠ²Π½ΠΎΠ΅ ΡΠ΄ΡΠΎ Π΄Π»Ρ Π³Π°ΠΌΠΌΠ°-ΠΈΠ·Π»ΡΡΠ΅Π½ΠΈΡ ΠΈ Ρ.ΠΏ. ΠΠ°ΠΌ ΡΠ΅ΠΏΠ΅ΡΡ Π½Π΅ Π½ΡΠΆΠ½ΠΎ Π΄ΡΠΌΠ°ΡΡ ΠΎ ΡΠ»ΠΎΠΆΠ½ΡΡ ΠΏΡΠΎΡΠ΅ΡΡΠ°Ρ , ΠΏΡΠΎΠΈΡΡ ΠΎΠ΄ΡΡΠΈΡ Π² ΠΈΡΡΠΎΡΠ½ΠΈΠΊΠ΅ ΠΈ ΠΏΡΠΈΠ²ΠΎΠ΄ΡΡΠΈΡ ΠΊ ΠΎΠ±ΡΠ°Π·ΠΎΠ²Π°Π½ΠΈΡ Π²ΠΎΠ»Π½, ΡΡΠΎΠ±Ρ ΠΏΠΎΠ½ΡΡΡ, ΡΡΠΎ ΠΏΡΠΎΠΈΡΡ ΠΎΠ΄ΠΈΡ ΠΏΡΠΈ ΠΏΠ°Π΄Π΅Π½ΠΈΠΈ Π²ΠΎΠ»Π½Ρ Π½Π° ΡΠ²ΡΡΠ΄ΠΎΠ΅ ΡΠ΅Π»ΠΎ. Π’Π΅ΠΌ Π½Π΅ ΠΌΠ΅Π½Π΅Π΅ ΠΌΡ ΠΊΠΎΡΠΎΡΠΊΠΎ ΠΊΠΎΡΠ½ΡΠΌΡΡ Π² ΡΠ»Π΅Π΄ΡΡΡΠ΅ΠΌ ΡΠ°Π·Π΄Π΅Π»Π΅ Π΄Π²ΡΡ ΡΠΏΠΎΡΠΎΠ±ΠΎΠ² Π³Π΅Π½Π΅ΡΠ°ΡΠΈΠΈ ΡΠ»Π΅ΠΊΡΡΠΎΠΌΠ°Π³Π½ΠΈΡΠ½ΡΡ Π²ΠΎΠ»Π½ Π²Π΅ΡΠ΅ΡΡΠ²ΠΎΠΌ, Ρ.Π΅. ΠΎΠ±ΡΡΠ΄ΠΈΠΌ, ΠΊΠ°ΠΊ Π²ΠΎΠ»Π½Π° ΠΏΠΎΠΊΠΈΠ΄Π°Π΅Ρ ΠΈΡΡΠΎΡΠ½ΠΈΠΊ.
4. ΠΠ΅Π½Π΅ΡΠ°ΡΠΈΡ ΡΠ²Π΅ΡΠ°
ΠΡΠ΅ Π½Π΅ΠΎΠ΄Π½ΠΎΠΊΡΠ°ΡΠ½ΠΎ Π²ΠΈΠ΄Π΅Π»ΠΈ Π½Π΅ΠΎΠ½ΠΎΠ²ΡΠ΅ ΡΡΡΠ±ΠΊΠΈ, ΠΈΡΠΏΡΡΠΊΠ°ΡΡΠΈΠ΅ ΠΊΡΠ°ΡΠ½ΡΠΉ ΡΠ²Π΅Ρ ΠΈ ΠΈΡΠΏΠΎΠ»ΡΠ·ΡΡΡΠΈΠ΅ΡΡ Π² ΡΠ΅ΠΊΠ»Π°ΠΌΠ½ΡΡ ΡΠΈΡΠ°Ρ . Π ΡΠ°ΠΊΠΎΠΉ ΡΡΡΠ±ΠΊΠ΅ ΡΠΎΠ΄Π΅ΡΠΆΠΈΡΡΡ Π³Π°Π· Π½Π΅ΠΎΠ½, ΠΈ ΠΎΠ½Π° ΡΡΡΡΠΎΠ΅Π½Π° ΡΠ°ΠΊ, ΡΡΠΎ ΠΏΡΠΈ ΠΏΡΠΈΠ»ΠΎΠΆΠ΅Π½ΠΈΠΈ Π½Π°ΠΏΡΡΠΆΠ΅Π½ΠΈΡ ΠΌΠ΅ΠΆΠ΄Ρ Π΅Ρ ΠΊΠΎΠ½ΡΠ°ΠΌΠΈ ΡΠ΅ΡΠ΅Π· ΡΡΡΠ±ΠΊΡ Π½Π°ΡΠΈΠ½Π°Π΅Ρ ΠΈΠ΄ΡΠΈ ΡΠΎΠΊ ΡΠ»Π΅ΠΊΡΡΠΎΠ½ΠΎΠ². ΠΡΠΈ ΡΠ»Π΅ΠΊΡΡΠΎΠ½Ρ ΡΡΠ°Π»ΠΊΠΈΠ²Π°ΡΡΡΡ Ρ Π°ΡΠΎΠΌΠ°ΠΌΠΈ Π½Π΅ΠΎΠ½Π° ΠΈ ΠΏΠ΅ΡΠ΅Π΄Π°ΡΡ ΠΈΠΌ ΡΠ°ΡΡΡ ΡΠ²ΠΎΠ΅ΠΉ ΠΊΠΈΠ½Π΅ΡΠΈΡΠ΅ΡΠΊΠΎΠΉ ΡΠ½Π΅ΡΠ³ΠΈΠΈ. Π ΡΠ΅Π·ΡΠ»ΡΡΠ°ΡΠ΅ Π°ΡΠΎΠΌΡ Π½Π΅ΠΎΠ½Π° ΠΏΠ΅ΡΠ΅Ρ ΠΎΠ΄ΡΡ Π² Π±ΠΎΠ»Π΅Π΅ Π²ΡΡΠΎΠΊΠΎΠ΅ ΠΊΠ²Π°Π½ΡΠΎΠ²ΠΎΠ΅ ΡΠ½Π΅ΡΠ³Π΅ΡΠΈΡΠ΅ΡΠΊΠΎΠ΅ ΡΠΎΡΡΠΎΡΠ½ΠΈΠ΅. ΠΠ°ΡΠ΅ΠΌ Π°ΡΠΎΠΌΡ ΡΠ±ΡΠ°ΡΡΠ²Π°ΡΡ Π»ΠΈΡΠ½ΡΡ ΡΠ½Π΅ΡΠ³ΠΈΡ, ΠΈΡΠΏΡΡΠΊΠ°Ρ ΡΠΎΡΠΎΠ½ ΠΈ Π²ΠΎΠ·Π²ΡΠ°ΡΠ°ΡΡΡ Π² ΠΈΡΡ ΠΎΠ΄Π½ΠΎΠ΅ Π½Π΅Π²ΠΎΠ·Π±ΡΠΆΠ΄ΡΠ½Π½ΠΎΠ΅ ΡΠΎΡΡΠΎΡΠ½ΠΈΠ΅, ΠΏΠΎΡΠ»Π΅ ΡΠ΅Π³ΠΎ ΠΏΡΠΎΡΠ΅ΡΡ ΠΏΠΎΠ²ΡΠΎΡΡΠ΅ΡΡΡ. Π’Π°ΠΊΠΈΠΌ ΠΎΠ±ΡΠ°Π·ΠΎΠΌ, ΡΠΎΡΠΎΠ½Ρ ΠΌΠΎΠ³ΡΡ ΠΈΠΌΠ΅ΡΡ ΡΠΎΠ»ΡΠΊΠΎ ΡΠ΅ ΡΠ½Π΅ΡΠ³ΠΈΠΈ (ΠΈ ΡΠΎΠΎΡΠ²Π΅ΡΡΡΠ²ΡΡΡΠΈΠ΅ ΡΠ°ΡΡΠΎΡΡ), ΠΊΠΎΡΠΎΡΡΠ΅ ΡΠ²Π»ΡΡΡΡΡ ΡΠ°Π·Π½ΠΎΡΡΡΠΌΠΈ ΡΠ½Π΅ΡΠ³ΠΈΠΉ Π²ΠΎΠ·Π±ΡΠΆΠ΄ΡΠ½Π½ΡΡ ΡΠ½Π΅ΡΠ³Π΅ΡΠΈΡΠ΅ΡΠΊΠΈΡ ΡΠΎΡΡΠΎΡΠ½ΠΈΠΉ Π°ΡΠΎΠΌΠ° ΠΈ ΠΎΡΠ½ΠΎΠ²Π½ΠΎΠ³ΠΎ ΡΠΎΡΡΠΎΡΠ½ΠΈΡ. Π ΡΠ»ΡΡΠ°Π΅ Π½Π΅ΠΎΠ½Π° Π±ΠΎΠ»ΡΡΠΈΠ½ΡΡΠ²ΠΎ ΡΠΎΡΠΎΠ½ΠΎΠ² ΠΈΠ·Π»ΡΡΠ°ΡΡΡΡ Π² ΡΠ΅Π·ΡΠ»ΡΡΠ°ΡΠ΅ ΠΏΠ΅ΡΠ΅Ρ ΠΎΠ΄Π° ΠΈΠ· ΡΠΎΡΡΠΎΡΠ½ΠΈΡ, ΡΠ½Π΅ΡΠ³ΠΈΡ ΠΊΠΎΡΠΎΡΠΎΠ³ΠΎ ΠΎΡΠ»ΠΈΡΠ°Π΅ΡΡΡ ΠΎΡ ΡΠ½Π΅ΡΠ³ΠΈΠΈ ΠΎΡΠ½ΠΎΠ²Π½ΠΎΠ³ΠΎ ΡΠΎΡΡΠΎΡΠ½ΠΈΡ Π½Π΅ΠΎΠ½Π° Π½Π° Π²Π΅Π»ΠΈΡΠΈΠ½Ρ, ΡΠΎΠΎΡΠ²Π΅ΡΡΡΠ²ΡΡΡΡΡ ΡΠ½Π΅ΡΠ³ΠΈΠΈ ΡΠΎΡΠΎΠ½ΠΎΠ² ΠΊΡΠ°ΡΠ½ΠΎΠ³ΠΎ ΡΠ²Π΅ΡΠ°. Π Π΄ΡΡΠ³ΠΈΡ Π³Π°Π·Π°Ρ Π΄ΠΎΠΌΠΈΠ½ΠΈΡΡΡΡ ΡΠΎΡΠΎΠ½Ρ Π΄ΡΡΠ³ΠΈΡ ΡΠ²Π΅ΡΠΎΠ².
ΠΠΎ ΡΠ²Π΅Ρ ΠΈΠ·Π»ΡΡΠ°Π΅ΡΡΡ ΠΈ ΡΠΈΠ»ΡΠ½ΠΎ Π½Π°Π³ΡΠ΅ΡΡΠΌ ΡΠ΅Π»ΠΎΠΌ. ΠΡΠΈΠΌΠ΅ΡΠ°ΠΌΠΈ ΠΌΠΎΠ³ΡΡ ΡΠ»ΡΠΆΠΈΡΡ Π»Π°ΠΌΠΏΠ° Π½Π°ΠΊΠ°Π»ΠΈΠ²Π°Π½ΠΈΡ, Π² ΠΊΠΎΡΠΎΡΠΎΠΉ ΡΠΎΠ½ΠΊΠ°Ρ Π²ΠΎΠ»ΡΡΡΠ°ΠΌΠΎΠ²Π°Ρ Π½ΠΈΡΡ ΡΠ°ΡΠΊΠ°Π»ΡΠ΅ΡΡΡ Π² ΡΠ΅Π·ΡΠ»ΡΡΠ°ΡΠ΅ ΠΏΡΠΎΡ ΠΎΠΆΠ΄Π΅Π½ΠΈΡ ΠΏΠΎ Π½Π΅ΠΉ ΡΠΎΠΊΠ°, ΠΈΠ»ΠΈ ΠΏΠΎΠ²Π΅ΡΡ Π½ΠΎΡΡΡ Π‘ΠΎΠ»Π½ΡΠ°, ΡΠ΅ΠΌΠΏΠ΅ΡΠ°ΡΡΡΠ° ΠΊΠΎΡΠΎΡΠΎΠΉ ΠΏΠΎΠ΄Π΄Π΅ΡΠΆΠΈΠ²Π°Π΅ΡΡΡ ΠΏΡΠΎΠΈΡΡ ΠΎΠ΄ΡΡΠΈΠΌΠΈ Π²Π½ΡΡΡΠΈ ΡΠ΅ΡΠΌΠΎΡΠ΄Π΅ΡΠ½ΡΠΌΠΈ ΠΏΡΠΎΡΠ΅ΡΡΠ°ΠΌΠΈ. ΠΡΠ»ΠΈ ΠΏΡΠΎΠΏΡΡΡΠΈΡΡ ΡΠ°ΠΊΠΎΠΉ ΡΠ²Π΅Ρ ΡΠ΅ΡΠ΅Π· ΡΡΠ΅ΠΊΠ»ΡΠ½Π½ΡΡ ΠΏΡΠΈΠ·ΠΌΡ, ΠΏΠΎ-ΡΠ°Π·Π½ΠΎΠΌΡ ΠΎΡΠΊΠ»ΠΎΠ½ΡΡΡΡΡ ΡΠ²Π΅Ρ ΡΠ°Π·Π½ΡΡ ΡΠ°ΡΡΠΎΡ, ΠΏΠΎΠ»ΡΡΠΈΡΡΡ Π½Π΅ΠΏΡΠ΅ΡΡΠ²Π½ΠΎΠ΅ ΡΠ°ΡΠΏΡΠ΅Π΄Π΅Π»Π΅Π½ΠΈΠ΅ ΡΠ²Π΅ΡΠΎΠ² ΠΎΡ ΡΠΈΠΎΠ»Π΅ΡΠΎΠ²ΠΎΠ³ΠΎ Π΄ΠΎ ΠΊΡΠ°ΡΠ½ΠΎΠ³ΠΎ. Π‘ ΠΏΠΎΠΌΠΎΡΡΡ ΡΠΎΠΎΡΠ²Π΅ΡΡΡΠ²ΡΡΡΠΈΡ ΠΏΡΠΈΠ±ΠΎΡΠΎΠ² ΠΌΠΎΠΆΠ½ΠΎ ΡΡΡΠ°Π½ΠΎΠ²ΠΈΡΡ, ΡΡΠΎ ΡΠ²Π΅Ρ ΡΠΎΠ΄Π΅ΡΠΆΠΈΡ ΡΠ°ΠΊΠΆΠ΅ ΠΊΠΎΠΌΠΏΠΎΠ½Π΅Π½ΡΡ Ρ ΡΠ°ΡΡΠΎΡΠ°ΠΌΠΈ, Π½Π΅Π²ΠΈΠ΄ΠΈΠΌΡΠΌΠΈ ΡΠ΅Π»ΠΎΠ²Π΅ΡΠ΅ΡΠΊΠΈΠΌ Π³Π»Π°Π·ΠΎΠΌ, β ΠΈΠ½ΡΡΠ°ΠΊΡΠ°ΡΠ½ΡΠ΅ (ΡΠ°ΡΡΠΎΡΡ Π½ΠΈΠΆΠ΅, ΡΠ΅ΠΌ Ρ ΠΊΡΠ°ΡΠ½ΠΎΠ³ΠΎ ΡΠ²Π΅ΡΠ°) ΠΈ ΡΠ»ΡΡΡΠ°ΡΠΈΠΎΠ»Π΅ΡΠΎΠ²ΡΠ΅ (ΡΠ°ΡΡΠΎΡΡ Π²ΡΡΠ΅, ΡΠ΅ΠΌ Ρ ΡΠΈΠΎΠ»Π΅ΡΠΎΠ²ΠΎΠ³ΠΎ ΡΠ²Π΅ΡΠ°). ΠΠ° ΡΠ°ΠΌΠΎΠΌ Π΄Π΅Π»Π΅ Π² ΡΠ²Π΅ΡΠ΅ Π»Π°ΠΌΠΏΠΎΡΠΊΠΈ ΠΈΠ»ΠΈ Π‘ΠΎΠ»Π½ΡΠ° ΡΠΎΠ΄Π΅ΡΠΆΠΈΡΡΡ ΠΏΠΎΠ»Π½ΡΠΉ ΡΠΏΠ΅ΠΊΡΡ ΡΠ»Π΅ΠΊΡΡΠΎΠΌΠ°Π³Π½ΠΈΡΠ½ΡΡ Π²ΠΎΠ»Π½, Ρ ΠΎΡΡ Π±ΠΎΠ»ΡΡΠΈΠ½ΡΡΠ²ΠΎ ΡΠΎΡΠΎΠ½ΠΎΠ² ΡΠΎΠ»Π½Π΅ΡΠ½ΠΎΠ³ΠΎ ΡΠ²Π΅ΡΠ° ΠΏΡΠΈΠ½Π°Π΄Π»Π΅ΠΆΠ°Ρ ΠΈΠ½ΡΠ΅ΡΠ²Π°Π»Ρ Π²ΠΈΠ΄ΠΈΠΌΠΎΠ³ΠΎ ΡΠ²Π΅ΡΠ°, Π° ΡΠ΅ΠΌ Π΄Π°Π»ΡΡΠ΅ ΠΎΡ ΡΡΠΎΠ³ΠΎ ΠΈΠ½ΡΠ΅ΡΠ²Π°Π»Π°, ΡΠ΅ΠΌ ΡΠΈΡΠ»ΠΎ ΡΠΎΡΠΎΠ½ΠΎΠ² ΡΠΎΠΎΡΠ²Π΅ΡΡΡΠ²ΡΡΡΠΈΡ ΡΠ°ΡΡΠΎΡ ΡΡΠ°Π½ΠΎΠ²ΠΈΡΡΡ ΠΌΠ΅Π½ΡΡΠ΅.
ΠΡΠΊΡΠ΄Π° ΠΆΠ΅ Π²ΠΎΠ·Π½ΠΈΠΊΠ°Π΅Ρ ΡΠ°ΠΊΠΎΠΉ Π½Π΅ΠΏΡΠ΅ΡΡΠ²Π½ΡΠΉ ΡΠΏΠ΅ΠΊΡΡ ΡΠ°ΡΡΠΎΡ? ΠΡ Π·Π½Π°Π΅ΠΌ, ΡΡΠΎ ΡΠ΅ΠΏΠ»ΠΎΠ²Π°Ρ ΡΠ½Π΅ΡΠ³ΠΈΡ ΡΠ΅Π»Π° β ΡΡΠΎ ΡΠ½Π΅ΡΠ³ΠΈΡ Π΄Π²ΠΈΠΆΠ΅Π½ΠΈΡ ΡΠΎΡΡΠ°Π²Π»ΡΡΡΠΈΡ ΡΠ΅Π»ΠΎ ΡΠ°ΡΡΠΈΡ (Π°ΡΠΎΠΌΠΎΠ², ΡΠ΄Π΅Ρ ΠΈ Ρ.ΠΏ.). ΠΠ½Π΅ΡΠ³ΠΈΠΈ ΠΎΡΠ΄Π΅Π»ΡΠ½ΡΡ ΡΠ°ΡΡΠΈΡ ΠΌΠ΅Π½ΡΡΡΡΡ ΠΎΡ Π½ΡΠ»Ρ Π΄ΠΎ ΠΎΡΠ΅Π½Ρ Π±ΠΎΠ»ΡΡΠΈΡ Π·Π½Π°ΡΠ΅Π½ΠΈΠΉ, Π° ΡΠ°ΡΠΏΡΠ΅Π΄Π΅Π»Π΅Π½ΠΈΠ΅ ΡΠ°ΡΡΠΈΡ ΠΏΠΎ ΡΠ°Π·Π½ΡΠΌ ΠΈΠ½ΡΠ΅ΡΠ²Π°Π»Π°ΠΌ ΡΠ½Π΅ΡΠ³ΠΈΠΈ Π·Π°Π²ΠΈΡΠΈΡ ΠΎΡ ΡΠ΅ΠΌΠΏΠ΅ΡΠ°ΡΡΡΡ ΡΠ΅Π»Π°. ΠΡΡ ΡΡΠΎ ΠΎΠ±ΡΡΠΆΠ΄Π°Π»ΠΎΡΡ Π² Π³Π». 6. ΠΠΌΠ΅Π½Π½ΠΎ ΡΡΠΎ ΠΎΡΠ΅Π½Ρ ΡΠ»ΠΎΠΆΠ½ΠΎΠ΅ ΡΠ΅ΠΏΠ»ΠΎΠ²ΠΎΠ΅ Π΄Π²ΠΈΠΆΠ΅Π½ΠΈΠ΅ ΡΠ»Π΅ΠΊΡΡΠΈΡΠ΅ΡΠΊΠΈΡ Π·Π°ΡΡΠ΄ΠΎΠ², ΠΊΠΎΠ½ΠΊΡΠ΅ΡΠ½ΠΎ ΡΠ»Π΅ΠΊΡΡΠΎΠ½ΠΎΠ² ΠΈ ΡΠ΄Π΅Ρ, ΠΏΠΎΡΠΎΠΆΠ΄Π°Π΅Ρ Π½Π΅ΠΏΡΠ΅ΡΡΠ²Π½ΡΠΉ ΡΠΏΠ΅ΠΊΡΡ ΡΠ»Π΅ΠΊΡΡΠΎΠΌΠ°Π³Π½ΠΈΡΠ½ΠΎΠ³ΠΎ ΠΈΠ·Π»ΡΡΠ΅Π½ΠΈΡ, ΠΊΠΎΡΠΎΡΠΎΠ΅ Π½Π°Π·ΡΠ²Π°Π΅ΡΡΡ ΠΏΠΎΡΡΠΎΠΌΡ ΡΠ΅ΠΏΠ»ΠΎΠ²ΡΠΌ ΠΈΠ·Π»ΡΡΠ΅Π½ΠΈΠ΅ΠΌ. ΠΡ Π·Π½Π°Π΅ΠΌ, ΡΡΠΎ Π΅ΡΠ»ΠΈ ΡΠ΅ΠΌΠΏΠ΅ΡΠ°ΡΡΡΠ° ΡΠ΅Π»Π° ΡΠ°Π²Π½Π° Π’Β ΠΊΠ΅Π»ΡΠ²ΠΈΠ½ΠΎΠ², ΡΠΎ ΡΡΠ΅Π΄Π½ΡΡ ΡΠ½Π΅ΡΠ³ΠΈΡ ΡΠ΅ΠΏΠ»ΠΎΠ²ΠΎΠ³ΠΎ Π΄Π²ΠΈΠΆΠ΅Π½ΠΈΡ, ΠΏΡΠΈΡ ΠΎΠ΄ΡΡΠ°ΡΡΡ Π½Π° ΠΎΠ΄Π½Ρ ΡΠ°ΡΡΠΈΡΡ, ΡΠΎΡΡΠ°Π²Π»ΡΠ΅Ρ ΠΎΠΊΠΎΠ»ΠΎ kT, Π³Π΄Π΅ k β ΠΏΠΎΡΡΠΎΡΠ½Π½Π°Ρ ΠΠΎΠ»ΡΡΠΌΠ°Π½Π°. ΠΡΠ΄Π΅ΠΌ Π½Π°Π·ΡΠ²Π°ΡΡ ΡΡΡ Π²Π΅Π»ΠΈΡΠΈΠ½Ρ ΡΠ΅ΠΏΠ»ΠΎΠ²ΠΎΠΉ ΡΠ½Π΅ΡΠ³ΠΈΠ΅ΠΉ, ΠΏΡΠΈΡ ΠΎΠ΄ΡΡΠ΅ΠΉΡΡ Π½Π° ΠΎΠ΄Π½Ρ ΡΠ°ΡΡΠΈΡΡ. ΠΡΠ»ΠΈ Π²ΡΡΠ°Π·ΠΈΡΡ ΡΠ½Π΅ΡΠ³ΠΈΡ ΡΠΎΡΠΎΠ½ΠΎΠ² Π² Π΄ΠΎΠ»ΡΡ ΡΠ΅ΠΏΠ»ΠΎΠ²ΠΎΠΉ ΡΠ½Π΅ΡΠ³ΠΈΠΈ, ΠΏΡΠΈΡ ΠΎΠ΄ΡΡΠ΅ΠΉΡΡ Π½Π° ΠΎΠ΄Π½Ρ ΡΠ°ΡΡΠΈΡΡ, ΠΈ ΠΏΠΎΡΡΡΠΎΠΈΡΡ ΡΠ°ΡΠΏΡΠ΅Π΄Π΅Π»Π΅Π½ΠΈΠ΅ ΡΠΎΡΠΎΠ½ΠΎΠ² ΡΠ΅ΠΏΠ»ΠΎΠ²ΠΎΠ³ΠΎ ΠΈΠ·Π»ΡΡΠ΅Π½ΠΈΡ Π² ΡΠ°Π·Π½ΡΡ Π΄ΠΈΠ°ΠΏΠ°Π·ΠΎΠ½Π°Ρ ΡΠ½Π΅ΡΠ³ΠΈΠΈ, ΡΠΎ ΠΏΠΎΠ»ΡΡΠΈΡΡΡ ΠΊΠ°ΡΡΠΈΠ½Π°, ΠΏΠΎΠΊΠ°Π·Π°Π½Π½Π°Ρ Π½Π° ΡΠΈΡ. 9-6. Π€ΠΎΡΠΌΠ° ΡΡΠΎΠ³ΠΎ ΡΠ°ΡΠΏΡΠ΅Π΄Π΅Π»Π΅Π½ΠΈΡ ΠΎΠ±ΡΡΠ»ΠΎΠ²Π»Π΅Π½Π° ΡΠ΅ΠΌ, ΡΡΠΎ ΡΠΎΡΠΎΠ½Ρ ΡΠ²Π»ΡΡΡΡΡ Π±ΠΎΠ·ΠΎΠ½Π°ΠΌΠΈ, ΠΈ ΠΎΡΡΠ°ΡΡΡΡ Π½Π΅ΠΈΠ·ΠΌΠ΅Π½Π½ΠΎΠΉ ΠΏΡΠΈ Π»ΡΠ±ΠΎΠΉ ΡΠ΅ΠΌΠΏΠ΅ΡΠ°ΡΡΡΠ΅ ΡΠ΅Π»Π°, Π±ΡΠ΄Ρ ΡΠΎ 3000 Π Ρ Π»Π°ΠΌΠΏΠΎΡΠΊΠΈ Π½Π°ΠΊΠ°Π»ΠΈΠ²Π°Π½ΠΈΡ, 6000 Π Ρ ΠΏΠΎΠ²Π΅ΡΡ Π½ΠΎΡΡΠΈ Π‘ΠΎΠ»Π½ΡΠ° ΠΈ Ρ.Π΄. ΠΠ°ΠΊ Π²ΠΈΠ΄Π½ΠΎ, Π½Π°ΠΈΠ±ΠΎΠ»ΡΡΠ΅Π΅ ΡΠΈΡΠ»ΠΎ ΡΠΎΡΠΎΠ½ΠΎΠ² ΠΈΠΌΠ΅ΡΡ ΡΠ½Π΅ΡΠ³ΠΈΠΈ, ΠΏΡΠΈΠΌΠ΅ΡΠ½ΠΎ Π² ΡΡΠΈ ΡΠ°Π·Π° Π±ΠΎΠ»ΡΡΠΈΠ΅ ΡΠ΅ΠΏΠ»ΠΎΠ²ΠΎΠΉ ΡΠ½Π΅ΡΠ³ΠΈΠΈ. Π’Π°ΠΊ ΠΊΠ°ΠΊ ΡΠ°ΡΡΠΎΡΠ° ΡΠΎΡΠΎΠ½ΠΎΠ² ΠΏΡΠΎΠΏΠΎΡΡΠΈΠΎΠ½Π°Π»ΡΠ½Π° ΠΈΡ ΡΠ½Π΅ΡΠ³ΠΈΠΈ, ΡΡΠΎ ΠΎΠ·Π½Π°ΡΠ°Π΅Ρ, Π½Π°ΠΏΡΠΈΠΌΠ΅Ρ, ΡΡΠΎ Π½Π°ΠΈΠ±ΠΎΠ»ΡΡΠ΅Π΅ ΡΠΈΡΠ»ΠΎ ΡΠΎΡΠΎΠ½ΠΎΠ² ΡΠΎΠ»Π½Π΅ΡΠ½ΠΎΠ³ΠΎ ΡΠ²Π΅ΡΠ° ΠΏΡΠΈΡ ΠΎΠ΄ΠΈΡΡΡ Π½Π° ΡΠ°ΡΡΠΎΡΡ, Π²Π΄Π²ΠΎΠ΅ Π±ΠΎΠ»ΡΡΡΡ, ΡΠ΅ΠΌ ΡΠ°ΡΡΠΎΡΠ° ΡΠ²Π΅ΡΠ° ΠΎΡ ΡΠ»Π΅ΠΊΡΡΠΈΡΠ΅ΡΠΊΠΎΠΉ Π»Π°ΠΌΠΏΠΎΡΠΊΠΈ. ΠΡΠ»ΠΈ ΠΏΠΎΠ΄ΡΡΠ°Π²ΠΈΡΡ ΡΠ΅Π°Π»ΡΠ½ΡΠ΅ ΡΠΈΡΡΡ, ΡΠΎ ΠΏΠΎΠ»ΡΡΠΈΡΡΡ, ΡΡΠΎ Π±ΠΎΠ»ΡΡΠΈΠ½ΡΡΠ²ΠΎ ΡΠΎΡΠΎΠ½ΠΎΠ² ΠΎΡ Π‘ΠΎΠ»Π½ΡΠ° ΠΎΡΠ½ΠΎΡΡΡΡΡ ΠΊ ΠΊΡΠ°ΡΠ½ΠΎΠΉ ΡΠ°ΡΡΠΈ ΡΠΏΠ΅ΠΊΡΡΠ° (Π΄Π»ΠΈΠ½Π° Π²ΠΎΠ»Π½Ρ ΠΏΡΠΈΠΌΠ΅ΡΠ½ΠΎ ΡΠ°Π²Π½Π° 8.10β5 ΡΠΌ), Π° Π±ΠΎΠ»ΡΡΠΈΠ½ΡΡΠ²ΠΎ ΡΠΎΡΠΎΠ½ΠΎΠ² ΡΠ²Π΅ΡΠ° ΡΠ»Π΅ΠΊΡΡΠΈΡΠ΅ΡΠΊΠΎΠΉ Π»Π°ΠΌΠΏΠΎΡΠΊΠΈ ΠΎΡΠ½ΠΎΡΡΡΡΡ ΠΊ ΠΈΠ½ΡΡΠ°ΠΊΡΠ°ΡΠ½ΠΎΠΉ ΡΠ°ΡΡΠΈ ΡΠΏΠ΅ΠΊΡΡΠ° (1,6.10β4 ΡΠΌ).
Π ΠΈΡ. 9-6. Π Π°ΡΠΏΡΠ΅Π΄Π΅Π»Π΅Π½ΠΈΠ΅ ΡΠ½Π΅ΡΠ³ΠΈΠΈ ΡΠ΅ΠΏΠ»ΠΎΠ²ΠΎΠ³ΠΎ ΠΈΠ·Π»ΡΡΠ΅Π½ΠΈΡ, ΠΏΠΎΠΊΠ°Π·ΡΠ²Π°ΡΡΠ΅Π΅ ΠΎΡΠ½ΠΎΡΠΈΡΠ΅Π»ΡΠ½ΠΎΠ΅ ΡΠΈΡΠ»ΠΎ ΡΠΎΡΠΎΠ½ΠΎΠ² Π² ΡΠ°Π·Π½ΡΡ ΠΈΠ½ΡΠ΅ΡΠ²Π°Π»Π°Ρ ΡΠ½Π΅ΡΠ³ΠΈΠΈ ΠΏΡΠΈ Π»ΡΠ±ΠΎΠΉ ΡΠ΅ΠΌΠΏΠ΅ΡΠ°ΡΡΡΠ΅ Π’. ΠΠ° Π³ΠΎΡΠΈΠ·ΠΎΠ½ΡΠ°Π»ΡΠ½ΠΎΠΉ ΠΎΡΠΈ ΠΎΡΠ»ΠΎΠΆΠ΅Π½ΠΎ ΠΎΡΠ½ΠΎΡΠ΅Π½ΠΈΠ΅ ΡΠ½Π΅ΡΠ³ΠΈΠΈ ΡΠΎΡΠΎΠ½Π° ΠΊ ΡΠ΅ΠΏΠ»ΠΎΠ²ΠΎΠΉ ΡΠ½Π΅ΡΠ³ΠΈΠΈ kT. ΠΠ΅ΡΠΌΠΎΡΡΡ Π½Π° ΡΠΎ, ΡΡΠΎ ΠΏΡΠΈ Π»ΡΠ±ΠΎΠΉ ΡΠ΅ΠΌΠΏΠ΅ΡΠ°ΡΡΡΠ΅ Π½Π°Π³ΡΠ΅ΡΠΎΠ³ΠΎ ΡΠ΅Π»Π° ΡΠ΅ΠΏΠ»ΠΎΠ²ΠΎΠ΅ ΠΈΠ·Π»ΡΡΠ΅Π½ΠΈΠ΅ ΡΠΎΠ΄Π΅ΡΠΆΠΈΡ ΡΠΎΡΠΎΠ½Ρ Π²ΡΠ΅Ρ ΡΠ°ΡΡΠΎΡ, Π±ΠΎΠ»ΡΡΠΈΠ½ΡΡΠ²ΠΎ ΠΈΠ· Π½ΠΈΡ ΠΏΠΎΠΏΠ°Π΄Π°Π΅Ρ Π² ΠΈΠ½ΡΠ΅ΡΠ²Π°Π» ΡΠ½Π΅ΡΠ³ΠΈΠΉ Π² ΠΎΠΊΡΠ΅ΡΡΠ½ΠΎΡΡΠΈ Π·Π½Π°ΡΠ΅Π½ΠΈΡ 3kT. ΠΠ»Ρ Π‘ΠΎΠ»Π½ΡΠ°, ΡΠ΅ΠΌΠΏΠ΅ΡΠ°ΡΡΡΠ° ΠΏΠΎΠ²Π΅ΡΡ Π½ΠΎΡΡΠΈ ΠΊΠΎΡΠΎΡΠΎΠ³ΠΎ ΡΠ°Π²Π½Π° 6000 Π, ΡΡΠΎΡ ΠΈΠ½ΡΠ΅ΡΠ²Π°Π» ΡΠΎΠΎΡΠ²Π΅ΡΡΡΠ²ΡΠ΅Ρ ΠΊΡΠ°ΡΠ½ΠΎΠΉ ΠΈ ΠΈΠ½ΡΡΠ°ΠΊΡΠ°ΡΠ½ΠΎΠΉ ΡΠ°ΡΡΡΠΌ Π²ΠΈΠ΄ΠΈΠΌΠΎΠ³ΠΎ ΡΠ²Π΅ΡΠ°
ΠΠΎΠΏΡΠΎΡ. Π§ΠΈΡΠ°Ρ ΡΡΠΎΡ ΡΠ΅ΠΊΡΡ, Π²Ρ, ΠΊΠ°ΠΊ ΠΈ ΠΎΠΊΡΡΠΆΠ°ΡΡΠΈΠ΅ Π²Π°Ρ ΡΠ΅Π»Π°, ΡΠ°ΠΊΠΆΠ΅ ΠΈΡΠΏΡΡΠΊΠ°Π΅ΡΠ΅ Π½Π΅ΠΏΡΠ΅ΡΡΠ²Π½ΡΠΉ ΡΠΏΠ΅ΠΊΡΡ ΡΠ»Π΅ΠΊΡΡΠΎΠΌΠ°Π³Π½ΠΈΡΠ½ΠΎΠ³ΠΎ ΠΈΠ·Π»ΡΡΠ΅Π½ΠΈΡ. Π§Π΅ΠΌΡ ΡΠ°Π²Π½Π° ΡΠ°ΡΡΠΎΡΠ° Π±ΠΎΠ»ΡΡΠΈΠ½ΡΡΠ²Π° ΡΠΎΡΠΎΠ½ΠΎΠ² ΡΡΠΎΠ³ΠΎ ΡΠ²Π΅ΡΠ° ΠΏΠΎ ΡΡΠ°Π²Π½Π΅Π½ΠΈΡ Ρ ΡΠ°ΡΡΠΎΡΠΎΠΉ Π±ΠΎΠ»ΡΡΠΈΠ½ΡΡΠ²Π° ΡΠΎΡΠΎΠ½ΠΎΠ² ΡΠ²Π΅ΡΠ° ΡΠ»Π΅ΠΊΡΡΠΈΡΠ΅ΡΠΊΠΎΠΉ Π»Π°ΠΌΠΏΠΎΡΠΊΠΈ?
ΠΡΠ²Π΅Ρ. Π’Π΅ΠΌΠΏΠ΅ΡΠ°ΡΡΡΠ° ΠΎΠΊΡΡΠΆΠ°ΡΡΠ΅ΠΉ Π²Π°Ρ ΡΡΠ΅Π΄Ρ ΡΠ°Π²Π½Π° ΠΏΡΠΈΠΌΠ΅ΡΠ½ΠΎ 20 Β°Π‘, Ρ. Π΅. 297 Π, ΡΡΠΎ ΡΠΎΡΡΠ°Π²Π»ΡΠ΅Ρ ΠΏΡΠΈΠΌΠ΅ΡΠ½ΠΎ ΠΎΠ΄Π½Ρ Π΄Π΅ΡΡΡΡΡ ΡΠ°ΡΡΡ ΡΠ΅ΠΌΠΏΠ΅ΡΠ°ΡΡΡΡ Π½ΠΈΡΠΈ Π½Π°ΠΊΠ°Π»ΠΈΠ²Π°Π½ΠΈΡ Π² Π»Π°ΠΌΠΏΠΎΡΠΊΠ΅. ΠΠΎΡΡΠΎΠΌΡ Π±ΠΎΠ»ΡΡΠΈΠ½ΡΡΠ²ΠΎ ΡΠΎΡΠΎΠ½ΠΎΠ² ΡΠ΅ΠΏΠ»ΠΎΠ²ΠΎΠ³ΠΎ ΠΈΠ·Π»ΡΡΠ΅Π½ΠΈΡ ΠΎΠΊΡΡΠΆΠ°ΡΡΠ΅ΠΉ ΡΡΠ΅Π΄Ρ ΠΈΠΌΠ΅ΡΡ ΡΠ½Π΅ΡΠ³ΠΈΡ, ΡΠ°Π²Π½ΡΡ ΠΎΠ΄Π½ΠΎΠΉ Π΄Π΅ΡΡΡΠΎΠΉ ΡΠ½Π΅ΡΠ³ΠΈΠΈ ΡΠΎΡΠΎΠ½ΠΎΠ² ΠΎΡ Π»Π°ΠΌΠΏΠΎΡΠΊΠΈ. ΠΠ·Π»ΡΡΠ΅Π½ΠΈΠ΅ Ρ ΡΠ°ΠΊΠΎΠΉ ΡΠ½Π΅ΡΠ³ΠΈΠ΅ΠΉ ΠΏΡΠΈΠ½Π°Π΄Π»Π΅ΠΆΠΈΡ Π΄Π°Π»ΡΠΊΠΎΠΉ ΡΠ°ΡΡΠΈ ΡΠΏΠ΅ΠΊΡΡΠ° ΠΈΠ½ΡΡΠ°ΠΊΡΠ°ΡΠ½ΠΎΠ³ΠΎ ΠΈΠ·Π»ΡΡΠ΅Π½ΠΈΡ. ΠΠΌΠ΅Π½Π½ΠΎ ΡΡΠΎ ΠΈΠ·Π»ΡΡΠ΅Π½ΠΈΠ΅ ΡΠΈΠΊΡΠΈΡΡΠ΅ΡΡΡ ΠΏΡΠΈΠ±ΠΎΡΠ°ΠΌΠΈ Π½ΠΎΡΠ½ΠΎΠ³ΠΎ Π²ΠΈΠ΄Π΅Π½ΠΈΡ.
ΠΠ°ΠΊ Π²ΠΈΠ΄Π½ΠΎ ΠΈΠ· ΡΠΈΡ. 9-6, ΡΠ΅ΠΌ Π½ΠΈΠΆΠ΅ ΡΠ΅ΠΌΠΏΠ΅ΡΠ°ΡΡΡΠ°, ΡΠ΅ΠΌ ΠΌΠ΅Π½ΡΡΠ΅ ΠΈΠ½ΡΠ΅ΡΠ²Π°Π» ΡΠ½Π΅ΡΠ³ΠΈΠΉ ΡΠΎΡΠΎΠ½ΠΎΠ², Π² ΠΊΠΎΡΠΎΡΠΎΠΌ ΠΎΠ±Π½Π°ΡΡΠΆΠΈΠ²Π°Π΅ΡΡΡ Π±ΠΎΠ»ΡΡΠ°Ρ ΡΠ°ΡΡΡ Π²ΡΠ΅Ρ ΡΠΎΡΠΎΠ½ΠΎΠ². ΠΡΠ΅Π»Π΅Π½Π½Π°Ρ Π·Π°ΠΏΠΎΠ»Π½Π΅Π½Π° ΡΠ»Π΅ΠΊΡΡΠΎΠΌΠ°Π³Π½ΠΈΡΠ½ΡΠΌ ΠΈΠ·Π»ΡΡΠ΅Π½ΠΈΠ΅ΠΌ, ΡΠΎΠΎΡΠ²Π΅ΡΡΡΠ²ΡΡΡΠΈΠΌ ΡΠ΅ΠΌΠΏΠ΅ΡΠ°ΡΡΡΠ΅ ΠΏΡΠΈΠΌΠ΅ΡΠ½ΠΎ 3 Π. ΠΡΠΎ ΠΎΠ·Π½Π°ΡΠ°Π΅Ρ, ΡΡΠΎ Π±ΠΎΠ»ΡΡΠ°Ρ ΡΠ°ΡΡΡ ΡΠΎΡΠΎΠ½ΠΎΠ² ΡΡΠΎΠ³ΠΎ ΠΈΠ·Π»ΡΡΠ΅Π½ΠΈΡ ΠΈΠΌΠ΅Π΅Ρ Π΄Π»ΠΈΠ½Ρ Π²ΠΎΠ»Π½Ρ ΠΏΠΎΡΡΠ΄ΠΊΠ° 1 ΠΌΠΌ. ΠΡΠΎ ΠΈΠ·Π»ΡΡΠ΅Π½ΠΈΠ΅ Π΅ΡΡΡ ΠΎΡΡΠ°ΡΠΎΠΊ ΠΠΎΠ»ΡΡΠΎΠ³ΠΎ Π²Π·ΡΡΠ²Π°, ΠΏΠΎΡΠΎΠ΄ΠΈΠ²ΡΠ΅Π³ΠΎ 10Β ΠΌΠ»ΡΠ΄ Π»Π΅Ρ ΡΠΎΠΌΡ Π½Π°Π·Π°Π΄ Π½Π°ΡΡ ΠΡΠ΅Π»Π΅Π½Π½ΡΡ.
5. ΠΠΎΠ³Π»ΠΎΡΠ΅Π½ΠΈΠ΅ ΡΠ²Π΅ΡΠ°
ΠΡΠ΅ ΡΠ²ΡΡΠ΄ΡΠ΅ ΡΠ΅Π»Π° ΡΠΎΡΡΠΎΡΡ ΠΈΠ· ΡΠ°ΡΡΠΈΡ β ΡΠ»Π΅ΠΊΡΡΠΎΠ½ΠΎΠ² ΠΈ ΡΠ΄Π΅Ρ, β ΠΎΠ±Π»Π°Π΄Π°ΡΡΠΈΡ Π΄Π²ΡΠΌΡ ΡΠ²ΠΎΠΉΡΡΠ²Π°ΠΌΠΈ, ΠΊΠΎΡΠΎΡΡΠ΅ ΠΏΠΎΠ·Π²ΠΎΠ»ΡΡΡ ΠΈΠΌ ΠΈΡΠΏΡΡΡΠ²Π°ΡΡ Π²Π»ΠΈΡΠ½ΠΈΠ΅ ΡΠ»Π΅ΠΊΡΡΠΈΡΠ΅ΡΠΊΠΈΡ ΠΈ ΠΌΠ°Π³Π½ΠΈΡΠ½ΡΡ ΠΏΠΎΠ»Π΅ΠΉ Π² ΡΠ»Π΅ΠΊΡΡΠΎΠΌΠ°Π³Π½ΠΈΡΠ½ΠΎΠΉ Π²ΠΎΠ»Π½Π΅: ΡΡΠΈ ΡΠ°ΡΡΠΈΡΡ Π·Π°ΡΡΠΆΠ΅Π½Ρ ΠΈ ΡΠ²Π»ΡΡΡΡΡ ΠΏΠΎΡΡΠΎΡΠ½Π½ΡΠΌΠΈ ΠΌΠ°Π³Π½ΠΈΡΠ°ΠΌΠΈ. ΠΡΠΎΠΌΠ΅ ΡΠΎΠ³ΠΎ, ΡΠΎΠ³Π»Π°ΡΠ½ΠΎ Π·Π°ΠΊΠΎΠ½Π°ΠΌ ΠΊΠ²Π°Π½ΡΠΎΠ²ΠΎΠΉ ΠΌΠ΅Ρ Π°Π½ΠΈΠΊΠΈ, ΡΠ½Π΅ΡΠ³ΠΈΠΈ ΡΠ»Π΅ΠΊΡΡΠΎΠ½ΠΎΠ² ΠΎΠΏΠΈΡΡΠ²Π°ΡΡΡΡ Ρ ΠΏΠΎΠΌΠΎΡΡΡ ΠΏΠΎΠ½ΡΡΠΈΡ ΡΠ½Π΅ΡΠ³Π΅ΡΠΈΡΠ΅ΡΠΊΠΈΡ Π·ΠΎΠ½, Π° ΡΠ½Π΅ΡΠ³ΠΈΡ ΡΠ»Π΅ΠΊΡΡΠΎΠΌΠ°Π³Π½ΠΈΡΠ½ΡΡ Π²ΠΎΠ»Π½ ΠΊΠ²Π°Π½ΡΠΎΠ²Π°Π½Π° ΠΈ ΡΠ°Π²Π½Π° ΡΡΠΌΠΌΠ΅ ΡΠ½Π΅ΡΠ³ΠΈΠΉ ΡΠΎΡΠΎΠ½ΠΎΠ². ΠΡΠΏΠΎΠ»ΡΠ·ΡΡ ΡΡΡ ΠΈΠ½ΡΠΎΡΠΌΠ°ΡΠΈΡ, ΠΏΠΎΠΏΡΠΎΠ±ΡΠ΅ΠΌ ΠΎΡΠ²Π΅ΡΠΈΡΡ Π½Π° Π½Π΅ΠΊΠΎΡΠΎΡΡΠ΅ Π²ΠΎΠΏΡΠΎΡΡ, Π½Π°ΠΏΡΠΈΠΌΠ΅Ρ, ΠΏΠΎΡΠ΅ΠΌΡ ΡΡΠ΅ΠΊΠ»ΠΎ ΠΏΡΠΎΠ·ΡΠ°ΡΠ½ΠΎ Π΄Π»Ρ ΡΠ²Π΅ΡΠ°, Π° Π°Π»ΡΠΌΠΈΠ½ΠΈΠ²Π°Ρ ΡΠΎΠ»ΡΠ³Π° Π½Π΅ΠΏΡΠΎΠ·ΡΠ°ΡΠ½Π°, ΠΈΠ»ΠΈ ΠΏΠΎΡΠ΅ΠΌΡ ΠΌΠ΅Π΄Ρ ΠΊΡΠ°ΡΠ½Π°Ρ, Π° ΡΠ°ΠΏΡΠΈΡ ΡΠΈΠ½ΠΈΠΉ.
Π ΡΡΠΎΠΌ ΡΠ°Π·Π΄Π΅Π»Π΅ ΡΠΎΡΡΠ΅Π΄ΠΎΡΠΎΡΠΈΠΌΡΡ Π½Π° ΡΠ°ΡΡΠΌΠΎΡΡΠ΅Π½ΠΈΠΈ ΡΠ΅Ρ ΡΡΡΠ΅ΠΊΡΠΎΠ², ΠΊΠΎΡΠΎΡΡΠ΅ Π²ΠΎΠ·Π½ΠΈΠΊΠ°ΡΡ ΠΏΡΠΈ Π²ΠΎΠ·Π΄Π΅ΠΉΡΡΠ²ΠΈΠΈ ΡΠ»Π΅ΠΊΡΡΠΈΡΠ΅ΡΠΊΠΎΠ³ΠΎ ΠΏΠΎΠ»Ρ ΡΠ²Π΅ΡΠΎΠ²ΠΎΠΉ Π²ΠΎΠ»Π½Ρ Π½Π° ΠΎΠ±ΡΠ°Π·ΡΡΡΠΈΠ΅ ΠΊΡΡΠΎΠΊ Π²Π΅ΡΠ΅ΡΡΠ²Π° Π°ΡΠΎΠΌΡ. ΠΠ»Π΅ΠΊΡΡΠΈΡΠ΅ΡΠΊΠΎΠ΅ ΠΏΠΎΠ»Π΅ Π΄Π΅ΠΉΡΡΠ²ΡΠ΅Ρ Π½Π° ΡΠ»Π΅ΠΊΡΡΠΈΡΠ΅ΡΠΊΠΈΠ΅ Π·Π°ΡΡΠ΄Ρ, Π° ΠΈΠΌΠ΅Π½Π½ΠΎ, Π½Π° ΡΠ»Π΅ΠΊΡΡΠΎΠ½Ρ ΠΈ ΡΠ΄ΡΠ°, Π²Ρ ΠΎΠ΄ΡΡΠΈΠ΅ Π² ΡΠΎΡΡΠ°Π² Π°ΡΠΎΠΌΠΎΠ². ΠΠΎΠ»Π΅, Π΄Π΅ΠΉΡΡΠ²ΡΡΡΠ΅Π΅ Π½Π° ΠΎΡΠ΄Π΅Π»ΡΠ½ΡΠΉ Π°ΡΠΎΠΌ, ΡΠ΄Π²ΠΈΠ³Π°Π΅Ρ ΡΠ΄ΡΠΎ (ΠΏΠΎΠ»ΠΎΠΆΠΈΡΠ΅Π»ΡΠ½ΡΠΉ Π·Π°ΡΡΠ΄) Π² ΠΎΠ΄Π½ΠΎΠΌ Π½Π°ΠΏΡΠ°Π²Π»Π΅Π½ΠΈΠΈ, Π° ΠΎΠΊΡΡΠΆΠ°ΡΡΠΈΠ΅ ΡΠ΄ΡΠΎ ΡΠ»Π΅ΠΊΡΡΠΎΠ½Ρ (ΠΎΡΡΠΈΡΠ°ΡΠ΅Π»ΡΠ½ΡΠΉ Π·Π°ΡΡΠ΄) β Π² ΠΏΡΠΎΡΠΈΠ²ΠΎΠΏΠΎΠ»ΠΎΠΆΠ½ΠΎΠΌ Π½Π°ΠΏΡΠ°Π²Π»Π΅Π½ΠΈΠΈ. ΠΡΠΎ ΠΎΡΠ½ΠΎΡΠΈΡΠ΅Π»ΡΠ½ΠΎΠ΅ ΡΠΌΠ΅ΡΠ΅Π½ΠΈΠ΅ Π΄Π²ΡΡ Π·Π°ΡΡΠ΄ΠΎΠ² ΠΏΡΠ΅ΠΊΡΠ°ΡΠ°Π΅ΡΡΡ Π² ΡΠΎΡ ΠΌΠΎΠΌΠ΅Π½Ρ, ΠΊΠΎΠ³Π΄Π° ΡΠΈΠ»Π°, Π΄Π΅ΠΉΡΡΠ²ΡΡΡΠ°Ρ ΡΠΎ ΡΡΠΎΡΠΎΠ½Ρ ΡΠ»Π΅ΠΊΡΡΠΈΡΠ΅ΡΠΊΠΎΠ³ΠΎ ΠΏΠΎΠ»Ρ, ΡΡΠ°Π²Π½ΠΎΠ²Π΅ΡΠΈΠ²Π°Π΅ΡΡΡ ΡΠΈΠ»Π°ΠΌΠΈ ΠΏΡΠΈΡΡΠΆΠ΅Π½ΠΈΡ ΠΌΠ΅ΠΆΠ΄Ρ ΡΠ΄ΡΠ°ΠΌΠΈ ΠΈ ΡΠ»Π΅ΠΊΡΡΠΎΠ½Π½ΡΠΌΠΈ ΠΎΠ±Π»Π°ΠΊΠ°ΠΌΠΈ Π² Π°ΡΠΎΠΌΠ°Ρ . ΠΡ Π΅ΡΡ Π²Π΅ΡΠ½ΡΠΌΡΡ ΠΊ ΡΡΠΎΠΌΡ ΡΡΡΠ΅ΠΊΡΡ.
Π‘Π²Π΅Ρ, ΠΏΠΎΠΌΠΈΠΌΠΎ ΠΏΡΠΎΡΠ΅Π³ΠΎ, ΡΠ²Π»ΡΠ΅ΡΡΡ ΠΏΠΎΡΠΎΠΊΠΎΠΌ ΡΠΎΡΠΎΠ½ΠΎΠ², ΠΊΠ²Π°Π½ΡΠΎΠ² ΡΠ½Π΅ΡΠ³ΠΈΠΈ. Π€ΠΎΡΠΎΠ½ ΠΌΠΎΠΆΠ΅Ρ Π±ΡΡΡ ΠΏΠΎΠ³Π»ΠΎΡΡΠ½ ΡΠ»Π΅ΠΊΡΡΠΎΠ½ΠΎΠΌ. Π ΡΠ΅Π·ΡΠ»ΡΡΠ°ΡΠ΅ ΡΠ½Π΅ΡΠ³ΠΈΡ ΡΠ»Π΅ΠΊΡΡΠΎΠ½Π° ΡΠ²Π΅Π»ΠΈΡΠΈΠ²Π°Π΅ΡΡΡ Π½Π° Π²Π΅Π»ΠΈΡΠΈΠ½Ρ, ΡΠ°Π²Π½ΡΡ ΡΠ½Π΅ΡΠ³ΠΈΠΈ ΠΏΠΎΠ³Π»ΠΎΡΡΠ½Π½ΠΎΠ³ΠΎ ΡΠΎΡΠΎΠ½Π°. Π§ΡΠΎΠ±Ρ ΡΠ°ΠΊΠΎΠ΅ ΠΏΠΎΠ³Π»ΠΎΡΠ΅Π½ΠΈΠ΅ ΡΡΠ°Π»ΠΎ Π²ΠΎΠ·ΠΌΠΎΠΆΠ½ΡΠΌ, Π΄ΠΎΠ»ΠΆΠ½ΠΎ ΡΡΡΠ΅ΡΡΠ²ΠΎΠ²Π°ΡΡ Π½Π΅Π·Π°Π½ΡΡΠΎΠ΅ ΡΠΎΡΡΠΎΡΠ½ΠΈΠ΅ Ρ Π±ΠΎΠ»ΡΡΠ΅ΠΉ ΡΠ½Π΅ΡΠ³ΠΈΠ΅ΠΉ, ΠΊΡΠ΄Π° ΡΠ»Π΅ΠΊΡΡΠΎΠ½ ΠΌΠΎΠ³ Π±Ρ ΠΏΠ΅ΡΠ΅ΠΉΡΠΈ. ΠΡΠ»ΠΈ ΡΠ°ΠΊΠΎΠ³ΠΎ ΡΠΎΡΡΠΎΡΠ½ΠΈΡ Π½Π΅Ρ, ΡΠΎΡΠΎΠ½ Π½Π΅ ΠΌΠΎΠΆΠ΅Ρ ΠΏΠΎΠ³Π»ΠΎΡΠΈΡΡΡΡ ΠΈ Π±ΡΠ΄Π΅Ρ ΠΏΡΠΎΠ΄ΠΎΠ»ΠΆΠ°ΡΡ ΡΠ²ΠΎΡ Π΄Π²ΠΈΠΆΠ΅Π½ΠΈΠ΅. ΠΠΎΡ ΠΌΡ ΠΈ ΠΏΠΎΠ΄ΠΎΡΠ»ΠΈ ΠΊ ΠΎΠ±ΡΡΡΠ½Π΅Π½ΠΈΡ ΡΠΎΠ³ΠΎ, ΠΏΠΎΡΠ΅ΠΌΡ ΠΎΡΠ΄Π΅Π»ΡΠ½ΡΠ΅ ΡΠ΅Π»Π° ΠΏΡΠΎΠ·ΡΠ°ΡΠ½Ρ Π΄Π»Ρ ΡΠ²Π΅ΡΠ°, Π° Π΄ΡΡΠ³ΠΈΠ΅ β Π½Π΅Ρ.
ΠΠ°ΠΊ ΠΏΡΠΈΠΌΠ΅Ρ ΠΏΡΠΎΠ·ΡΠ°ΡΠ½ΠΎΠ³ΠΎ Π²Π΅ΡΠ΅ΡΡΠ²Π° ΡΠ°ΡΡΠΌΠΎΡΡΠΈΠΌ ΡΡΠ΅ΠΊΠ»ΠΎ. ΠΠ½ΠΎ ΡΠ²Π»ΡΠ΅ΡΡΡ Π΄ΠΈΡΠ»Π΅ΠΊΡΡΠΈΠΊΠΎΠΌ, ΠΈ ΠΌΡ ΠΊΠ°ΡΠ΅ΡΡΠ²Π΅Π½Π½ΠΎ ΠΏΡΠ΅Π΄ΡΡΠ°Π²Π»ΡΠ΅ΠΌ ΡΠ΅Π±Π΅, ΠΊΠ°ΠΊ Π²ΡΠ³Π»ΡΠ΄ΠΈΡ ΡΠ»Π΅ΠΊΡΡΠΎΠ½Π½Π°Ρ ΡΡΡΡΠΊΡΡΡΠ° ΡΡΠ΅ΠΊΠ»Π°, Π΅ΡΠ»ΠΈ ΠΈΡΠΏΠΎΠ»ΡΠ·ΠΎΠ²Π°ΡΡ ΠΏΠΎΠ½ΡΡΠΈΠ΅ ΡΠ½Π΅ΡΠ³Π΅ΡΠΈΡΠ΅ΡΠΊΠΈΡ Π·ΠΎΠ½. ΠΠ° ΡΠΈΡ. 9-7 ΠΏΠΎΠΊΠ°Π·Π°Π½Ρ Π΄Π²Π΅ ΡΠ½Π΅ΡΠ³Π΅ΡΠΈΡΠ΅ΡΠΊΠΈΠ΅ Π·ΠΎΠ½Ρ, Π΄ΠΎΡΡΠ°ΡΠΎΡΠ½ΡΠ΅ Π΄Π»Ρ ΠΎΠ±ΡΡΡΠ½Π΅Π½ΠΈΡ ΡΠ²Π»Π΅Π½ΠΈΡ. ΠΠΈΠΆΠ½ΡΡ Π·ΠΎΠ½Π° Π·Π°ΠΏΠΎΠ»Π½Π΅Π½Π°: ΠΊΠ°ΠΆΠ΄ΠΎΠ΅ ΠΊΠ²Π°Π½ΡΠΎΠ²ΠΎΠ΅ ΡΠΎΡΡΠΎΡΠ½ΠΈΠ΅ Π·Π°Π½ΡΡΠΎ ΡΠ»Π΅ΠΊΡΡΠΎΠ½ΠΎΠΌ. ΠΠ°Π΄ ΡΡΠΎΠΉ Π·ΠΎΠ½ΠΎΠΉ Π½Π°Ρ ΠΎΠ΄ΠΈΡΡΡ ΠΎΡΠ΄Π΅Π»ΡΠ½Π½Π°Ρ ΡΠ½Π΅ΡΠ³Π΅ΡΠΈΡΠ΅ΡΠΊΠΎΠΉ ΡΠ΅Π»ΡΡ (Π·ΠΎΠ½ΠΎΠΉ Π·Π°ΠΏΡΠ΅ΡΡΠ½Π½ΡΡ ΡΠ½Π΅ΡΠ³ΠΈΠΉ) ΠΏΡΡΡΠ°Ρ Π·ΠΎΠ½Π°, Π² ΠΊΠΎΡΠΎΡΠΎΠΉ Π½Π΅Ρ ΡΠ»Π΅ΠΊΡΡΠΎΠ½ΠΎΠ². Π Π°ΡΡΡΠΎΡΠ½ΠΈΡ ΠΏΠΎ Π²Π΅ΡΡΠΈΠΊΠ°Π»ΠΈ Π½Π° ΡΠΈΡΡΠ½ΠΊΠ΅ ΡΠΎΠΎΡΠ²Π΅ΡΡΡΠ²ΡΡΡ Π·Π½Π°ΡΠ΅Π½ΠΈΡΠΌ ΡΠ½Π΅ΡΠ³ΠΈΠΉ. ΠΠ°ΡΠ°Π»ΡΠ½ΡΠΉ ΡΠΎΡΠΎΠ½ Ρ ΡΠ½Π΅ΡΠ³ΠΈΠ΅ΠΉ, ΡΠΎΠΎΡΠ²Π΅ΡΡΡΠ²ΡΡΡΠ΅ΠΉ Π΄Π»ΠΈΠ½Π΅ ΠΎΡΡΠ΅Π·ΠΊΠ° ΠΠ, ΠΊΠΎΡΠΎΡΠ°Ρ ΠΌΠ΅Π½ΡΡΠ΅, ΡΠ΅ΠΌ Π²Π΅Π»ΠΈΡΠΈΠ½Π° ΡΠ½Π΅ΡΠ³Π΅ΡΠΈΡΠ΅ΡΠΊΠΎΠΉ ΡΠ΅Π»ΠΈ, Π½Π΅ ΠΌΠΎΠΆΠ΅Ρ Π±ΡΡΡ ΠΏΠΎΠ³Π»ΠΎΡΡΠ½ Π½ΠΈ ΠΎΠ΄Π½ΠΈΠΌ ΠΈΠ· ΡΠ»Π΅ΠΊΡΡΠΎΠ½ΠΎΠ² Π² Π·Π°ΠΏΠΎΠ»Π½Π΅Π½Π½ΠΎΠΉ Π·ΠΎΠ½Π΅, Ρ.ΠΊ. ΠΊΠΎΠ½Π΅ΡΠ½ΠΎΠ΅ ΡΠΎΡΡΠΎΡΠ½ΠΈΠ΅ ΡΠ»Π΅ΠΊΡΡΠΎΠ½Π° Π΄ΠΎΠ»ΠΆΠ½ΠΎ Π½Π°Ρ ΠΎΠ΄ΠΈΡΡΡΡ Π»ΠΈΠ±ΠΎ Π² Π·Π°ΠΏΠΎΠ»Π½Π΅Π½Π½ΠΎΠΉ Π·ΠΎΠ½Π΅, Π³Π΄Π΅ Π²ΡΠ΅ ΠΌΠ΅ΡΡΠ° Π·Π°Π½ΡΡΡ, Π»ΠΈΠ±ΠΎ ΠΏΠΎΠΏΠ°Π΄Π°Π΅Ρ Π² ΡΠ΅Π»Ρ, Π³Π΄Π΅ Π²ΠΎΠΎΠ±ΡΠ΅ Π½Π΅ ΠΌΠΎΠΆΠ΅Ρ Π±ΡΡΡ Π½ΠΈΠΊΠ°ΠΊΠΈΡ ΠΊΠ²Π°Π½ΡΠΎΠ²ΡΡ ΡΠΎΡΡΠΎΡΠ½ΠΈΠΉ ΡΠ»Π΅ΠΊΡΡΠΎΠ½ΠΎΠ². ΠΠΎΡΡΠΎΠΌΡ ΡΠ°ΠΊΠΎΠΉ ΡΠΎΡΠΎΠ½ Π±ΡΠ΄Π΅Ρ ΠΏΡΠΎΡ ΠΎΠ΄ΠΈΡΡ ΡΠΊΠ²ΠΎΠ·Ρ Π²Π΅ΡΠ΅ΡΡΠ²ΠΎ. ΠΡΡΡΡ ΡΠ΅ΠΏΠ΅ΡΡ ΡΠΎΡΠΎΠ½Ρ ΠΈΠΌΠ΅ΡΡ ΡΠ½Π΅ΡΠ³ΠΈΡ Π±ΠΎΠ»ΡΡΡΡ, ΡΠ΅ΠΌ ΡΠΈΡΠΈΠ½Π° ΡΠ½Π΅ΡΠ³Π΅ΡΠΈΡΠ΅ΡΠΊΠΎΠΉ ΡΠ΅Π»ΠΈ, Π½Π°ΠΏΡΠΈΠΌΠ΅Ρ, CD. Π’ΠΎΠ³Π΄Π° ΡΠ»Π΅ΠΊΡΡΠΎΠ½ Π² Π½ΠΈΠΆΠ½Π΅ΠΉ Π·ΠΎΠ½Π΅ ΠΌΠΎΠΆΠ΅Ρ ΠΏΠΎΠ³Π»ΠΎΡΠΈΡΡ ΡΠΎΡΠΎΠ½ ΠΈ ΠΏΠ΅ΡΠ΅ΠΉΡΠΈ Π² Π½Π΅Π·Π°Π½ΡΡΠΎΠ΅ ΡΠΎΡΡΠΎΡΠ½ΠΈΠ΅ Π² Π²Π΅ΡΡ Π½Π΅ΠΉ Π·ΠΎΠ½Π΅.
Π ΠΈΡ. 9-7. ΠΠ½Π΅ΡΠ³Π΅ΡΠΈΡΠ΅ΡΠΊΠΈΠ΅ Π·ΠΎΠ½Ρ ΡΠ»Π΅ΠΊΡΡΠΎΠ½ΠΎΠ² Π² ΠΏΡΠΎΠ·ΡΠ°ΡΠ½ΠΎΠΌ Π²Π΅ΡΠ΅ΡΡΠ²Π΅ ΡΠΈΠΏΠ° ΡΡΠ΅ΠΊΠ»Π°. Π€ΠΎΡΠΎΠ½Ρ Ρ ΡΠ½Π΅ΡΠ³ΠΈΡΠΌΠΈ ΠΠ (Π²Π΅Π»ΠΈΡΠΈΠ½Π΅ ΡΠ½Π΅ΡΠ³ΠΈΠΈ ΡΠΎΠΎΡΠ²Π΅ΡΡΡΠ²ΡΠ΅Ρ Π΄Π»ΠΈΠ½Π° ΠΎΡΡΠ΅Π·ΠΊΠ° ΠΠ), ΠΌΠ΅Π½ΡΡΠΈΠΌΠΈ ΡΠΈΡΠΈΠ½Ρ ΡΠ΅Π»ΠΈ ΠΌΠ΅ΠΆΠ΄Ρ Π·Π°Π½ΡΡΠΎΠΉ ΠΈ Π½Π΅Π·Π°ΠΏΠΎΠ»Π½Π΅Π½Π½ΠΎΠΉ Π·ΠΎΠ½Π°ΠΌΠΈ, Π½Π΅ ΠΌΠΎΠ³ΡΡ ΠΏΠΎΠ³Π»ΠΎΡΠΈΡΡΡΡ ΡΠ»Π΅ΠΊΡΡΠΎΠ½Π°ΠΌΠΈ, Ρ.ΠΊ. Π½Π΅Ρ ΡΠ°ΠΊΠΈΡ ΠΊΠΎΠ½Π΅ΡΠ½ΡΡ ΡΠΎΡΡΠΎΡΠ½ΠΈΠΉ, Π² ΠΊΠΎΡΠΎΡΠΎΠ΅ ΡΠ΅ ΠΌΠΎΠ³Π»ΠΈ Π±Ρ ΠΏΠ΅ΡΠ΅ΠΉΡΠΈ. Π’Π°ΠΊΠΈΠ΅ ΡΠΎΡΠΎΠ½Ρ ΠΏΡΠΎΡ ΠΎΠ΄ΡΡ ΡΠΊΠ²ΠΎΠ·Ρ Π²Π΅ΡΠ΅ΡΡΠ²ΠΎ. ΠΡΠ»ΠΈ ΠΆΠ΅ ΡΠ½Π΅ΡΠ³ΠΈΠΈ ΡΠΎΡΠΎΠ½ΠΎΠ² ΠΏΡΠ΅Π²ΡΡΠ°ΡΡ ΡΠΈΡΠΈΠ½Ρ ΡΠ΅Π»ΠΈ (CD Π½Π° ΡΠΈΡΡΠ½ΠΊΠ΅), ΡΠΎ ΡΠ°ΠΊΠΈΠ΅ ΡΠΎΡΠΎΠ½Ρ ΠΏΠΎΠ³Π»ΠΎΡΠ°ΡΡΡΡ Π²Π΅ΡΠ΅ΡΡΠ²ΠΎΠΌ
Π’Π΅ΠΏΠ΅ΡΡ ΠΌΠΎΠΆΠ½ΠΎ ΠΏΠΎΠ½ΡΡΡ, ΠΏΠΎΡΠ΅ΠΌΡ ΡΡΠ΅ΠΊΠ»ΠΎ ΠΏΡΠΎΠ·ΡΠ°ΡΠ½ΠΎ Π΄Π»Ρ Π²ΠΈΠ΄ΠΈΠΌΠΎΠ³ΠΎ ΡΠ²Π΅ΡΠ°. ΠΠΎΠ½Π½Π°Ρ ΡΡΡΡΠΊΡΡΡΠ° ΡΡΠ΅ΠΊΠ»Π° ΡΠ°ΠΊΠΎΠ²Π°, ΡΡΠΎ Ρ ΡΠΎΡΠΎΠ½ΠΎΠ² Π²ΠΈΠ΄ΠΈΠΌΠΎΠ³ΠΎ ΡΠ²Π΅ΡΠ° Π½Π΅ Ρ Π²Π°ΡΠ°Π΅Ρ ΡΠ½Π΅ΡΠ³ΠΈΠΈ, ΡΡΠΎΠ±Ρ ΠΏΠ΅ΡΠ΅Π±ΡΠΎΡΠΈΡΡ ΡΠ»Π΅ΠΊΡΡΠΎΠ½Ρ ΠΈΠ· Π½ΠΈΠΆΠ½Π΅ΠΉ Π·ΠΎΠ½Ρ Π² Π²Π΅ΡΡ Π½ΡΡ, ΠΏΡΠ΅ΠΎΠ΄ΠΎΠ»Π΅Π² ΡΠ΅Π»Ρ. ΠΠΎΡΡΠΎΠΌΡ ΡΠΎΡΠΎΠ½Ρ Π²ΠΈΠ΄ΠΈΠΌΠΎΠ³ΠΎ ΡΠ²Π΅ΡΠ° ΠΏΡΠΎΡ ΠΎΠ΄ΡΡ ΡΠΊΠ²ΠΎΠ·Ρ ΡΡΠ΅ΠΊΠ»ΠΎ Π±Π΅Π· ΠΏΠΎΠ³Π»ΠΎΡΠ΅Π½ΠΈΡ. ΠΠΎ ΡΠΎΡΠΎΠ½Ρ ΡΠ»ΡΡΡΠ°ΡΠΈΠΎΠ»Π΅ΡΠΎΠ²ΠΎΠ³ΠΎ ΡΠ²Π΅ΡΠ° ΠΈΠΌΠ΅ΡΡ Π±ΠΎΠ»Π΅Π΅ Π²ΡΡΠΎΠΊΡΡ ΡΠ°ΡΡΠΎΡΡ, ΡΠ»Π΅Π΄ΠΎΠ²Π°ΡΠ΅Π»ΡΠ½ΠΎ, Π±ΠΎΠ»ΡΡΡΡ ΡΠ½Π΅ΡΠ³ΠΈΡ, ΠΊΠΎΡΠΎΡΠΎΠΉ ΠΎΠΊΠ°Π·ΡΠ²Π°Π΅ΡΡΡ Π΄ΠΎΡΡΠ°ΡΠΎΡΠ½ΠΎ, ΡΡΠΎΠ±Ρ Π²ΡΡΠ²Π°ΡΡ ΡΠ»Π΅ΠΊΡΡΠΎΠ½Ρ ΠΈΠ· Π·Π°ΠΏΠΎΠ»Π½Π΅Π½Π½ΠΎΠΉ Π·ΠΎΠ½Ρ ΠΈ ΠΏΠ΅ΡΠ΅Π±ΡΠΎΡΠΈΡΡ ΠΈΡ Π² Π½Π΅Π·Π°Π½ΡΡΡΡ Π·ΠΎΠ½Ρ. ΠΠΎΡΡΠΎΠΌΡ ΡΠ°ΠΊΠΎΠΉ ΡΠ²Π΅Ρ Π±ΡΠ΄Π΅Ρ ΠΏΠΎΠ³Π»ΠΎΡΠ°ΡΡΡΡ ΡΡΠ΅ΠΊΠ»ΠΎΠΌ. ΠΠ° ΡΠΈΡ. 9-7 ΠΎΡΡΠ΅Π·ΠΊΡ ΠΠ ΡΠΎΠΎΡΠ²Π΅ΡΡΡΠ²ΡΠ΅Ρ ΡΠΎΡΠΎΠ½ Π²ΠΈΠ΄ΠΈΠΌΠΎΠ³ΠΎ ΡΠ²Π΅ΡΠ°, Π° ΠΎΡΡΠ΅Π·ΠΊΡ CD β ΡΠ»ΡΡΡΠ°ΡΠΈΠΎΠ»Π΅ΡΠΎΠ²ΠΎΠ³ΠΎ. Π ΠΊΠ²Π°ΡΡΠ΅ ΡΠ½Π΅ΡΠ³Π΅ΡΠΈΡΠ΅ΡΠΊΠ°Ρ ΡΠ΅Π»Ρ ΡΠΈΡΠ΅, ΡΠ΅ΠΌ Ρ ΠΎΠ±ΡΡΠ½ΠΎΠ³ΠΎ ΡΡΠ΅ΠΊΠ»Π°, ΠΏΠΎΡΡΠΎΠΌΡ ΡΠΎΡΠΎΠ½Ρ Π½Π΅ ΡΠΎΠ»ΡΠΊΠΎ Π²ΠΈΠ΄ΠΈΠΌΠΎΠ³ΠΎ, Π½ΠΎ ΠΈ ΡΠ»ΡΡΡΠ°ΡΠΈΠΎΠ»Π΅ΡΠΎΠ²ΠΎΠ³ΠΎ ΡΠ²Π΅ΡΠ° Π½Π΅ ΠΎΠ±Π»Π°Π΄Π°ΡΡ Π΄ΠΎΡΡΠ°ΡΠΎΡΠ½ΠΎΠΉ ΡΠ½Π΅ΡΠ³ΠΈΠ΅ΠΉ, ΡΡΠΎΠ±Ρ ΡΠ»Π΅ΠΊΡΡΠΎΠ½Ρ Π² Π·Π°ΠΏΠΎΠ»Π½Π΅Π½Π½ΠΎΠΉ Π·ΠΎΠ½Π΅ ΠΌΠΎΠ³Π»ΠΈ ΠΈΡ ΠΏΠΎΠ³Π»ΠΎΡΠΈΡΡ ΠΈ ΠΏΠ΅ΡΠ΅ΠΉΡΠΈ Π² Π²Π΅ΡΡ Π½ΡΡ ΠΏΡΡΡΡΡ Π·ΠΎΠ½Ρ. Π ΡΡΠΎΠΌ ΠΏΡΠΈΡΠΈΠ½Π° ΡΠΎΠ³ΠΎ, ΡΡΠΎ ΡΠ»ΡΡΡΠ°ΡΠΈΠΎΠ»Π΅ΡΠΎΠ²ΡΠ΅ Π»Π°ΠΌΠΏΡ Π΄Π΅Π»Π°ΡΡΡΡ Π½Π΅ ΠΈΠ· ΠΎΠ±ΡΡΠ½ΠΎΠ³ΠΎ ΡΡΠ΅ΠΊΠ»Π°, Π° ΠΈΠ· ΠΊΠ²Π°ΡΡΠ°.
Π‘ΡΠΌΠΌΠΈΡΡΡ, ΠΌΠΎΠΆΠ½ΠΎ ΡΡΠ²Π΅ΡΠΆΠ΄Π°ΡΡ, ΡΡΠΎ ΡΠ²Π΅Ρ ΠΌΠΎΠΆΠ΅Ρ ΠΏΠΎΠ³Π»ΠΎΡΠΈΡΡΡΡ ΡΠ»Π΅ΠΊΡΡΠΎΠ½Π°ΠΌΠΈ Π² Π²Π΅ΡΠ΅ΡΡΠ²Π΅, Π΅ΡΠ»ΠΈ ΠΎΠ½ΠΈ ΡΠΏΠΎΡΠΎΠ±Π½Ρ ΠΏΠ΅ΡΠ΅ΠΉΡΠΈ Π² ΡΠΎΡΡΠΎΡΠ½ΠΈΡ, ΡΠ½Π΅ΡΠ³ΠΈΡ ΠΊΠΎΡΠΎΡΡΡ Π±ΠΎΠ»ΡΡΠ΅ Π½Π°ΡΠ°Π»ΡΠ½ΠΎΠΉ ΡΠ½Π΅ΡΠ³ΠΈΠΈ Π½Π° Π²Π΅Π»ΠΈΡΠΈΠ½Ρ, ΡΠ°Π²Π½ΡΡ ΡΠ½Π΅ΡΠ³ΠΈΠΈ ΡΠΎΡΠΎΠ½Π°. ΠΡΠ»ΠΈ Π² Π·ΠΎΠ½Π½ΠΎΠΉ ΡΡΡΡΠΊΡΡΡΠ΅ Π΄Π°Π½Π½ΠΎΠ³ΠΎ Π²Π΅ΡΠ΅ΡΡΠ²Π° ΡΠ°ΠΊΠΈΡ ΡΠΎΡΡΠΎΡΠ½ΠΈΠΉ Π½Π΅Ρ, ΡΠΎΡΠΎΠ½Ρ Π±Π΅ΡΠΏΡΠ΅ΠΏΡΡΡΡΠ²Π΅Π½Π½ΠΎ ΠΏΡΠΎΡ ΠΎΠ΄ΡΡ ΡΠΊΠ²ΠΎΠ·Ρ Π²Π΅ΡΠ΅ΡΡΠ²ΠΎ, ΠΎΠ½ΠΎ ΠΏΡΠΎΠ·ΡΠ°ΡΠ½ΠΎ Π΄Π»Ρ ΡΠ²Π΅ΡΠ° Π΄Π°Π½Π½ΠΎΠ³ΠΎ ΡΠ²Π΅ΡΠ°. ΠΠΎΠ΄ΠΎΠ±Π½Π°Ρ ΠΊΠ°ΡΡΠΈΠ½Π° ΠΎΠ±ΡΡΡΠ½ΡΠ΅Ρ ΡΠ°ΠΊΠΆΠ΅, ΠΏΠΎΡΠ΅ΠΌΡ Π²Π΅ΡΠ΅ΡΡΠ²ΠΎ ΠΌΠΎΠΆΠ΅Ρ Π±ΡΡΡ ΠΏΡΠΎΠ·ΡΠ°ΡΠ½ΡΠΌ Π΄Π»Ρ Π²ΠΎΠ»Π½ ΠΎΠ΄Π½ΠΎΠΉ ΡΠ°ΡΡΠΈ ΡΠ»Π΅ΠΊΡΡΠΎΠΌΠ°Π³Π½ΠΈΡΠ½ΠΎΠ³ΠΎ ΡΠΏΠ΅ΠΊΡΡΠ° ΠΈ Π½Π΅ΠΏΡΠΎΠ·ΡΠ°ΡΠ½ΡΠΌ Π΄Π»Ρ Π΄ΡΡΠ³ΠΎΠΉ ΡΠ°ΡΡΠΈ.
Π Π°ΡΡΠΌΠΎΡΡΠΈΠΌ ΡΠ΅ΠΏΠ΅ΡΡ ΡΠ»ΡΡΠ°ΠΉ ΡΠ²Π΅ΡΠ°, ΠΏΠ°Π΄Π°ΡΡΠ΅Π³ΠΎ Π½Π° ΠΏΠΎΠ²Π΅ΡΡ Π½ΠΎΡΡΡ ΠΌΠ΅ΡΠ°Π»Π»Π°. ΠΡ Π·Π½Π°Π΅ΠΌ, ΡΡΠΎ ΠΌΠ΅ΡΠ°Π»Π» Ρ ΠΎΡΠΎΡΠΎ ΠΏΡΠΎΠ²ΠΎΠ΄ΠΈΡ ΡΠ»Π΅ΠΊΡΡΠΈΡΠ΅ΡΠΊΠΈΠΉ ΡΠΎΠΊ, Ρ.ΠΊ. Π²Π½Π΅ΡΠ½ΠΈΠ΅ ΡΠ»Π΅ΠΊΡΡΠΎΠ½Ρ ΠΊΠ°ΠΆΠ΄ΠΎΠ³ΠΎ Π°ΡΠΎΠΌΠ° ΡΠ²ΠΎΠ±ΠΎΠ΄Π½ΠΎ ΠΏΡΡΠ΅ΡΠ΅ΡΡΠ²ΡΡΡ ΠΏΠΎ Π²ΡΠ΅ΠΌΡ ΠΌΠ΅ΡΠ°Π»Π»Ρ. ΠΠ° ΡΠΈΡ. 9-8 ΠΏΠΎΠΊΠ°Π·Π°Π½ ΠΏΡΡΠΎΠΊ ΡΠ²Π΅ΡΠ°, ΠΎΡΡΠ°ΠΆΠ°ΡΡΠΈΠΉΡΡ ΠΎΡ ΠΏΠΎΠ²Π΅ΡΡ Π½ΠΎΡΡΠΈ ΠΌΠ΅ΡΠ°Π»Π»Π°. ΠΠ»ΠΈΠΆΠ°ΠΉΡΠΈΠ΅ ΠΊ ΠΏΠΎΠ²Π΅ΡΡ Π½ΠΎΡΡΠΈ ΡΠ»Π΅ΠΊΡΡΠΎΠ½Ρ ΠΈΡΠΏΡΡΡΠ²Π°ΡΡ Π΄Π΅ΠΉΡΡΠ²ΠΈΠ΅ ΡΠΈΠ»Ρ ΡΠΎ ΡΡΠΎΡΠΎΠ½Ρ ΠΊΠΎΠ»Π΅Π±Π»ΡΡΠ΅Π³ΠΎΡΡ ΡΠ»Π΅ΠΊΡΡΠΈΡΠ΅ΡΠΊΠΎΠ³ΠΎ ΠΏΠΎΠ»Ρ ΡΠ²Π΅ΡΠΎΠ²ΠΎΠΉ Π²ΠΎΠ»Π½Ρ ΠΈ Π½Π°ΡΠΈΠ½Π°ΡΡ ΡΠ°ΠΌΠΈ ΠΊΠΎΠ»Π΅Π±Π°ΡΡΡΡ Ρ ΡΠΎΠΉ ΠΆΠ΅ ΡΠ°ΡΡΠΎΡΠΎΠΉ, ΡΡΠΎ ΠΈ ΡΠ²Π΅Ρ. ΠΠ΄ΠΈΠ½ ΠΈΠ· ΡΡΠΈΡ ΡΠ»Π΅ΠΊΡΡΠΎΠ½ΠΎΠ², ΠΏΠΎΠΌΠ΅ΡΠ΅Π½Π½ΡΠΉ ΡΠΈΡΡΠΎΠΉ 1, ΠΏΠΎΠΊΠ°Π·Π°Π½ Π½Π° ΡΠΈΡΡΠ½ΠΊΠ΅. Π’Π°ΠΊΠΎΠΉ ΠΊΠΎΠ»Π΅Π±Π»ΡΡΠΈΠΉΡΡ ΡΠ»ΠΎΠΉ ΡΠ»Π΅ΠΊΡΡΠΎΠ½ΠΎΠ² ΠΎΡΠ±ΠΈΡΠ°Π΅Ρ ΡΠ½Π΅ΡΠ³ΠΈΡ Ρ ΡΠ²Π΅ΡΠΎΠ²ΠΎΠΉ Π²ΠΎΠ»Π½Ρ, ΠΏΠΎΡΡΠΎΠΌΡ ΠΎΠ½Π° ΠΎΡΠ»Π°Π±Π»ΡΠ΅ΡΡΡ ΠΈ Π·Π°ΡΡΠ°Π²Π»ΡΠ΅Ρ ΠΊΠΎΠ»Π΅Π±Π°ΡΡΡΡ ΡΠ»Π΅Π΄ΡΡΡΠΈΠΉ ΡΠ»ΠΎΠΉ ΡΠ»Π΅ΠΊΡΡΠΎΠ½ΠΎΠ² (ΠΏΠΎΠΌΠ΅ΡΠ΅Π½Π½ΡΠΉ ΡΠΈΡΡΠΎΠΉ 2) Ρ ΠΌΠ΅Π½ΡΡΠ΅ΠΉ ΡΠ°ΡΡΠΎΡΠΎΠΉ ΠΈ Ρ.Π΄., Π΄ΠΎ ΡΠ΅Ρ ΠΏΠΎΡ, ΠΏΠΎΠΊΠ° Π½Π° ΠΎΠΏΡΠ΅Π΄Π΅Π»ΡΠ½Π½ΠΎΠΉ Π³Π»ΡΠ±ΠΈΠ½Π΅ ΠΎΡ ΠΏΠΎΠ²Π΅ΡΡ Π½ΠΎΡΡΠΈ ΠΊΠΎΠ»Π΅Π±Π°Π½ΠΈΡ ΠΏΠΎΠ»Ρ Π² ΡΠ²Π΅ΡΠΎΠ²ΠΎΠΉ Π²ΠΎΠ»Π½Π΅ ΠΈ ΡΠΎΠΎΡΠ²Π΅ΡΡΡΠ²ΡΡΡΠΈΠ΅ ΠΊΠΎΠ»Π΅Π±Π°Π½ΠΈΡ ΡΠ»Π΅ΠΊΡΡΠΎΠ½ΠΎΠ² Π½Π΅ Π·Π°ΡΡΡ Π½ΡΡ. Π Π±ΠΎΠ»ΡΡΠΈΠ½ΡΡΠ²Π΅ ΠΌΠ΅ΡΠ°Π»Π»ΠΎΠ² ΡΡΠ° Π³Π»ΡΠ±ΠΈΠ½Π° ΠΎΡΠ΅Π½Ρ ΠΌΠ°Π»Π° ΠΈ ΡΠΎΡΡΠ°Π²Π»ΡΠ΅Ρ Π²Π΅Π»ΠΈΡΠΈΠ½Ρ ΠΏΠΎΡΡΠ΄ΠΊΠ° 10β6ΡΠΌ. Π’Π΅ΠΏΠ΅ΡΡ ΠΏΠΎΠ½ΡΡΠ½ΠΎ, ΠΏΠΎΡΠ΅ΠΌΡ Π΄Π°ΠΆΠ΅ ΠΎΡΠ΅Π½Ρ ΡΠΎΠ½ΠΊΠΈΠΉ ΡΠ»ΠΎΠΉ ΠΌΠ΅ΡΠ°Π»Π»Π° Π½Π΅ ΠΏΡΠΎΠΏΡΡΠΊΠ°Π΅Ρ ΡΠ²Π΅Ρ.
Π ΠΈΡ. 9-8. ΠΠΎΠ³Π»ΠΎΡΠ΅Π½ΠΈΠ΅ ΠΈ ΠΎΡΡΠ°ΠΆΠ΅Π½ΠΈΠ΅ ΡΠ²Π΅ΡΠ° ΠΎΡ ΠΌΠ΅ΡΠ°Π»Π»Π°. ΠΡΡΠΈΠ»Π»ΠΈΡΡΡΡΠ΅Π΅ ΡΠ»Π΅ΠΊΡΡΠΈΡΠ΅ΡΠΊΠΎΠ΅ ΠΏΠΎΠ»Π΅ ΡΠ²Π΅ΡΠΎΠ²ΠΎΠΉ Π²ΠΎΠ»Π½Ρ Π·Π°ΡΡΠ°Π²Π»ΡΠ΅Ρ ΠΊΠΎΠ»Π΅Π±Π°ΡΡΡΡ ΡΠ»Π΅ΠΊΡΡΠΎΠ½Ρ ΠΏΡΠΎΠ²ΠΎΠ΄ΠΈΠΌΠΎΡΡΠΈ (ΠΏΠΎΠΊΠ°Π·Π°Π½Ρ ΡΠ΅ΡΡΡΠ΅ ΡΠ»Π΅ΠΊΡΡΠΎΠ½Π°, ΠΏΡΠΈΡΡΠΌ Π΄Π²Π° ΠΏΠ΅ΡΠ²ΡΡ ΠΏΡΠΎΠ½ΡΠΌΠ΅ΡΠΎΠ²Π°Π½Ρ). ΠΠΌΠΏΠ»ΠΈΡΡΠ΄Ρ ΠΊΠΎΠ»Π΅Π±Π°Π½ΠΈΠΉ ΡΠ»Π΅ΠΊΡΡΠΎΠ½ΠΎΠ² ΠΏΠΎΠΊΠ°Π·Π°Π½Ρ ΡΡΡΠ΅Π»ΠΊΠ°ΠΌΠΈ. ΠΡΠΈ ΡΡΡΠ΅Π»ΠΊΠΈ ΡΠΌΠ΅Π½ΡΡΠ°ΡΡΡΡ ΠΏΡΠΈ ΠΏΡΠΎΠ΄Π²ΠΈΠΆΠ΅Π½ΠΈΠΈ Π² Π³Π»ΡΠ±Ρ ΠΌΠ΅ΡΠ°Π»Π»Π°, ΡΡΠΎ ΡΠΎΠΎΡΠ²Π΅ΡΡΡΠ²ΡΠ΅Ρ Π·Π°ΡΡΡ Π°Π½ΠΈΡ ΠΊΠΎΠ»Π΅Π±Π°Π½ΠΈΠΉ. Π’Π°ΠΊΠΈΠΌ ΠΎΠ±ΡΠ°Π·ΠΎΠΌ, ΡΠ»Π΅ΠΊΡΡΠΎΠ½ ΡΠ°ΡΡΠΈΡΠ½ΠΎ ΠΏΠΎΠ³Π»ΠΎΡΠ°Π΅Ρ ΡΠ½Π΅ΡΠ³ΠΈΡ ΠΏΠ°Π΄Π°ΡΡΠ΅Π³ΠΎ ΡΠΎΡΠΎΠ½Π°, Π° ΡΠ°ΡΡΠΈΡΠ½ΠΎ ΠΏΠ΅ΡΠ΅ΠΈΠ·Π»ΡΡΠ°Π΅Ρ Π΅Ρ Π² ΠΎΠΊΡΡΠΆΠ°ΡΡΠ΅Π΅ ΠΏΡΠΎΡΡΡΠ°Π½ΡΡΠ²ΠΎ
ΠΡ ΠΌΠΎΠΆΠ΅ΡΠ΅ Π·Π°Π΄Π°ΡΡ Π²ΠΎΠΏΡΠΎΡ: ΠΎΡΠΊΡΠ΄Π° ΠΆΠ΅ Π±Π΅ΡΡΡΡΡ ΡΠ²Π΅Ρ, ΠΎΡΡΠ°ΠΆΡΠ½Π½ΡΠΉ ΠΎΡ ΠΏΠΎΠ²Π΅ΡΡ Π½ΠΎΡΡΠΈ ΠΌΠ΅ΡΠ°Π»Π»Π°, Ρ. ΠΊ. ΡΠΎΠ·Π΄Π°ΡΡΡΡ Π²ΠΏΠ΅ΡΠ°ΡΠ»Π΅Π½ΠΈΠ΅, ΡΡΠΎ Π²ΡΡ ΡΠ½Π΅ΡΠ³ΠΈΡ ΠΏΠ°Π΄Π°ΡΡΠ΅Π³ΠΎ ΡΠΎΡΠΎΠ½Π° ΠΏΠ΅ΡΠ΅Ρ ΠΎΠ΄ΠΈΡ Π² ΡΠ½Π΅ΡΠ³ΠΈΡ ΠΊΠΎΠ»Π΅Π±Π°Π½ΠΈΠΉ ΡΠ»Π΅ΠΊΡΡΠΎΠ½ΠΎΠ²? ΠΠ΄Π½Π°ΠΊΠΎ Π²ΡΠΏΠΎΠΌΠ½ΠΈΠΌ, ΡΡΠΎ ΡΠ»Π΅ΠΊΡΡΠΎΠΌΠ°Π³Π½ΠΈΡΠ½ΡΠ΅ Π²ΠΎΠ»Π½Ρ ΠΏΠΎΡΠΎΠΆΠ΄Π°ΡΡΡΡ ΠΊΠΎΠ»Π΅Π±Π»ΡΡΠΈΠΌΠΈΡΡ Π·Π°ΡΡΠ΄Π°ΠΌΠΈ. ΠΠ»Π΅ΠΊΡΡΠΎΠ½Ρ Π½Π° ΠΏΠΎΠ²Π΅ΡΡ Π½ΠΎΡΡΠΈ ΠΌΠ΅ΡΠ°Π»Π»Π° ΠΊΠΎΠ»Π΅Π±Π»ΡΡΡΡ Ρ ΡΠΎΠΉ ΠΆΠ΅ ΡΠ°ΡΡΠΎΡΠΎΠΉ, ΡΡΠΎ ΠΈ ΠΏΠ°Π΄Π°ΡΡΠΈΠΉ ΡΠ²Π΅Ρ. ΠΠΌΠ΅Π½Π½ΠΎ ΡΡΠΈ ΠΎΡΡΠΈΠ»Π»ΡΡΠΈΠΈ ΠΈ ΠΏΠΎΡΠΎΠΆΠ΄Π°ΡΡ ΡΠ»Π΅ΠΊΡΡΠΎΠΌΠ°Π³Π½ΠΈΡΠ½ΡΠ΅ Π²ΠΎΠ»Π½Ρ ΡΠΎΠΉ ΠΆΠ΅ ΡΠ°ΡΡΠΎΡΡ, ΡΡΠΎ ΠΈ ΠΏΠ°Π΄Π°ΡΡΠ°Ρ Π²ΠΎΠ»Π½Π°, Π²ΠΎΡ ΠΎΠ½ΠΈ-ΡΠΎ ΠΈ ΠΎΠ±ΡΠ°Π·ΡΡΡ ΠΎΡΡΠ°ΠΆΡΠ½Π½ΡΠΉ ΡΠ²Π΅Ρ.
Π‘ΠΎΠΊΡ. ΠΏΠ΅Ρ. Ρ Π°Π½Π³Π». Π.Π.ΠΠ΅ΡΠΊΠΎΠ²Π°
ΠΡΠΎΠ΄ΠΎΠ»ΠΆΠ΅Π½ΠΈΠ΅ Π² β 10
2.9 ΠΠ²ΠΈΠΆΠ΅Π½ΠΈΠ΅ ΡΠ°ΡΡΠΈΡ Π²Π΅ΡΠ΅ΡΡΠ²Π° Π² ΡΠ²ΠΎΠ±ΠΎΠ΄Π½ΠΎΠΌ ΠΏΡΠΎΡΡΡΠ°Π½ΡΡΠ²Π΅ ΠΏΠΎΠ΄ Π΄Π΅ΠΉΡΡΠ²ΠΈΠ΅ΠΌ ΡΠ»Π΅ΠΊΡΡΠΎΡΡΠ°ΡΠΈΡΠ΅ΡΠΊΠΈΡ ΠΈ ΠΌΠ°Π³Π½ΠΈΡΠΎΡΡΠ°ΡΠΈΡΠ΅ΡΠΊΠΈΡ ΠΏΠΎΠ»Π΅ΠΉ
2.9.1 ΠΠ° ΠΊΠ°ΠΊΠΈΠ΅ ΡΠ°ΡΡΠΈΡΡ Π²Π΅ΡΠ΅ΡΡΠ²Π° Π΄Π΅ΠΉΡΡΠ²ΡΠ΅Ρ ΡΠ»Π΅ΠΊΡΡΠΈΡΠ΅ΡΠΊΠΎΠ΅ ΠΏΠΎΠ»Π΅?
β» ΠΠ»Π΅ΠΊΡΡΠΈΡΠ΅ΡΠΊΠΎΠ΅ ΠΏΠΎΠ»Π΅ ΠΏΡΠΎΠΈΠ·Π²ΠΎΠ΄ΠΈΡ ΡΠΈΠ»ΠΎΠ²ΠΎΠ΅ Π΄Π΅ΠΉΡΡΠ²ΠΈΠ΅ Π½Π° ΡΠ°ΡΡΠΈΡΡ Π²Π΅ΡΠ΅ΡΡΠ²Π°, ΠΎΠ±Π»Π°Π΄Π°ΡΡΠΈΠ΅ ΡΠ»Π΅ΠΊΡΡΠΈΡΠ΅ΡΠΊΠΈΠΌ Π·Π°ΡΡΠ΄ΠΎΠΌ. ΠΡΠΎ ΠΌΠΎΠ³ΡΡ Π±ΡΡΡ Π½Π΅ ΡΠΎΠ»ΡΠΊΠΎ ΠΌΠΈΠΊΡΠΎΡΠ°ΡΡΠΈΡΡ — ΡΠ»Π΅ΠΊΡΡΠΎΠ½Ρ, ΠΏΡΠΎΡΠΎΠ½Ρ, ΠΈΠΎΠ½Ρ ΠΈ Ρ.ΠΏ., Π½ΠΎ ΠΈ ΠΌΠ°ΠΊΡΠΎΡΠ°ΡΡΠΈΡΡ β ΡΠ°ΡΡΠΈΡΡ ΠΏΡΠ»ΠΈ, Π΄ΡΠΌΠ°, ΡΡΠΌΠ°Π½Π° ΠΈ Ρ. Π΄. ΠΡΠΎΠΌΠ΅ ΡΠΎΠ³ΠΎ, ΡΠ»Π΅ΠΊΡΡΠΈΡΠ΅ΡΠΊΠΎΠ΅ ΠΏΠΎΠ»Π΅ ΡΠ°ΠΊΠΆΠ΅ Π΄Π΅ΠΉΡΡΠ²ΡΠ΅Ρ ΠΈ Π½Π° ΡΠ»Π΅ΠΊΡΡΠΎΠ½Π΅ΠΉΡΡΠ°Π»ΡΠ½ΡΠ΅ ΡΠ°ΡΡΠΈΡΡ Ρ ΠΎΠ±ΡΠΈΠΌ Π½ΡΠ»Π΅Π²ΡΠΌ Π·Π°ΡΡΠ΄ΠΎΠΌ, Π½ΠΎ ΠΎΠ±Π»Π°Π΄Π°ΡΡΠΈΠ΅ ΡΠ»Π΅ΠΊΡΡΠΈΡΠ΅ΡΠΊΠΎΠΉ ΠΏΠΎΠ»ΡΡΠ½ΠΎΡΡΡΡ ΠΈΠ»ΠΈ ΡΠΏΠΎΡΠΎΠ±Π½ΡΠ΅ ΠΏΡΠΈΠΎΠ±ΡΠ΅ΡΡΠΈ ΠΏΠΎΠ»ΡΡΠ½ΠΎΡΡΡ Π² ΡΠ»Π΅ΠΊΡΡΠΈΡΠ΅ΡΠΊΠΎΠΌ ΠΏΠΎΠ»Π΅. ΠΡΠΎ ΡΠΎΠΆΠ΅ ΠΌΠΎΠ³ΡΡ Π±ΡΡΡ ΠΈ ΠΌΠΈΠΊΡΠΎ — ΠΈ ΠΌΠ°ΠΊΡΠΎ ΡΠ°ΡΡΠΈΡΡ.
2.9.2 ΠΠ²ΠΈΠΆΠ΅Π½ΠΈΠ΅ ΡΠ²ΠΎΠ±ΠΎΠ΄Π½ΠΎΠΉ Π·Π°ΡΡΠΆΠ΅Π½Π½ΠΎΠΉ ΡΠ°ΡΡΠΈΡΡ Π² ΡΠ»Π΅ΠΊΡΡΠΎΡΡΠ°ΡΠΈΡΠ΅ΡΠΊΠΎΠΌ ΠΏΠΎΠ»Π΅
β» ΠΠ²ΠΈΠΆΠ΅Π½ΠΈΠ΅ ΡΠ°ΡΡΠΈΡΡ Π·Π°Π²ΠΈΡΠΈΡ ΠΎΡ Π΅Π΅ ΠΌΠ°ΡΡΡ ΠΈ Π΄Π΅ΠΉΡΡΠ²ΡΡΡΠ΅ΠΉ Π½Π° Π½Π΅Π΅ ΡΠΈΠ»Ρ ΠΈ ΠΏΡΠΎΠΈΡΡ ΠΎΠ΄ΠΈΡ ΠΏΠΎ Π·Π°ΠΊΠΎΠ½Ρ . ΠΠ΅ΠΉΡΡΠ²ΡΡ Π½Π° Π·Π°ΡΡΠ΄ ΡΠ°ΡΡΠΈΡΡ ΡΠΈΠ»ΠΎΠΉ, ΡΠ»Π΅ΠΊΡΡΠΎΡΡΠ°ΡΠΈΡΠ΅ΡΠΊΠΎΠ΅ ΠΏΠΎΠ»Π΅ Π²ΠΎΠ·Π±ΡΠΆΠ΄Π°Π΅Ρ Π΅Π΅ Π΄Π²ΠΈΠΆΠ΅Π½ΠΈΠ΅ Π² ΡΠ²ΠΎΠ±ΠΎΠ΄Π½ΠΎΠΌ ΠΏΡΠΎΡΡΡΠ°Π½ΡΡΠ²Π΅ ΠΏΠΎ Π·Π°ΠΊΠΎΠ½Ρ, ΡΡΠΎ ΡΠΎΠΎΡΠ²Π΅ΡΡΡΠ²ΡΠ΅Ρ Ρ ΡΡΠ°Π²Π½Π΅Π½ΠΈΡ.
,
Π³Π΄Π΅ — ΡΠ°Π΄ΠΈΡΡ β Π²Π΅ΠΊΡΠΎΡ ΡΠ°ΡΡΠΈΡΡ.
Π£ΡΠ°Π²Π½Π΅Π½ΠΈΠ΅ ΡΠΎΠ΄Π΅ΡΠΆΠΈΡ ΠΈΠ½ΡΠΎΡΠΌΠ°ΡΠΈΡ ΠΎ ΡΠΊΠΎΡΠΎΡΡΠΈ ΠΈ ΡΡΠ°Π΅ΠΊΡΠΎΡΠΈΠΈ Π΄Π²ΠΈΠΆΠ΅Π½ΠΈΡ ΡΠ°ΡΡΠΈΡΡ ΠΈ, Π² Π·Π°Π²ΠΈΡΠΈΠΌΠΎΡΡΠΈ ΠΎΡ ΠΊΠΎΠ½ΠΊΡΠ΅ΡΠ½ΡΡ ΠΎΡΠΎΠ±Π΅Π½Π½ΠΎΡΡΠ΅ΠΉ ΠΏΠΎΠ»Ρ ΠΈ ΠΎΡ Π½Π°ΡΠ°Π»ΡΠ½ΡΡ ΡΡΠ»ΠΎΠ²ΠΈΠΉΠΈ
ΠΏΡΠΈ t=0. ΠΡΠΎΠ±Π΅Π½Π½ΠΎΡΡΠΈ ΡΠ»Π΅ΠΊΡΡΠΎΡΡΠ°ΡΠΈΡΠ΅ΡΠΊΠΈΡ ΠΏΠΎΠ»Π΅ΠΉ ΠΏΡΠ°ΠΊΡΠΈΡΠ΅ΡΠΊΠΈ Π±Π΅Π·Π³ΡΠ°Π½ΠΈΡΠ½Ρ. ΠΡΠΎ ΠΌΠΎΠ³ΡΡ Π±ΡΡΡ ΠΎΠ΄Π½ΠΎΡΠΎΠ΄Π½ΡΠ΅ ΠΈ ΡΠ°Π·Π½ΠΎΠΎΠ±ΡΠ°Π·Π½ΡΠ΅ Π½Π΅ΠΎΠ΄Π½ΠΎΡΠΎΠ΄Π½ΡΠ΅ ΠΏΠΎΠ»Ρ. Π ΡΠ°ΠΊΠΈΡ ΠΏΠΎΠ»ΡΡ ΡΡΠ°Π΅ΠΊΡΠΎΡΠΈΠΈ ΡΠ°ΡΡΠΈΡ ΠΌΠΎΠ³ΡΡ Π±ΡΡΡ ΠΈ ΠΏΡΡΠΌΡΠΌΠΈ, ΠΈ ΠΏΠ°ΡΠ°Π±ΠΎΠ»Π°ΠΌΠΈ ΠΈ Π΄ΡΡΠ³ΠΈΠΌΠΈ Π±ΠΎΠ»Π΅Π΅ ΡΠ»ΠΎΠΆΠ½ΡΠΌΠΈ ΠΊΡΠΈΠ²ΡΠΌΠΈ. ΠΠ΄Π½Π°ΠΊΠΎ Π½Π° Π»ΡΠ±ΠΎΠΉ ΠΈΠ· Π²ΠΎΠ·ΠΌΠΎΠΆΠ½ΡΡ ΡΡΠ°Π΅ΠΊΡΠΎΡΠΈΠΉ ΡΡΠΊΠΎΡΠ΅Π½ΠΈΠ΅ ΡΠ°ΡΡΠΈΡΡ
.
Π’Π°ΠΊΠΈΠΌ ΠΎΠ±ΡΠ°Π·ΠΎΠΌ ΡΡΠΊΠΎΡΠ΅Π½ΠΈΠ΅ ΡΠ°ΡΡΠΈΡΡ ΠΏΠΎΠ΄ Π΄Π΅ΠΉΡΡΠ²ΠΈΠ΅ΠΌ ΠΏΠΎΠ»Ρ ΠΎΠΏΡΠ΅Π΄Π΅Π»ΡΠ΅ΡΡΡ Π² Π·Π½Π°ΡΠΈΡΠ΅Π»ΡΠ½ΠΎΠΉ ΠΌΠ΅ΡΠ΅ ΠΎΡΠ½ΠΎΡΠ΅Π½ΠΈΠ΅ΠΌ Π·Π°ΡΡΠ΄Π° ΡΠ°ΡΡΠΈΡΡ ΠΊ Π΅Π΅ ΠΌΠ°ΡΡΠ΅, ΠΈΠ»ΠΈ ΠΈΠ½Π°ΡΠ΅ Π΅Π΅ ΡΠ΄Π΅Π»ΡΠ½ΡΠΌ Π·Π°ΡΡΠ΄ΠΎΠΌ. ΠΠΈΠΊΡΠΎΡΠ°ΡΡΠΈΡΡ ΠΎΠ±Π»Π°Π΄Π°ΡΡ Π²Π΅ΡΡΠΌΠ° Π±ΠΎΠ»ΡΡΠΈΠΌ ΡΠ΄Π΅Π»ΡΠ½ΡΠΌ Π·Π°ΡΡΠ΄ΠΎΠΌ, Π½Π°ΠΏΡΠΈΠΌΠ΅Ρ Π΄Π»Ρ ΡΠ»Π΅ΠΊΡΡΠΎΠ½Π°ΡΠΎΠ³Π΄Π° ΠΊΠ°ΠΊ Ρ ΠΌΠ°ΠΊΡΠΎΡΠ°ΡΡΠΈΡΞ³ Π²Π΅ΡΡΠΌΠ° ΠΌΠ°Π»ΠΎ, ΠΎΠ±ΡΡΠ½ΠΎ .
2.9.3 ΠΠ° ΠΊΠ°ΠΊΠΈΠ΅ ΡΠ°ΡΡΠΈΡΡ Π²Π΅ΡΠ΅ΡΡΠ²Π° Π΄Π΅ΠΉΡΡΠ²ΡΠ΅Ρ ΠΌΠ°Π³Π½ΠΈΡΠ½ΠΎΠ΅ ΠΏΠΎΠ»Π΅
β» ΠΠ°Π³Π½ΠΈΡΠ½ΠΎΠ΅ ΠΏΠΎΠ»Π΅ Π΄Π΅ΠΉΡΡΠ²ΡΠ΅Ρ Π½Π° ΠΌΠ°ΠΊΡΠΎ- ΠΈ ΠΌΠΈΠΊΡΠΎΡΠ°ΡΡΠΈΡΡ, ΠΎΠ±Π»Π°Π΄Π°ΡΡΠΈΠ΅ ΡΠ»Π΅ΠΊΡΡΠΈΡΠ΅ΡΠΊΠΈΠΌ Π·Π°ΡΡΠ΄ΠΎΠΌ, Π½ΠΎ ΡΠΎΠ»ΡΠΊΠΎ Π² ΡΠΎΡΡΠΎΡΠ½ΠΈΠΈ ΠΈΡ Π΄Π²ΠΈΠΆΠ΅Π½ΠΈΡ. Π‘ΠΈΠ»ΠΎΠ²ΠΎΠΌΡ Π²ΠΎΠ·Π΄Π΅ΠΉΡΡΠ²ΠΈΡ ΡΠΎ ΡΡΠΎΡΠΎΠ½Ρ ΠΌΠ°Π³Π½ΠΈΡΠ½ΠΎΠ³ΠΎ ΠΏΠΎΠ»Ρ ΠΏΠΎΠ΄Π²Π΅ΡΠ³Π°ΡΡΡΡ ΡΠ°ΠΊΠΆΠ΅ ΡΠ»Π΅ΠΊΡΡΠΎΠ½Π΅ΠΉΡΡΠ°Π»ΡΠ½ΡΠ΅ ΡΠ°ΡΡΠΈΡΡ, ΠΎΠ±Π»Π°Π΄Π°ΡΡΠΈΠ΅ ΠΌΠ°Π³Π½ΠΈΡΠ½ΡΠΌ ΠΌΠΎΠΌΠ΅Π½ΡΠΎΠΌ. Π‘Π²ΠΎΠΈΠΌ Π΄Π΅ΠΉΡΡΠ²ΠΈΠ΅ΠΌ Π½Π° Π°ΡΠΎΠΌ ΠΌΠ°Π³Π½ΠΈΡΠ½ΠΎΠ΅ ΠΏΠΎΠ»Π΅ ΠΌΠΎΠΆΠ΅Ρ Π²Π»ΠΈΡΡΡ Π½Π° Π²Π½ΡΡΡΠ΅Π½Π½Π΅Π΅ ΡΠΎΡΡΠΎΡΠ½ΠΈΠ΅ Π°ΡΠΎΠΌΠ°.
2.9.4 ΠΠ²ΠΈΠΆΠ΅Π½ΠΈΠ΅ ΡΠ²ΠΎΠ±ΠΎΠ΄Π½ΠΎΠΉ Π·Π°ΡΡΠΆΠ΅Π½Π½ΠΎΠΉ ΡΠ°ΡΡΠΈΡΡ Π² ΠΎΠ΄Π½ΠΎΡΠΎΠ΄Π½ΠΎΠΌ ΠΌΠ°Π³Π½ΠΈΡΠΎΡΡΠ°ΡΠΈΡΠ΅ΡΠΊΠΎΠΌ ΠΏΠΎΠ»Π΅
Π ΠΈΡ. 2.9.4
ΠΠ²ΠΈΠΆΠ΅Π½ΠΈΠ΅ ΠΏΠΎΠ΄ Π΄Π΅ΠΉΡΡΠ²ΠΈΠ΅ΠΌ ΡΠΈΠ»Ρ ΠΠΎΡΠ΅Π½ΡΠ° Π² ΠΎΠ΄Π½ΠΎΡΠΎΠ΄Π½ΠΎΠΌ ΠΌΠ°Π³Π½ΠΈΡΠ½ΠΎΠΌ ΠΏΠΎΠ»Π΅
β» ΠΡΠ΅ ΠΎΡΠΎΠ±Π΅Π½Π½ΠΎΡΡΠΈ ΡΠΈΠ»ΠΎΠ²ΠΎΠ³ΠΎ Π΄Π΅ΠΉΡΡΠ²ΠΈΡ ΠΌΠ°Π³Π½ΠΈΡΠ½ΠΎΠ³ΠΎ ΠΏΠΎΠ»Ρ Π½Π° Π·Π°ΡΡΠΆΠ΅Π½Π½ΡΡ ΡΠ°ΡΡΠΈΡΡ Π²ΡΡΠ°ΠΆΠ°ΡΡΡΡ ΡΠΈΠ»ΠΎΠΉ ΠΠΎΡΠ΅Π½ΡΠ° ΠΈ ΠΎΠΏΡΠ΅Π΄Π΅Π»ΡΡΡΡΡ ΡΡΠ°Π²Π½Π΅Π½ΠΈΠ΅ΠΌ:
Π² ΠΊΠΎΡΠΎΡΠΎΠΌ Π²Π·Π°ΠΈΠΌΠ½Π°Ρ ΠΎΡΠΈΠ΅Π½ΡΠ°ΡΠΈΡ Π²Π΅ΠΊΡΠΎΡΠΎΠ² ΠΈΡΠΎΡΡΠ°Π²Π»ΡΠ΅Ρ ΠΏΡΠ°Π²ΡΠΉ Π²ΠΈΠ½Ρ. ΠΠΎΡΠΊΠΎΠ»ΡΠΊΡ ΡΠΈΠ»Π° ΠΠΎΡΠ΅Π½ΡΠ°, ΡΠΎ Π²ΡΠ·Π²Π°Π½Π½ΠΎΠ΅ Π΅Ρ ΡΡΠΊΠΎΡΠ΅Π½ΠΈΠ΅ ΡΠ°ΡΡΠΈΡΡΡΠΎΠΆΠ΅ ΠΏΠ΅ΡΠΏΠ΅Π½Π΄ΠΈΠΊΡΠ»ΡΡΠ½ΠΎ ΠΊ ΡΠΊΠΎΡΠΎΡΡΠΈΠΡΡΡΠ΄Π° ΡΠ»Π΅Π΄ΡΠ΅Ρ, ΡΡΠΎ Π΄Π²ΠΈΠΆΠ΅Π½ΠΈΠ΅ ΡΠ°ΡΡΠΈΡΡ Π² ΠΌΠ°Π³Π½ΠΈΡΠ½ΠΎΠΌ ΠΏΠΎΠ»Π΅ Π΄ΠΎΠ»ΠΆΠ½ΠΎ ΠΏΡΠΎΠΈΡΡ ΠΎΠ΄ΠΈΡΡ Π»ΠΈΠ±ΠΎ ΠΏΠΎ ΠΎΠΊΡΡΠΆΠ½ΠΎΡΡΠΈ, Π»ΠΈΠ±ΠΎ ΠΏΠΎ ΡΠΏΠΈΡΠ°Π»ΠΈ, ΠΊΠ°ΠΊ ΡΡΠΎ ΠΏΠΎΠΊΠ°Π·Π°Π½ΠΎ Π½Π° ΡΠΈΡ.2.9.4. Π΄Π»Ρ ΡΠ»ΡΡΠ°Ρ ΠΎΠ΄Π½ΠΎΡΠΎΠ΄Π½ΠΎΠ³ΠΎ ΠΏΠΎΠ»Ρ.
ΠΠΎΠΏΠ΅ΡΠ΅ΡΠ½ΠΎΠ΅ ΠΊ ΡΠΊΠΎΡΠΎΡΡΠΈ ΠΌΠ°Π³Π½ΠΈΡΠ½ΠΎΠ΅ ΠΏΠΎΠ»Π΅ Π·Π°Ρ Π²Π°ΡΡΠ²Π°Π΅Ρ ΡΠ°ΡΡΠΈΡΡ ΠΈ Π²ΠΎΠ²Π»Π΅ΠΊΠ°Π΅Ρ Π΅Π΅ Π² Π΄Π²ΠΈΠΆΠ΅Π½ΠΈΠ΅ ΠΏΠΎ ΠΊΡΡΠ³ΠΎΠ²ΠΎΠΉ ΠΎΡΠ±ΠΈΡΠ΅ Ρ ΠΏΠ΅ΡΠΈΠΎΠ΄ΠΈΡΠ΅ΡΠΊΠΈΠΌ ΠΏΠΎΠ²ΡΠΎΡΠ΅Π½ΠΈΠ΅ΠΌ Π΄Π²ΠΈΠΆΠ΅Π½ΠΈΡ Π½Π° Π΄Π°Π½Π½ΠΎΠΉ ΠΎΡΠ±ΠΈΡΠ΅, ΠΏΡΠΈ ΡΡΠΎΠΌ
ΠΡΠ»ΠΈ ΡΠΊΠΎΡΠΎΡΡΡ ΡΠ°ΡΡΠΈΡΡ Π½Π΅ ΠΏΠ΅ΡΠΏΠ΅Π½Π΄ΠΈΠΊΡΠ»ΡΡΠ½Π° ΠΊ Π½Π°ΠΏΡΠ°Π²Π»Π΅Π½ΠΈΡ ΠΏΠΎΠ»Ρ, ΡΠΎ Π΅Π΅ Π΄Π²ΠΈΠΆΠ΅Π½ΠΈΠ΅ Π±ΡΠ΄Π΅Ρ ΠΏΡΠΎΠΈΡΡ ΠΎΠ΄ΠΈΡΡ ΠΏΠΎ ΡΠΏΠΈΡΠ°Π»ΠΈ, ΠΏΠ°ΡΠ°ΠΌΠ΅ΡΡΡ ΠΊΠΎΡΠΎΡΠΎΠΉ Π±ΡΠ΄ΡΡ ΠΎΠΏΡΠ΅Π΄Π΅Π»ΡΡΡΡΡ ΠΏΠΎΠΏΠ΅ΡΠ΅ΡΠ½ΠΎΠΉ ΠΈ ΠΏΡΠΎΠ΄ΠΎΠ»ΡΠ½ΠΎΠΉ ΡΠΎΡΡΠ°Π²Π»ΡΡΡΠΈΠΌΠΈ ΡΠΊΠΎΡΠΎΡΡΠΈ:
Π Π°Π΄ΠΈΡΡ Π²ΠΈΡΠΊΠΎΠ² ΡΠΏΠΈΡΠ°Π»ΠΈ ΠΈ ΡΠ°Π³ ΠΌΠ΅ΠΆΠ΄Ρ Π²ΠΈΡΠΊΠ°ΠΌΠΈ ΠΎΠΏΡΠ΅Π΄Π΅Π»ΡΡΡΡΡ ΠΏΡΠΈ ΡΡΠΎΠΌ Π²ΡΡΠ°ΠΆΠ΅Π½ΠΈΡΠΌΠΈ
Π ΡΠ»ΡΡΠ°Π΅ Π½Π΅ΠΎΠ΄Π½ΠΎΡΠΎΠ΄Π½ΠΎΠ³ΠΎ ΠΌΠ°Π³Π½ΠΈΡΠ½ΠΎΠ³ΠΎ ΠΏΠΎΠ»Ρ Π΄Π²ΠΈΠΆΠ΅Π½ΠΈΠ΅ Π·Π°ΡΡΠΆΠ΅Π½Π½ΠΎΠΉ ΡΠ°ΡΡΠΈΡΡ ΡΡΡΠ΅ΡΡΠ²Π΅Π½Π½ΠΎ ΡΡΠ»ΠΎΠΆΠ½ΡΠ΅ΡΡΡ. Π‘ΠΏΠΈΡΠ°Π»ΡΠ½Π°Ρ ΡΡΠ°Π΅ΠΊΡΠΎΡΠΈΡ ΡΠ°ΡΡΠΈΡΡ ΡΡΡΡΠ΅ΠΌΠ»ΡΠ΅ΡΡΡ Π² ΠΎΠ±Π»Π°ΡΡΡ ΡΠΈΠ»ΡΠ½ΠΎΠ³ΠΎ ΠΏΠΎΠ»Ρ Ρ ΡΠΌΠ΅Π½ΡΡΠ΅Π½ΠΈΠ΅ΠΌ ΡΠ°Π΄ΠΈΡΡΠ° Π²ΠΈΡΠΊΠ° Π½Π° ΠΊΠ°ΠΆΠ΄ΠΎΠΌ ΠΎΠ±ΠΎΡΠΎΡΠ΅. ΠΠΎ Π΄ΠΎΡΡΠΈΠΆΠ΅Π½ΠΈΠΈ Π² ΡΠΈΠ»ΡΠ½ΠΎΠΌ ΠΏΠΎΠ»Π΅ ΡΠ°ΠΊΠΎΠΉ ΡΠΎΡΠΊΠΈ, Π³Π΄Π΅ ΠΏΡΠΎΠ΄ΠΎΠ»ΡΠ½Π°Ρ ΡΠΎΡΡΠ°Π²Π»ΡΡΡΠ°Ρ ΡΠΊΠΎΡΠΎΡΡΠΈ ΠΎΠ±ΡΠ°ΡΠ°Π΅ΡΡΡ Π² Π½ΡΠ»Ρ ΠΈ ΠΌΠ΅Π½ΡΠ΅Ρ Π·Π½Π°ΠΊ Π½Π° ΠΏΡΠΎΡΠΈΠ²ΠΎΠΏΠΎΠ»ΠΎΠΆΠ½ΡΠΉ, ΡΠ°ΡΡΠΈΡΠ° Π½Π°ΡΠΈΠ½Π°Π΅Ρ Π΄Π²ΠΈΠ³Π°ΡΡΡΡ Π² ΠΎΠ±ΡΠ°ΡΠ½ΡΡ ΡΡΠΎΡΠΎΠ½Ρ ΡΠΆΠ΅ ΠΏΠΎ ΡΠ°ΡΡΠΈΡΡΡΡΠ΅ΠΉΡΡ ΡΠΏΠΈΡΠ°Π»ΠΈ. ΠΡΠΎΠΈΡΡ ΠΎΠ΄ΠΈΡ ΡΠ²ΠΎΠ΅ΠΎΠ±ΡΠ°Π·Π½ΠΎΠ΅ ΠΎΡΡΠ°ΠΆΠ΅Π½ΠΈΠ΅ Π·Π°ΡΡΠΆΠ΅Π½Π½ΠΎΠΉ ΡΠ°ΡΡΠΈΡΡ ΡΠΈΠ»ΡΠ½ΡΠΌ Π½Π΅ΠΎΠ΄Π½ΠΎΡΠΎΠ΄Π½ΡΠΌ ΠΌΠ°Π³Π½ΠΈΡΠ½ΡΠΌ ΠΏΠΎΠ»Π΅ΠΌ. ΠΡΠ»ΠΈ ΡΠ°ΠΊΠΈΠ΅ ΠΎΡΡΠ°ΠΆΠ°ΡΠ΅Π»ΡΠ½ΡΠ΅ ΠΏΠΎΠ»Ρ ΠΎΠ±ΡΠ°Π·ΠΎΠ²Π°ΡΡ Ρ Π΄Π²ΡΡ ΡΡΠΎΡΠΎΠ½, ΡΠΎ ΡΠ°ΡΡΠΈΡΠ° ΠΎΠΊΠ°ΠΆΠ΅ΡΡΡ Π² ΠΌΠ°Π³Π½ΠΈΡΠ½ΠΎΠΉ Π»ΠΎΠ²ΡΡΠΊΠ΅, ΠΊΠΎΡΠΎΡΡΡ ΠΎΠ½Π° Π½Π΅ ΡΠΌΠΎΠΆΠ΅Ρ ΠΏΠΎΠΊΠΈΠ½ΡΡΡ.
2.9.5 ΠΠ²ΠΈΠΆΠ΅Π½ΠΈΠ΅ ΡΠ²ΠΎΠ±ΠΎΠ΄Π½ΠΎΠΉ Π·Π°ΡΡΠΆΠ΅Π½Π½ΠΎΠΉ ΡΠ°ΡΡΠΈΡΡ Π² ΠΏΠΎΡΡΠΎΡΠ½Π½ΠΎΠΌ ΡΠ»Π΅ΠΊΡΡΠΎΠΌΠ°Π³Π½ΠΈΡΠ½ΠΎΠΌ ΠΏΠΎΠ»Π΅
β» ΠΠΎΠ»Π΅ ΡΡΠΈΡΠ°Π΅ΡΡΡ ΡΠ»Π΅ΠΊΡΡΠΎΠΌΠ°Π³Π½ΠΈΡΠ½ΡΠΌ ΠΈ ΠΏΠΎΡΡΠΎΡΠ½Π½ΡΠΌ, Π΅ΡΠ»ΠΈ Π² ΠΊΠ°ΠΆΠ΄ΠΎΠΉ ΡΠΎΡΠΊΠ΅ Π΅Π³ΠΎ ΠΏΡΠΎΡΡΡΠ°Π½ΡΡΠ²Π° ΠΎΠ΄Π½ΠΎΠ²ΡΠ΅ΠΌΠ΅Π½Π½ΠΎ Π΅ΡΡΡ Π΄Π²Π° ΠΏΠΎΠ»Ρ β ΡΠ»Π΅ΠΊΡΡΠΈΡΠ΅ΡΠΊΠΎΠ΅ ΠΈ ΠΌΠ°Π³Π½ΠΈΡΠ½ΠΎΠ΅, ΠΏΡΠΈΡΠ΅ΠΌ ΠΊΠ°ΠΆΠ΄ΠΎΠ΅ ΠΈΠ· Π½ΠΈΡ ΠΏΠΎΡΡΠΎΡΠ½Π½ΠΎ. Π§Π°ΡΡΠΈΡΠ° Π²Π΅ΡΠ΅ΡΡΠ²Π°, ΠΎΠ±Π»Π°Π΄Π°ΡΡΠ°Ρ ΡΠ»Π΅ΠΊΡΡΠΈΡΠ΅ΡΠΊΠΈΠΌ Π·Π°ΡΡΠ΄ΠΎΠΌq, Π±ΡΠ΄Π΅Ρ ΠΈΡΠΏΡΡΡΠ²Π°ΡΡ Π² ΡΠ°ΠΊΠΈΡ ΡΡΠ»ΠΎΠ²ΠΈΡΡ ΡΠΈΠ»ΠΎΠ²ΠΎΠ΅ Π΄Π΅ΠΉΡΡΠ²ΠΈΠ΅ ΠΊΠ°ΠΊ ΡΠΎ ΡΡΠΎΡΠΎΠ½Ρ ΡΠ»Π΅ΠΊΡΡΠΈΡΠ΅ΡΠΊΠΎΠ³ΠΎ, ΡΠ°ΠΊ ΠΈ ΡΠΎ ΡΡΠΎΡΠΎΠ½Ρ ΠΌΠ°Π³Π½ΠΈΡΠ½ΠΎΠ³ΠΎ ΠΏΠΎΠ»Ρ. Π Π΅Π·ΡΠ»ΡΡΠΈΡΡΡΡΠ΅Π΅ Π΄Π΅ΠΉΡΡΠ²ΠΈΠ΅ Π½Π° ΡΠ°ΡΡΠΈΡΡ ΠΎΠΏΡΠ΅Π΄Π΅Π»ΡΠ΅ΡΡΡ ΠΏΡΠΈ ΡΡΠΎΠΌ ΠΎΠ±ΠΎΠ±ΡΠ΅Π½Π½ΠΎΠΉ ΡΠΈΠ»ΠΎΠΉ ΠΠΎΡΠ΅Π½ΡΠ°:
Π³Π΄Π΅ ΠΈ- ΡΡΠΎ ΡΠ»Π΅ΠΊΡΡΠΈΡΠ΅ΡΠΊΠ°Ρ ΠΈ ΠΌΠ°Π³Π½ΠΈΡΠ½Π°Ρ ΡΠΎΡΡΠ°Π²Π»ΡΡΡΠΈΠ΅ ΡΡΠΎΠΉ ΡΠΈΠ»Ρ.
ΠΠ²ΠΈΠΆΠ΅Π½ΠΈΠ΅ ΡΠ°ΡΡΠΈΡΡ Π±ΡΠ΄Π΅Ρ ΠΏΡΠΎΠΈΡΡ ΠΎΠ΄ΠΈΡΡ ΠΏΠΎ Π·Π°ΠΊΠΎΠ½Ρ , ΠΈΠ· ΠΊΠΎΡΠΎΡΠΎΠ³ΠΎ ΡΠ»Π΅Π΄ΡΠ΅Ρ
ΠΈΠ»ΠΈ ΡΡΠΎ ΡΠΎΠΆΠ΅
ΠΠ· ΡΠ΅ΡΠ΅Π½ΠΈΡ Π΄Π°Π½Π½ΠΎΠ³ΠΎ ΡΡΠ°Π²Π½Π΅Π½ΠΈΡ ΠΏΡΠΈ Π·Π°Π΄Π°Π½Π½ΡΡ Π½Π°ΡΠ°Π»ΡΠ½ΡΡ ΡΡΠ»ΠΎΠ²ΠΈΡΡ Π½Π°Ρ ΠΎΠ΄ΡΡΡΡ ΡΠΊΠΎΡΠΎΡΡΡ, ΡΡΠΊΠΎΡΠ΅Π½ΠΈΠ΅ ΠΈ ΡΡΠ°Π΅ΠΊΡΠΎΡΠΈΡ Π΄Π²ΠΈΠΆΠ΅Π½ΠΈΡ ΡΠ°ΡΡΠΈΡΡ ΠΠΎΠ»ΡΠΈΠ² ΡΡΠ°ΡΠΈΠΎΠ½Π°ΡΠ½ΡΡ ΡΡΠ»ΠΎΠ²ΠΈΡΡ ΠΌΠΎΠ³ΡΡ Π±ΡΡΡ Π½Π΅Π·Π°Π²ΠΈΡΠΈΠΌΡΠΌΠΈ ΠΈ ΡΠΏΡΠ°Π²Π»ΡΠ΅ΠΌΡΠΌΠΈ ΠΊΠ°ΠΊ ΠΏΠΎ ΠΌΠΎΠ΄ΡΠ»Ρ, ΡΠ°ΠΊ ΠΈ ΠΏΠΎ Π²Π·Π°ΠΈΠΌΠ½ΠΎΠΉ ΠΎΡΠΈΠ΅Π½ΡΠ°ΡΠΈΠΈ. ΠΡΠΈ ΡΠ°ΠΊΠΈΡ ΡΡΠ»ΠΎΠ²ΠΈΡΡ ΠΌΠΎΠΆΠ½ΠΎ ΡΠΏΡΠ°Π²Π»ΡΡΡ ΡΡΠ°Π΅ΠΊΡΠΎΡΠΈΠ΅ΠΉ ΡΠ°ΡΡΠΈΡΡ
ΠΠ²ΠΈΠΆΠ΅Π½ΠΈΠ΅ Π·Π°ΡΡΠΆΠ΅Π½Π½ΠΎΠΉ ΡΠ°ΡΡΠΈΡΡ Π² ΡΠ»Π΅ΠΊΡΡΠΈΡΠ΅ΡΠΊΠΎΠΌ ΠΈ ΠΌΠ°Π³Π½ΠΈΡΠ½ΠΎΠΌ ΠΏΠΎΠ»ΡΡ .
Β
ΠΡΠ»ΠΈ ΡΠ°ΡΡΠΈΡΠ°, ΠΎΠ±Π»Π°Π΄Π°ΡΡΠ°Ρ Π·Π°ΡΡΠ΄ΠΎΠΌ Π΅, Π΄Π²ΠΈΠΆΠ΅ΡΡΡ Π² ΠΏΡΠΎΡΡΡΠ°Π½ΡΡΠ²Π΅, Π³Π΄Π΅ ΠΈΠΌΠ΅Π΅ΡΡΡ ΡΠ»Π΅ΠΊΡΡΠΈΡΠ΅ΡΠΊΠΎΠ΅ ΠΏΠΎΠ»Π΅ Ρ Π½Π°ΠΏΡΡΠΆΡΠ½Π½ΠΎΡΡΡΡ E ΡΠΎ Π½Π° Π½Π΅Ρ Π΄Π΅ΠΉΡΡΠ²ΡΠ΅Ρ ΡΠΈΠ»Π° eE. ΠΡΠ»ΠΈ, ΠΊΡΠΎΠΌΠ΅ ΡΠ»Π΅ΠΊΡΡΠΈΡΠ΅ΡΠΊΠΎΠ³ΠΎ, ΠΈΠΌΠ΅Π΅ΡΡΡ ΠΌΠ°Π³Π½ΠΈΡΠ½ΠΎΠ΅ ΠΏΠΎΠ»Π΅, ΡΠΎ Π½Π° ΡΠ°ΡΡΠΈΡΡ Π΄Π΅ΠΉΡΡΠ²ΡΠ΅Ρ Π΅ΡΡ ΡΠΈΠ»Π° ΠΠΎΡΠ΅Π½ΡΠ°, ΡΠ°Π²Π½Π°Ρ e[uB] , Π³Π΄Π΅ u — ΡΠΊΠΎΡΠΎΡΡΡ Π΄Π²ΠΈΠΆΠ΅Π½ΠΈΡ ΡΠ°ΡΡΠΈΡΡ ΠΎΡΠ½ΠΎΡΠΈΡΠ΅Π»ΡΠ½ΠΎ ΠΏΠΎΠ»Ρ, B — ΠΌΠ°Π³Π½ΠΈΡΠ½Π°Ρ ΠΈΠ½Π΄ΡΠΊΡΠΈΡ. ΠΠΎΡΡΠΎΠΌΡ ΡΠΎΠ³Π»Π°ΡΠ½ΠΎ Π²ΡΠΎΡΠΎΠΌΡ Π·Π°ΠΊΠΎΠ½Ρ ΠΡΡΡΠΎΠ½Π° ΡΡΠ°Π²Π½Π΅Π½ΠΈΠ΅ Π΄Π²ΠΈΠΆΠ΅Π½ΠΈΡ ΡΠ°ΡΡΠΈΡ ΠΈΠΌΠ΅Π΅Ρ Π²ΠΈΠ΄:
ΠΠ°ΠΏΠΈΡΠ°Π½Π½ΠΎΠ΅ Π²Π΅ΠΊΡΠΎΡΠ½ΠΎΠ΅ ΡΡΠ°Π²Π½Π΅Π½ΠΈΠ΅ ΡΠ°ΡΠΏΠ°Π΄Π°Π΅ΡΡΡ Π½Π° ΡΡΠΈ ΡΠΊΠ°Π»ΡΡΠ½ΡΡ ΡΡΠ°Π²Π½Π΅Π½ΠΈΡ, ΠΊΠ°ΠΆΠ΄ΠΎΠ΅ ΠΈΠ· ΠΊΠΎΡΠΎΡΡΡ ΠΎΠΏΠΈΡΡΠ²Π°Π΅Ρ Π΄Π²ΠΈΠΆΠ΅Π½ΠΈΠ΅ Π²Π΄ΠΎΠ»Ρ ΡΠΎΠΎΡΠ²Π΅ΡΡΡΠ²ΡΡΡΠ΅ΠΉ ΠΊΠΎΠΎΡΠ΄ΠΈΠ½Π°ΡΠ½ΠΎΠΉ ΠΎΡΠΈ.
ΠΡΠ΅Π΄ΠΏΠΎΠ»ΠΎΠΆΠΈΠΌ, ΡΡΠΎ Π·Π°ΡΡΠΆΠ΅Π½Π½ΡΠ΅ ΡΠ°ΡΡΠΈΡΡ, Π΄Π²ΠΈΠ³Π°Π²ΡΠΈΠ΅ΡΡ ΠΏΠ΅ΡΠ²ΠΎΠ½Π°ΡΠ°Π»ΡΠ½ΠΎ Π²Π΄ΠΎΠ»Ρ ΠΎΡΠΈ Π₯ ΡΠΎ ΡΠΊΠΎΡΠΎΡΡΡΡ ΠΏΠΎΠΏΠ°Π΄Π°ΡΡ Π² ΡΠ»Π΅ΠΊΡΡΠΈΡΠ΅ΡΠΊΠΎΠ΅ ΠΏΠΎΠ»Π΅ ΠΏΠ»ΠΎΡΠΊΠΎΠ³ΠΎ ΠΊΠΎΠ½Π΄Π΅Π½ΡΠ°ΡΠΎΡΠ°.
ΠΡΠ»ΠΈ Π·Π°Π·ΠΎΡ ΠΌΠ΅ΠΆΠ΄Ρ ΠΏΠ»Π°ΡΡΠΈΠ½Π°ΠΌΠΈ ΠΌΠ°Π» ΠΏΠΎ ΡΡΠ°Π²Π½Π΅Π½ΠΈΡ Ρ ΠΈΡ Π΄Π»ΠΈΠ½ΠΎΠΉ, ΡΠΎ ΠΊΡΠ°Π΅Π²ΡΠΌΠΈ ΡΡΡΠ΅ΠΊΡΠ°ΠΌΠΈ ΠΌΠΎΠΆΠ½ΠΎ ΠΏΡΠ΅Π½Π΅Π±ΡΠ΅ΡΡ ΠΈ ΡΡΠΈΡΠ°ΡΡ ΡΠ»Π΅ΠΊΡΡΠΈΡΠ΅ΡΠΊΠΎΠ΅ ΠΏΠΎΠ»Π΅ ΠΌΠ΅ΠΆΠ΄Ρ ΠΏΠ»Π°ΡΡΠΈΠ½Π°ΠΌΠΈ ΠΎΠ΄Π½ΠΎΡΠΎΠ΄Π½ΡΠΌ. ΠΠ°ΠΏΡΠ°Π²Π»ΡΡ ΠΎΡΡ Y ΠΏΠ°ΡΠ°Π»Π»Π΅Π»ΡΠ½ΠΎ ΠΏΠΎΠ»Ρ, ΠΌΡ ΠΈΠΌΠ΅Π΅ΠΌ: . Π’Π°ΠΊ ΠΊΠ°ΠΊ ΠΌΠ°Π³Π½ΠΈΡΠ½ΠΎΠ³ΠΎ ΠΏΠΎΠ»Ρ Π½Π΅Ρ, ΡΠΎ . Π ΡΠ°ΡΡΠΌΠ°ΡΡΠΈΠ²Π°Π΅ΠΌΠΎΠΌ ΡΠ»ΡΡΠ°Π΅ Π½Π° Π·Π°ΡΡΠΆΠ΅Π½Π½ΡΠ΅ ΡΠ°ΡΡΠΈΡΡ Π΄Π΅ΠΉΡΡΠ²ΡΠ΅Ρ ΡΠΎΠ»ΡΠΊΠΎ ΡΠΈΠ»Π° ΡΠΎ ΡΡΠΎΡΠΎΠ½Ρ ΡΠ»Π΅ΠΊΡΡΠΈΡΠ΅ΡΠΊΠΎΠ³ΠΎ ΠΏΠΎΠ»Ρ, ΠΊΠΎΡΠΎΡΠ°Ρ ΠΏΡΠΈ Π²ΡΠ±ΡΠ°Π½Π½ΠΎΠΌ Π½Π°ΠΏΡΠ°Π²Π»Π΅Π½ΠΈΠΈ ΠΊΠΎΠΎΡΠ΄ΠΈΠ½Π°ΡΠ½ΡΡ ΠΎΡΠ΅ΠΉ ΡΠ΅Π»ΠΈΠΊΠΎΠΌ Π½Π°ΠΏΡΠ°Π²Π»Π΅Π½Π° ΠΏΠΎ ΠΎΡΠΈ Y. ΠΠΎΡΡΠΎΠΌΡ ΡΡΠ°Π΅ΠΊΡΠΎΡΠΈΡ Π΄Π²ΠΈΠΆΠ΅Π½ΠΈΡ ΡΠ°ΡΡΠΈΡ Π»Π΅ΠΆΠΈΡ Π² ΠΏΠ»ΠΎΡΠΊΠΎΡΡΠΈ XY ΠΈ ΡΡΠ°Π²Π½Π΅Π½ΠΈΡ Π΄Π²ΠΈΠΆΠ΅Π½ΠΈΡ ΠΏΡΠΈΠ½ΠΈΠΌΠ°ΡΡ Π²ΠΈΠ΄:
ΠΠ²ΠΈΠΆΠ΅Π½ΠΈΠ΅ ΡΠ°ΡΡΠΈΡ Π² ΡΡΠΎΠΌ ΡΠ»ΡΡΠ°Π΅ ΠΏΡΠΎΠΈΡΡ ΠΎΠ΄ΠΈΡ ΠΏΠΎΠ΄ Π΄Π΅ΠΉΡΡΠ²ΠΈΠ΅ΠΌ ΠΏΠΎΡΡΠΎΡΠ½Π½ΠΎΠΉ ΡΠΈΠ»Ρ ΠΈ ΠΏΠΎΠ΄ΠΎΠ±Π½ΠΎ Π΄Π²ΠΈΠΆΠ΅Π½ΠΈΡ Π³ΠΎΡΠΈΠ·ΠΎΠ½ΡΠ°Π»ΡΠ½ΠΎ Π±ΡΠΎΡΠ΅Π½Π½ΠΎΠ³ΠΎ ΡΠ΅Π»Π° Π² ΠΏΠΎΠ»Π΅ ΡΡΠΆΠ΅ΡΡΠΈ. ΠΠΎΡΡΠΎΠΌΡ ΡΡΠ½ΠΎ Π±Π΅Π· Π΄Π°Π»ΡΠ½Π΅ΠΉΡΠΈΡ ΡΠ°ΡΡΠ΅ΡΠΎΠ², ΡΡΠΎ ΡΠ°ΡΡΠΈΡΡ Π±ΡΠ΄ΡΡ Π΄Π²ΠΈΠ³Π°ΡΡΡΡ ΠΏΠΎ ΠΏΠ°ΡΠ°Π±ΠΎΠ»Π°ΠΌ.
ΠΡΡΠΈΡΠ»ΠΈΠΌ ΡΠ³ΠΎΠ» , Π½Π° ΠΊΠΎΡΠΎΡΡΠΉ ΠΎΡΠΊΠ»ΠΎΠ½ΠΈΡΡΡ ΠΏΡΡΠΎΠΊ ΡΠ°ΡΡΠΈΡ ΠΏΠΎΡΠ»Π΅ ΠΏΡΠΎΡ ΠΎΠΆΠ΄Π΅Π½ΠΈΡ ΡΠ΅ΡΠ΅Π· ΠΊΠΎΠ½Π΄Π΅Π½ΡΠ°ΡΠΎΡ. ΠΠ½ΡΠ΅Π³ΡΠΈΡΡΡ ΠΏΠ΅ΡΠ²ΠΎΠ΅ ΠΈΠ· ΡΡΠ°Π²Π½Π΅Π½ΠΈΠΉ, Π½Π°Ρ ΠΎΠ΄ΠΈΠΌ:
ΠΠ½ΡΠ΅Π³ΡΠ°ΡΠΈΡ Π²ΡΠΎΡΠΎΠ³ΠΎ ΡΡΠ°Π²Π½Π΅Π½ΠΈΡ Π΄Π°ΡΡ:
Π’Π°ΠΊ ΠΊΠ°ΠΊ ΠΏΡΠΈ t=0 (ΠΌΠΎΠΌΠ΅Π½Ρ Π²ΡΡΡΠΏΠ»Π΅Π½ΠΈΡ ΡΠ°ΡΡΠΈΡΡ Π² ΠΊΠΎΠ½Π΄Π΅Π½ΡΠ°ΡΠΎΡ) u(y)=0, ΡΠΎ c=0, ΠΈ ΠΏΠΎΡΡΠΎΠΌΡ
ΠΡΡΡΠ΄Π° ΠΏΠΎΠ»ΡΡΠ°Π΅ΠΌ Π΄Π»Ρ ΡΠ³Π»Π° ΠΎΡΠΊΠ»ΠΎΠ½Π΅Π½ΠΈΡ:
ΠΡΡΠ΅ΠΊΡ Π₯ΠΎΠ»Π»Π° β ΡΠ²Π»Π΅Π½ΠΈΠ΅ Π²ΠΎΠ·Π½ΠΈΠΊΠ½ΠΎΠ²Π΅Π½ΠΈΡ ΠΏΠΎΠΏΠ΅ΡΠ΅ΡΠ½ΠΎΠΉ ΡΠ°Π·Π½ΠΎΡΡΠΈ ΠΏΠΎΡΠ΅Π½ΡΠΈΠ°Π»ΠΎΠ² (Π½Π°Π·ΡΠ²Π°Π΅ΠΌΠΎΠΉ ΡΠ°ΠΊΠΆΠ΅ Π₯ΠΎΠ»Π»ΠΎΠ²ΡΠΊΠΈΠΌ Π½Π°ΠΏΡΡΠΆΠ΅Π½ΠΈΠ΅ΠΌ) ΠΏΡΠΈ ΠΏΠΎΠΌΠ΅ΡΠ΅Π½ΠΈΠΈ ΠΏΡΠΎΠ²ΠΎΠ΄Π½ΠΈΠΊΠ° Ρ ΠΏΠΎΡΡΠΎΡΠ½Π½ΡΠΌ ΡΠΎΠΊΠΎΠΌ Π² ΠΌΠ°Π³Π½ΠΈΡΠ½ΠΎΠ΅ ΠΏΠΎΠ»Π΅.
Π ΠΏΡΠΎΡΡΠ΅ΠΉΡΠ΅ΠΌ ΡΠ°ΡΡΠΌΠΎΡΡΠ΅Π½ΠΈΠΈ ΡΡΡΠ΅ΠΊΡ Π₯ΠΎΠ»Π»Π° Π²ΡΠ³Π»ΡΠ΄ΠΈΡ ΡΠ»Π΅Π΄ΡΡΡΠΈΠΌ ΠΎΠ±ΡΠ°Π·ΠΎΠΌ. ΠΡΡΡΡ ΡΠ΅ΡΠ΅Π· ΠΌΠ΅ΡΠ°Π»Π»ΠΈΡΠ΅ΡΠΊΠΈΠΉ Π±ΡΡΡ Π² ΡΠ»Π°Π±ΠΎΠΌ ΠΌΠ°Π³Π½ΠΈΡΠ½ΠΎΠΌ ΠΏΠΎΠ»Π΅ B ΡΠ΅ΡΡΡ ΡΠ»Π΅ΠΊΡΡΠΈΡΠ΅ΡΠΊΠΈΠΉ ΡΠΎΠΊ ΠΏΠΎΠ΄ Π΄Π΅ΠΉΡΡΠ²ΠΈΠ΅ΠΌ Π½Π°ΠΏΡΡΠΆΡΠ½Π½ΠΎΡΡΠΈ E. ΠΠ°Π³Π½ΠΈΡΠ½ΠΎΠ΅ ΠΏΠΎΠ»Π΅ Π±ΡΠ΄Π΅Ρ ΠΎΡΠΊΠ»ΠΎΠ½ΡΡΡΠ½ΠΎΡΠΈΡΠ΅Π»ΠΈ Π·Π°ΡΡΠ΄Π° (Π΄Π»Ρ ΠΎΠΏΡΠ΅Π΄Π΅Π»ΡΠ½Π½ΠΎΡΡΠΈ ΡΠ»Π΅ΠΊΡΡΠΎΠ½Ρ) ΠΎΡ ΠΈΡ Π΄Π²ΠΈΠΆΠ΅Π½ΠΈΡ Π²Π΄ΠΎΠ»Ρ ΠΈΠ»ΠΈ ΠΏΡΠΎΡΠΈΠ² ΡΠ»Π΅ΠΊΡΡΠΈΡΠ΅ΡΠΊΠΎΠ³ΠΎ ΠΏΠΎΠ»Ρ ΠΊ ΠΎΠ΄Π½ΠΎΠΉ ΠΈΠ· Π³ΡΠ°Π½Π΅ΠΉ Π±ΡΡΡΠ°. ΠΡΠΈ ΡΡΠΎΠΌ ΠΊΡΠΈΡΠ΅ΡΠΈΠ΅ΠΌ ΠΌΠ°Π»ΠΎΡΡΠΈ Π±ΡΠ΄Π΅Ρ ΡΠ»ΡΠΆΠΈΡΡ ΡΡΠ»ΠΎΠ²ΠΈΠ΅, ΡΡΠΎ ΠΏΡΠΈ ΡΡΠΎΠΌ ΡΠ»Π΅ΠΊΡΡΠΎΠ½ Π½Π΅ Π½Π°ΡΠ½ΡΡ Π΄Π²ΠΈΠ³Π°ΡΡΡΡ ΠΏΠΎ ΡΠΏΠΈΡΠ°Π»ΠΈ.
Π’Π°ΠΊΠΈΠΌ ΠΎΠ±ΡΠ°Π·ΠΎΠΌ, ΡΠΈΠ»Π° ΠΠΎΡΠ΅Π½ΡΠ° ΠΏΡΠΈΠ²Π΅Π΄ΡΡ ΠΊ Π½Π°ΠΊΠΎΠΏΠ»Π΅Π½ΠΈΡ ΠΎΡΡΠΈΡΠ°ΡΠ΅Π»ΡΠ½ΠΎΠ³ΠΎ Π·Π°ΡΡΠ΄Π° Π²ΠΎΠ·Π»Π΅ ΠΎΠ΄Π½ΠΎΠΉ Π³ΡΠ°Π½ΠΈ Π±ΡΡΡΠΊΠ° ΠΈ ΠΏΠΎΠ»ΠΎΠΆΠΈΡΠ΅Π»ΡΠ½ΠΎΠ³ΠΎ Π²ΠΎΠ·Π»Π΅ ΠΏΡΠΎΡΠΈΠ²ΠΎΠΏΠΎΠ»ΠΎΠΆΠ½ΠΎΠΉ. ΠΠ°ΠΊΠΎΠΏΠ»Π΅Π½ΠΈΠ΅ Π·Π°ΡΡΠ΄Π° Π±ΡΠ΄Π΅Ρ ΠΏΡΠΎΠ΄ΠΎΠ»ΠΆΠ°ΡΡΡΡ Π΄ΠΎ ΡΠ΅Ρ ΠΏΠΎΡ, ΠΏΠΎΠΊΠ° Π²ΠΎΠ·Π½ΠΈΠΊΡΠ΅Π΅ ΡΠ»Π΅ΠΊΡΡΠΈΡΠ΅ΡΠΊΠΎΠ΅ ΠΏΠΎΠ»Π΅ Π·Π°ΡΡΠ΄ΠΎΠ² E1 Π½Π΅ ΡΠΊΠΎΠΌΠΏΠ΅Π½ΡΠΈΡΡΠ΅Ρ ΠΌΠ°Π³Π½ΠΈΡΠ½ΡΡ ΡΠΎΡΡΠ°Π²Π»ΡΡΡΡΡ ΡΠΈΠ»Ρ ΠΠΎΡΠ΅Π½ΡΠ°:
Π‘ΠΊΠΎΡΠΎΡΡΡ ΡΠ»Π΅ΠΊΡΡΠΎΠ½ΠΎΠ² v ΠΌΠΎΠΆΠ½ΠΎ Π²ΡΡΠ°Π·ΠΈΡΡ ΡΠ΅ΡΠ΅Π· ΠΏΠ»ΠΎΡΠ½ΠΎΡΡΡ ΡΠΎΠΊΠ°:
Π³Π΄Π΅ n β ΠΊΠΎΠ½ΡΠ΅Π½ΡΡΠ°ΡΠΈΡ Π½ΠΎΡΠΈΡΠ΅Π»Π΅ΠΉ Π·Π°ΡΡΠ΄Π°. Π’ΠΎΠ³Π΄Π°
ΠΠΎΡΡΡΠΈΡΠΈΠ΅Π½Ρ ΠΏΡΠΎΠΏΠΎΡΡΠΈΠΎΠ½Π°Π»ΡΠ½ΠΎΡΡΠΈ ΠΌΠ΅ΠΆΠ΄Ρ E1 ΠΈ jB Π½Π°Π·ΡΠ²Π°Π΅ΡΡΡ ΠΊΠΎΡΡΡΠΈΡΠΈΠ΅Π½ΡΠΎΠΌ (ΠΊΠΎΠ½ΡΡΠ°Π½ΡΠΎΠΉ) Π₯ΠΎΠ»Π»Π°. Π ΡΠ°ΠΊΠΎΠΌ ΠΏΡΠΈΠ±Π»ΠΈΠΆΠ΅Π½ΠΈΠΈ Π·Π½Π°ΠΊ ΠΏΠΎΡΡΠΎΡΠ½Π½ΠΎΠΉ Π₯ΠΎΠ»Π»Π° Π·Π°Π²ΠΈΡΠΈΡ ΠΎΡ Π·Π½Π°ΠΊΠ° Π½ΠΎΡΠΈΡΠ΅Π»Π΅ΠΉ Π·Π°ΡΡΠ΄Π°, ΡΡΠΎ ΠΏΠΎΠ·Π²ΠΎΠ»ΡΠ΅Ρ ΠΎΠΏΡΠ΅Π΄Π΅Π»ΡΡΡ ΠΈΡ ΡΠΈΠΏ Π΄Π»Ρ Π±ΠΎΠ»ΡΡΠΎΠ³ΠΎ ΡΠΈΡΠ»Π° ΠΌΠ΅ΡΠ°Π»Π»ΠΎΠ². ΠΠ»Ρ Π½Π΅ΠΊΠΎΡΠΎΡΡΡ ΠΌΠ΅ΡΠ°Π»Π»ΠΎΠ² (Π² ΡΠΈΠ»ΡΠ½ΡΡ ΠΏΠΎΠ»ΡΡ ), ΡΠ°ΠΊΠΈΡ ΠΊΠ°ΠΊ Π°Π»ΡΠΌΠΈΠ½ΠΈΠΉ, ΡΠΈΠ½ΠΊ, ΠΆΠ΅Π»Π΅Π·ΠΎ, ΠΊΠΎΠ±Π°Π»ΡΡ, Π½Π°Π±Π»ΡΠ΄Π°Π΅ΡΡΡ ΠΏΠΎΠ»ΠΎΠΆΠΈΡΠ΅Π»ΡΠ½ΡΠΉ Π·Π½Π°ΠΊ RH, ΡΡΠΎ ΠΎΠ±ΡΡΡΠ½ΡΠ΅ΡΡΡ Π² ΠΏΠΎΠ»ΡΠΊΠ»Π°ΡΡΠΈΡΠ΅ΡΠΊΠΎΠΉ ΠΈ ΠΊΠ²Π°Π½ΡΠΎΠ²ΠΎΠΉ ΡΠ΅ΠΎΡΠΈΡΡ ΡΠ²ΡΡΠ΄ΠΎΠ³ΠΎ ΡΠ΅Π»Π°.
ΠΠ°ΡΡ-ΡΠΏΠ΅ΠΊΡΡΠΎΠΌΠ΅ΡΡΠΈΡ (ΠΌΠ°ΡΡ-ΡΠΏΠ΅ΠΊΡΡΠΎΡΠΊΠΎΠΏΠΈΡ, ΠΌΠ°ΡΡ-ΡΠΏΠ΅ΠΊΡΡΠΎΠ³ΡΠ°ΡΠΈΡ, ΠΌΠ°ΡΡ-ΡΠΏΠ΅ΠΊΡΡΠ°Π»ΡΠ½ΡΠΉ Π°Π½Π°Π»ΠΈΠ·, ΠΌΠ°ΡΡ-ΡΠΏΠ΅ΠΊΡΡΠΎΠΌΠ΅ΡΡΠΈΡΠ΅ΡΠΊΠΈΠΉ Π°Π½Π°Π»ΠΈΠ·) β ΠΌΠ΅ΡΠΎΠ΄ ΠΈΡΡΠ»Π΅Π΄ΠΎΠ²Π°Π½ΠΈΡ Π²Π΅ΡΠ΅ΡΡΠ²Π° ΠΏΡΡΡΠΌ ΠΎΠΏΡΠ΅Π΄Π΅Π»Π΅Π½ΠΈΡ ΠΎΡΠ½ΠΎΡΠ΅Π½ΠΈΡ ΠΌΠ°ΡΡΡ ΠΊ Π·Π°ΡΡΠ΄Ρ (ΠΊΠ°ΡΠ΅ΡΡΠ²Π°) ΠΈ ΠΊΠΎΠ»ΠΈΡΠ΅ΡΡΠ²Π°Π·Π°ΡΡΠΆΠ΅Π½Π½ΡΡ ΡΠ°ΡΡΠΈΡ, ΠΎΠ±ΡΠ°Π·ΡΡΡΠΈΡ ΡΡ ΠΏΡΠΈ ΡΠΎΠΌ ΠΈΠ»ΠΈ ΠΈΠ½ΠΎΠΌ ΠΏΡΠΎΡΠ΅ΡΡΠ΅ Π²ΠΎΠ·Π΄Π΅ΠΉΡΡΠ²ΠΈΡ Π½Π° Π²Π΅ΡΠ΅ΡΡΠ²ΠΎ (ΡΠΌ. ΠΌΠ΅ΡΠΎΠ΄Ρ ΠΈΠΎΠ½ΠΈΠ·Π°ΡΠΈΠΈ). ΠΠ°ΡΡ-ΡΠΏΠ΅ΠΊΡΡΠΎΠΌΠ΅ΡΡ β ΡΡΠΎ Π²Π°ΠΊΡΡΠΌΠ½ΡΠΉ ΠΏΡΠΈΠ±ΠΎΡ, ΠΈΡΠΏΠΎΠ»ΡΠ·ΡΡΡΠΈΠΉ ΡΠΈΠ·ΠΈΡΠ΅ΡΠΊΠΈΠ΅ Π·Π°ΠΊΠΎΠ½Ρ Π΄Π²ΠΈΠΆΠ΅Π½ΠΈΡ Π·Π°ΡΡΠΆΠ΅Π½Π½ΡΡ ΡΠ°ΡΡΠΈΡ Π² ΠΌΠ°Π³Π½ΠΈΡΠ½ΡΡ ΠΈ ΡΠ»Π΅ΠΊΡΡΠΈΡΠ΅ΡΠΊΠΈΡ ΠΏΠΎΠ»ΡΡ , ΠΈ Π½Π΅ΠΎΠ±Ρ ΠΎΠ΄ΠΈΠΌΡΠΉ Π΄Π»Ρ ΠΏΠΎΠ»ΡΡΠ΅Π½ΠΈΡ ΠΌΠ°ΡΡ-ΡΠΏΠ΅ΠΊΡΡΠ°.
Π£ΡΠΊΠΎΡΠΈΡΠ΅Π»Ρ Π·Π°ΡΡΠΆΠ΅Π½Π½ΡΡ ΡΠ°ΡΡΠΈΡ β ΠΊΠ»Π°ΡΡ ΡΡΡΡΠΎΠΉΡΡΠ² Π΄Π»Ρ ΠΏΠΎΠ»ΡΡΠ΅Π½ΠΈΡ Π·Π°ΡΡΠΆΠ΅Π½Π½ΡΡ ΡΠ°ΡΡΠΈΡ (ΡΠ»Π΅ΠΌΠ΅Π½ΡΠ°ΡΠ½ΡΡ ΡΠ°ΡΡΠΈΡ, ΠΈΠΎΠ½ΠΎΠ²) Π²ΡΡΠΎΠΊΠΈΡ ΡΠ½Π΅ΡΠ³ΠΈΠΉ.
Π ΠΎΡΠ½ΠΎΠ²Π΅ ΡΠ°Π±ΠΎΡΡ ΡΡΠΊΠΎΡΠΈΡΠ΅Π»Ρ Π·Π°Π»ΠΎΠΆΠ΅Π½ΠΎ Π²Π·Π°ΠΈΠΌΠΎΠ΄Π΅ΠΉΡΡΠ²ΠΈΠ΅ Π·Π°ΡΡΠΆΠ΅Π½Π½ΡΡ ΡΠ°ΡΡΠΈΡ Ρ ΡΠ»Π΅ΠΊΡΡΠΈΡΠ΅ΡΠΊΠΈΠΌ ΠΈ ΠΌΠ°Π³Π½ΠΈΡΠ½ΡΠΌ ΠΏΠΎΠ»ΡΠΌΠΈ. ΠΠ»Π΅ΠΊΡΡΠΈΡΠ΅ΡΠΊΠΎΠ΅ ΠΏΠΎΠ»Π΅ ΡΠΏΠΎΡΠΎΠ±Π½ΠΎ Π½Π°ΠΏΡΡΠΌΡΡ ΡΠΎΠ²Π΅ΡΡΠ°ΡΡ ΡΠ°Π±ΠΎΡΡ Π½Π°Π΄ ΡΠ°ΡΡΠΈΡΠ΅ΠΉ, ΡΠΎ Π΅ΡΡΡ ΡΠ²Π΅Π»ΠΈΡΠΈΠ²Π°ΡΡ Π΅Ρ ΡΠ½Π΅ΡΠ³ΠΈΡ. ΠΠ°Π³Π½ΠΈΡΠ½ΠΎΠ΅ ΠΆΠ΅ ΠΏΠΎΠ»Π΅, ΡΠΎΠ·Π΄Π°Π²Π°Ρ ΡΠΈΠ»Ρ ΠΠΎΡΠ΅Π½ΡΠ°, Π»ΠΈΡΡ ΠΎΡΠΊΠ»ΠΎΠ½ΡΠ΅Ρ ΡΠ°ΡΡΠΈΡΡ, Π½Π΅ ΠΈΠ·ΠΌΠ΅Π½ΡΡ Π΅Ρ ΡΠ½Π΅ΡΠ³ΠΈΠΈ, ΠΈ Π·Π°Π΄Π°ΡΡ ΠΎΡΠ±ΠΈΡΡ, ΠΏΠΎ ΠΊΠΎΡΠΎΡΠΎΠΉ Π΄Π²ΠΈΠΆΡΡΡΡ ΡΠ°ΡΡΠΈΡΡ.
Π£ΡΠΊΠΎΡΠΈΡΠ΅Π»ΠΈ ΠΌΠΎΠΆΠ½ΠΎ ΠΏΡΠΈΠ½ΡΠΈΠΏΠΈΠ°Π»ΡΠ½ΠΎ ΡΠ°Π·Π΄Π΅Π»ΠΈΡΡ Π½Π° Π΄Π²Π΅ Π±ΠΎΠ»ΡΡΠΈΠ΅ Π³ΡΡΠΏΠΏΡ. ΠΡΠΎ Π»ΠΈΠ½Π΅ΠΉΠ½ΡΠ΅ ΡΡΠΊΠΎΡΠΈΡΠ΅Π»ΠΈ, Π³Π΄Π΅ ΠΏΡΡΠΎΠΊ ΡΠ°ΡΡΠΈΡ ΠΎΠ΄Π½ΠΎΠΊΡΠ°ΡΠ½ΠΎ ΠΏΡΠΎΡ ΠΎΠ΄ΠΈΡ ΡΡΠΊΠΎΡΡΡΡΠΈΠ΅ ΠΏΡΠΎΠΌΠ΅ΠΆΡΡΠΊΠΈ, ΠΈ ΡΠΈΠΊΠ»ΠΈΡΠ΅ΡΠΊΠΈΠ΅ ΡΡΠΊΠΎΡΠΈΡΠ΅Π»ΠΈ, Π² ΠΊΠΎΡΠΎΡΡΡ ΠΏΡΡΠΊΠΈ Π΄Π²ΠΈΠΆΡΡΡΡ ΠΏΠΎ Π·Π°ΠΌΠΊΠ½ΡΡΡΠΌ ΠΊΡΠΈΠ²ΡΠΌ ΡΠΈΠΏΠ° ΠΎΠΊΡΡΠΆΠ½ΠΎΡΡΠ΅ΠΉ, ΠΏΡΠΎΡ ΠΎΠ΄Ρ ΡΡΠΊΠΎΡΡΡΡΠΈΠ΅ ΠΏΡΠΎΠΌΠ΅ΠΆΡΡΠΊΠΈ ΠΏΠΎ ΠΌΠ½ΠΎΠ³Ρ ΡΠ°Π·. ΠΠΎΠΆΠ½ΠΎ ΡΠ°ΠΊΠΆΠ΅ ΠΊΠ»Π°ΡΡΠΈΡΠΈΡΠΈΡΠΎΠ²Π°ΡΡ ΡΡΠΊΠΎΡΠΈΡΠ΅Π»ΠΈ ΠΏΠΎ Π½Π°Π·Π½Π°ΡΠ΅Π½ΠΈΡ: ΠΊΠΎΠ»Π»Π°ΠΉΠ΄Π΅ΡΡ, ΠΈΡΡΠΎΡΠ½ΠΈΠΊΠΈ Π½Π΅ΠΉΡΡΠΎΠ½ΠΎΠ², Π±ΡΡΡΠ΅ΡΡ, ΠΈΡΡΠΎΡΠ½ΠΈΠΊΠΈ ΡΠΈΠ½Ρ ΡΠΎΡΡΠΎΠ½Π½ΠΎΠ³ΠΎ ΠΈΠ·Π»ΡΡΠ΅Π½ΠΈΡ, ΡΡΡΠ°Π½ΠΎΠ²ΠΊΠΈ Π΄Π»Ρ ΡΠ΅ΡΠ°ΠΏΠΈΠΈ ΡΠ°ΠΊΠ°, ΠΏΡΠΎΠΌΡΡΠ»Π΅Π½Π½ΡΠ΅ ΡΡΠΊΠΎΡΠΈΡΠ΅Π»ΠΈ.
Β
Β
ΠΠ»ΠΈΡΠ΅Ρ Π»ΠΈ Π½Π° Π΄Π²ΠΈΠΆΡΡΠΈΠ΅ΡΡ Π·Π°ΡΡΠ΄Ρ ΠΌΠ°Π³Π½ΠΈΡΠ½ΠΎΠ΅ ΠΏΠΎΠ»Π΅, ΠΊΠΎΡΠΎΡΠΎΠ΅ ΠΎΠ½ΠΈ ΡΠΎΠ·Π΄Π°ΡΡ ΠΏΡΠΈ Π΄Π²ΠΈΠΆΠ΅Π½ΠΈΠΈ, ΡΠΎΠ·Π΄Π°Π²Π°Ρ ΡΠΎΠΊ? ΠΡΠ»ΠΈ Π½Π΅Ρ, ΡΠΎ ΠΊΠ°ΠΊ Π²ΠΎΠ·ΠΌΠΎΠΆΠ½Π° ΡΠ°ΠΌΠΎΠΈΠ½Π΄ΡΠΊΡΠΈΡ?
Π‘ΠΏΡΠΎΡΠΈΠ»
ΠΠ·ΠΌΠ΅Π½Π΅Π½ΠΎ 6 Π»Π΅Ρ, 8 ΠΌΠ΅ΡΡΡΠ΅Π² Π½Π°Π·Π°Π΄
ΠΡΠΎΡΠΌΠΎΡΡΠ΅Π½ΠΎ 5ΠΊ ΡΠ°Π·
$\begingroup$
ΠΡΡΡΡ ΡΠ³ΡΡΡΠΎΠΊ Π·Π°ΡΡΠ΄ΠΎΠ² Π΄Π²ΠΈΠΆΠ΅ΡΡΡ Ρ ΠΏΠΎΡΡΠΎΡΠ½Π½ΠΎΠΉ ΡΠΊΠΎΡΠΎΡΡΡΡ $\mathbf v\;. $ ΠΠΎΡΠΊΠΎΠ»ΡΠΊΡ Π·Π°ΡΡΠ΄Ρ Π΄Π²ΠΈΠΆΡΡΡΡ, ΠΎΠ½ΠΈ Π±ΡΠ΄ΡΡ ΡΠΎΠ·Π΄Π°Π²Π°ΡΡ ΠΌΠ°Π³Π½ΠΈΡΠ½ΠΎΠ΅ ΠΏΠΎΠ»Π΅ $\bf B$, ΡΠ°ΠΊ ΠΊΠ°ΠΊ ΠΈΠΌΠ΅Π½Π½ΠΎ ΡΠΎΠΊ ΡΠΎΠ·Π΄Π°Π΅Ρ ΠΌΠ°Π³Π½ΠΈΡΠ½ΠΎΠ΅ ΠΏΠΎΠ»Π΅. 92} \;;$$, ΡΠ°ΠΊ ΠΊΠ°ΠΊ $d\mathbf l$ ΠΈ $\hat{\mathbf r}$ Π½Π°ΠΏΡΠ°Π²Π»Π΅Π½Ρ Π² ΠΎΠ΄Π½ΠΎΠΌ Π½Π°ΠΏΡΠ°Π²Π»Π΅Π½ΠΈΠΈ, ΡΠΈΡΠ»ΠΈΡΠ΅Π»Ρ ΡΠ°Π²Π΅Π½ Π½ΡΠ»Ρ, Π° Π² Π·Π½Π°ΠΌΠ΅Π½Π°ΡΠ΅Π»Π΅ $r$ ΡΠ°Π²Π΅Π½ Π½ΡΠ»Ρ, ΡΡΠΎ ΡΠ½ΠΎΠ²Π° Π΄Π΅Π»Π°Π΅Ρ ΡΠΈΠ»Ρ Π±Π΅ΡΠΊΠΎΠ½Π΅ΡΠ½ΠΎΠΉ . ΠΡΠΎ Π½Π΅Π²ΠΎΠ·ΠΌΠΎΠΆΠ½ΠΎ.
ΠΠ΄Π½Π°ΠΊΠΎ, ΠΊΠΎΠ³Π΄Π° ΡΠΎΠΊ Π½Π΅ΠΏΠΎΡΡΠΎΡΠ½Π΅Π½, Π΄Π²ΠΈΠΆΡΡΠΈΠ΅ΡΡ Π·Π°ΡΡΠ΄Ρ ΡΠΎΠ·Π΄Π°ΡΡ Π·Π°Π²ΠΈΡΡΡΠ΅Π΅ ΠΎΡ Π²ΡΠ΅ΠΌΠ΅Π½ΠΈ ΠΌΠ°Π³Π½ΠΈΡΠ½ΠΎΠ΅ ΠΏΠΎΠ»Π΅, ΡΡΠΎ ΠΏΡΠΈΠ²ΠΎΠ΄ΠΈΡ ΠΊ ΡΠ°ΠΌΠΎΠΈΠ½Π΄ΡΠΊΡΠΈΠΈ.
ΠΠΠ‘ ΡΠ°ΠΌΠΎΠΈΠ½Π΄ΡΠΊΡΠΈΠΈ, Π²ΡΠ·Π²Π°Π½Π½Π°Ρ ΠΈΠ·ΠΌΠ΅Π½Π΅Π½ΠΈΠ΅ΠΌ ΠΏΠΎΡΠΎΠΊΠ° ΠΌΠ°Π³Π½ΠΈΡΠ½ΠΎΠ³ΠΎ ΠΏΠΎΠ»Ρ, ΡΠΎΠ·Π΄Π°Π²Π°Π΅ΠΌΠΎΠ³ΠΎ Π·Π°ΡΡΠ΄Π°ΠΌΠΈ, Π΄Π΅ΠΉΡΡΠ²ΡΠ΅Ρ Π½Π° Π·Π°ΡΡΠ΄Ρ; ΠΊΠ°ΠΆΠ΄ΡΠΉ Π·Π°ΡΡΠ΄ ΠΈΡΠΏΡΡΡΠ²Π°Π΅Ρ ΡΠΈΠ»Ρ $\mathbf F= q\; \mathbf v\times \mathbf B $, Π³Π΄Π΅ $\mathbf B$ ΡΠΎΠ·Π΄Π°Π΅ΡΡΡ ΠΈΠΌΠΈ ΡΠ°ΠΌΠΈΠΌΠΈ.
ΠΡΠ°ΠΊ, ΡΡΠΎ ΠΎΠ·Π½Π°ΡΠ°Π΅Ρ, ΡΡΠΎ Π·Π°ΡΡΠ΄Ρ Π΄Π²ΠΈΠΆΡΡΡΡ ΠΈ ΡΠΎΠ·Π΄Π°ΡΡ ΠΌΠ°Π³Π½ΠΈΡΠ½ΠΎΠ΅ ΠΏΠΎΠ»Π΅, ΠΊΠΎΡΠΎΡΠΎΠ΅ ΡΠ½ΠΎΠ²Π° Π²ΠΎΠ·Π΄Π΅ΠΉΡΡΠ²ΡΠ΅Ρ Π½Π° Π·Π°ΡΡΠ΄Ρ, ΡΠΎΠ·Π΄Π°Π²ΡΠΈΠ΅ ΠΏΠΎΠ»Π΅.
Π ΡΡΠ°ΡΠΈΠΎΠ½Π°ΡΠ½ΠΎΠΌ ΡΠΎΡΡΠΎΡΠ½ΠΈΠΈ Π·Π°ΡΡΠ΄Ρ Π½Π΅ ΠΏΠΎΠ΄Π²Π΅ΡΠΆΠ΅Π½Ρ Π²Π»ΠΈΡΠ½ΠΈΡ ΡΠΎΠ±ΡΡΠ²Π΅Π½Π½ΠΎΠ³ΠΎ ΠΌΠ°Π³Π½ΠΈΡΠ½ΠΎΠ³ΠΎ ΠΏΠΎΠ»Ρ, ΡΠΎΠ³Π΄Π° ΠΊΠ°ΠΊ, Ρ Π΄ΡΡΠ³ΠΎΠΉ ΡΡΠΎΡΠΎΠ½Ρ, Π² Π½Π΅ΡΡΠ°ΡΠΈΠΎΠ½Π°ΡΠ½ΠΎΠΌ ΡΠΎΡΡΠΎΡΠ½ΠΈΠΈ ΠΌΠ°Π³Π½ΠΈΡΠ½ΠΎΠ΅ ΠΏΠΎΠ»Π΅, ΡΠΎΠ·Π΄Π°Π²Π°Π΅ΠΌΠΎΠ΅ ΡΡΠΈΠΌΠΈ Π΄Π²ΠΈΠΆΡΡΠΈΠΌΠΈΡΡ Π·Π°ΡΡΠ΄Π°ΠΌΠΈ, ΠΏΡΠΈΠ΄Π°Π΅Ρ ΡΠΈΠ»Ρ ΠΊΠ°ΠΆΠ΄ΠΎΠΌΡ Π·Π°ΡΡΠ΄Ρ, ΡΡΠΎ ΠΏΡΠΈΠ²ΠΎΠ΄ΠΈΡ ΠΊ Π²ΠΎΠ·Π½ΠΈΠΊΠ½ΠΎΠ²Π΅Π½ΠΈΡ ΡΠ°ΠΌΠΎ- Ρ. Π΄.Ρ.
ΠΠΎ ΠΏΠΎΡΠ΅ΠΌΡ ΡΠ°ΠΊ?
ΠΠΎΠ»Ρ ΡΠ²Π»ΡΡΡΡΡ Π½Π΅Π·Π°Π²ΠΈΡΠΈΠΌΡΠΌΠΈ ΠΎΠ±ΡΠ΅ΠΊΡΠ°ΠΌΠΈ; ΠΎΠ½ΠΈ ΠΌΠΎΠ³ΡΡ Π΄Π΅Π»ΠΈΡΡΡΡ ΠΈΠΌΠΏΡΠ»ΡΡΠΎΠΌ ΠΈ ΡΠ½Π΅ΡΠ³ΠΈΠ΅ΠΉ Ρ Π·Π°ΡΡΠ΄Π°ΠΌΠΈ, ΠΊΠ°ΠΊ ΡΠΊΠ°Π·Π°Π» Π’ΠΈΠΌΠ΅ΠΉ.
ΠΠ° Π·Π°ΡΡΠ΄Ρ Π΄Π΅ΠΉΡΡΠ²ΠΈΡΠ΅Π»ΡΠ½ΠΎ Π²Π»ΠΈΡΠ΅Ρ ΠΈΡ ΠΏΠΎΠ»Π΅, ΠΊΠΎΠ³Π΄Π° ΠΎΠ½ΠΈ ΡΠΈΡΠ°ΡΡΡΡ. ΠΠΎ, Π½Π°ΡΠΊΠΎΠ»ΡΠΊΠΎ Ρ ΡΠΈΡΠ°Π», Ρ Π½Π΅ ΠΎΠ±Π½Π°ΡΡΠΆΠΈΠ», ΡΡΠΎ ΠΏΡΠΈ Π½Π΅ΡΡΠ°ΡΠΈΠΎΠ½Π°ΡΠ½ΠΎΠΌ ΡΠΎΡΡΠΎΡΠ½ΠΈΠΈ ΡΠΎΠΊΠ° ΠΈΠ·Π»ΡΡΠ°ΡΡΡΡ Π·Π°ΡΡΠ΄Ρ. Π’Π΅ΠΌ Π½Π΅ ΠΌΠ΅Π½Π΅Π΅ Π½Π° Π½ΠΈΡ Π²Π»ΠΈΡΠ΅Ρ ΠΈΠ·ΠΌΠ΅Π½ΡΡΡΠΈΠΉΡΡ ΠΏΠΎΡΠΎΠΊ ΡΠΎΠ·Π΄Π°Π²Π°Π΅ΠΌΠΎΠ³ΠΎ ΠΈΠΌΠΈ ΠΌΠ°Π³Π½ΠΈΡΠ½ΠΎΠ³ΠΎ ΠΏΠΎΠ»Ρ. ΠΡΠΎ ΡΠΎΠ·Π΄Π°Π΅Ρ ΡΠΎΠ±ΡΡΠ²Π΅Π½Π½ΡΡ ΠΠΠ‘; ΡΡΠΎ ΠΎΠ·Π½Π°ΡΠ°Π΅Ρ, ΡΡΠΎ ΠΌΠ°Π³Π½ΠΈΡΠ½ΠΎΠ΅ ΠΏΠΎΠ»Π΅ Π²ΠΎΠ·Π΄Π΅ΠΉΡΡΠ²ΡΠ΅Ρ Π½Π° ΡΠ΅ Π΄Π²ΠΈΠΆΡΡΠΈΠ΅ΡΡ Π·Π°ΡΡΠ΄Ρ, ΠΊΠΎΡΠΎΡΡΠ΅ ΡΠΎΠ·Π΄Π°ΡΡ ΠΏΠΎΠ»Π΅.
ΠΠ΄Π½Π°ΠΊΠΎ, ΠΏΠΎΡΠ΅ΠΌΡ Π½Π° Π΄Π²ΠΈΠΆΡΡΠΈΠ΅ΡΡ Π·Π°ΡΡΠ΄Ρ Π½Π΅ Π΄Π΅ΠΉΡΡΠ²ΡΠ΅Ρ ΠΈΡ ΡΠΎΠ±ΡΡΠ²Π΅Π½Π½ΠΎΠ΅ ΠΌΠ°Π³Π½ΠΈΡΠ½ΠΎΠ΅ ΠΏΠΎΠ»Π΅, ΠΊΠΎΠ³Π΄Π° ΡΠΎΠΊ Π½Π°Ρ ΠΎΠ΄ΠΈΡΡΡ Π² ΡΡΡΠ°Π½ΠΎΠ²ΠΈΠ²ΡΠ΅ΠΌΡΡ ΡΠΎΡΡΠΎΡΠ½ΠΈΠΈ, Π½ΠΎ Π² Π½Π΅ΡΡΠ°ΡΠΈΠΎΠ½Π°ΡΠ½ΠΎΠΌ ΡΠΎΡΡΠΎΡΠ½ΠΈΠΈ Π½Π° Π·Π°ΡΡΠ΄Ρ Π΄Π΅ΠΉΡΡΠ²ΡΠ΅Ρ ΡΠΈΠ»Π° ΠΌΠ°Π³Π½ΠΈΡΠ½ΠΎΠ³ΠΎ ΠΏΠΎΠ»Ρ, ΠΊΠΎΡΠΎΡΠΎΠ΅ ΠΎΠ½ΠΈ ΡΠΎΠ·Π΄Π°ΡΡ?
ΠΡΠ»ΠΈ Π½Π° Π·Π°ΡΡΠ΄Ρ Π΄Π΅ΠΉΡΡΠ²ΠΈΡΠ΅Π»ΡΠ½ΠΎ Π²Π»ΠΈΡΠ΅Ρ ΠΌΠ°Π³Π½ΠΈΡΠ½ΠΎΠ΅ ΠΏΠΎΠ»Π΅, ΠΊΠΎΡΠΎΡΠΎΠ΅ ΠΎΠ½ΠΈ ΡΠΎΠ·Π΄Π°ΡΡ, Π΅ΡΡΡ Π»ΠΈ ΡΠΏΠΎΡΠΎΠ± ΠΌΠ°ΡΠ΅ΠΌΠ°ΡΠΈΡΠ΅ΡΠΊΠΈ Π΄ΠΎΠΊΠ°Π·Π°ΡΡ, ΡΡΠΎ Π²ΠΎ Π²ΡΠ΅ΠΌΡ ΡΠ°ΠΌΠΎΠΈΠ½Π΄ΡΠΊΡΠΈΠΈ Π΄Π²ΠΈΠΆΡΡΠΈΠ΅ΡΡ Π·Π°ΡΡΠ΄Ρ ΠΏΡΠΈΠ½ΡΠΆΠ΄Π°ΡΡΡΡ ΡΠΎΠ·Π΄Π°Π²Π°Π΅ΠΌΡΠΌ ΠΈΠΌΠΈ ΠΌΠ°Π³Π½ΠΈΡΠ½ΡΠΌ ΠΏΠΎΠ»Π΅ΠΌ?
Π― ΡΠΎΠ²ΡΠ΅ΠΌ Π·Π°ΠΏΡΡΠ°Π»ΡΡ; Ρ ΡΡΠΎ-ΡΠΎ ΠΏΡΠΎΠΏΡΡΡΠΈΠ» Π·Π΄Π΅ΡΡ? ΠΠΎΠΆΠ°Π»ΡΠΉΡΡΠ° ΠΏΠΎΠΌΠΎΠ³ΠΈ.
- ΠΌΠ°Π³Π½ΠΈΡΠ½ΡΠ΅ ΠΏΠΎΠ»Ρ
- ΡΠ»Π΅ΠΊΡΡΠΎΠΌΠ°Π³Π½ΠΈΡΠ½ΡΠ΅ ΠΈΠ½Π΄ΡΠΊΡΠΈΠΎΠ½Π½ΡΠ΅
$\endgroup$
12
$\begingroup$
ΠΠΈ ΡΡΠ°Π²Π½Π΅Π½ΠΈΡ ΠΡΠ»ΠΎΠ½Π°, Π½ΠΈ ΡΡΠ°Π²Π½Π΅Π½ΠΈΡ ΠΠΈΠΎ-Π‘Π°Π²Π°ΡΠ° Π½Π΅ ΡΠ²Π»ΡΡΡΡΡ ΠΏΡΠ°Π²ΠΈΠ»ΡΠ½ΡΠΌΠΈ ΡΡΠ°Π²Π½Π΅Π½ΠΈΡΠΌΠΈ Π΄Π»Ρ ΡΠ»Π΅ΠΊΡΡΠΎΠΌΠ°Π³Π½ΠΈΡΠ½ΠΎΠ³ΠΎ ΠΏΠΎΠ»Ρ, Π·Π° ΠΈΡΠΊΠ»ΡΡΠ΅Π½ΠΈΠ΅ΠΌ ΡΡΠ°ΡΠΈΠΊΠΈ. Π‘ΡΡΠ΅ΡΡΠ²ΡΡΡ Π·Π°Π²ΠΈΡΡΡΠΈΠ΅ ΠΎΡ Π²ΡΠ΅ΠΌΠ΅Π½ΠΈ ΠΎΠ±ΠΎΠ±ΡΠ΅Π½ΠΈΡ, ΡΠ°ΠΊΠΈΠ΅ ΠΊΠ°ΠΊ 93\vec r’$$ Π³Π΄Π΅ $t_r$ Π½Π° ΡΠ°ΠΌΠΎΠΌ Π΄Π΅Π»Π΅ ΡΠ²Π»ΡΠ΅ΡΡΡ ΡΡΠ½ΠΊΡΠΈΠ΅ΠΉ $\vec r’$, Π° ΠΈΠΌΠ΅Π½Π½ΠΎ $t_r=t-\frac{|\vec r-\vec r’|}{c}.$
ΠΠ½ΠΈ ΡΠ²ΠΎΠ΄ΡΡΡΡ ΠΊ ΠΡΠ»ΠΎΠ½Ρ ΠΈ ΠΠΈΠΎ- Π‘Π°Π²Π°ΡΠ° ΡΠΎΠ»ΡΠΊΠΎ ΡΠΎΠ³Π΄Π°, ΠΊΠΎΠ³Π΄Π° ΡΡΠΈ ΠΏΡΠΎΠΈΠ·Π²ΠΎΠ΄Π½ΡΠ΅ ΠΏΠΎ Π²ΡΠ΅ΠΌΠ΅Π½ΠΈ ΡΠΎΡΠ½ΠΎ ΡΠ°Π²Π½Ρ Π½ΡΠ»Ρ, ΡΡΠΎ ΡΠ²Π»ΡΠ΅ΡΡΡ ΡΡΠ°ΡΠΈΠΊΠΎΠΉ. Π’Π°ΠΊΠΈΠΌ ΠΎΠ±ΡΠ°Π·ΠΎΠΌ, ΠΡΠΈΠΌΠ΅Π½ΠΊΠΎ ΡΠ²Π»ΡΠ΅ΡΡΡ ΠΏΡΠΈΠΌΠ΅ΡΠΎΠΌ ΡΠΎΠ±ΡΡΠ²Π΅Π½Π½ΡΡ Π²ΡΠ΅ΠΌΠ΅Π½Π½ΡΡ Π·Π°ΠΊΠΎΠ½ΠΎΠ² Π΄Π»Ρ ΡΠ»Π΅ΠΊΡΡΠΎΠΌΠ°Π³Π½ΠΈΡΠ½ΠΎΠ³ΠΎ ΠΏΠΎΠ»Ρ. ΠΠ±ΡΠ°ΡΠΈΡΠ΅ Π²Π½ΠΈΠΌΠ°Π½ΠΈΠ΅, ΡΡΠΎ ΠΈ ΡΠ»Π΅ΠΊΡΡΠΈΡΠ΅ΡΠΊΠ°Ρ, ΠΈ ΠΌΠ°Π³Π½ΠΈΡΠ½Π°Ρ ΡΠ°ΡΡΠΈ ΡΠ»Π΅ΠΊΡΡΠΎΠΌΠ°Π³Π½ΠΈΡΠ½ΠΎΠ³ΠΎ ΠΏΠΎΠ»Ρ ΠΈΠΌΠ΅ΡΡ ΡΠ°ΡΡΠΈ, ΠΊΠΎΡΠΎΡΡΠ΅ Π·Π°Π²ΠΈΡΡΡ ΠΎΡ ΠΈΠ·ΠΌΠ΅Π½Π΅Π½ΠΈΡ ΡΠΎΠΊΠ° Π²ΠΎ Π²ΡΠ΅ΠΌΠ΅Π½ΠΈ. ΠΠ°ΠΊΠΎΠ½ Π€Π°ΡΠ°Π΄Π΅Ρ ΡΠ²ΡΠ·ΡΠ²Π°Π΅Ρ ΠΈΡ Π²ΠΌΠ΅ΡΡΠ΅.
Π ΡΠ»Π΅Π΄ΡΡΡΠΈΠΉ ΡΠ°ΠΊΡ ΡΠ°ΠΊΠΆΠ΅ ΡΠ²Π»ΡΠ΅ΡΡΡ ΠΊΠ»ΡΡΠ΅Π²ΡΠΌ: ΠΠΠ‘ Π²ΠΎΠΊΡΡΠ³ Π½Π΅ΠΏΠΎΠ΄Π²ΠΈΠΆΠ½ΠΎΠ³ΠΎ ΠΏΡΠΎΠ²ΠΎΠ΄Π° ΡΠΎΡΡΠ°Π²Π»ΡΠ΅Ρ 100% ΠΈΠ·-Π·Π° ΡΠ»Π΅ΠΊΡΡΠΈΡΠ΅ΡΠΊΠΎΠ³ΠΎ ΠΏΠΎΠ»Ρ. Π Π΅ΡΠ»ΠΈ Ρ Π²Π°Ρ Π΅ΡΡΡ Π²ΡΠ΅Π»Π΅Π½Π½Π°Ρ Ρ Π½Π΅ΠΉΡΡΠ°Π»ΡΠ½ΡΠΌΠΈ ΠΎΠ±ΡΠ΅ΠΊΡΠ°ΠΌΠΈ ΠΏΠΎΠ²ΡΡΠ΄Ρ, ΡΠΎ ΠΡΠΈΠΌΠ΅Π½ΠΊΠΎ ΠΏΡΠ΅Π΄ΡΠΊΠ°Π·ΡΠ²Π°Π΅Ρ, ΡΡΠΎ ΡΠ»Π΅ΠΊΡΡΠΈΡΠ΅ΡΠΊΠΎΠ΅ ΠΏΠΎΠ»Π΅ ΠΈΡΠΊΠ»ΡΡΠΈΡΠ΅Π»ΡΠ½ΠΎ ΠΈ Π½Π° 100% Π²ΡΠ·Π²Π°Π½ΠΎ Π²ΡΠ΅ΠΌΠ΅Π½Π½ΡΠΌΠΈ Π²Π°ΡΠΈΠ°ΡΠΈΡΠΌΠΈ ΡΠΎΠΊΠ°. Π Ρ Π΄Π΅ΠΉΡΡΠ²ΠΈΡΠ΅Π»ΡΠ½ΠΎ ΠΈΠΌΠ΅Ρ Π² Π²ΠΈΠ΄Ρ ΠΏΡΠΈΡΠΈΠ½Ρ, ΠΊΠ°ΠΊ Π² ΠΏΡΠΈΡΠΈΠ½Π΅ ΠΈ ΡΠ»Π΅Π΄ΡΡΠ²ΠΈΠΈ. Π£ΡΠ°Π²Π½Π΅Π½ΠΈΡ ΡΠ²Π»ΡΡΡΡΡ ΠΏΡΠΈΡΠΈΠ½Π½ΠΎ-ΡΠ»Π΅Π΄ΡΡΠ²Π΅Π½Π½ΡΠΌΠΈ, ΠΏΠΎΡΠΊΠΎΠ»ΡΠΊΡ Π·Π°ΡΡΠ΄ (ΠΈ ΠΈΠ·ΠΌΠ΅Π½Π΅Π½ΠΈΠ΅ Π·Π°ΡΡΠ΄Π°) ΠΈ ΡΠΎΠΊ (ΠΈ ΠΈΠ·ΠΌΠ΅Π½Π΅Π½ΠΈΠ΅ ΡΠΎΠΊΠ°) ΠΎΡΠ½ΠΎΡΡΡΡΡ ΠΊ ΠΏΡΠΎΡΠ»ΠΎΠΌΡ ($t_r\leq t$), ΠΊΠΎΡΠΎΡΡΠ΅ Π²Π»ΠΈΡΡΡ Π½Π° Π½Π°ΡΡΠΎΡΡΠ΅Π΅.
ΠΡΠ°ΠΊ, Π΄Π°Π²Π°ΠΉΡΠ΅ ΠΏΠΎΡΠΌΠΎΡΡΠΈΠΌ. ΠΡΠ»ΠΎΠ½ ΠΈ ΠΠΈΠΎ-Π‘Π°Π²Π°Ρ Π½Π΅Π²Π΅ΡΠ½Ρ, Π΅ΡΠ»ΠΈ Π½Π΅ Π² ΡΡΠ°ΡΠΈΠΊΠ΅. Π‘Π°ΠΌΠΎΠΈΠ½Π΄ΡΠΊΡΠΈΡ Π½Π΅ΠΏΠΎΠ΄Π²ΠΈΠΆΠ½ΠΎΠ³ΠΎ ΠΏΡΠΎΠ²ΠΎΠ΄Π° Π²ΡΠ·ΡΠ²Π°Π΅ΡΡΡ ΡΠ»Π΅ΠΊΡΡΠΈΡΠ΅ΡΠΊΠΈΠΌΠΈ ΠΏΠΎΠ»ΡΠΌΠΈ, ΡΠΎΠ·Π΄Π°ΡΡΠΈΠΌΠΈ Π½Π΅Π½ΡΠ»Π΅Π²ΡΡ ΠΠΠ‘. ΠΡΠΈ ΡΠ»Π΅ΠΊΡΡΠΈΡΠ΅ΡΠΊΠΈΠ΅ ΠΏΠΎΠ»Ρ Π²ΡΠ·Π²Π°Π½Ρ ΠΈΠ·ΠΌΠ΅Π½ΡΡΡΠΈΠΌΠΈΡΡ Π²ΠΎ Π²ΡΠ΅ΠΌΠ΅Π½ΠΈ ΡΠΎΠΊΠ°ΠΌΠΈ . Π Π΄Π°, ΡΡΠΈ ΠΈΠ·ΠΌΠ΅Π½ΡΡΡΠΈΠ΅ΡΡ Π²ΠΎ Π²ΡΠ΅ΠΌΠ΅Π½ΠΈ ΡΠΎΠΊΠΈ ΠΈ Π²ΡΠ·ΡΠ²Π°ΡΡ (ΠΈ Π΄Π°, Ρ ΠΈΠΌΠ΅Ρ Π² Π²ΠΈΠ΄Ρ ΠΏΡΠΈΡΠΈΠ½Ρ) ΠΌΠ°Π³Π½ΠΈΡΠ½ΡΠ΅ ΠΏΠΎΠ»Ρ.
Π ΠΊΠΎΠ³Π΄Π° Π»ΡΠ΄ΠΈ Π³ΠΎΠ²ΠΎΡΡΡ, ΡΡΠΎ ΠΏΠ΅ΡΠ΅ΠΌΠ΅Π½Π½ΡΠ΅ ΡΠ»Π΅ΠΊΡΡΠΈΡΠ΅ΡΠΊΠΈΠ΅ ΠΏΠΎΠ»Ρ Π²ΡΠ·ΡΠ²Π°ΡΡ ΠΌΠ°Π³Π½ΠΈΡΠ½ΡΠ΅ ΠΏΠΎΠ»Ρ ΠΈ Π½Π°ΠΎΠ±ΠΎΡΠΎΡ, ΡΡΠΎ Π½Π΅ ΡΠ°ΠΊΠ°Ρ ΠΏΡΠΈΡΠΈΠ½Π½ΠΎΡΡΡ, Π³Π΄Π΅ ΠΎΠ΄Π½ΠΎ Π²ΡΠ·ΡΠ²Π°Π΅Ρ Π΄ΡΡΠ³ΠΎΠ΅, ΡΡΠΎ ΠΏΡΠΎΡΡΠΎ ΡΠ°Π²Π΅Π½ΡΡΠ²ΠΎ , ΡΠ°ΠΊ ΠΊΠ°ΠΊ ΠΎΠ½ΠΈ ΡΠ²ΡΠ·ΡΠ²Π°ΡΡ Π΄Π²Π΅ Π²Π΅ΡΠΈ, ΠΏΡΠΎΠΈΡΡ ΠΎΠ΄ΡΡΠΈΠ΅ ΠΎΠ΄Π½ΠΎΠ²ΡΠ΅ΠΌΠ΅Π½Π½ΠΎ. ΠΡΠΈΡΠΈΠ½Π½ΠΎΡΡΡ ΡΠ²ΡΠ·ΡΠ²Π°Π΅Ρ Π½Π°ΡΡΠΎΡΡΠ΅Π΅ (ΡΠ»Π΅Π΄ΡΡΠ²ΠΈΡ) Ρ ΠΏΡΠΎΡΠ»ΡΠΌ (ΠΏΡΠΈΡΠΈΠ½Ρ). Π¦ΠΈΡΠΈΡΠΎΠ²Π°ΡΡ ΠΡΠΈΠΌΠ΅Π½ΠΊΠΎ:
Π½ΠΈ ΡΡΠ°Π²Π½Π΅Π½ΠΈΡ ΠΠ°ΠΊΡΠ²Π΅Π»Π»Π°, Π½ΠΈ ΠΈΡ ΡΠ΅ΡΠ΅Π½ΠΈΡ Π½Π΅ ΡΠΊΠ°Π·ΡΠ²Π°ΡΡ Π½Π° Π½Π°Π»ΠΈΡΠΈΠ΅ ΠΏΡΠΈΡΠΈΠ½Π½ΠΎΠΉ ΡΠ²ΡΠ·ΠΈ ΠΌΠ΅ΠΆΠ΄Ρ ΡΠ»Π΅ΠΊΡΡΠΈΡΠ΅ΡΠΊΠΈΠΌΠΈ ΠΈ ΠΌΠ°Π³Π½ΠΈΡΠ½ΡΠΌΠΈ ΠΏΠΎΠ»ΡΠΌΠΈ. Π‘Π»Π΅Π΄ΠΎΠ²Π°ΡΠ΅Π»ΡΠ½ΠΎ, ΠΌΡ Π΄ΠΎΠ»ΠΆΠ½Ρ Π·Π°ΠΊΠ»ΡΡΠΈΡΡ, ΡΡΠΎ ΡΠ»Π΅ΠΊΡΡΠΎΠΌΠ°Π³Π½ΠΈΡΠ½ΠΎΠ΅ ΠΏΠΎΠ»Π΅ Π΅ΡΡΡ Π΄Π²ΠΎΠΉΡΡΠ²Π΅Π½Π½Π°Ρ ΡΡΡΠ½ΠΎΡΡΡ, Π²ΡΠ΅Π³Π΄Π° ΠΈΠΌΠ΅ΡΡΠ°Ρ ΡΠ»Π΅ΠΊΡΡΠΈΡΠ΅ΡΠΊΡΡ ΠΈ ΠΌΠ°Π³Π½ΠΈΡΠ½ΡΡ ΡΠΎΡΡΠ°Π²Π»ΡΡΡΠΈΠ΅ ΠΎΠ΄Π½ΠΎΠ²ΡΠ΅ΠΌΠ΅Π½Π½ΠΎ , ΡΠΎΠ·Π΄Π°Π²Π°Π΅ΠΌΡΠ΅ ΠΈΡ ΠΎΠ±ΡΠΈΠΌΠΈ ΠΈΡΡΠΎΡΠ½ΠΈΠΊΠ°ΠΌΠΈ: ΠΏΠ΅ΡΠ΅ΠΌΠ΅Π½Π½ΡΠΌΠΈ Π²ΠΎ Π²ΡΠ΅ΠΌΠ΅Π½ΠΈ ΡΠ»Π΅ΠΊΡΡΠΈΡΠ΅ΡΠΊΠΈΠΌΠΈ Π·Π°ΡΡΠ΄Π°ΠΌΠΈ ΠΈ ΡΠΎΠΊΠ°ΠΌΠΈ (Π²ΡΠ΄Π΅Π»Π΅Π½ΠΎ ΠΌΠ½ΠΎΠΉ).
Π’Π΅ΠΏΠ΅ΡΡ ΠΏΠ΅ΡΠ΅ΠΉΠ΄Π΅ΠΌ ΠΊ ΠΏΠΎΠ»ΡΠΌ self. ΠΠ΅Π³ΠΊΠΎ ΠΏΡΠ΅Π΄ΡΡΠ°Π²ΠΈΡΡ, ΡΡΠΎ ΠΊΠ°ΠΆΠ΄ΡΠΉ Π·Π°ΡΡΠ΄ ΡΠ΅Π°Π³ΠΈΡΡΠ΅Ρ ΡΠΎΠ»ΡΠΊΠΎ Π½Π° ΠΏΠΎΠ»Ρ Π΄ΡΡΠ³ΠΈΡ Π·Π°ΡΡΠ΄ΠΎΠ². ΠΠΎ Π½Π΅ ΡΠ»Π΅Π΄ΡΠ΅Ρ ΡΠ»ΠΈΡΠΊΠΎΠΌ ΡΠΏΡΠΎΡΠ°ΡΡ ΠΏΡΠΈ ΡΠ°ΡΡΠΌΠΎΡΡΠ΅Π½ΠΈΠΈ Π»ΡΡΠ΅Π²ΠΎΠΉ ΡΠ΅Π°ΠΊΡΠΈΠΈ ΠΈ Π΄ΡΡΠ³ΠΈΡ ΠΏΠΎΠ΄ΠΎΠ±Π½ΡΡ ΠΎΡΠ»ΠΎΠΆΠ½Π΅Π½ΠΈΠΉ.
ΠΠ° ΡΠ°ΠΌΠΎΠΌ Π΄Π΅Π»Π΅ Ρ Π²Π°Ρ Π΅ΡΡΡ Π·Π°ΡΡΠ΄Ρ ΠΈ ΠΏΠΎΠ»Ρ. Π Π²Ρ Π΄ΠΎΠ»ΠΆΠ½Ρ ΡΠΊΠ°Π·Π°ΡΡ ΠΎΠ±Π°. Π£ ΠΊΠ°ΠΆΠ΄ΠΎΠ³ΠΎ Π΅ΡΡΡ ΡΠ½Π΅ΡΠ³ΠΈΡ, Ρ ΠΊΠ°ΠΆΠ΄ΠΎΠ³ΠΎ Π΅ΡΡΡ ΠΈΠΌΠΏΡΠ»ΡΡ, ΠΈ ΠΎΠ½ΠΈ ΠΎΠ±ΠΌΠ΅Π½ΠΈΠ²Π°ΡΡΡΡ Π΄ΡΡΠ³ Ρ Π΄ΡΡΠ³ΠΎΠΌ ΡΠ½Π΅ΡΠ³ΠΈΠ΅ΠΉ ΠΈ ΠΈΠΌΠΏΡΠ»ΡΡΠΎΠΌ. ΠΠΎΠ»Ρ β ΡΡΠΎ Π½Π΅ ΠΏΡΠΎΡΡΠΎ ΠΌΠ°ΡΠ΅ΠΌΠ°ΡΠΈΡΠ΅ΡΠΊΠΈΠ΅ Π²ΡΠ΄ΡΠΌΠΊΠΈ Π΄Π»Ρ Π²ΡΡΠΈΡΠ»Π΅Π½ΠΈΡ ΡΠΈΠ» ΠΌΠ΅ΠΆΠ΄Ρ ΡΠ°ΡΡΠΈΡΠ°ΠΌΠΈ, ΡΡΠΎ ΡΠ΅Π°Π»ΡΠ½ΡΠ΅ Π²Π΅ΡΠΈ ΡΠΎ ΡΠ²ΠΎΠΈΠΌΠΈ ΡΡΠ΅ΠΏΠ΅Π½ΡΠΌΠΈ ΡΠ²ΠΎΠ±ΠΎΠ΄Ρ, ΡΠΎΠ±ΡΡΠ²Π΅Π½Π½ΠΎΠΉ ΡΠ½Π΅ΡΠ³ΠΈΠ΅ΠΉ, ΠΈΠΌΠΏΡΠ»ΡΡΠΎΠΌ ΠΈ Π΄Π°ΠΆΠ΅ Π½Π°ΠΏΡΡΠΆΠ΅Π½ΠΈΠ΅ΠΌ.
Π’ΠΎΠ³Π΄Π° ΠΌΠΎΠΆΠ½ΠΎ ΡΠ·Π½Π°ΡΡ ΡΠ½Π΅ΡΠ³ΠΈΡ ΠΈ ΠΈΠΌΠΏΡΠ»ΡΡ ΠΏΠΎΠ»Π΅ΠΉ ΠΈ Π·Π°ΡΡΠ΄ΠΎΠ² ΠΈ ΠΏΠΎΡΠΎΠΌ ΡΠ·Π½Π°ΡΡ, ΠΊΠ°ΠΊ ΠΎΠ½ΠΈ Π²Π·Π°ΠΈΠΌΠ½ΠΎ ΠΎΠ±ΠΌΠ΅Π½ΠΈΠ²Π°ΡΡΡΡ ΡΠ½Π΅ΡΠ³ΠΈΠ΅ΠΉ ΠΈ ΠΈΠΌΠΏΡΠ»ΡΡΠΎΠΌ ΠΌΠ΅ΠΆΠ΄Ρ ΡΠΎΠ±ΠΎΠΉ.
ΠΠΎ, ΠΊΠ°ΠΊ Π²ΡΠ΅ Π³ΠΎΠ²ΠΎΡΠΈΠ»ΠΈ, ΡΠ°ΠΌΠΎΡΠΈΠ»Π° ΠΌΠΎΠΆΠ΅Ρ Π±ΡΡΡ, ΡΠΎΠ»ΡΠΊΠΎ Π΅ΡΠ»ΠΈ Π·Π°ΡΡΠ΄Ρ ΠΈΠ·Π»ΡΡΠ°ΡΡ.
ΠΠ΅ Π²ΡΠ΅ ΡΠ°ΠΊ Π³ΠΎΠ²ΠΎΡΡΡ. Π‘ΠΎΠ±ΡΡΠ²Π΅Π½Π½Π°Ρ ΡΠΈΠ»Π° ΠΈΠΌΠ΅Π΅Ρ Π³ΠΎΡΠ°Π·Π΄ΠΎ Π±ΠΎΠ»Π΅Π΅ ΠΎΠ±ΡΠ΅Π΅ Π·Π½Π°ΡΠ΅Π½ΠΈΠ΅, ΡΠ΅ΠΌ ΡΠΈΠ»Ρ ΡΠ°Π΄ΠΈΠ°ΡΠΈΠΎΠ½Π½ΠΎΠΉ ΡΠ΅Π°ΠΊΡΠΈΠΈ. Π Π½Π°ΠΈΠ±ΠΎΠ»Π΅Π΅ ΡΠ°ΡΠΏΡΠΎΡΡΡΠ°Π½Π΅Π½Π½ΠΎΠΌ ΡΠ»ΡΡΠ°Π΅ Π³Π°ΡΠΌΠΎΠ½ΠΈΡΠ΅ΡΠΊΠΎΠ³ΠΎ Π΄Π²ΠΈΠΆΠ΅Π½ΠΈΡ ΡΠ°ΡΡΠΈΡΠ° ΠΏΠΎΠ»ΡΡΠ°Π΅Ρ ΡΡΠΎΠ»ΡΠΊΠΎ ΠΆΠ΅ ΡΠ½Π΅ΡΠ³ΠΈΠΈ, ΡΠΊΠΎΠ»ΡΠΊΠΎ ΡΠ΅ΡΡΠ΅Ρ ΠΎΡ ΠΈΠ½Π΄ΡΠΊΡΠΈΠ²Π½ΡΡ ΠΏΠΎΠ»Π΅ΠΉ (ΠΏΠΎΠ»Ρ Π¨ΠΎΡΡΠ°, ΠΊΠΎΡΠΎΡΡΠ΅ ΠΎΡΠ»Π°Π±Π΅Π²Π°ΡΡ Π±ΡΡΡΡΠ΅Π΅, ΡΠ΅ΠΌ ΠΏΠΎΠ»Ρ ΠΈΠ·Π»ΡΡΠ΅Π½ΠΈΡ, Π½ΠΎ ΡΠΎΡ ΡΠ°Π½ΡΡΡ ΡΠ½Π΅ΡΠ³ΠΈΡ) ΠΈ ΡΠΎΠ»ΡΠΊΠΎ Π² ΡΡΠ΅Π΄Π½Π΅ΠΌ ΡΠ΅ΡΡΠ΅Ρ ΡΠ½Π΅ΡΠ³ΠΈΡ ΡΠ°Π΄ΠΈΠ°ΡΠΈΠΎΠ½Π½ΡΠΌ ΠΏΠΎΠ»ΡΠΌ.
ΠΠ½ΠΎΠ³ΠΈΠ΅ Π»ΡΠ΄ΠΈ ΠΈΠ·ΡΡΠ°ΡΡ ΡΡΠΎ. Π‘ΡΠΈΠ²Π΅Π½ ΠΠ°ΠΉΠ» Π½Π°ΠΏΠΈΡΠ°Π» ΡΠ΅Π»ΡΡ ΠΊΠ½ΠΈΠ³Ρ ΡΠΎΠ»ΡΠΊΠΎ ΠΎ ΡΠ°Π²Π½ΠΎΠΌΠ΅ΡΠ½ΠΎ ΡΡΠΊΠΎΡΡΡΡΠ΅ΠΉΡΡ Π·Π°ΡΡΠΆΠ΅Π½Π½ΠΎΠΉ ΡΠ°ΡΡΠΈΡΠ΅ ΠΈ ΡΠ΅Π»ΡΡ ΠΊΠ½ΠΈΠ³Ρ ΠΎ Π²Π½ΡΡΡΠ΅Π½Π½ΠΈΡ ΡΠΈΠ»Π°Ρ . ΠΠ΅ΡΠ±Π΅ΡΡ Π¨ΠΏΠΎΠ½ Π½Π°ΠΏΠΈΡΠ°Π» ΡΠ΅Π»ΡΡ ΠΊΠ½ΠΈΠ³Ρ ΠΎ Π²Π·Π°ΠΈΠΌΠ½ΠΎ ΡΠ²ΡΠ·Π°Π½Π½ΠΎΠΉ Π΄ΠΈΠ½Π°ΠΌΠΈΠΊΠ΅ ΡΠ»Π΅ΠΊΡΡΠΎΠΌΠ°Π³Π½ΠΈΡΠ½ΡΡ ΠΏΠΎΠ»Π΅ΠΉ ΠΈ Π·Π°ΡΡΠ΄ΠΎΠ². Π Π€ΡΠΈΡ Π ΠΎΡΠ»ΠΈΡ Π½Π°ΠΏΠΈΡΠ°Π» ΠΊΠ»Π°ΡΡΠΈΡΠ΅ΡΠΊΡΡ ΠΊΠ½ΠΈΠ³Ρ ΠΎ ΠΊΠ»Π°ΡΡΠΈΡΠ΅ΡΠΊΠΈΡ Π·Π°ΡΡΠΆΠ΅Π½Π½ΡΡ ΡΠ°ΡΡΠΈΡΠ°Ρ . Π ΡΡΠΎ Π²ΡΠ΅Π³ΠΎ Π»ΠΈΡΡ ΠΊΠ½ΠΈΠ³ΠΈ, ΠΏΠΎ ΡΡΠΎΠΉ ΡΠ΅ΠΌΠ΅ Π΅ΠΆΠ΅Π³ΠΎΠ΄Π½ΠΎ ΠΏΡΠ±Π»ΠΈΠΊΡΠ΅ΡΡΡ, Π½Π°Π²Π΅ΡΠ½ΠΎΠ΅, Π½Π΅ ΠΌΠ΅Π½Π΅Π΅ Π΄Π΅ΡΡΡΠΊΠ° ΡΡΠ°ΡΠ΅ΠΉ, ΠΏΠΎ ΠΊΡΠ°ΠΉΠ½Π΅ΠΉ ΠΌΠ΅ΡΠ΅, Π·Π° ΠΏΠΎΡΠ»Π΅Π΄Π½ΠΈΠ΅ ΡΡΠΎ Π»Π΅Ρ.
ΠΡΠ°ΠΊ. ΠΡΡΡ ΠΌΠ½ΠΎΠ³ΠΎ ΡΠΎΠ±ΡΡΠ²Π΅Π½Π½ΡΡ ΡΠΈΠ». ΠΠ΅Π±Π°ΡΡ ΠΏΡΠΎΠ΄ΠΎΠ»ΠΆΠ°ΡΡΡΡ ΠΈ ΠΏΠΎ ΡΠ΅ΠΉ Π΄Π΅Π½Ρ ΠΎ ΡΠΎΠΌ, Π²ΡΠ·ΡΠ²Π°ΡΡ Π»ΠΈ ΡΠΎΠ±ΡΡΠ²Π΅Π½Π½ΡΠ΅ ΡΠΈΠ»Ρ ΠΈΠ·ΠΌΠ΅Π½Π΅Π½ΠΈΠ΅ ΡΠΈΠ», ΡΡΠ²ΠΎΠΊ (ΠΈΠ·ΠΌΠ΅Π½Π΅Π½ΠΈΠ΅ ΡΡΠΊΠΎΡΠ΅Π½ΠΈΡ) ΠΈΠ»ΠΈ ΡΠ°ΠΌΠΎ ΡΡΠΊΠΎΡΠ΅Π½ΠΈΠ΅. ΠΠ»ΠΈ ΡΡΠΎ ΡΡΠΎ-ΡΠΎ ΡΠΎΠ²ΡΠ΅ΠΌ Π΄ΡΡΠ³ΠΎΠ΅.
ΠΠΌΠ΅ΠΉΡΠ΅ Π² Π²ΠΈΠ΄Ρ, ΡΡΠΎ Π² ΡΡΠ΅Π±Π½ΠΈΠΊΠ΅ ΠΎ ΠΏΡΠΎΠ²ΠΎΠ΄Π΅ ΠΊΠ»Π°ΡΡΠΈΡΠ΅ΡΠΊΠΎΠ³ΠΎ ΡΠ°Π·ΠΌΠ΅ΡΠ° ΠΎΠ±ΡΡΠ½ΠΎ ΠΎΠ±ΡΡΠΆΠ΄Π°ΡΡΡΡ ΠΌΠ°ΠΊΡΠΎΡΠΊΠΎΠΏΠΈΡΠ΅ΡΠΊΠΈΠ΅ ΠΏΠΎΠ»Ρ, Π³Π΄Π΅ Π²Ρ ΡΡΡΠ΅Π΄Π½ΡΠ΅ΡΠ΅ Π΄ΠΎΡΡΠ°ΡΠΎΡΠ½ΠΎ Π±ΠΎΠ»ΡΡΠΎΠ΅ ΠΊΠΎΠ»ΠΈΡΠ΅ΡΡΠ²ΠΎ Π°ΡΠΎΠΌΠΎΠ², ΡΡΠΎΠ±Ρ ΠΏΠΎΠ»Ρ, Π·Π°ΡΡΠ΄ ΠΈ ΡΠΎΠΊ ΡΡΠ°Π»ΠΈ Π±ΠΎΠ»Π΅Π΅ Π³Π»Π°Π΄ΠΊΠΈΠΌΠΈ ΠΏΠΎΠ»ΡΠΌΠΈ, ΠΊΠΎΡΠΎΡΡΠ΅ Π½Π΅ ΠΏΡΡΠ³Π°ΡΡ ΠΈΠ·-Π·Π° ΡΠ΅ΠΏΠ»ΠΎΠ²ΡΡ ΡΡΡΠ΅ΠΊΡΠΎΠ². ΠΈΠ»ΠΈ Π² Π·Π°Π²ΠΈΡΠΈΠΌΠΎΡΡΠΈ ΠΎΡ ΡΠΎΠ³ΠΎ, Π³Π΄Π΅ Π²Ρ Π½Π°Ρ ΠΎΠ΄ΠΈΡΠ΅ΡΡ Π² ΡΠ΅ΡΠ΅ΡΠΊΠ΅ ΡΠ²Π΅ΡΠ΄ΠΎΠ³ΠΎ ΡΠ΅Π»Π°. Π ΠΊΠΎΠ³Π΄Π° ΠΊΠΎΠ³ΠΎ-ΡΠΎ Π²ΠΎΠ»Π½ΡΡΡ ΡΡΠΈ ΡΡΡΠ΅ΠΊΡΡ, ΠΎΠ½ΠΈ ΠΈΡΠΏΠΎΠ»ΡΠ·ΡΡΡ ΡΡΠ°ΡΠΈΡΡΠΈΡΠ΅ΡΠΊΡΡ ΡΠΈΠ·ΠΈΠΊΡ ΠΈ/ΠΈΠ»ΠΈ ΠΊΠ²Π°Π½ΡΠΎΠ²ΡΡ ΡΠΈΠ·ΠΈΠΊΡ. ΠΠ΅ ΡΠΎΠ»ΡΠΊΠΎ ΡΠΈΡΡΡΠΉ ΠΠ°ΠΊΡΠ²Π΅Π»Π».
$\endgroup$
28
$\begingroup$
ΠΠ°Π΄ΡΠΌΡΠ²Π°Π»ΠΈΡΡ Π»ΠΈ Π²Ρ ΠΊΠΎΠ³Π΄Π°-Π½ΠΈΠ±ΡΠ΄Ρ ΡΠ°ΠΌΠΈ, ΡΡΠΎ ΠΌΠ°Π³Π½ΠΈΡΠ½ΡΠ΅ Π΄ΠΈΠΏΠΎΠ»ΡΠ½ΡΠ΅ ΠΌΠΎΠΌΠ΅Π½ΡΡ ΡΠ²Π»ΡΡΡΡΡ ΠΏΡΠΈΡΠΈΠ½ΠΎΠΉ Π»ΡΠ±ΠΎΠ³ΠΎ ΠΌΠ°Π³Π½ΠΈΡΠ½ΠΎΠ³ΠΎ ΠΏΠΎΠ»Ρ? Π΅ΡΠ»ΠΈ ΠΎΠ½ΠΈ Π²ΡΡΠΎΠ²Π½Π΅Π½Ρ, ΠΎΠ½ΠΈ ΠΎΠ±ΡΠ°Π·ΡΡΡ Π²Π½Π΅ΡΠ½Π΅Π΅ ΠΏΠΎΠ»Π΅. ΠΡΡΠ°Π²Π½ΠΈΠ²Π°Π½ΠΈΠ΅ ΠΏΡΠΎΠΈΡΡ ΠΎΠ΄ΠΈΡ Π·Π° ΡΡΠ΅Ρ ΡΡΠΊΠΎΡΠ΅Π½ΠΈΡ, ΠΏΠΎΡΠΎΠΌΡ ΡΡΠΎ ΠΌΠ°Π³Π½ΠΈΡΠ½ΡΠΉ Π΄ΠΈΠΏΠΎΠ»ΡΠ½ΡΠΉ ΠΌΠΎΠΌΠ΅Π½Ρ ΡΠ²ΡΠ·Π°Π½ Ρ ΡΠΎΠ±ΡΡΠ²Π΅Π½Π½ΡΠΌ ΡΠΏΠΈΠ½ΠΎΠΌ (Π½Π° ΡΠ°ΠΌΠΎΠΌ Π΄Π΅Π»Π΅ ΡΡΠΎ Π²ΡΠ°ΡΠ΅Π½ΠΈΠ΅ ΡΠΎΠ³Π»Π°ΡΠ½ΠΎ ΡΠ΅Π°Π»ΡΠ½ΠΎ ΠΏΡΠΎΠ²Π΅Π΄Π΅Π½Π½ΠΎΠΌΡ ΡΠΊΡΠΏΠ΅ΡΠΈΠΌΠ΅Π½ΡΡ ΠΠΉΠ½ΡΡΠ΅ΠΉΠ½Π° ΠΈ Π΄Π΅ Π₯Π°Π°ΡΠ°). Π ΡΡΠΎ ΡΡΠ²Π΅ΡΠΆΠ΄Π΅Π½ΠΈΠ΅ ΠΎ Π΄Π²ΠΈΠΆΡΡΠΈΡ ΡΡ Π·Π°ΡΡΠ΄Π°Ρ ΡΠ²Π»ΡΠ΅ΡΡΡ Π½Π΅ΠΏΡΠ°Π²ΠΈΠ»ΡΠ½ΡΠΌ ΡΠΎΠ»ΠΊΠΎΠ²Π°Π½ΠΈΠ΅ΠΌ, Π·Π°ΡΡΠ΄Ρ Π΄ΠΎΠ»ΠΆΠ½Ρ Π±ΡΡΡ ΡΡΠΊΠΎΡΠ΅Π½Ρ (ΠΊΠ°ΠΊ Π² ΠΊΠ°ΡΡΡΠΊΠ΅). ΠΠΊΡΠΏΠ΅ΡΠΈΠΌΠ΅Π½Ρ Ρ ΠΌΠ°Π³Π½ΠΈΡΠ½ΡΠΌ ΠΏΠΎΠ»Π΅ΠΌ ΠΌΠ΅ΠΆΠ΄Ρ Π΄Π²ΡΠΌΡ ΠΏΡΠΎΠ²ΠΎΠ΄Π°ΠΌΠΈ ΠΏΡΠΎΠ²ΠΎΠ΄ΠΈΠ»ΡΡ ΡΠ½Π°ΡΠ°Π»Π° ΠΏΡΠΈ Π²ΠΊΠ»ΡΡΠ΅Π½ΠΈΠΈ ΡΠΎΠΊΠΎΠ² (ΡΠ°Π·Π³ΠΎΠ½ Π·Π°ΡΡΠ΄ΠΎΠ²).
$\endgroup$
4
$\begingroup$
ΠΡΡΡΡ ΡΠ³ΡΡΡΠΎΠΊ Π·Π°ΡΡΠ΄ΠΎΠ² Π΄Π²ΠΈΠΆΠ΅ΡΡΡ Ρ ΠΏΠΎΡΡΠΎΡΠ½Π½ΠΎΠΉ ΡΠΊΠΎΡΠΎΡΡΡΡ $\mathbf v\;.$ ΠΠΎΡΠΊΠΎΠ»ΡΠΊΡ Π·Π°ΡΡΠ΄Ρ Π΄Π²ΠΈΠΆΡΡΡΡ, ΠΎΠ½ΠΈ Π±ΡΠ΄ΡΡ ΡΠΎΠ·Π΄Π°Π²Π°ΡΡ ΠΌΠ°Π³Π½ΠΈΡΠ½ΠΎΠ΅ ΠΏΠΎΠ»Π΅ $\bf B$, ΡΠ°ΠΊ ΠΊΠ°ΠΊ ΠΈΠΌΠ΅Π½Π½ΠΎ ΡΠΎΠΊ ΡΠΎΠ·Π΄Π°Π΅Ρ ΠΌΠ°Π³Π½ΠΈΡΠ½ΠΎΠ΅ ΠΏΠΎΠ»Π΅.
ΠΡ Π²ΡΠ΅ Π·Π½Π°ΠΊΠΎΠΌΡ Ρ ΡΠΎΠΊΠΎΠΌ Π² ΠΏΡΠΎΠ²ΠΎΠ΄Π΅, ΠΊΠ°ΠΊ ΠΏΠΎΠΊΠ°Π·Π°Π½ΠΎ Π½Π° ΡΠΈΡΡΠ½ΠΊΠ΅ Π½ΠΈΠΆΠ΅, Π½ΠΎ Π΄Π²ΠΈΠΆΡΡΠΈΠ΅ΡΡ Π·Π°ΡΡΠ΄Ρ Π½Π° ΡΠ°ΠΌΠΎΠΌ Π΄Π΅Π»Π΅ Π½Π΅ Β«ΡΠΎΠ·Π΄Π°ΡΡΒ» ΠΌΠ°Π³Π½ΠΈΡΠ½ΠΎΠ΅ ΠΏΠΎΠ»Π΅. Π‘ΠΌ. Π½Π΅ΠΌΠ½ΠΎΠ³ΠΎ ΠΎΠ± ΡΡΠΎΠΌ Ρ ΠΡΠΈΠΌΠ΅Π½ΠΊΠΎ: «. ..Π½ΠΈ ΡΡΠ°Π²Π½Π΅Π½ΠΈΡ ΠΠ°ΠΊΡΠ²Π΅Π»Π»Π°, Π½ΠΈ ΠΈΡ ΡΠ΅ΡΠ΅Π½ΠΈΡ Π½Π΅ ΡΠΊΠ°Π·ΡΠ²Π°ΡΡ Π½Π° Π½Π°Π»ΠΈΡΠΈΠ΅ ΠΏΡΠΈΡΠΈΠ½Π½ΠΎ-ΡΠ»Π΅Π΄ΡΡΠ²Π΅Π½Π½ΠΎΠΉ ΡΠ²ΡΠ·ΠΈ ΠΌΠ΅ΠΆΠ΄Ρ ΡΠ»Π΅ΠΊΡΡΠΈΡΠ΅ΡΠΊΠΈΠΌ ΠΈ ΠΌΠ°Π³Π½ΠΈΡΠ½ΡΠΌ ΠΏΠΎΠ»ΡΠΌΠΈ. Π‘Π»Π΅Π΄ΠΎΠ²Π°ΡΠ΅Π»ΡΠ½ΠΎ, ΠΌΡ Π΄ΠΎΠ»ΠΆΠ½Ρ Π·Π°ΠΊΠ»ΡΡΠΈΡΡ, ΡΡΠΎ ΡΠ»Π΅ΠΊΡΡΠΎΠΌΠ°Π³Π½ΠΈΡΠ½ΠΎΠ΅ ΠΏΠΎΠ»Π΅ ΡΠ²Π»ΡΠ΅ΡΡΡ Π΄Π²ΠΎΠΉΡΡΠ²Π΅Π½Π½ΠΎΠΉ ΡΡΡΠ½ΠΎΡΡΡΡ, Π²ΡΠ΅Π³Π΄Π° ΠΈΠΌΠ΅ΡΡΠ΅ΠΉ ΡΠ»Π΅ΠΊΡΡΠΈΡΠ΅ΡΠΊΡΡ ΠΈ ΠΌΠ°Π³Π½ΠΈΡΠ½ΡΡ ΡΠΎΡΡΠ°Π²Π»ΡΡΡΠΈΠ΅ ΠΎΠ΄Π½ΠΎΠ²ΡΠ΅ΠΌΠ΅Π½Π½ΠΎ…»
ΠΡ ΠΌΠΎΠΆΠ΅ΡΠ΅ ΠΏΠΎΠ½ΡΡΡ ΡΡΠΎ, ΠΏΡΠ΅Π΄ΡΡΠ°Π²ΠΈΠ², ΡΡΠΎ Π²Ρ ΠΏΠΎΠ·ΠΈΡΡΠΎΠ½, ΠΈ Ρ ΡΠ°ΠΆΠ°Ρ Π²Π°Ρ ΠΏΠ΅ΡΠ΅Π΄ ΠΏΡΠΈΠ±ΠΈΡΡΠΌ ΠΊ Π·Π΅ΠΌΠ»Π΅ ΡΠ»Π΅ΠΊΡΡΠΎΠ½ΠΎΠΌ. ΠΡΠΎΡ ΡΠ»Π΅ΠΊΡΡΠΎΠ½ ΠΈΠΌΠ΅Π΅Ρ ΡΠ»Π΅ΠΊΡΡΠΎΠΌΠ°Π³Π½ΠΈΡΠ½ΠΎΠ΅ ΠΏΠΎΠ»Π΅. Π£ Π½Π΅Π³ΠΎ Π½Π΅Ρ ΡΠ»Π΅ΠΊΡΡΠΈΡΠ΅ΡΠΊΠΎΠ³ΠΎ ΠΏΠΎΠ»Ρ ΠΈΠ»ΠΈ ΠΌΠ°Π³Π½ΠΈΡΠ½ΠΎΠ³ΠΎ ΠΏΠΎΠ»Ρ, Ρ Π½Π΅Π³ΠΎ Π΅ΡΡΡ ΡΠ»Π΅ΠΊΡΡΠΎΠΌΠ°Π³Π½ΠΈΡΠ½ΠΎΠ΅ ΠΏΠΎΠ»Π΅ . ΠΠ°ΠΊ ΠΈ ΠΏΠΎΠ·ΠΈΡΡΠΎΠ½. ΠΠ·-Π·Π° ΡΡΠΎΠ³ΠΎ, ΠΊΠΎΠ³Π΄Π° Ρ ΠΎΡΠΏΡΡΠΊΠ°Ρ ΡΠ΅Π±Ρ, ΡΡ ΠΏΡΡΠΌΠΎΠ»ΠΈΠ½Π΅ΠΉΠ½ΠΎ Π΄Π²ΠΈΠΆΠ΅ΡΡΡΡ ΠΊ ΡΠ»Π΅ΠΊΡΡΠΎΠ½Ρ. ΠΠ°ΡΠ΅ΠΌ, ΠΏΠΎΡΠΊΠΎΠ»ΡΠΊΡ Π²Ρ Π²ΠΈΠ΄ΠΈΡΠ΅ ΡΠΎΠ»ΡΠΊΠΎ Π»ΠΈΠ½Π΅ΠΉΠ½ΠΎΠ΅ Π΄Π²ΠΈΠΆΠ΅Π½ΠΈΠ΅, Π²Ρ Π΄ΡΠΌΠ°Π΅ΡΠ΅ ΠΎ ΡΠ°Π΄ΠΈΠ°Π»ΡΠ½ΠΎΠΌ ΡΠ»Π΅ΠΊΡΡΠΈΡΠ΅ΡΠΊΠΎΠΌ ΠΏΠΎΠ»Π΅ ΡΠΎΠ³Π»Π°ΡΠ½ΠΎ ΠΊΡΡΡΡ ΠΠ½Π΄ΡΡ ΠΠ°ΡΡΠΈ ΠΏΠΎ ΡΠΈΠ·ΠΈΠΊΠ΅ 106:
ΠΠΎ ΡΡΠΎ ΠΏΡΠΎΠΈΠ·ΠΎΠΉΠ΄Π΅Ρ, Π΅ΡΠ»ΠΈ Ρ Π±ΡΠΎΡΡ Π²Π°Ρ ΠΌΠΈΠΌΠΎ ΡΠ»Π΅ΠΊΡΡΠΎΠ½Π°? ΠΡ ΠΏΠΎ-ΠΏΡΠ΅ΠΆΠ½Π΅ΠΌΡ Π΄Π²ΠΈΠΆΠ΅ΡΠ΅ΡΡ ΠΊ Π½Π΅ΠΌΡ, Π½ΠΎ Π²Ρ ΡΠ°ΠΊΠΆΠ΅ ΠΈΡΠΏΡΡΡΠ²Π°Π΅ΡΠ΅ Π²ΡΠ°ΡΠ°ΡΠ΅Π»ΡΠ½ΠΎΠ΅ Π΄Π²ΠΈΠΆΠ΅Π½ΠΈΠ΅, ΠΊΠΎΡΠΎΡΠΎΠ΅ Π²Ρ ΠΏΡΠΈΠΏΠΈΡΡΠ²Π°Π΅ΡΠ΅ ΠΌΠ°Π³Π½ΠΈΡΠ½ΠΎΠΌΡ ΠΏΠΎΠ»Ρ ΡΠ»Π΅ΠΊΡΡΠΎΠ½Π°. ΠΠΎ Π·Π°ΠΌΠ΅ΡΡΡΠ΅, ΡΡΠΎ Π²Ρ ΡΠΎΠ·Π΄Π°Π»ΠΈ ΠΌΠ°Π³Π½ΠΈΡΠ½ΠΎΠ΅ ΠΏΠΎΠ»Π΅ ΡΠ»Π΅ΠΊΡΡΠΎΠ½Π° Π½Π΅ ΡΠΎΠ»ΡΠΊΠΎ ΠΏΠΎΡΠΎΠΌΡ, ΡΡΠΎ Π²Ρ ΠΏΠ΅ΡΠ΅Π΅Ρ Π°Π»ΠΈ. 2} \;;$$, ΡΠ°ΠΊ ΠΊΠ°ΠΊ $d\mathbf l$ & $\hat{\ mathbf r}$ Π½Π°ΠΏΡΠ°Π²Π»Π΅Π½Ρ Π² ΠΎΠ΄Π½ΠΎΠΌ Π½Π°ΠΏΡΠ°Π²Π»Π΅Π½ΠΈΠΈ, ΡΠΈΡΠ»ΠΈΡΠ΅Π»Ρ ΡΠ°Π²Π΅Π½ Π½ΡΠ»Ρ, Π° Π·Π½Π°ΠΌΠ΅Π½Π°ΡΠ΅Π»Ρ $r$ ΡΠ°Π²Π΅Π½ Π½ΡΠ»Ρ, ΡΡΠΎ ΡΠ½ΠΎΠ²Π° Π΄Π΅Π»Π°Π΅Ρ ΡΠΈΠ»Ρ Π±Π΅ΡΠΊΠΎΠ½Π΅ΡΠ½ΠΎΠΉ. ΠΡΠΎ Π½Π΅Π²ΠΎΠ·ΠΌΠΎΠΆΠ½ΠΎ.
Π‘ΠΎΠ³Π»Π°ΡΠ΅Π½. Π’ΠΎΡΠ΅ΡΠ½ΡΠ΅ ΡΠ°ΡΡΠΈΡΡ ΡΠΏΡΠΎΡΠ°ΡΡ ΠΌΠ°ΡΠ΅ΠΌΠ°ΡΠΈΠΊΡ, Π½ΠΎ ΡΡΠΎ Π²ΡΠ΅Π³ΠΎ Π»ΠΈΡΡ ΠΏΡΠΈΠ±Π»ΠΈΠΆΠ΅Π½ΠΈΠ΅, Π° ΡΠΏΡΠΎΡΠ΅Π½ΠΈΠ΅ ΠΌΠ°ΡΠ΅ΠΌΠ°ΡΠΈΠΊΠΈ Π½Π΅ Π΄ΠΎΠ»ΠΆΠ½ΠΎ ΡΠΊΡΡΠ²Π°ΡΡ ΠΎΡ Π»ΡΠ΄Π΅ΠΉ ΡΠ²Π΅ΡΠ΄ΡΠ΅ Π½Π°ΡΡΠ½ΡΠ΅ Π΄ΠΎΠΊΠ°Π·Π°ΡΠ΅Π»ΡΡΡΠ²Π° ΡΠ°ΠΊΠΈΡ Π²Π΅ΡΠ΅ΠΉ, ΠΊΠ°ΠΊ Π΄ΠΈΡΡΠ°ΠΊΡΠΈΡ ΡΠ»Π΅ΠΊΡΡΠΎΠ½ΠΎΠ² ΠΈ ΡΡΡΠ΅ΠΊΡ ΠΠΉΠ½ΡΡΠ΅ΠΉΠ½Π°-Π΄Π΅-ΠΠ°Π°Π·Π°. ΠΠΎ ΡΡΠΎ ΡΠ°ΠΊ.
ΠΡΠ²ΠΎΠ΄: ΠΠ° Π·Π°ΡΡΠ΄Ρ Π½Π΅ ΠΌΠΎΠΆΠ΅Ρ Π²Π»ΠΈΡΡΡ ΠΈΡ ΡΠΎΠ±ΡΡΠ²Π΅Π½Π½ΠΎΠ΅ ΡΠ»Π΅ΠΊΡΡΠΈΡΠ΅ΡΠΊΠΎΠ΅ ΠΏΠΎΠ»Π΅ ΠΈ ΠΌΠ°Π³Π½ΠΈΡΠ½ΠΎΠ΅ ΠΏΠΎΠ»Π΅.
Π‘ΠΎΠ³Π»Π°ΡΠ΅Π½.
ΠΠΎ ΡΠΎΠ³Π΄Π° ΠΊΠ°ΠΊ ΠΌΠΎΠΆΠ΅Ρ Π±ΡΡΡ ΡΠ°ΠΌΠΎΠΈΠ½Π΄ΡΠΊΡΠΈΡ , Π΅ΡΠ»ΠΈ Π½Π° Π΄Π²ΠΈΠΆΡΡΠΈΠ΅ΡΡ Π·Π°ΡΡΠ΄Ρ Π½Π΅ Π΄Π΅ΠΉΡΡΠ²ΡΡΡ ΠΈΡ ΡΠΎΠ±ΡΡΠ²Π΅Π½Π½ΡΠ΅ ΠΌΠ°Π³Π½ΠΈΡΠ½ΡΠ΅ ΠΏΠΎΠ»Ρ?
Π‘Π°ΠΌΠΎΠΈΠ½Π΄ΡΠΊΡΠΈΡ — ΡΡΠΎ «ΠΈΠ½Π΄ΡΠΊΡΠΈΡ Π½Π°ΠΏΡΡΠΆΠ΅Π½ΠΈΡ Π² ΠΏΡΠΎΠ²ΠΎΠ΄Π΅ Ρ ΡΠΎΠΊΠΎΠΌ ΠΏΡΠΈ ΠΈΠ·ΠΌΠ΅Π½Π΅Π½ΠΈΠΈ ΡΠΎΠΊΠ° Π² ΡΠ°ΠΌΠΎΠΌ ΠΏΡΠΎΠ²ΠΎΠ΄Π΅» . Π ΡΡΠΎΡ ΡΠΎΠΊ ΠΌΠ΅Π½ΡΠ΅ΡΡΡ, ΠΏΠΎΡΠΎΠΌΡ ΡΡΠΎ ΠΏΡΠΎΠΈΡΡ ΠΎΠ΄ΠΈΡ ΠΊΠ°ΠΊΠΎΠ΅-ΡΠΎ Π²Π·Π°ΠΈΠΌΠΎΠ΄Π΅ΠΉΡΡΠ²ΠΈΠ΅ Ρ Π΄ΡΡΠ³ΠΈΠΌΠΈ Π·Π°ΡΡΠΆΠ΅Π½Π½ΡΠΌΠΈ ΡΠ°ΡΡΠΈΡΠ°ΠΌΠΈ. ΠΠ°ΠΊ ΡΠ΅Π»Π°Ρ ΠΊΡΡΠ° Π·Π°ΡΡΠΆΠ΅Π½Π½ΡΡ ΡΠ°ΡΡΠΈΡ Π² ΠΎΠ±Π»ΠΈΠΊΠ΅ ΠΌΠΎΠ΅ΠΉ ΡΡΠΊΠΈ, ΡΠ΅Π»ΠΊΠ°ΡΡΠ΅ΠΉ Π²ΡΠΊΠ»ΡΡΠ°ΡΠ΅Π»Π΅ΠΌ.
ΠΠΠ‘ ΡΠ°ΠΌΠΎΠΈΠ½Π΄ΡΠΊΡΠΈΠΈ, Π²ΡΠ·Π²Π°Π½Π½Π°Ρ ΠΈΠ·ΠΌΠ΅Π½Π΅Π½ΠΈΠ΅ΠΌ ΠΏΠΎΡΠΎΠΊΠ° ΠΌΠ°Π³Π½ΠΈΡΠ½ΠΎΠ³ΠΎ ΠΏΠΎΠ»Ρ, ΡΠΎΠ·Π΄Π°Π²Π°Π΅ΠΌΠΎΠ³ΠΎ Π·Π°ΡΡΠ΄Π°ΠΌΠΈ, Π΄Π΅ΠΉΡΡΠ²ΡΠ΅Ρ Π½Π° Π·Π°ΡΡΠ΄Ρ; ΠΊΠ°ΠΆΠ΄ΡΠΉ Π·Π°ΡΡΠ΄ ΠΈΡΠΏΡΡΡΠ²Π°Π΅Ρ ΡΠΈΠ»Ρ $\mathbf F= q\; mathbf v\times \mathbf B $, Π³Π΄Π΅ $\mathbf B$ ΡΠΎΠ·Π΄Π°Π΅ΡΡΡ ΠΈΠΌΠΈ ΡΠ°ΠΌΠΈΠΌΠΈ. ΠΡΠ°ΠΊ, ΡΡΠΎ ΠΎΠ·Π½Π°ΡΠ°Π΅Ρ, ΡΡΠΎ Π·Π°ΡΡΠ΄Ρ Π΄Π²ΠΈΠΆΡΡΡΡ ΠΈ ΡΠΎΠ·Π΄Π°ΡΡ ΠΌΠ°Π³Π½ΠΈΡΠ½ΠΎΠ΅ ΠΏΠΎΠ»Π΅, ΠΊΠΎΡΠΎΡΠΎΠ΅ ΡΠ½ΠΎΠ²Π° Π΄Π΅ΠΉΡΡΠ²ΡΠ΅Ρ Π½Π° Π·Π°ΡΡΠ΄Ρ, ΡΠΎΠ·Π΄Π°Π²ΡΠΈΠ΅ ΡΡΠΎ ΠΏΠΎΠ»Π΅.
ΠΡΠΎΡΡΠΎ ΡΠΏΡΠΎΡΡΠΈΡΠ΅ ΡΡΠΎΡ ΡΠΎΠΊ Π΄ΠΎ ΠΎΠ΄Π½ΠΎΠ³ΠΎ ΡΠ»Π΅ΠΊΡΡΠΎΠ½Π°, Π·Π°ΡΠ΅ΠΌ ΠΏΡΠ΅Π΄ΡΡΠ°Π²ΡΡΠ΅, ΡΡΠΎ ΠΎΠ½ Π½Π΅ Π΄Π²ΠΈΠΆΠ΅ΡΡΡ, Π° Π²ΠΌΠ΅ΡΡΠΎ ΡΡΠΎΠ³ΠΎ Π΄Π²ΠΈΠ³Π°Π΅ΡΠ΅ΡΡ Π²Ρ. ΠΠΈΠΊΡΠΎ Π½Π΅ ΡΠΎΠ·Π΄Π°Π²Π°Π» ΠΌΠ°Π³Π½ΠΈΡΠ½ΠΎΠ΅ ΠΏΠΎΠ»Π΅ Π΄Π»Ρ ΡΠ»Π΅ΠΊΡΡΠΎΠ½Π° ΡΠΎΠ»ΡΠΊΠΎ ΠΏΠΎΡΠΎΠΌΡ, ΡΡΠΎ ΠΎΠ½ΠΈ Π΄Π²ΠΈΠ³Π°Π»ΠΈΡΡ. ΠΠΎΡΠΎΠΌΡ ΡΡΠΎ Ρ Π½Π΅Π³ΠΎ ΡΠ»Π΅ΠΊΡΡΠΎΠΌΠ°Π³Π½ΠΈΡΠ½ΠΎΠ΅ ΠΏΠΎΠ»Π΅ . Π Π²Π·Π°ΠΈΠΌΠΎΠ΄Π΅ΠΉΡΡΠ²ΠΈΡ ΡΠ»Π΅ΠΊΡΡΠΎΠΌΠ°Π³Π½ΠΈΡΠ½ΠΎΠ³ΠΎ ΠΏΠΎΠ»Ρ ΠΏΡΠΈΠ²ΠΎΠ΄ΡΡ ΠΊ Π»ΠΈΠ½Π΅ΠΉΠ½ΠΎΠΌΡ ΠΈ/ΠΈΠ»ΠΈ Π²ΡΠ°ΡΠ°ΡΠ΅Π»ΡΠ½ΠΎΠΌΡ Π΄Π²ΠΈΠΆΠ΅Π½ΠΈΡ. ΠΠΎΠ³Π΄Π° ΠΌΡ Π²ΠΈΠ΄ΠΈΠΌ ΡΠΎΠ»ΡΠΊΠΎ ΠΏΠ΅ΡΠ²ΠΎΠ΅, ΠΌΡ Π½Π°Π·ΡΠ²Π°Π΅ΠΌ Π΅Π³ΠΎ ΡΠ»Π΅ΠΊΡΡΠΈΡΠ΅ΡΠΊΠΈΠΌ ΠΏΠΎΠ»Π΅ΠΌ, ΠΊΠΎΠ³Π΄Π° ΠΌΡ Π²ΠΈΠ΄ΠΈΠΌ ΡΠΎΠ»ΡΠΊΠΎ Π²ΡΠΎΡΠΎΠ΅, ΠΌΡ Π½Π°Π·ΡΠ²Π°Π΅ΠΌ Π΅Π³ΠΎ ΠΌΠ°Π³Π½ΠΈΡΠ½ΡΠΌ ΠΏΠΎΠ»Π΅ΠΌ, Π½ΠΎ ΡΠ°ΡΡΠΌΠ°ΡΡΠΈΠ²Π°Π΅ΠΌΡΠ΅ ΠΏΠΎΠ»Ρ β ΡΡΠΎ ΡΠ»Π΅ΠΊΡΡΠΎΠΌΠ°Π³Π½ΠΈΡΠ½ΡΠ΅ ΠΏΠΎΠ»Ρ. Π ΡΠ°ΠΌΡΠΉ ΠΏΡΠΎΡΡΠΎΠΉ ΠΏΡΠΈΠΌΠ΅Ρ Π²Π·Π°ΠΈΠΌΠΎΠ΄Π΅ΠΉΡΡΠ²ΠΈΡ ΡΠ»Π΅ΠΊΡΡΠΎΠΌΠ°Π³Π½ΠΈΡΠ½ΠΎΠ³ΠΎ ΠΏΠΎΠ»Ρ ΡΠ²ΡΠ·Π°Π½ Ρ ΠΏΠΎΠ·ΠΈΡΡΠΎΠ½ΠΈΠ΅ΠΌ. ΠΡΡΡ Π»ΠΈΠ½Π΅ΠΉΠ½ΠΎΠ΅ Π΄Π²ΠΈΠΆΠ΅Π½ΠΈΠ΅, ΠΊΠΎΠ³Π΄Π° ΡΠ»Π΅ΠΊΡΡΠΎΠ½ ΠΈ ΠΏΠΎΠ·ΠΈΡΡΠΎΠ½ Π΄Π²ΠΈΠΆΡΡΡΡ Π½Π°Π²ΡΡΡΠ΅ΡΡ Π΄ΡΡΠ³ Π΄ΡΡΠ³Ρ ΠΈ Π°Π½Π½ΠΈΠ³ΠΈΠ»ΠΈΡΡΡΡ, Π° ΡΠ°ΠΊΠΆΠ΅ Π΅ΡΡΡ Π²ΡΠ°ΡΠ°ΡΠ΅Π»ΡΠ½ΠΎΠ΅ Π΄Π²ΠΈΠΆΠ΅Π½ΠΈΠ΅:
ΠΠ·ΠΎΠ±ΡΠ°ΠΆΠ΅Π½ΠΈΠ΅ CCASA ΠΎΡ Manticorp/Rubber Duck, ΡΠΌ. Wikipedia
ΠΠΎ ΠΊΠ°ΠΊ ΠΆΠ΅ ΡΠ°ΠΊ? ΠΡΠΎ Π΄Π°Π»ΠΎ Π±Ρ Π±Π΅ΡΠΊΠΎΠ½Π΅ΡΠ½ΡΡ ΡΠΈΠ»Ρ (Π΄Π°ΠΆΠ΅ Π΅ΡΠ»ΠΈ Π±Ρ ΠΎΠ½Π° Π½Π΅ Π±ΡΠ»Π° Π±Π΅ΡΠΊΠΎΠ½Π΅ΡΠ½ΠΎΠΉ, ΠΎΠ½Π° Π΄ΠΎΠ»ΠΆΠ½Π° Π±ΡΠ»Π° Π±Ρ Π±ΡΡΡ ΡΠ°Π²Π½Π° Π½ΡΠ»Ρ, ΠΏΠΎΡΠΊΠΎΠ»ΡΠΊΡ $d\mathbf l$ ΠΈ $\hat{\mathbf r}$ ΠΏΠ°ΡΠ°Π»Π»Π΅Π»ΡΠ½Ρ).
ΠΠ΅ΡΠΊΠΎΠ½Π΅ΡΠ½ΡΡ ΡΠΈΠ» Π½Π΅ ΡΡΡΠ΅ΡΡΠ²ΡΠ΅Ρ, ΡΠ»Π΅ΠΊΡΡΠΎΠ½ ΠΈΠΌΠ΅Π΅Ρ ΡΠ²ΠΎΡ ΠΊΠΎΠΌΠΏΡΠΎΠ½ΠΎΠ²ΡΠΊΡΡ Π΄Π»ΠΈΠ½Ρ Π²ΠΎΠ»Π½Ρ ΠΈ ΠΈΠΌΠ΅Π΅Ρ «ΡΠΏΠΈΠ½ΠΎΡΠ½ΡΡ» ΠΏΡΠΈΡΠΎΠ΄Ρ.
ΠΡΠ°ΠΊ, Π²ΠΎΠΏΡΠΎΡ Π½Π° ΠΌΠΈΠ»Π»ΠΈΠΎΠ½ Π΄ΠΎΠ»Π»Π°ΡΠΎΠ²: Π²Π»ΠΈΡΠ΅Ρ Π»ΠΈ Π½Π° Π΄Π²ΠΈΠΆΡΡΠΈΠ΅ΡΡ Π·Π°ΡΡΠ΄Ρ ΠΌΠ°Π³Π½ΠΈΡΠ½ΠΎΠ΅ ΠΏΠΎΠ»Π΅, ΠΊΠΎΡΠΎΡΠΎΠ΅ ΠΎΠ½ΠΈ ΡΠΎΠ·Π΄Π°ΡΡ ΠΏΡΠΈ Π΄Π²ΠΈΠΆΠ΅Π½ΠΈΠΈ, ΡΠΎΠ·Π΄Π°Π²Π°Ρ ΡΠΎΠΊ? ΠΠ°ΠΊ ΠΌΠΎΠΆΠ΅Ρ Π±ΡΡΡ, Π΅ΡΠ»ΠΈ ΡΠΎΡΠΌΡΠ»Π° Π³ΠΎΠ²ΠΎΡΠΈΡ ΠΎΠ± ΠΎΠ±ΡΠ°ΡΠ½ΠΎΠΌ?
ΠΠ΅Ρ, ΠΏΠΎΡΠΎΠΌΡ ΡΡΠΎ Π²Π°ΡΠ΅ Π΄Π²ΠΈΠΆΠ΅Π½ΠΈΠ΅ ΠΎΡΠ½ΠΎΡΠΈΡΠ΅Π»ΡΠ½ΠΎ Π·Π°ΡΡΠΆΠ΅Π½Π½ΠΎΠΉ ΡΠ°ΡΡΠΈΡΡ Π½Π΅ ΡΠΎΠ·Π΄Π°Π΅Ρ Π΄Π»Ρ Π½Π΅Π΅ ΠΌΠ°Π³Π½ΠΈΡΠ½ΠΎΠ³ΠΎ ΠΏΠΎΠ»Ρ. ΠΡΠΎ ΠΏΡΠΎΡΡΠΎ ΠΏΡΠΈΠ²ΠΎΠ΄ΠΈΡ ΠΊ Π²ΡΠ°ΡΠ°ΡΠ΅Π»ΡΠ½ΠΎΠΉ ΡΠΈΠ»Π΅ Π² Π΄ΠΎΠΏΠΎΠ»Π½Π΅Π½ΠΈΠ΅ ΠΊ Π»ΠΈΠ½Π΅ΠΉΠ½ΠΎΠΉ ΡΠΈΠ»Π΅ ΠΈΠ·-Π·Π° Β«ΡΠΏΠΈΠ½ΠΎΡΠ½ΠΎΠΉΒ» ΠΏΡΠΈΡΠΎΠ΄Ρ Π·Π°ΡΡΠΆΠ΅Π½Π½ΡΡ ΡΠ°ΡΡΠΈΡ.
$\endgroup$
0
ΠΠ°ΡΠ΅Π³ΠΈΡΡΡΠΈΡΡΠΉΡΠ΅ΡΡ ΠΈΠ»ΠΈ Π²ΠΎΠΉΠ΄ΠΈΡΠ΅ Π² ΡΠΈΡΡΠ΅ΠΌΡ
ΠΠ°ΡΠ΅Π³ΠΈΡΡΡΠΈΡΡΠΉΡΠ΅ΡΡ Ρ ΠΏΠΎΠΌΠΎΡΡΡ Google
ΠΠ°ΡΠ΅Π³ΠΈΡΡΡΠΈΡΠΎΠ²Π°ΡΡΡΡ ΡΠ΅ΡΠ΅Π· Facebook
ΠΠ°ΡΠ΅Π³ΠΈΡΡΡΠΈΡΡΠΉΡΠ΅ΡΡ, ΠΈΡΠΏΠΎΠ»ΡΠ·ΡΡ ΡΠ»Π΅ΠΊΡΡΠΎΠ½Π½ΡΡ ΠΏΠΎΡΡΡ ΠΈ ΠΏΠ°ΡΠΎΠ»Ρ
ΠΠΏΡΠ±Π»ΠΈΠΊΠΎΠ²Π°ΡΡ ΠΊΠ°ΠΊ Π³ΠΎΡΡΡ
ΠΠ»Π΅ΠΊΡΡΠΎΠ½Π½Π°Ρ ΠΏΠΎΡΡΠ°
Π’ΡΠ΅Π±ΡΠ΅ΡΡΡ, Π½ΠΎ Π½Π΅ ΠΎΡΠΎΠ±ΡΠ°ΠΆΠ°Π΅ΡΡΡ
ΠΠΏΡΠ±Π»ΠΈΠΊΠΎΠ²Π°ΡΡ ΠΊΠ°ΠΊ Π³ΠΎΡΡΡ
ΠΠ»Π΅ΠΊΡΡΠΎΠ½Π½Π°Ρ ΠΏΠΎΡΡΠ°
Π’ΡΠ΅Π±ΡΠ΅ΡΡΡ, Π½ΠΎ Π½Π΅ ΠΎΡΠΎΠ±ΡΠ°ΠΆΠ°Π΅ΡΡΡ
ΠΠ°ΠΆΠΈΠΌΠ°Ρ Β«ΠΠΏΡΠ±Π»ΠΈΠΊΠΎΠ²Π°ΡΡ ΡΠ²ΠΎΠΉ ΠΎΡΠ²Π΅ΡΒ», Π²Ρ ΡΠΎΠ³Π»Π°ΡΠ°Π΅ΡΠ΅ΡΡ Ρ Π½Π°ΡΠΈΠΌΠΈ ΡΡΠ»ΠΎΠ²ΠΈΡΠΌΠΈ ΠΎΠ±ΡΠ»ΡΠΆΠΈΠ²Π°Π½ΠΈΡ, ΠΏΠΎΠ»ΠΈΡΠΈΠΊΠΎΠΉ ΠΊΠΎΠ½ΡΠΈΠ΄Π΅Π½ΡΠΈΠ°Π»ΡΠ½ΠΎΡΡΠΈ ΠΈ ΠΏΠΎΠ»ΠΈΡΠΈΠΊΠΎΠΉ ΠΈΡΠΏΠΎΠ»ΡΠ·ΠΎΠ²Π°Π½ΠΈΡ ΡΠ°ΠΉΠ»ΠΎΠ² cookie
ΠΠ°Π³Π½ΠΈΡΠ½ΠΎΠ΅ ΠΏΠΎΠ»Π΅ ΡΠΎΡΡΠΎΠΈΡ ΠΈΠ· ΡΠΎΡΠΎΠ½ΠΎΠ² | Π€ΠΈΠ·ΠΈΠΊΠ° Π€ΡΡΠ³ΠΎΠ½
ΠΠ°ΡΠ΅Π³ΠΎΡΠΈΡ ΠΡΠ±Π΅ΡΠΈΡΠ΅ ΠΊΠ°ΡΠ΅Π³ΠΎΡΠΈΡΠ ΡΡΡΠ³ΠΎΠ½Π΅ ΡΠΈΠ·ΠΈΠΊΠΈΠΠ»Π΅ΠΊΡΡΠΈΡΠ΅ΡΡΠ²ΠΎ ΠΈ ΠΌΠ°Π³Π½ΠΈΡΡΠΡΠ΅ ΠΎΡΡΠ°Π»ΡΠ½ΠΎΠ΅Π‘Π²Π΅Ρ ΠΈ Π·Π²ΡΠΊΠΠ²ΠΈΠΆΠ΅Π½ΠΈΠ΅ Π²Π΅ΡΠ΅ΠΉΠΠΎΠ²Π°Ρ ΠΈ Π·Π°Ρ Π²Π°ΡΡΠ²Π°ΡΡΠ°Ρ ΡΠΈΠ·ΠΈΠΊΠ°Π‘ΠΎΡΡΠΎΡΠ½ΠΈΡ Π²Π΅ΡΠ΅ΡΡΠ²Π° ΠΈ ΡΠ½Π΅ΡΠ³ΠΈΠΈΠΠΎΡΠΌΠΎΡΠΠΎΠ΄ Π²ΠΎΠ΄ΠΎΠΉ ΠΈ Π² Π²ΠΎΠ·Π΄ΡΡ Π΅
ΠΠΎΠ΄ΠΊΠ°ΡΠ΅Π³ΠΎΡΠΈΡ
ΠΠΎΠΈΡΠΊΠΠ°Π΄Π°ΠΉΡΠ΅ Π²ΠΎΠΏΡΠΎΡ
ΠΠΎΡΠ»Π΅Π΄Π½ΠΈΠΉ ΠΎΡΠ²Π΅Ρ: 22. 10.2007
Q:
ΠΠΎΠΉ Π²ΠΎΠΏΡΠΎΡ: ΠΈΠ· ΡΠ΅Π³ΠΎ ΡΠΎΡΡΠΎΠΈΡ ΠΌΠ°Π³Π½ΠΈΡΠ½ΠΎΠ΅ ΠΏΠΎΠ»Π΅? Π― ΠΌΠ½ΠΎΠ³ΠΎ ΡΠΈΡΠ°Π» ΠΎ ΠΌΠ°Π³Π½ΠΈΡΠ°Ρ
ΠΈ ββΠΏΠΎΠ»ΡΡ
, ΠΊΠΎΡΠΎΡΡΠ΅ ΠΎΠ½ΠΈ ΡΠΎΠ·Π΄Π°ΡΡ, ΠΈ Π΄Π°ΠΆΠ΅ ΠΎ ΡΠΎΠΌ, ΡΡΠΎ Π²ΠΎΠΊΡΡΠ³ ΡΠ°ΠΌΠΈΡ
ΡΠ»Π΅ΠΊΡΡΠΎΠ½ΠΎΠ² Π΅ΡΡΡ ΠΌΠ°Π³Π½ΠΈΡΠ½ΡΠ΅ ΠΏΠΎΠ»Ρ, Π½ΠΎ Ρ ΠΏΠΎΠΊΠ° Π½Π΅ Π²ΡΡΡΠ΅ΡΠ°Π» Π½ΠΈΡΠ΅Π³ΠΎ, ΡΡΠΎ Π³ΠΎΠ²ΠΎΡΠΈΠ»ΠΎ Π±Ρ ΠΎ ΡΠΎΠΌ, ΠΈΠ· ΡΠ΅Π³ΠΎ ΡΠΎΡΡΠΎΠΈΡ ΡΠ°ΠΌΠΎ ΠΏΠΎΠ»Π΅. ΠΡΠΎ ΠΌΠ°ΡΠ΅ΡΠΈΡ, ΠΏΠΎΡΡΠΎΠΌΡ ΠΎΠ½Π° Π΄ΠΎΠ»ΠΆΠ½Π° Π±ΡΡΡ ΡΠ΄Π΅Π»Π°Π½Π° ΠΈΠ· ΡΠ΅Π³ΠΎ-ΡΠΎ. ΠΡΡΡ Π»ΠΈ Π½Π°Π·Π²Π°Π½ΠΈΠ΅ Π΄Π»Ρ ΡΡΠΈΡ
Β«ΡΠ°ΡΡΠΈΡΒ»? ΠΠ»ΠΈ ΠΎΠ½ΠΈ ΡΠ°ΠΌΠΈ ΠΏΡΠΎΡΡΠΎ ΡΠ»Π΅ΠΊΡΡΠΎΠ½Ρ?
— ΠΡΠ³Π»Π°Ρ (35 Π»Π΅Ρ)
ΠΡΠΈΠ·ΠΈΠ°Π½Π° Π‘Π¨Π
A:
ΠΡΠΈΠ²Π΅Ρ, ΠΡΠ³Π»Π°Ρ!
ΠΠ»Π΅ΠΊΡΡΠΎΠΌΠ°Π³Π½ΠΈΡΠ½ΠΎΠ΅ Π²Π·Π°ΠΈΠΌΠΎΠ΄Π΅ΠΉΡΡΠ²ΠΈΠ΅ ΠΎΠΏΠΎΡΡΠ΅Π΄ΠΎΠ²Π°Π½ΠΎ ΠΊΠΎΠ½ΡΡΠ°Π½ΡΠΎΠΉ ΠΎΠ±ΠΌΠ΅Π½ ΡΠΎΡΠΎΠ½Π°ΠΌΠΈ ΠΎΡ ΠΎΠ΄Π½ΠΎΠ³ΠΎ Π·Π°ΡΡΠΆΠ΅Π½Π½ΠΎΠ³ΠΎ ΠΎΠ±ΡΠ΅ΠΊΡΠ° ΠΊ Π΄ΡΡΠ³ΠΎΠΌΡ. ΠΠ°Π³Π½ΠΈΡΠ½ΡΠΉ ΠΏΠΎΠ»Π΅ Π½Π° ΡΠ°ΠΌΠΎΠΌ Π΄Π΅Π»Π΅ ΠΏΡΠΎΡΡΠΎ ΠΊΠ»Π°ΡΡΠΈΡΠ΅ΡΠΊΠΎΠ΅ ΠΏΡΠΈΠ±Π»ΠΈΠΆΠ΅Π½ΠΈΠ΅ ΠΊ ΡΠΎΡΠΎΠ½Π½ΠΎΠΌΡ ΠΎΠ±ΠΌΠ΅Π½Ρ ΠΊΠ°ΡΡΠΈΠ½Π°. Π Π΄Π²ΠΈΠΆΡΡΠ΅ΠΉΡΡ ΡΠΈΡΡΠ΅ΠΌΠ΅ ΠΎΡΡΡΠ΅ΡΠ° Π²ΠΌΠ΅ΡΡΠΎ Π½Π΅Π΅ ΠΏΠΎΡΠ²Π»ΡΠ΅ΡΡΡ ΠΌΠ°Π³Π½ΠΈΡΠ½ΠΎΠ΅ ΠΏΠΎΠ»Π΅ ΠΊΠ°ΠΊ ΠΊΠΎΠΌΠ±ΠΈΠ½Π°ΡΠΈΡ ΠΌΠ°Π³Π½ΠΈΡΠ½ΠΎΠ³ΠΎ ΠΏΠΎΠ»Ρ ΠΈ ΡΠ»Π΅ΠΊΡΡΠΈΡΠ΅ΡΠΊΠΎΠ³ΠΎ ΠΏΠΎΠ»Ρ, ΠΏΠΎΡΡΠΎΠΌΡ ΡΠ»Π΅ΠΊΡΡΠΈΡΠ΅ΡΠΊΠΎΠ΅ Π° ΠΌΠ°Π³Π½ΠΈΡΠ½ΡΠ΅ ΠΏΠΎΠ»Ρ ΡΠΎΡΡΠΎΡΡ ΠΈΠ· ΠΎΠ΄Π½ΠΎΠ³ΠΎ ΠΈ ΡΠΎΠ³ΠΎ ΠΆΠ΅ Β«Π²Π΅ΡΠ΅ΡΡΠ²Π°Β» (ΡΠΎΡΠΎΠ½ΠΎΠ²).
Π Π½Π΅ΠΊΠΎΡΠΎΡΡΡ ΡΠ»Π΅ΠΊΡΡΠΎΠΌΠ°Π³Π½ΠΈΡΠ½ΡΡ Π²Π·Π°ΠΈΠΌΠΎΠ΄Π΅ΠΉΡΡΠ²ΠΈΡΡ ΡΡΠ°ΡΡΠ²ΡΡΡ Β«Π½Π°ΡΡΠΎΡΡΠΈΠ΅Β» ΡΠΎΡΠΎΠ½Ρ Ρ ΠΎΠΏΡΠ΅Π΄Π΅Π»Π΅Π½Π½ΡΠ΅ ΡΠ°ΡΡΠΎΡΡ, ΡΠ½Π΅ΡΠ³ΠΈΠΈ ΠΈ ΠΈΠΌΠΏΡΠ»ΡΡΡ. ΠΠ»Π΅ΠΊΡΡΠΎΡΡΠ°ΡΠΈΡΠ΅ΡΠΊΠΈΠ΅ ΠΈ ΠΌΠ°Π³Π½ΠΈΡΠ½ΡΠ΅ Π²ΠΌΠ΅ΡΡΠΎ ΡΡΠΎΠ³ΠΎ ΠΏΠΎΠ»Ρ Π²ΠΊΠ»ΡΡΠ°ΡΡ ΠΎΠ±ΠΌΠ΅Π½ Β«Π²ΠΈΡΡΡΠ°Π»ΡΠ½ΡΠΌΠΈΒ» ΡΠΎΡΠΎΠ½Π°ΠΌΠΈ. ΠΡΠ΅Π½Ρ Π±Π»ΠΈΠ·ΠΊΠΎ ΠΊ ΡΠ»Π΅ΠΊΡΡΠΎΠ½ ΠΏΡΠ΅Π΄ΡΡΠ°Π²Π»ΡΠ΅Ρ ΡΠΎΠ±ΠΎΠΉ ΠΏΠ»ΠΎΡΠ½ΠΎΠ΅ ΠΎΠ±Π»Π°ΠΊΠΎ Π²ΠΈΡΡΡΠ°Π»ΡΠ½ΡΡ ΡΠΎΡΠΎΠ½ΠΎΠ², ΠΊΠΎΡΠΎΡΡΠ΅ ΠΏΠΎΡΡΠΎΡΠ½Π½ΠΎ ΠΈΡΠΏΡΡΠΊΠ°Π΅ΡΡΡ ΠΈ Π²Π½ΠΎΠ²Ρ ΠΏΠΎΠ³Π»ΠΎΡΠ°Π΅ΡΡΡ ΡΠ»Π΅ΠΊΡΡΠΎΠ½ΠΎΠΌ. ΠΠ΅ΠΊΠΎΡΠΎΡΡΠ΅ ΠΈΠ· ΡΡΠΈΡ ΡΠΎΡΠΎΠ½ΠΎΠ² ΡΠ°ΡΡΠ΅ΠΏΠ»ΡΡΡΡΡ Π½Π° ΡΠ»Π΅ΠΊΡΡΠΎΠ½-ΠΏΠΎΠ·ΠΈΡΡΠΎΠ½Π½ΡΠ΅ ΠΏΠ°ΡΡ (ΠΈΠ»ΠΈ ΠΏΠ°ΡΡ Π΅ΡΠ΅ Π±ΠΎΠ»Π΅Π΅ ΡΡΠΆΠ΅Π»ΡΡ Π²Π΅ΡΠ΅ΡΡΠ²), ΠΊΠΎΡΠΎΡΡΠ΅ ΡΠ΅ΠΊΠΎΠΌΠ±ΠΈΠ½ΠΈΡΡΡΡ Π² ΡΠΎΡΠΎΠ½Ρ, ΠΊΠΎΡΠΎΡΡΠ΅ ΠΏΠΎΠ²ΡΠΎΡΠ½ΠΎ ΠΏΠΎΠ³Π»ΠΎΡΠ°ΡΡΡΡ ΠΈΡΡ ΠΎΠ΄Π½ΡΠΌ ΡΠ»Π΅ΠΊΡΡΠΎΠ½. ΠΡΠΈ Π²ΠΈΡΡΡΠ°Π»ΡΠ½ΡΠ΅ ΠΏΠ΅ΡΠ»ΠΈ ΡΠ°ΡΡΠΈΡ ΡΠΊΡΠ°Π½ΠΈΡΡΡΡ Π·Π°ΡΡΠ΄ ΡΠ»Π΅ΠΊΡΡΠΎΠ½ ΡΠ°ΠΊ, ΡΡΠΎ Π²Π΄Π°Π»ΠΈ ΠΎΡ ΡΠ»Π΅ΠΊΡΡΠΎΠ½Π° ΠΊΠ°ΠΆΠ΅ΡΡΡ, ΡΡΠΎ ΠΎΠ½ ΠΈΠΌΠ΅Π΅Ρ ΠΌΠ΅Π½ΡΡΠ΅ Π·Π°ΡΡΠ΄ΠΈΡΡ ΡΠ΅ΠΌ ΡΡΠ΄ΠΎΠΌ. 92 (Π² ΡΠΈΡΡΠ΅ΠΌΠ΅ ΠΎΡΡΡΠ΅ΡΠ°, Π² ΠΊΠΎΡΠΎΡΠΎΠΉ ΡΠ»Π΅ΠΊΡΡΠΎΠ½ Π½Π΅ ΠΈΠΌΠ΅Π΅Ρ ΠΈΠΌΠΏΡΠ»ΡΡ).
Π’ΠΎΠΌ
(ΠΎΠΏΡΠ±Π»ΠΈΠΊΠΎΠ²Π°Π½ΠΎ 22.10.2007)
ΠΠΎΠΏΠΎΠ»Π½Π΅Π½ΠΈΠ΅ β1: ΠΏΡΠ΅Π΄ΡΡΠ°Π²Π»Π΅Π½ΠΈΠ΅ ΠΎ ΠΌΠ°Π³Π½Π΅ΡΠΈΠ·ΠΌΠ΅
ΠΠΎΠΏΡΠΎΡ:
ΠΠΎΠΆΠ½ΠΎ Π»ΠΈ ΡΠ΅ΡΠΊΠΎ ΠΎΡΠ²Π΅ΡΠΈΡΡ Π½Π° Π²ΠΎΠΏΡΠΎΡ, ΠΈΡΠΏΠΎΠ»ΡΠ·ΡΡ Π±ΠΎΠ»Π΅Π΅ ΡΠ»Π΅ΠΌΠ΅Π½ΡΠ°ΡΠ½ΡΠ΅ ΡΠ΅ΡΠΌΠΈΠ½Ρ Π΄Π»Ρ ΡΠ»Π΅ΠΌΠ΅Π½ΡΠ°ΡΠ½ΡΡ
ΡΠ°ΡΡΠΈΡ? ΠΠ»ΠΈ, ΠΌΠΎΠΆΠ΅Ρ Π±ΡΡΡ, ΡΡ
Π΅ΠΌΠ° ΡΠ°ΡΡΠΈΡ ΠΊΠ°ΠΊ ΠΊΠ°ΡΠΊΠ°Π΄Π° Π² ΡΠ°Π·Π»ΠΈΡΠ½ΡΡ
ΡΠ½Π΅ΡΠ³Π΅ΡΠΈΡΠ΅ΡΠΊΠΈΡ
ΡΠΎΡΡΠΎΡΠ½ΠΈΡΡ
, ΠΏΡΠΎΠΈΠ·Π²ΠΎΠ΄ΡΡΠΈΡ
Π²Π·Π°ΠΈΠΌΠ½ΡΠ΅ ΡΠΈΠ»Ρ ΠΈΠ»ΠΈ Π΄Π΅ΠΉΡΡΠ²ΠΈΡ Π½Π° Π΄ΡΡΠ³ΡΡ ΠΌΠ°ΡΠ΅ΡΠΈΡ?
— ΠΠΆ. Π‘. ΠΠΠ Π (67 Π»Π΅Ρ)
Davao City,Philippines
A:
ΠΠΎΡ ΠΎΠΆΠ΅, Π²Ρ ΠΏΡΠΎΡΠΈΡΠ΅ Π΄ΠΈΠ°Π³ΡΠ°ΠΌΠΌΡ Π€Π΅ΠΉΠ½ΠΌΠ°Π½Π° Π΄Π»Ρ ΠΏΡΠ΅Π΄ΡΡΠ°Π²Π»Π΅Π½ΠΈΡ ΡΠ»Π΅ΠΊΡΡΠΎΠΌΠ°Π³Π½ΠΈΡΠ½ΡΡ Π²Π·Π°ΠΈΠΌΠΎΠ΄Π΅ΠΉΡΡΠ²ΠΈΠΉ. ΠΡ ΠΌΠΎΠΆΠ΅ΡΠ΅ ΠΏΠΎΠ»ΡΡΠΈΡΡ ΡΡΠΎ Π² ΠΎΠ±ΡΡΠΆΠ΄Π΅Π½ΠΈΠΈ Π² ΠΠΈΠΊΠΈΠΏΠ΅Π΄ΠΈΠΈ:Β https://en.wikipedia.org/wiki/Quantum_electrodynamics.
Π ΠΏΠΎΠΊΠ° Ρ Π²ΠΎΡΠΏΠΎΠ»ΡΠ·ΡΡΡΡ ΡΠ»ΡΡΠ°Π΅ΠΌ, ΡΡΠΎΠ±Ρ Π½Π΅ΠΌΠ½ΠΎΠ³ΠΎ ΠΈΠ·ΠΌΠ΅Π½ΠΈΡΡ ΠΏΡΠ΅Π·Π΅Π½ΡΠ°ΡΠΈΡ Π’ΠΎΠΌΠ°. ΠΡ ΠΎΠ±ΡΡΠ½ΠΎ Π³ΠΎΠ²ΠΎΡΠΈΠΌ ΡΡΠΎ-ΡΠΎ Π²ΡΠΎΠ΄Π΅ Β«Π²ΠΈΡΡΡΠ°Π»ΡΠ½ΡΠ΅ ΡΠΎΡΠΎΠ½Ρβ¦ ΠΏΠΎΡΡΠΎΡΠ½Π½ΠΎ ΠΈΡΠΏΡΡΠΊΠ°ΡΡΡΡ ΠΈ ΠΏΠΎΠ²ΡΠΎΡΠ½ΠΎ ΠΏΠΎΠ³Π»ΠΎΡΠ°ΡΡΡΡ ΡΠ»Π΅ΠΊΡΡΠΎΠ½ΠΎΠΌΒ», Π½ΠΎ Π½Π° ΡΠ°ΠΌΠΎΠΌ Π΄Π΅Π»Π΅ ΠΌΡ ΠΈΠΌΠ΅Π΅ΠΌ Π² Π²ΠΈΠ΄Ρ Π½Π΅ ΡΡΠΎ. ΠΠ²Π΅ ΡΠ°ΡΡΠΈΡΡ, Π²Π·Π°ΠΈΠΌΠΎΠ΄Π΅ΠΉΡΡΠ²ΡΡΡΠΈΠ΅ ΡΠ»Π΅ΠΊΡΡΠΎΠΌΠ°Π³Π½ΠΈΡΠ½ΡΠΌ ΠΎΠ±ΡΠ°Π·ΠΎΠΌ, Π΄Π΅ΠΉΡΡΠ²ΠΈΡΠ΅Π»ΡΠ½ΠΎ ΠΎΠΊΡΡΠΆΠ΅Π½Ρ Π²ΠΈΡΡΡΠ°Π»ΡΠ½ΡΠΌ ΡΠΎΡΠΎΠ½Π½ΡΠΌ ΠΎΠ±Π»Π°ΠΊΠΎΠΌ. ΠΠ΄Π½Π°ΠΊΠΎ Π² ΠΈΠ·Π²Π΅ΡΡΠ½ΡΡ ΡΠ»ΡΡΠ°ΡΡ (Π½Π°ΠΏΡΠΈΠΌΠ΅Ρ, Π°ΡΠΎΠΌ Π²ΠΎΠ΄ΠΎΡΠΎΠ΄Π°) ΡΡΠΎ ΠΎΠ±Π»Π°ΠΊΠΎ ΡΠΎΠ²Π΅ΡΡΠ΅Π½Π½ΠΎ Π½Π΅ΠΈΠ·ΠΌΠ΅Π½Π½ΠΎ Π²ΠΎ Π²ΡΠ΅ΠΌΠ΅Π½ΠΈ. ΠΠΎΠΎΠ±ΡΠ΅ Π½ΠΈΡΠ΅Π³ΠΎ Π½Π΅ ΠΏΡΠΎΠΈΡΡ ΠΎΠ΄ΠΈΡ. Π‘Π»ΠΎΠ²Π° ΠΎ ΠΊΠΎΠ»Π΅Π±Π»ΡΡΠΈΡ ΡΡ Π²ΠΎΠΊΡΡΠ³ Π²Π΅ΡΠ°Ρ β Π³ΡΡΠ±ΡΠΉ ΡΠΏΠΎΡΠΎΠ± ΠΏΠ΅ΡΠ΅Π΄Π°ΡΡ ΠΎΠ΄Π½ΠΎ ΠΈΠ· ΡΠΏΠ΅ΡΠΈΡΠΈΡΠ΅ΡΠΊΠΈΡ ΡΠ²ΠΎΠΉΡΡΠ² ΠΊΠ²Π°Π½ΡΠΎΠ²ΡΡ ΠΏΠΎΠ»Π΅ΠΉ. ΠΠ»Π΅ΠΊΡΡΠΈΡΠ΅ΡΠΊΠΈΠ΅ ΠΈ ΠΌΠ°Π³Π½ΠΈΡΠ½ΡΠ΅ ΠΏΠΎΠ»Ρ ΠΈΠΌΠ΅ΡΡ Π½Π΅ ΡΠΎΠ»ΡΠΊΠΎ ΡΡΠ΅Π΄Π½ΠΈΠ΅ Π·Π½Π°ΡΠ΅Π½ΠΈΡ, Π½ΠΎ ΠΈ Π΄ΠΈΠ°ΠΏΠ°Π·ΠΎΠ½Ρ Π²ΠΎΠ·ΠΌΠΎΠΆΠ½ΡΡ Π·Π½Π°ΡΠ΅Π½ΠΈΠΉ Π²ΠΎΠΊΡΡΠ³ ΡΡΠ΅Π΄Π½Π΅Π³ΠΎ. ΠΠΎΡ ΡΡΠΎ ΡΠ°ΠΊ ΠΎΡΠ»ΠΈΡΠ°Π΅ΡΡΡ ΠΎΡ ΠΊΠ»Π°ΡΡΠΈΡΠ΅ΡΠΊΠΈΡ ΠΏΠΎΠ»Π΅ΠΉ. (ΠΡΠΎ ΠΏΠΎΡ ΠΎΠΆΠ΅ Π½Π° ΠΏΠΎΠ»ΠΎΠΆΠ΅Π½ΠΈΡ ΠΊΠ²Π°Π½ΡΠΎΠ²ΡΡ ΡΠ°ΡΡΠΈΡ, ΠΊΠΎΡΠΎΡΡΠ΅ ΠΈΠΌΠ΅ΡΡ Π΄ΠΈΠ°ΠΏΠ°Π·ΠΎΠ½ ΠΎΠΊΠΎΠ»ΠΎ ΡΡΠ΅Π΄Π½Π΅Π³ΠΎ ΠΏΠΎΠ»ΠΎΠΆΠ΅Π½ΠΈΡ, Π² ΠΎΡΠ»ΠΈΡΠΈΠ΅ ΠΎΡ ΠΊΠ»Π°ΡΡΠΈΡΠ΅ΡΠΊΠΈΡ ΡΠ°ΡΡΠΈΡ. ) ΠΡ ΠΌΠΎΠΆΠ΅ΡΠ΅ ΠΏΠ΅ΡΠ΅Π΄Π°ΡΡ ΠΎΠ±ΡΠ°Π· ΡΡΠΎΠ³ΠΎ Π΄ΠΈΠ°ΠΏΠ°Π·ΠΎΠ½Π°, ΠΏΡΠΈΡΠ²ΠΎΡΠΈΠ²ΡΠΈΡΡ, ΡΡΠΎ ΠΏΠΎΠ»Ρ ΠΏΡΡΠ³Π°ΡΡ ΠΌΠ΅ΠΆΠ΄Ρ ΡΠ°Π·Π»ΠΈΡΠ½ΡΠΌΠΈ Π²ΠΎΠ·ΠΌΠΎΠΆΠ½ΡΠΌΠΈ Π·Π½Π°ΡΠ΅Π½ΠΈΡΠΌΠΈ, ΡΠΎΡΠ½ΠΎ ΡΠ°ΠΊ ΠΆΠ΅, ΠΊΠ°ΠΊ Π²Ρ ΠΌΠΎΠΆΠ΅ΡΠ΅ ΠΏΡΠ΅Π΄ΡΡΠ°Π²ΠΈΡΡ ΡΡΠΎ ΡΠ°ΡΡΠΈΡΠ° ΠΏΡΡΠ³Π°Π΅Ρ ΠΌΠ΅ΠΆΠ΄Ρ ΡΠ°Π·Π»ΠΈΡΠ½ΡΠΌΠΈ ΠΏΠΎΠ»ΠΎΠΆΠ΅Π½ΠΈΡΠΌΠΈ Π² ΡΠ²ΠΎΠ΅ΠΌ ΠΎΠ±Π»Π°ΠΊΠ΅. ΠΠ΄Π½Π°ΠΊΠΎ ΠΏΠΎΠ»Ρ (Π²ΠΊΠ»ΡΡΠ°Ρ ΠΈΡ ΡΠ°Π·Π±ΡΠΎΡ Π·Π½Π°ΡΠ΅Π½ΠΈΠΉ) Π΄ΠΎΠ»ΠΆΠ½Ρ ΠΏΡΡΠ³Π°ΡΡ Π½Π΅ Π±ΠΎΠ»ΡΡΠ΅, ΡΠ΅ΠΌ ΡΠ°ΡΡΠΈΡΡ Π² ΠΏΡΠΎΡΡΡΠ°Π½ΡΡΠ²Π΅. ΠΠ°ΠΏΡΠΈΠΌΠ΅Ρ, Π² Ρ ΠΎΡΠΎΡΠ΅ΠΌ ΡΡΠ°Π±ΠΈΠ»ΡΠ½ΠΎΠΌ Π°ΡΠΎΠΌΠ½ΠΎΠΌ ΡΠΎΡΡΠΎΡΠ½ΠΈΠΈ Π½ΠΈΡΠ΅Π³ΠΎ Π½Π΅ ΠΌΠ΅Π½ΡΠ΅ΡΡΡ Π²ΠΎ Π²ΡΠ΅ΠΌΠ΅Π½ΠΈ. Π‘ΡΠ°ΡΠΈΡΠ΅ΡΠΊΠΈΠΉ Π΄ΠΈΠ°ΠΏΠ°Π·ΠΎΠ½ Π²ΠΎΠ·ΠΌΠΎΠΆΠ½ΠΎΡΡΠ΅ΠΉ ΠΏΡΠ΅Π²ΡΠ°ΡΠ°Π΅ΡΡΡ Π² Π΄Π΅ΠΉΡΡΠ²ΠΈΡΠ΅Π»ΡΠ½ΡΠΉ Π΄ΠΈΠ°ΠΏΠ°Π·ΠΎΠ½ ΡΠ΅Π·ΡΠ»ΡΡΠ°ΡΠΎΠ² ΡΠΎΠ»ΡΠΊΠΎ ΡΠΎΠ³Π΄Π°, ΠΊΠΎΠ³Π΄Π° ΡΠΈΡΡΠ΅ΠΌΠ° ΠΎΠΏΡΠ΅Π΄Π΅Π»Π΅Π½Π½ΡΠΌ ΠΎΠ±ΡΠ°Π·ΠΎΠΌ Π²Π·Π°ΠΈΠΌΠΎΠ΄Π΅ΠΉΡΡΠ²ΡΠ΅Ρ Ρ Π±ΠΎΠ»ΡΡΠΈΠΌ Π²Π½Π΅ΡΠ½ΠΈΠΌ ΠΌΠΈΡΠΎΠΌ.
ΠΠ°ΠΉΠΊ Π.
(ΠΎΠΏΡΠ±Π»ΠΈΠΊΠΎΠ²Π°Π½ΠΎ 27.04.2011)
ΠΠΎΠΏΠΎΠ»Π½Π΅Π½ΠΈΠ΅ β2: Π²Π΅ΡΠ½Π΅ΠΌΡΡ ΠΊ ΠΌΠ΅Ρ Π°Π½ΠΈΡΠ΅ΡΠΊΠΈΠΌ ΠΏΠΎΠ»ΡΠΌ ΠΠ°ΠΊΡΠ²Π΅Π»Π»Π°
ΠΠΎΠΏΡΠΎΡ:
Π― Π΄Π΅ΠΉΡΡΠ²ΠΈΡΠ΅Π»ΡΠ½ΠΎ Ρ
ΠΎΡΡ Π·Π½Π°ΡΡ, ΠΈΠ· ΡΠ΅Π³ΠΎ ΡΠΎΡΡΠΎΠΈΡ ΠΌΠ°Π³Π½ΠΈΡΠ½ΠΎΠ΅ ΠΏΠΎΠ»Π΅ ? ΠΠΎΠΆΠ°Π»ΡΠΉΡΡΠ°, Π½Π΅ ΠΈΡΠΏΠΎΠ»ΡΠ·ΡΠΉΡΠ΅ ΡΠΎΡΠΎΠ½Ρ Π² ΡΠ²ΠΎΠ΅ΠΌ ΠΎΡΠ²Π΅ΡΠ΅, ΠΏΠΎΡΠΊΠΎΠ»ΡΠΊΡ Π²ΡΠ΅ ΠΌΡ Π·Π½Π°Π΅ΠΌ, ΡΡΠΎ ΠΌΠ°Π³Π½ΠΈΡΠ½ΡΠ΅ ΠΏΠΎΠ»Ρ Π½Π΅ ΡΠΎΡΡΠΎΡΡ ΠΈΠ· ΡΠΎΡΠΎΠ½ΠΎΠ². ΠΠ° Π²ΠΎΠΏΡΠΎΡ ΠΡΠ³Π»Π°ΡΠ° ΡΠ°ΠΊ ΠΈ Π½Π΅ ΠΎΡΠ²Π΅ΡΠΈΠ»ΠΈ. ΠΡΠ³Π»Π°Ρ Π½Π΅ ΡΠΏΡΠ°ΡΠΈΠ²Π°Π» ΠΎΠ± ΡΠ»Π΅ΠΊΡΡΠΎΠΌΠ°Π³Π½ΠΈΡΠ½ΠΎΠΌ ΠΏΠΎΠ»Π΅, ΠΊΠΎΡΠΎΡΠΎΠ΅ Π½Π°ΠΈΠ±ΠΎΠ»Π΅Π΅ ΡΠ΅ΡΠ½ΠΎ ΡΠ²ΡΠ·Π°Π½ΠΎ Ρ ΡΠΎΡΠΎΠ½ΠΎΠΌ. ΠΡΠ΅Π΄ΠΎΡΡΠ°Π²Π»Π΅Π½Π½ΡΠ΅ ΠΎΡΠ²Π΅ΡΡ Π±ΡΠ»ΠΈ Π½Π΅ΠΏΠΎΠ»Π½ΡΠΌΠΈ ΠΈ Π½Π΅ ΠΎΡΠ½ΠΎΡΠΈΠ»ΠΈΡΡ ΠΊ Π²ΠΎΠΏΡΠΎΡΡ ΠΡΠ³Π»Π°ΡΠ°. ΠΠ· ΡΠ΅Π³ΠΎ ΡΠΎΡΡΠΎΠΈΡ ΠΌΠ°Π³Π½ΠΈΡΠ½ΠΎΠ΅ ΠΏΠΎΠ»Π΅? ΠΠΎΡ ΡΡΠΎ ΡΠΎΠ²ΡΠ΅ΠΌΠ΅Π½Π½Π°Ρ Π½Π°ΡΠΊΠ° Π·Π½Π°Π΅Ρ ΠΎ ΠΌΠ°Π³Π½ΠΈΡΠ½ΡΡ
ΠΏΠΎΠ»ΡΡ
. Π§Π΅ΡΡΠ½ΡΠΉ ΠΎΡΠ²Π΅Ρ: ΠΌΡ Π½Π΅ Π·Π½Π°Π΅ΠΌ, ΡΡΠΎ ΡΠ°ΠΊΠΎΠ΅ ΠΌΠ°Π³Π½ΠΈΡΠ½ΠΎΠ΅ ΠΏΠΎΠ»Π΅. Π§ΡΠΎ ΠΌΡ Π·Π½Π°Π΅ΠΌ, ΡΠ°ΠΊ ΡΡΠΎ ΡΠΎ, ΡΡΠΎ ΠΏΠΎΠ»Π΅ ΠΠ°Π³Π½ΠΈΡΠ° Π³Π΅Π½Π΅ΡΠΈΡΡΠ΅ΡΡΡ Π΄Π²ΠΈΠΆΠ΅Π½ΠΈΠ΅ΠΌ ΡΠ»Π΅ΠΊΡΡΠΎΡΡΠ°ΡΠΈΡΠ΅ΡΠΊΠΈΡ
Π·Π°ΡΡΠ΄ΠΎΠ² Π²Π½ΡΡΡΠΈ ΡΠ°ΠΌΠΎΠ³ΠΎ ΠΌΠ°Π³Π½ΠΈΡΠ°. ΠΠ»Π΅ΠΊΡΡΠΈΡΠ΅ΡΠΊΠΈΠ΅ Π·Π°ΡΡΠ΄Ρ — ΡΡΠΎ ΡΠ»Π΅ΠΊΡΡΠΎΠ½Ρ. ΠΠ»Π΅ΠΊΡΡΠΎΠ½Ρ Π΄Π²ΠΈΠΆΡΡΡΡ ΠΊΠΎΠ³Π΅ΡΠ΅Π½ΡΠ½ΠΎ ΠΈ ΡΠΈΠ½Ρ
ΡΠΎΠ½Π½ΠΎ, ΡΡΠΎ Π²ΡΠ·ΡΠ²Π°Π΅Ρ Π²ΡΠ±ΡΠΎΡ ΡΠΈΠ»ΡΠ½ΠΎΠ³ΠΎ ΠΌΠ°Π³Π½ΠΈΡΠ½ΠΎΠ³ΠΎ ΠΏΠΎΠ»Ρ ΠΈΠ· ΠΌΠ°Π³Π½ΠΈΡΠ°. Π§Π΅Π³ΠΎ ΠΌΡ Π½Π΅ Π·Π½Π°Π΅ΠΌ, ΡΠ°ΠΊ ΡΡΠΎ ΡΠΎΠ³ΠΎ, ΠΈΠ· ΡΠ΅Π³ΠΎ ΡΠΎΡΡΠΎΠΈΡ ΡΡΠΎ ΠΏΠΎΠ»Π΅. ΠΠ΅ΠΊΠΎΡΠΎΡΡΠ΅ Π»ΡΠ΄ΠΈ Π³ΠΎΠ²ΠΎΡΠΈΠ»ΠΈ, ΡΡΠΎ ΠΎΠ½ ΡΠΎΡΡΠΎΠΈΡ ΠΈΠ· ΠΌΠ°Π³Π½ΠΈΡΠ½ΡΡ
ΠΌΠΎΠ½ΠΎΠΏΠΎΠ»Π΅ΠΉ. ΠΠ°Π³Π½ΠΈΡΠ½ΡΠ΅ ΠΌΠΎΠ½ΠΎΠΏΠΎΠ»ΠΈ Π½ΠΈΠΊΠΎΠ³Π΄Π° Π½Π΅ ΠΎΡΠΊΡΡΠ²Π°Π»ΠΈΡΡ, ΡΠ°ΠΊ ΡΡΠΎ Π΅ΡΡΡ Π±ΠΎΠ»ΡΡΠ°Ρ Π²Π΅ΡΠΎΡΡΠ½ΠΎΡΡΡ, ΠΎΡΠ΅Π½Ρ Π±ΠΎΠ»ΡΡΠ°Ρ Π²Π΅ΡΠΎΡΡΠ½ΠΎΡΡΡ ΡΠΎΠ³ΠΎ, ΡΡΠΎ ΡΠ΅ΠΎΡΠΈΡ Π½Π΅Π²Π΅ΡΠ½Π°. Π― ΡΡΠΈΡΠ°Ρ, ΡΡΠΎ ΠΌΠ°Π³Π½ΠΈΡΠ½ΠΎΠ΅ ΠΏΠΎΠ»Π΅ Π²ΠΎΠΎΠ±ΡΠ΅ Π½Π΅ ΡΠΎΡΡΠΎΠΈΡ ΠΈΠ· ΠΏΠΎΠ»Ρ ΠΊΠ°ΠΊΠΎΠΉ-Π»ΠΈΠ±ΠΎ ΡΠ°ΡΡΠΈΡΡ. ΠΡΠ³Π»Π°Ρ, Π΄ΡΠΌΠ°ΠΉΡΠ΅ ΠΎ ΠΌΠ°Π³Π½ΠΈΡΠ½ΠΎΠΌ ΠΏΠΎΠ»Π΅ ΠΊΠ°ΠΊ ΠΎ ΠΏΡΡΠΌΠΎΠΉ Π΄Π΅ΡΠΎΡΠΌΠ°ΡΠΈΠΈ ΡΠΈΠ·ΠΈΡΠ΅ΡΠΊΠΎΠ³ΠΎ ΠΏΡΠΎΡΡΡΠ°Π½ΡΡΠ²Π°. Π’Π°ΠΊ Π΄ΠΎΠ»ΠΆΠ½Ρ ΡΠ°Π±ΠΎΡΠ°ΡΡ Π²ΡΠ΅ ΡΠΈΡΡΡΠ΅ ΠΏΠΎΠ»Ρ. ΠΡΠΎ Π΄ΠΎΠ»ΠΆΠ½Ρ Π±ΡΡΡ ΠΌΠ΅Ρ
Π°Π½ΠΈΡΠ΅ΡΠΊΠΈΠ΅ Π΄Π΅ΡΠΎΡΠΌΠ°ΡΠΈΠΈ ΠΏΡΠΎΡΡΡΠ°Π½ΡΡΠ²Π°. ΠΡ ΠΌΠΎΠΆΠ΅ΡΠ΅ Π΄ΡΠΌΠ°ΡΡ ΠΎ ΠΏΡΠΎΡΡΡΠ°Π½ΡΡΠ²Π΅ ΠΊΠ°ΠΊ ΠΎ ΡΠ²Π΅ΡΠ΄ΠΎΠΉ ΡΠΏΡΡΠ³ΠΎΠΉ ΡΠΊΠ°Π½ΠΈ Ρ Π½ΠΈΠ·ΠΊΠΎΠΉ ΠΏΠ»ΠΎΡΠ½ΠΎΡΡΡΡ ΠΈ Π²ΡΡΠΎΠΊΠΈΠΌ Π½Π°ΡΡΠΆΠ΅Π½ΠΈΠ΅ΠΌ. ΠΠ°Π³Π½ΠΈΡΠ½ΠΎΠ΅ ΠΏΠΎΠ»Π΅ Π΅ΡΡΡ ΠΌΠ΅Ρ
Π°Π½ΠΈΡΠ΅ΡΠΊΠ°Ρ Π΄Π΅ΡΠΎΡΠΌΠ°ΡΠΈΡ ΡΠ°ΠΌΠΎΠ³ΠΎ ΠΏΡΠΎΡΡΡΠ°Π½ΡΡΠ²Π°. Π― Π±Ρ Ρ
ΠΎΡΠ΅Π», ΡΡΠΎΠ±Ρ Π²Ρ Π½Π°ΠΏΠ΅ΡΠ°ΡΠ°Π»ΠΈ ΡΡΠΎΡ Π²ΠΎΠΏΡΠΎΡ ΠΈ ΠΎΡΠ²Π΅ΡΠΈΠ»ΠΈ, Π½ΠΎ ΠΌΡ ΠΎΠ±Π° Π·Π½Π°Π΅ΠΌ, ΡΡΠΎ Π²Ρ ΡΡΠΎΠ³ΠΎ Π½Π΅ ΡΠ΄Π΅Π»Π°Π΅ΡΠ΅.
— ΠΠ°ΡΠΊ (58 Π»Π΅Ρ)
Π€Π»ΠΎΡΠΈΠ΄Π°
A:
ΠΠΎΠ³Π΄Π° ΠΠ°ΠΊΡΠ²Π΅Π»Π» Π²ΠΏΠ΅ΡΠ²ΡΠ΅ ΠΏΡΠΈΠ΄ΡΠΌΠ°Π» ΡΠ²ΠΎΠΈ Π·Π½Π°ΠΌΠ΅Π½ΠΈΡΡΠ΅ ΡΡΠ°Π²Π½Π΅Π½ΠΈΡ ΡΠ»Π΅ΠΊΡΡΠΎΠΌΠ°Π³Π½Π΅ΡΠΈΠ·ΠΌΠ°, ΠΎΠ½ ΠΏΠΎΠΏΡΡΠ°Π»ΡΡ ΡΠ΄Π΅Π»Π°ΡΡ ΠΌΠ΅Ρ Π°Π½ΠΈΡΠ΅ΡΠΊΡΡ ΠΌΠΎΠ΄Π΅Π»Ρ Ρ ΠΌΠ°Π»Π΅Π½ΡΠΊΠΈΠΌΠΈ ΡΠ΅ΡΡΠ΅ΡΠ½ΡΠΌΠΈ, ΠΊΠΎΠ»Π΅ΡΠ°ΠΌΠΈ ΠΈ ΠΏΡΠΎΡΠΈΠΌ. ΠΡ Π½Π΅Π³ΠΎ Π±ΡΡΡΡΠΎ ΠΎΡΠΊΠ°Π·Π°Π»ΠΈΡΡ, ΠΏΠΎΡΠΊΠΎΠ»ΡΠΊΡ ΠΎΠ½ Π½Π΅ Π΄ΠΎΠ±Π°Π²Π»ΡΠ» Π½ΠΈΡΠ΅Π³ΠΎ, ΠΊΡΠΎΠΌΠ΅ ΡΡΠ»ΠΎΠΆΠ½Π΅Π½ΠΈΡ ΡΡΠ°Π²Π½Π΅Π½ΠΈΠΉ ΠΏΠΎΠ»Ρ.
Π£ Π²Π°Ρ Π΅ΡΡΡ ΡΠ°Π·Π»ΠΈΡΠ½ΡΠ΅ ΡΠ»ΠΎΠ²Π΅ΡΠ½ΡΠ΅ ΡΡΠ²Π΅ΡΠΆΠ΄Π΅Π½ΠΈΡ ΠΎ ΡΠΎΠΌ, ΡΡΠΎ ΡΠ°ΠΊΠΎΠ΅ ΠΏΠΎΠ»Π΅ «Π½Π° ΡΠ°ΠΌΠΎΠΌ Π΄Π΅Π»Π΅». ΠΡ Π³ΠΎΠ²ΠΎΡΠΈΡΠ΅, ΡΡΠΎ ΠΎΠ½ ΡΠ΄Π΅Π»Π°Π½ Π½Π΅ ΠΈΠ· ΠΌΠΎΠ½ΠΎΠΏΠΎΠ»Π΅ΠΉ, Π½ΠΎ Ρ Π½ΠΈΠΊΠΎΠ³Π΄Π° Π½Π΅ ΡΠ»ΡΡΠ°Π», ΡΡΠΎΠ±Ρ ΠΊΡΠΎ-ΡΠΎ ΠΏΡΠ΅Π΄ΠΏΠΎΠ»Π°Π³Π°Π», ΡΡΠΎ ΡΡΠΎ ΡΠ°ΠΊ. ΠΡ Π³ΠΎΠ²ΠΎΡΠΈΡΠ΅, ΡΡΠΎ ΡΡΠΎ Π½Π΅ ΠΈΠΌΠ΅Π΅Ρ Π½ΠΈΡΠ΅Π³ΠΎ ΠΎΠ±ΡΠ΅Π³ΠΎ Ρ ΡΠΎΡΠΎΠ½Π°ΠΌΠΈ, ΡΠΎ Π΅ΡΡΡ Ρ ΠΊΠ²Π°Π½ΡΠΎΠ²ΠΎΠΉ ΠΌΠ΅Ρ Π°Π½ΠΈΠΊΠΎΠΉ. ΠΠ°ΡΠ° ΠΊΠ°ΡΡΠΈΠ½Π° Π·Π²ΡΡΠΈΡ ΡΠ°ΠΊ, Π±ΡΠ΄ΡΠΎ Π½Π΅ ΡΠΎΠΎΡΠ²Π΅ΡΡΡΠ²ΡΠ΅Ρ ΡΠΏΠ΅ΡΠΈΠ°Π»ΡΠ½ΠΎΠΉ ΡΠ΅ΠΎΡΠΈΠΈ ΠΎΡΠ½ΠΎΡΠΈΡΠ΅Π»ΡΠ½ΠΎΡΡΠΈ.
Π‘ΠΎΡΠ΅ΡΠ°Π½ΠΈΠ΅ ΡΠΏΠ΅ΡΠΈΠ°Π»ΡΠ½ΠΎΠΉ ΡΠ΅ΠΎΡΠΈΠΈ ΠΎΡΠ½ΠΎΡΠΈΡΠ΅Π»ΡΠ½ΠΎΡΡΠΈ ΠΈ ΠΊΠ²Π°Π½ΡΠΎΠ²ΠΎΠΉ ΠΌΠ΅Ρ Π°Π½ΠΈΠΊΠΈ ΠΏΠΎΠ·Π²ΠΎΠ»ΡΠ΅Ρ ΠΏΡΠΎΠ²ΠΎΠ΄ΠΈΡΡ ΡΠ°ΡΡΠ΅ΡΡ Π²Π΅ΡΠ΅ΠΉ, ΠΊΠΎΡΠΎΡΡΠ΅ ΠΌΠΎΠΆΠ½ΠΎ ΠΈΠ·ΠΌΠ΅ΡΠΈΡΡ. ΠΠ°ΠΏΡΠΈΠΌΠ΅Ρ, ΠΎΠ½ Π΄Π°Π΅Ρ ΠΏΡΠ΅Π΄ΡΠΊΠ°Π·Π°Π½ΠΈΠ΅ Π³ΠΈΡΠΎΠΌΠ°Π³Π½ΠΈΡΠ½ΠΎΠ³ΠΎ ΠΎΡΠ½ΠΎΡΠ΅Π½ΠΈΡ ΡΠ»Π΅ΠΊΡΡΠΎΠ½ΠΎΠ². ΠΠΊΡΠΏΠ΅ΡΠΈΠΌΠ΅Π½ΡΠ°Π»ΡΠ½ΠΎ Π·Π½Π°ΡΠ΅Π½ΠΈΠ΅ ΡΠ°Π²Π½ΠΎ 2,00231930462 Ρ Π½Π΅Π±ΠΎΠ»ΡΡΠΎΠΉ ΠΏΠΎΠ³ΡΠ΅ΡΠ½ΠΎΡΡΡΡ Π² ΠΏΠΎΡΠ»Π΅Π΄Π½Π΅ΠΌ Π΄Π΅ΡΡΡΠΈΡΠ½ΠΎΠΌ ΡΠ°Π·ΡΡΠ΄Π΅. «ΠΡΠ΅Π΄ΡΠΊΠ°Π·Π°Π½ΠΈΠ΅ ΠΠΠ ΡΠΎΠ³Π»Π°ΡΡΠ΅ΡΡΡ Ρ ΡΠΊΡΠΏΠ΅ΡΠΈΠΌΠ΅Π½ΡΠ°Π»ΡΠ½ΠΎ ΠΈΠ·ΠΌΠ΅ΡΠ΅Π½Π½ΡΠΌ Π·Π½Π°ΡΠ΅Π½ΠΈΠ΅ΠΌ Π±ΠΎΠ»Π΅Π΅ ΡΠ΅ΠΌ Π½Π° 10Β Π·Π½Π°ΡΠ°ΡΠΈΡ ΡΠΈΡΡ…» Β Β
ΠΠ°ΠΊΠΎΠ΅ Π·Π½Π°ΡΠ΅Π½ΠΈΠ΅ Π΄Π°Π΅Ρ Π²Π°ΡΠ° ΠΌΠΎΠ΄Π΅Π»Ρ?
ΠΠ°ΠΉΠΊ Π.
ΡΡ. Π― Π½Π΅ ΠΌΠΎΠ³Ρ ΡΠ΄Π΅ΡΠΆΠ°ΡΡΡΡ ΠΎΡ ΡΠΎΠ³ΠΎ, ΡΡΠΎΠ±Ρ Π½Π΅ ΠΏΡΠΈΠ²Π΅ΡΡΠΈ ΠΏΠ΅ΡΠ²ΡΠ΅ Π΄Π²Π΅ ΡΡΡΠΎΡΠΊΠΈ ΠΈΠ· ΠΏΡΠΈΠΌΠ΅ΡΠ½ΠΎ 100-ΡΡΡΠ°Π½ΠΈΡΠ½ΠΎΠ³ΠΎ ΠΈΠ½ΡΠ΅ΡΠ²ΡΡ, ΠΊΠΎΡΠΎΡΠΎΠ΅ Π€ΠΎΠ½Π΄ Ρ ΠΈΠΌΠΈΡΠ΅ΡΠΊΠΎΠ³ΠΎ Π½Π°ΡΠ»Π΅Π΄ΠΈΡ ΠΏΡΠΎΠ²Π΅Π» Ρ ΡΡΠ΅Π½ΡΠΌ (ΠΌΠΎΠΉ ΠΎΡΠ΅Ρ), ΠΊΠΎΡΠΎΡΡΠΉ ΡΠ°Π±ΠΎΡΠ°Π» Ρ ΠΌΠ°Π³Π½ΠΈΡΠ°ΠΌΠΈ Π² ΡΠ΅ΡΠ΅Π½ΠΈΠ΅ ~ 9 Π»Π΅Ρ.0 Π»Π΅Ρ. Β«Π Π΅Π±Π΅Π½ΠΊΠΎΠΌ Ρ ΠΎΠ±Π½Π°ΡΡΠΆΠΈΠ», ΡΡΠΎ ΠΌΠΎΠ³Ρ Π·Π°ΡΡΠ°Π²ΠΈΡΡ Π±ΡΠ»Π°Π²ΠΊΠΈ Π² ΠΊΠΎΡΠΎΠ±ΠΊΠ΅ ΡΡΠΎΡΡΡ, ΠΈ, ΠΏΠ΅ΡΠ΅ΠΌΠ΅ΡΠ°Ρ ΠΌΠ°Π³Π½ΠΈΡ, Ρ ΠΌΠΎΠ³Ρ Π·Π°ΡΡΠ°Π²ΠΈΡΡ ΠΈΡ Π΄Π²ΠΈΠ³Π°ΡΡΡΡ. Π― ΠΏΠΎΠ½ΡΡΠΈΡ Π½Π΅ ΠΈΠΌΠ΅Π», ΡΡΠΎ ΡΠ°ΠΊΠΎΠ΅ ΠΌΠ°Π³Π½ΠΈΡΠ½ΠΎΠ΅ ΠΏΠΎΠ»Π΅, ΠΈ ΠΏΠΎΠ΄ΠΎΠ·ΡΠ΅Π²Π°Ρ, ΡΡΠΎ Π΄ΠΎ ΡΠΈΡ ΠΏΠΎΡ ΠΏΠΎΠ½ΡΡΠΈΡ Π½Π΅ ΠΈΠΌΠ΅Ρ, ΡΡΠΎ ΡΠ°ΠΊΠΎΠ΅ ΠΌΠ°Π³Π½ΠΈΡΠ½ΠΎΠ΅ ΠΏΠΎΠ»Π΅. ΠΌΠ°Π³Π½ΠΈΡΠ½ΠΎΠ΅ ΠΏΠΎΠ»Π΅ Π΅ΡΡΡ, Π·Π° ΠΈΡΠΊΠ»ΡΡΠ΅Π½ΠΈΠ΅ΠΌ Π½Π΅ΠΊΠΎΡΠΎΡΡΡ Π²Π΅ΡΠ΅ΠΉ, ΠΊΠΎΡΠΎΡΡΠ΅ ΠΎΠ½ΠΎ Π΄Π΅Π»Π°Π΅ΡΒ».
(ΠΎΠΏΡΠ±Π»ΠΈΠΊΠΎΠ²Π°Π½ΠΎ 04.09.2013)
ΠΠΎΠΏΠΎΠ»Π½Π΅Π½ΠΈΠ΅ β3: ΡΠ»Π΅ΠΊΡΡΠΈΡΠ΅ΡΠΊΠΈΠ΅ ΠΏΠΎΠ»Ρ ΠΊΠ°ΠΊ Π²ΠΈΡΡΡΠ°Π»ΡΠ½ΡΠ΅ ΡΠΎΡΠΎΠ½Π½ΡΠ΅ ΠΎΠ±Π»Π°ΠΊΠ°
ΠΠΎΠΏΡΠΎΡ:
ΠΠ°ΠΆΠ΅ΡΡΡ, ΡΡΠΎ Π²ΠΈΡΡΡΠ°Π»ΡΠ½ΠΎΠ΅ ΡΠΎΡΠΎΠ½Π½ΠΎΠ΅ ΠΎΠ±Π»Π°ΠΊΠΎ ΡΡΡΠ΅ΡΡΠ²ΡΠ΅Ρ ΡΠΎΠ»ΡΠΊΠΎ ΠΎΡΠ΅Π½Ρ Π±Π»ΠΈΠ·ΠΊΠΎ ΠΊ ΡΡΠ°ΡΠΈΡΠ΅ΡΠΊΠΎΠΌΡ Π·Π°ΡΡΠ΄Ρ. ΠΠ°ΠΊ ΠΌΡ ΠΏΡΠΈΠΌΠΈΡΠΈΠΌ ΡΡΠΎ Ρ (Π°) ΡΠ΅ΠΌ ΡΠ°ΠΊΡΠΎΠΌ, ΡΡΠΎ ΡΠΈΠ»ΠΎΠ²ΡΠ΅ Π»ΠΈΠ½ΠΈΠΈ ΡΠ»Π΅ΠΊΡΡΠΈΡΠ΅ΡΠΊΠΎΠ³ΠΎ ΠΏΠΎΠ»Ρ ΡΡΠ½ΡΡΡΡ Π²Π΅ΡΠ½ΠΎ ΠΈ (Π±) ΡΠ΅ΠΌ ΡΠ°ΠΊΡΠΎΠΌ, ΡΡΠΎ Π²ΠΈΡΡΡΠ°Π»ΡΠ½ΡΠ΅ ΡΠΎΡΠΎΠ½Ρ ΠΎΠ±ΠΌΠ΅Π½ΠΈΠ²Π°ΡΡΡΡ Π½Π° ΡΠ°ΡΡΡΠΎΡΠ½ΠΈΡΡ
, ΠΏΡΠ΅Π²ΡΡΠ°ΡΡΠΈΡ
ΡΠ°ΡΡΡΠΎΡΠ½ΠΈΠ΅ Π²ΠΈΡΡΡΠ°Π»ΡΠ½ΠΎΠ³ΠΎ ΡΠΎΡΠΎΠ½Π½ΠΎΠ³ΠΎ ΠΎΠ±Π»Π°ΠΊΠ°? Π― ΠΏΡΠ΅Π΄ΡΡΠ°Π²Π»ΡΡ ΡΠ΅Π±Π΅ Π²ΠΈΡΡΡΠ°Π»ΡΠ½ΠΎΠ΅ ΡΠΎΡΠΎΠ½Π½ΠΎΠ΅ ΠΎΠ±Π»Π°ΠΊΠΎ, ΠΎΠΊΡΡΠΆΠ°ΡΡΠ΅Π΅ ΠΈΠ·ΠΎΠ»ΠΈΡΠΎΠ²Π°Π½Π½ΡΠΉ ΡΠ»Π΅ΠΊΡΡΠΎΠ½. ΡΠ΄Π°ΡΠ°. ΠΠ½Π΅Π·Π°ΠΏΠ½ΠΎ ΠΏΡΠΎΡΠΎΠ½ ΠΎΠΊΠ°Π·ΡΠ²Π°Π΅ΡΡΡ ΡΡΠ΄ΠΎΠΌ Ρ ΡΠ»Π΅ΠΊΡΡΠΎΠ½ΠΎΠΌ. ΠΡΠ΅ΠΆΠ΄Π΅ ΡΠ΅ΠΌ ΠΏΡΠΎΡΠΎΠ½ ΠΏΡΠΈΠ±ΡΠ΄Π΅Ρ Π½Π° ΡΡΠ΅Π½Ρ, Π²ΠΈΡΡΡΠ°Π»ΡΠ½ΠΎΠ΅ ΡΠΎΡΠΎΠ½Π½ΠΎΠ΅ ΠΎΠ±Π»Π°ΠΊΠΎ ΠΏΠ»ΠΎΡΠ½ΠΎ ΡΠΏΠ°ΠΊΠΎΠ²ΡΠ²Π°Π΅ΡΡΡ Π²ΠΎΠΊΡΡΠ³ ΡΠ»Π΅ΠΊΡΡΠΎΠ½Π°. (ΠΠΈΡΡΡΠ°Π»ΡΠ½ΠΎΠ΅ ΡΠΎΡΠΎΠ½Π½ΠΎΠ΅ ΠΎΠ±Π»Π°ΠΊΠΎ ΠΏΡΠ΅Π΄ΡΡΠ°Π²Π»ΡΠ΅Ρ ΡΠΎΠ±ΠΎΠΉ ΡΡΠ°ΡΠΈΡΠ΅ΡΠΊΠΎΠ΅ ΠΊΠ²Π°Π½ΡΠΎΠ²ΠΎΠ΅ ΡΠΎΡΡΠΎΡΠ½ΠΈΠ΅, Π½ΠΎ ΡΡΠΎ Π½Π΅ ΠΎΠ·Π½Π°ΡΠ°Π΅Ρ, ΡΡΠΎ Π²ΠΈΡΡΡΠ°Π»ΡΠ½ΡΠ΅ ΡΠΎΡΠΎΠ½Ρ Π½Π΅ Π΄Π²ΠΈΠΆΡΡΡΡ β ΠΏΠΎΠ΄ΠΎΠ±Π½ΠΎ ΡΠ»Π΅ΠΊΡΡΠΎΠ½Π½ΠΎΠΌΡ ΠΎΠ±Π»Π°ΠΊΡ Π²ΠΎΠΊΡΡΠ³ Π°ΡΠΎΠΌΠ° Π²ΠΎΠ΄ΠΎΡΠΎΠ΄Π°). Π²ΠΈΡΡΡΠ°Π»ΡΠ½ΠΎΠ΅ ΡΠΎΡΠΎΠ½Π½ΠΎΠ΅ ΠΎΠ±Π»Π°ΠΊΠΎ ΠΌΠ΅Π½ΡΠ΅Ρ ΡΠΎΡΠΌΡ? ΠΠ°ΠΏΡΠΈΠΌΠ΅Ρ, Β«ΡΠ°ΡΡΡΠ³ΠΈΠ²Π°Π΅ΡΡΡ Π»ΠΈΒ» Π²ΠΈΡΡΡΠ°Π»ΡΠ½ΠΎΠ΅ ΡΠΎΡΠΎΠ½Π½ΠΎΠ΅ ΠΎΠ±Π»Π°ΠΊΠΎ, ΡΡΠΎΠ±Ρ Π΄ΠΎΡΡΠΈΡΡ ΠΏΡΠΎΡΠΎΠ½Π°, ΠΏΡΠ΅Π΄ΡΡΠ°Π²Π»ΡΡ ΠΏΠΎΠ²ΡΡΠ΅Π½Π½ΠΎΠ΅ ΠΏΡΠΈΡΡΡΡΡΠ²ΠΈΠ΅ Π²ΠΈΡΡΡΠ°Π»ΡΠ½ΡΡ
ΡΠΎΡΠΎΠ½ΠΎΠ² Π½Π° ΠΏΡΡΠΈ, Π³Π΄Π΅ ΠΎΠ½ΠΈ ΠΎΠ±ΠΌΠ΅Π½ΠΈΠ²Π°ΡΡΡΡ ΠΌΠ΅ΠΆΠ΄Ρ Π΄Π²ΡΠΌΡ Π·Π°ΡΡΠΆΠ΅Π½Π½ΡΠΌΠΈ ΡΠ°ΡΡΠΈΡΠ°ΠΌΠΈ?
— JD (29 Π»Π΅Ρ)
ΠΡΠΈΡΠ²ΠΈΠ»Π», ΠΠ΅Π½ΡΡΠΊΠΊΠΈ, Π‘Π¨Π
A:
ΠΠ»Π°ΡΡΠΈΡΠ΅ΡΠΊΠΎΠ΅ Π²ΡΡΠ°ΠΆΠ΅Π½ΠΈΠ΅ ΠΏΠΎΠ»Ρ Π³ΠΎΠ²ΠΎΡΠΈΡ Π²Π°ΠΌ, ΠΊΠ°ΠΊ ΡΠ°ΡΠΏΡΠΎΡΡΡΠ°Π½ΡΠ΅ΡΡΡ Π²ΠΈΡΡΡΠ°Π»ΡΠ½ΠΎΠ΅ ΡΠΎΡΠΎΠ½Π½ΠΎΠ΅ ΠΎΠ±Π»Π°ΠΊΠΎ. Π’Π°ΠΊΠΈΠΌ ΠΎΠ±ΡΠ°Π·ΠΎΠΌ, ΡΡΠΈ Π²ΠΈΡΡΡΠ°Π»ΡΠ½ΡΠ΅ ΡΠΎΡΠΎΠ½Ρ Π½Π΅ Π±ΠΎΠ»Π΅Π΅ ΠΈ Π½Π΅ ΠΌΠ΅Π½Π΅Π΅ ΡΠΊΠΎΠ½ΡΠ΅Π½ΡΡΠΈΡΠΎΠ²Π°Π½Ρ Π½Π° Π·Π°ΡΡΠ΄Π΅, ΡΠ΅ΠΌ ΠΊΠ»Π°ΡΡΠΈΡΠ΅ΡΠΊΠΈΠ΅ ΠΏΠΎΠ»Ρ. ΠΠ·ΠΌΠ΅Π½Π΅Π½ΠΈΡ ΡΠΎΡΠΌΡ, ΠΊΠΎΠ³Π΄Π° ΠΏΡΠΈΡΡΡΡΡΠ²ΡΡΡ Π΄Π²Π°Β Π·Π°ΡΡΠ΄Π°, ΠΎΠΏΡΠ΅Π΄Π΅Π»ΡΡΡΡΡ ΡΡΠΌΠΌΠΈΡΠΎΠ²Π°Π½ΠΈΠ΅ΠΌ ΠΊΠ»Π°ΡΡΠΈΡΠ΅ΡΠΊΠΈΡ Π²Π΅ΠΊΡΠΎΡΠ½ΡΡ ΠΏΠΎΠ»Π΅ΠΉ. Π’Π°ΠΊ ΡΡΠΎ Π΄Π°, ΠΌΠ΅ΠΆΠ΄Ρ Π·Π°ΡΡΠ΄Π°ΠΌΠΈ ΡΡΡΠ΅ΡΡΠ²ΡΠ΅Ρ ΠΎΡΠΎΠ±Π΅Π½Π½ΠΎ ΡΠΈΠ»ΡΠ½ΠΎΠ΅ ΠΏΠΎΠ»Π΅, Π½ΠΎ Π΄Π°Π»Π΅ΠΊΠΎ ΠΎΡ Π½ΠΈΡ ΠΏΠΎΠ»Ρ ΠΈΠΌΠ΅ΡΡ ΡΠ΅Π½Π΄Π΅Π½ΡΠΈΡ Π³Π°ΡΠΈΡΡΡΡ.
ΠΠ°ΠΉΠΊ Π.
(ΠΎΠΏΡΠ±Π»ΠΈΠΊΠΎΠ²Π°Π½ΠΎ 11.04.2015)
ΠΠΎΠΏΠΎΠ»Π½Π΅Π½ΠΈΠ΅ β4: ΡΠΎΡΠΎΠ½Ρ ΠΈ ΠΌΠ°Π³Π½ΠΈΡΠ½ΡΠ΅ ΠΏΠΎΠ»Ρ
ΠΠΎΠΏΡΠΎΡ:
ΠΠ°Π³Π½ΠΈΡΠ½ΡΠ΅ ΠΏΠΎΠ»Ρ Π½Π΅Π»ΡΠ·Ρ ΠΎΠ±ΡΡΡΠ½ΠΈΡΡ, ΠΏΡΠΎΡΡΠΎ ΡΠΊΠ°Π·Π°Π², ΡΡΠΎ ΠΎΠ½ΠΈ ΡΠΎΡΡΠΎΡΡ ΠΈΠ· Β«ΡΠΎΡΠΎΠ½ΠΎΠ²Β».
ΠΠ°ΠΊΠ°Ρ Π΄Π°ΡΠ° ΠΈΡ
ΡΠΈΠ·ΠΈΡΠ΅ΡΠΊΠΎΠ³ΠΎ ΠΌΠ°ΠΊΠΈΡΠΆΠ°?
— Π ΠΎΠ±Π΅ΡΡ ΠΠΎΠ½ΡΠ΅ (69 Π»Π΅Ρ)
Port Hueneme
A:
Π ΠΎΠ΄Π½ΠΎΠΌ ΡΠΌΡΡΠ»Π΅ Π²Ρ ΠΏΡΠ°Π²Ρ, ΡΡΠΎ Π½Π΅ Π΄ΠΎΠ»ΠΆΠ½Ρ Π΄ΡΠΌΠ°ΡΡ, ΡΡΠΎ ΠΌΠ°Π³Π½ΠΈΡΠ½ΡΠ΅ ΠΏΠΎΠ»Ρ ΡΠΎΡΡΠΎΡΡ ΠΈΠ· ΡΠΎΡΠΎΠ½ΠΎΠ². ΠΡΠ»ΠΈ Ρ Π²Π°Ρ Π΅ΡΡΡ ΠΎΠΏΡΠ΅Π΄Π΅Π»Π΅Π½Π½ΠΎΠ΅ ΠΊΠΎΠ»ΠΈΡΠ΅ΡΡΠ²ΠΎ ΡΠΎΡΠΎΠ½ΠΎΠ² ΠΊΠ°ΠΆΠ΄ΠΎΠ³ΠΎ ΡΠΈΠΏΠ°, ΠΎΠΆΠΈΠ΄Π°Π΅ΠΌΠΎΠ΅ ΠΌΠ°Π³Π½ΠΈΡΠ½ΠΎΠ΅ ΠΏΠΎΠ»Π΅ ΡΠ°Π²Π½ΠΎ Π½ΡΠ»Ρ. Π§ΡΠΎΠ±Ρ ΠΏΠΎΠ»ΡΡΠΈΡΡ ΡΡΠΎ-ΡΠΎ Π²ΡΠΎΠ΄Π΅ ΠΊΠ»Π°ΡΡΠΈΡΠ΅ΡΠΊΠΎΠ³ΠΎ ΡΠ΅ΡΠΊΠΎ ΠΎΠΏΡΠ΅Π΄Π΅Π»Π΅Π½Π½ΠΎΠ³ΠΎ ΠΌΠ°Π³Π½ΠΈΡΠ½ΠΎΠ³ΠΎ ΠΏΠΎΠ»Ρ, Π²Π°ΠΌ Π½ΡΠΆΠ΅Π½ Π±ΠΎΠ»ΡΡΠΎΠΉ ΡΠ°Π·Π±ΡΠΎΡ Π²ΠΎΠ·ΠΌΠΎΠΆΠ½ΡΡ ΡΠΈΡΠ΅Π» ΡΠΎΡΠΎΠ½ΠΎΠ².* ΠΡΠΎ ΡΠΈΠ»ΡΠ½ΠΎ ΠΎΡΠ»ΠΈΡΠ°Π΅ΡΡΡ ΠΎΡ ΠΏΡΠ΅Π΄ΠΏΠΎΠ»ΠΎΠΆΠ΅Π½ΠΈΡ, ΡΡΠΎ ΠΊΠ°ΠΆΠ΄ΡΠΉ ΡΠΎΡΠΎΠ½ Π²Π½ΠΎΡΠΈΡ ΠΊΠ°ΠΊΠΎΠ΅-ΡΠΎ ΠΏΠΎΠ»Π΅.
Π‘ Π΄ΡΡΠ³ΠΎΠΉ ΡΡΠΎΡΠΎΠ½Ρ, Π²ΠΎΠ·ΠΌΠΎΠΆΠ½ΠΎ, Π²Ρ Π΄ΡΠΌΠ°Π΅ΡΠ΅, ΡΡΠΎ ΠΏΠΎΠΌΠΈΠΌΠΎ ΡΠΎΡΠΎΠ½ΠΎΠ² Π΅ΡΡΡ Π΅ΡΠ΅ ΠΊΠ°ΠΊΠΎΠΉ-ΡΠΎ ΠΈΠ½Π³ΡΠ΅Π΄ΠΈΠ΅Π½Ρ. ΠΠ΅Ρ.
ΠΠ°ΠΉΠΊ Π.
*ΠΡΠ»ΠΈ Π²Ρ ΠΏΠΎΡΠΌΠΎΡΡΠΈΡΠ΅, Π½Π°ΠΏΡΠΈΠΌΠ΅Ρ, Π½Π° ΡΡΠ°Π²Π½Π΅Π½ΠΈΠ΅Β 21 Π² ΡΡΠΎΠΉ ΡΡΠ°ΡΡΠ΅ (https://www.phys.ksu.edu/personal/wysin/notes/quantumEM.pdf), Π²Ρ Π½Π°ΠΉΠ΄Π΅ΡΠ΅ Π²ΡΡΠ°ΠΆΠ΅Π½ΠΈΠ΅ Π΄Π»Ρ ΠΌΠ°Π³Π½ΠΈΡΠ½ΠΎΠ΅ ΠΏΠΎΠ»Π΅ Π² ΡΠ΅ΡΠΌΠΈΠ½Π°Ρ ΠΎΠΏΠ΅ΡΠ°ΡΠΎΡΠΎΠ² ΡΠΎΠΆΠ΄Π΅Π½ΠΈΡ ΠΈ ΡΠ½ΠΈΡΡΠΎΠΆΠ΅Π½ΠΈΡ ΡΠΎΡΠΎΠ½ΠΎΠ².
(ΠΎΠΏΡΠ±Π»ΠΈΠΊΠΎΠ²Π°Π½ΠΎ 13.12.2015)
ΠΠΎΠΏΠΎΠ»Π½Π΅Π½ΠΈΠ΅ β 5: ΡΠΎΡΠΎΠ½Ρ ΠΈ ΠΌΠ°Π³Π½Π΅ΡΠΈΠ·ΠΌ ΠΏΠΎΠ»Π΅ ΡΠ°Π²Π½ΠΎ Π½ΡΠ»Ρ. Π§ΡΠΎΠ±Ρ ΠΏΠΎΠ»ΡΡΠΈΡΡ ΡΡΠΎ-ΡΠΎ Π²ΡΠΎΠ΄Π΅ ΠΊΠ»Π°ΡΡΠΈΡΠ΅ΡΠΊΠΎΠ³ΠΎ ΡΠ΅ΡΠΊΠΎ ΠΎΠΏΡΠ΅Π΄Π΅Π»Π΅Π½Π½ΠΎΠ³ΠΎ ΠΌΠ°Π³Π½ΠΈΡΠ½ΠΎΠ³ΠΎ ΠΏΠΎΠ»Ρ, Π²Π°ΠΌ Π½ΡΠΆΠ΅Π½ Π±ΠΎΠ»ΡΡΠΎΠΉ ΡΠ°Π·Π±ΡΠΎΡ Π²ΠΎΠ·ΠΌΠΎΠΆΠ½ΡΡ ΡΠΈΡΠ΅Π» ΡΠΎΡΠΎΠ½ΠΎΠ²Β». ΠΌΠ΅Ρ Π°Π½ΠΈΠΊΠ°. ΠΡΠ° ΡΠ΅ΠΌΠ° ΡΠ±ΠΈΠ²Π°Π΅Ρ ΠΌΠ΅Π½Ρ Ρ ΡΠΎΠ»ΠΊΡ, ΠΏΠΎΡΠΎΠΌΡ ΡΡΠΎ Ρ Π΄ΡΠΌΠ°Ρ, ΡΡΠΎ Ρ ΠΌΠ΅Π½Ρ Π΅ΡΡΡ Π±Π°Π·ΠΎΠ²ΠΎΠ΅ ΠΏΠΎΠ½ΠΈΠΌΠ°Π½ΠΈΠ΅ Β«ΠΎΠ±ΡΡΠ½ΡΡ Β» ΡΠΎΡΠΎΠ½ΠΎΠ² (Π½Π°ΠΏΡΠΈΠΌΠ΅Ρ, Π²ΠΈΠ΄ΠΈΠΌΠΎΠ³ΠΎ ΡΠ²Π΅ΡΠ°), Π½ΠΎ ΡΠ°ΠΊΠΈΠ΅ ΡΠΎΡΠΎΠ½Ρ ΠΈΠΌΠ΅ΡΡ ΠΎΠΏΡΠ΅Π΄Π΅Π»Π΅Π½Π½ΡΠ΅ ΡΠ½Π΅ΡΠ³ΠΈΠΈ ΠΈ Π΄Π»ΠΈΠ½Ρ Π²ΠΎΠ»Π½, ΠΈ ΠΎΠ½ΠΈ Π½Π΅ Π΄Π²ΠΈΠΆΡΡΡΡ ΠΏΠΎ Π·Π°ΠΌΠΊΠ½ΡΡΠΎΠΌΡ ΠΊΠΎΠ½ΡΡΡΡ, ΠΊΠ°ΠΊ ΠΌΠ°Π³Π½ΠΈΡΠ½ΠΎΠ΅ ΠΏΠΎΠ»Π΅. ΠΊ. Β«ΠΠ°Π³Π½ΠΈΡΠ½ΡΠΉΒ» ΡΠΎΡΠΎΠ½ ΠΊΠ°ΠΆΠ΅ΡΡΡ Π΄ΡΡΠ³ΠΈΠΌ, ΡΠ΅ΠΌ Β«ΠΎΠ±ΡΡΠ½ΡΠΉΒ» ΡΠΎΡΠΎΠ½, ΠΈ Ρ Ρ ΠΎΡΠ΅Π» Π±Ρ ΠΏΠΎΠ½ΡΡΡ, ΠΏΠΎΡΠ΅ΠΌΡ, Π΅ΡΠ»ΠΈ ΡΡΠΎ Π²ΠΎΠ·ΠΌΠΎΠΆΠ½ΠΎ Π±Π΅Π· ΠΏΠΎΠ½ΠΈΠΌΠ°Π½ΠΈΡ ΠΌΠ°ΡΠ΅ΠΌΠ°ΡΠΈΠΊΠΈ. Π‘ΠΏΠ°ΡΠΈΠ±ΠΎ!
— ΠΠΆΠ΅ΠΉΠΌΡ (30 Π»Π΅Ρ)
A:
Π― Π½Π΅ ΡΠΌΠΎΠ³Ρ ΡΡΠΎ ΠΎΠ±ΡΡΡΠ½ΠΈΡΡ, Π½ΠΎ ΠΌΠΎΠ³Ρ ΠΏΡΠΎΡΡΠ½ΠΈΡΡ Π½Π΅ΠΊΠΎΡΠΎΡΡΠ΅ ΠΌΠΎΠΌΠ΅Π½ΡΡ.
Π€ΠΎΡΠΎΠ½Ρ, Π²Π½ΠΎΡΡΡΠΈΠ΅ Π²ΠΊΠ»Π°Π΄ Π² ΠΌΠ°Π³Π½ΠΈΡΠ½ΡΠ΅ ΠΏΠΎΠ»ΡΒ , Π½ΠΈΡΠ΅ΠΌ Π½Π΅ ΠΎΡΠ»ΠΈΡΠ°ΡΡΡΡ ΠΎΡ ΡΠΎΡΠΎΠ½ΠΎΠ², Π²Π½ΠΎΡΡΡΠΈΡ Π²ΠΊΠ»Π°Π΄ Π² ΡΠ»Π΅ΠΊΡΡΠΈΡΠ΅ΡΠΊΠΈΠ΅ ΠΏΠΎΠ»Ρ. ΠΠΎΠ½ΠΊΡΠ΅ΡΠ½ΡΠΉ ΠΎΠ±ΡΠ°Π·Π΅Ρ ΡΠΎΠ³ΠΎ, ΠΊΠ°ΠΊΠΈΠ΅ ΡΠ°Π·Ρ ΠΏΡΠΈΡΡΡΡΡΠ²ΡΡΡ Π΄Π»Ρ ΡΠ°Π·Π½ΠΎΠ³ΠΎ ΠΊΠΎΠ»ΠΈΡΠ΅ΡΡΠ²Π° ΡΠΎΡΠΎΠ½ΠΎΠ², ΠΎΠΏΡΠ΅Π΄Π΅Π»ΡΠ΅Ρ, ΠΊΠ°ΠΊΠΈΠ΅ ΠΊΠ»Π°ΡΡΠΈΡΠ΅ΡΠΊΠΈΠ΅ ΠΏΠΎΠ»Ρ ΠΏΡΠΈΡΡΡΡΡΠ²ΡΡΡ.
Π§ΡΠΎ ΠΊΠ°ΡΠ°Π΅ΡΡΡ ΠΎΡΠ½ΠΎΡΠ΅Π½ΠΈΡ ΠΊΠΎΠ»ΠΈΡΠ΅ΡΡΠ²Π° ΡΠΎΡΠΎΠ½ΠΎΠ² ΠΊ ΠΊΠ»Π°ΡΡΠΈΡΠ΅ΡΠΊΠΈΠΌ ΠΏΠΎΠ»ΡΠΌ, Ρ ΠΌΠΎΠ³Ρ ΠΏΡΠ΅Π΄Π»ΠΎΠΆΠΈΡΡ Π°Π½Π°Π»ΠΎΠ³ΠΈΡ, ΠΊΠΎΡΠΎΡΡΡ Π²Ρ ΠΌΠΎΠ³Π»ΠΈ Π±Ρ ΠΈΠ·ΡΡΠΈΡΡ, ΠΈ ΠΊΠΎΡΠΎΡΡΡ Π±ΡΠ»ΠΎ Π±Ρ Π»Π΅Π³ΡΠ΅ ΠΈΠ·ΠΎΠ±ΡΠ°Π·ΠΈΡΡ. ΠΠΎΡΠΌΠΎΡΡΠΈΡΠ΅ Π½Π° Π²ΠΎΠ»Π½ΠΎΠ²ΡΠ΅ ΡΡΠ½ΠΊΡΠΈΠΈ, ΠΊΠΎΡΠΎΡΡΠ΅ ΠΏΡΠ΅Π΄ΡΡΠ°Π²Π»ΡΡΡ ΡΠΎΡΡΠΎΡΠ½ΠΈΡ ΠΏΡΠΎΡΡΠΎΠ³ΠΎ Π³Π°ΡΠΌΠΎΠ½ΠΈΡΠ΅ΡΠΊΠΎΠ³ΠΎ ΠΎΡΡΠΈΠ»Π»ΡΡΠΎΡΠ° (ΠΌΠ°ΡΡΠ° Π½Π° ΠΏΡΡΠΆΠΈΠ½Π΅). (https://en.wikipedia.org/wiki/Quantum_harmonic_oscillator) Π’Π΅, Ρ ΠΊΠΎΠ³ΠΎ ΠΎΠΏΡΠ΅Π΄Π΅Π»Π΅Π½Π½Π°Ρ ΡΠ½Π΅ΡΠ³ΠΈΡ, ΠΏΠΎΠ΄ΠΎΠ±Π½Ρ ΡΠΎΡΡΠΎΡΠ½ΠΈΡΠΌ ΡΠΎΡΠΎΠ½ΠΎΠ² Ρ ΠΎΠΏΡΠ΅Π΄Π΅Π»Π΅Π½Π½ΡΠΌΠΈ ΡΠΈΡΠ»Π°ΠΌΠΈ ΠΈ, ΡΠ»Π΅Π΄ΠΎΠ²Π°ΡΠ΅Π»ΡΠ½ΠΎ, ΠΎΠΏΡΠ΅Π΄Π΅Π»Π΅Π½Π½ΡΠΌΠΈ ΡΠ½Π΅ΡΠ³ΠΈΡΠΌΠΈ. ΠΠ΄Π½ΠΎΡΠ½Π΅ΡΠ³Π΅ΡΠΈΡΠ΅ΡΠΊΠΈΠ΅ ΡΠΎΡΡΠΎΡΠ½ΠΈΡ SHO Π²ΡΠ΅Π³Π΄Π° ΡΠ°Π²Π½ΠΎΠΌΠ΅ΡΠ½ΠΎ ΡΠ°ΡΠΏΡΠ΅Π΄Π΅Π»Π΅Π½Ρ Π²ΠΎΠΊΡΡΠ³ ΡΡΠ΅Π΄Π½Π΅ΠΉ ΡΠΎΡΠΊΠΈ Π±Π΅Π· ΡΡΠ΅Π΄Π½Π΅Π³ΠΎ ΡΠΌΠ΅ΡΠ΅Π½ΠΈΡ ΠΈ ΡΡΠ΅Π΄Π½Π΅ΠΉ ΡΠΊΠΎΡΠΎΡΡΠΈ. ΠΠΎ ΡΠΎΡΠ½ΠΎΠΉ Π°Π½Π°Π»ΠΎΠ³ΠΈΠΈ, ΡΠΎΡΠΎΠ½Π½ΡΠ΅ ΡΠΎΡΡΠΎΡΠ½ΠΈΡ Ρ ΠΎΠΏΡΠ΅Π΄Π΅Π»Π΅Π½Π½ΡΠΌ ΡΠΈΡΠ»ΠΎΠΌ Π½Π΅ ΠΈΠΌΠ΅ΡΡ ΡΡΠ΅Π΄Π½Π΅Π³ΠΎ ΠΌΠ°Π³Π½ΠΈΡΠ½ΠΎΠ³ΠΎ ΠΏΠΎΠ»Ρ ΠΈΠ»ΠΈ ΡΠ»Π΅ΠΊΡΡΠΈΡΠ΅ΡΠΊΠΎΠ³ΠΎ ΠΏΠΎΠ»Ρ. Π§ΡΠΎΠ±Ρ Π·Π°ΡΡΠ°Π²ΠΈΡΡ SHO Π²ΡΠ°ΡΠ°ΡΡΡΡ ΠΊΠ»Π°ΡΡΠΈΡΠ΅ΡΠΊΠΈ, Π²Π°ΠΌ Π½ΡΠΆΠ½ΠΎ ΡΠΎΠ·Π΄Π°ΡΡ ΡΠΎΡΡΠΎΡΠ½ΠΈΡ, Π² ΠΊΠΎΡΠΎΡΡΡ ΠΈΠ½ΡΠ΅ΡΡΠ΅ΡΠ΅Π½ΡΠΈΡ ΠΌΠ΅ΠΆΠ΄Ρ ΡΠ°ΡΡΡΠΌΠΈ Ρ ΡΠ°Π·Π½ΡΠΌΠΈ ΡΠ½Π΅ΡΠ³ΠΈΡΠΌΠΈ Π²ΡΠ·ΡΠ²Π°Π΅Ρ ΠΎΡΠΌΠ΅Π½Ρ Π² ΠΎΠ΄Π½ΠΈΡ ΡΠ°ΡΡΡΡ ΠΈ ΡΡΠΈΠ»Π΅Π½ΠΈΠ΅ Π² Π΄ΡΡΠ³ΠΈΡ . ΠΠΎ ΠΌΠ΅ΡΠ΅ ΡΠΎΠ³ΠΎ, ΠΊΠ°ΠΊ Π²ΠΎΠ»Π½Π° ΠΈΠ·ΠΌΠ΅Π½ΡΠ΅ΡΡΡ Π²ΠΎ Π²ΡΠ΅ΠΌΠ΅Π½ΠΈ, ΠΏΠΎΠ»ΠΎΠΆΠ΅Π½ΠΈΡ ΡΡΠΈΡ ΡΠΎΡΠ΅ΠΊ Ρ Π½ΠΈΠ·ΠΊΠΎΠΉ ΠΈ Π²ΡΡΠΎΠΊΠΎΠΉ ΠΏΠ»ΠΎΡΠ½ΠΎΡΡΡΡ ΠΊΠΎΠ»Π΅Π±Π»ΡΡΡΡ Π²Π·Π°Π΄ ΠΈ Π²ΠΏΠ΅ΡΠ΅Π΄, ΡΡΠΎ ΠΎΠ·Π½Π°ΡΠ°Π΅Ρ ΠΈΠ·ΠΌΠ΅Π½Π΅Π½ΠΈΠ΅ ΠΏΠΎΠ»ΠΎΠΆΠ΅Π½ΠΈΡ ΠΈ ΡΠΊΠΎΡΠΎΡΡΠΈ. ΠΠ½Π°Π»ΠΎΠ³ΠΎΠΌ Π΄Π»Ρ ΡΠΎΡΠΎΠ½ΠΎΠ² ΡΠ²Π»ΡΠ΅ΡΡΡ ΠΈΠ·ΠΌΠ΅Π½Π΅Π½ΠΈΠ΅ ΠΏΠΎΠ»Π΅ΠΉ.
ΠΠ°ΠΉΠΊ Π.
(ΠΎΠΏΡΠ±Π»ΠΈΠΊΠΎΠ²Π°Π½ΠΎ 16.01.2017)
ΠΠΎΠΏΠΎΠ»Π½Π΅Π½ΠΈΠ΅ β6: ΠΈΠ½Π³ΡΠ΅Π΄ΠΈΠ΅Π½ΡΡ ΠΌΠ°Π³Π½Π΅ΡΠΈΠ·ΠΌΠ°?
Q:
ΠΠ°ΠΉΠΊ Π. ΡΠΊΠ°Π·Π°Π»: «Π‘ Π΄ΡΡΠ³ΠΎΠΉ ΡΡΠΎΡΠΎΠ½Ρ, ΠΌΠΎΠΆΠ΅Ρ Π±ΡΡΡ, Π²Ρ Π΄ΡΠΌΠ°Π΅ΡΠ΅, ΡΡΠΎ ΠΊΡΠΎΠΌΠ΅ ΡΠΎΡΠΎΠ½ΠΎΠ² Π΅ΡΡΡ ΠΊΠ°ΠΊΠΎΠΉ-ΡΠΎ Π΄ΡΡΠ³ΠΎΠΉ ΠΈΠ½Π³ΡΠ΅Π΄ΠΈΠ΅Π½Ρ. ΠΠ΅Ρ, ΠΌΠ°Π»ΡΡΠΈΠΊ! ΠΡΠΎ ΡΠΌΠ΅Π»ΠΎΠ΅ Π·Π°ΡΠ²Π»Π΅Π½ΠΈΠ΅. ΠΠΎΠ½Π΅ΡΠ½ΠΎ, Π²Ρ ΠΈΠΌΠ΅Π΅ΡΠ΅ Π² Π²ΠΈΠ΄Ρ Β«ΠΌΡ Π½Π΅ Π·Π½Π°Π΅ΠΌ Π½ΠΈ ΠΎ ΠΊΠ°ΠΊΠΎΠΌ Π΄ΡΡΠ³ΠΎΠΌ ΠΈΠ½Π³ΡΠ΅Π΄ΠΈΠ΅Π½ΡΠ΅Β» β ΠΊΠ°ΠΊ ΠΌΡ Π½ΠΈΡΠ΅Π³ΠΎ Π½Π΅ Π·Π½Π°Π΅ΠΌ ΠΎ ΡΠ΅ΠΌΠ½ΠΎΠΉ ΡΠ½Π΅ΡΠ³ΠΈΠΈ ΠΈ ΠΎΡΠ΅Π½Ρ ΠΌΠ°Π»ΠΎ ΠΎ ΡΠ΅ΠΌΠ½ΠΎΠΉ ΠΌΠ°ΡΠ΅ΡΠΈΠΈ. ΠΠ½ΡΠ΅ΡΠ΅ΡΠ½ΠΎ, ΡΡΠΎ Π±Ρ ΠΎΠ½ ΡΠΊΠ°Π·Π°Π» Π΄ΠΎ ΠΏΠΎΡΡΡΠ»ΠΈΡΠΎΠ²Π°Π½ΠΈΡ ΡΠΎΡΠΎΠ½ΠΎΠ². ΠΡΠΎΡΠΈΡΠ°Π² Π²ΡΡ Π΄ΠΈΡΠΊΡΡΡΠΈΡ, ΠΌΠ½Π΅ ΠΊΠ°ΠΆΠ΅ΡΡΡ, ΡΡΠΎ Π΅Π΄ΠΈΠ½ΡΡΠ²Π΅Π½Π½ΡΠΉ Π²Π΅ΡΠ½ΡΠΉ ΠΎΡΠ²Π΅Ρ: Β«ΠΡ Π½Π΅ Π·Π½Π°Π΅ΠΌΒ». Π‘ΠΊΠΎΠ»ΡΠΊΠΎ Π°ΡΠΎΠΌΠΎΠ²/ΡΠΌ Π² ΠΌΠ΅ΠΆΠ³Π°Π»Π°ΠΊΡΠΈΡΠ΅ΡΠΊΠΎΠΌ ΠΏΡΠΎΡΡΡΠ°Π½ΡΡΠ²Π΅? ΠΠ΅ ΡΠ°ΠΊ ΡΠΆ ΠΈ ΠΌΠ½ΠΎΠ³ΠΎ, Π³Π°ΡΠ°Π½ΡΠΈΡΡΡ, ΠΏΠΎΡΡΠΎΠΌΡ ΠΌΠΎΠΆΠ½ΠΎ Π±ΡΠ»ΠΎ Π±Ρ ΠΎΠΆΠΈΠ΄Π°ΡΡ, ΡΡΠΎ Π±ΡΠ΄Π΅Ρ ΠΌΠ΅Π½ΡΡΠ΅ ΡΡΠΈΡ
ΠΆΠΈΠ·Π½Π΅Π½Π½ΠΎ Π²Π°ΠΆΠ½ΡΡ
ΡΠΎΡΠΎΠ½ΠΎΠ², Π²Ρ
ΠΎΠ΄ΡΡΠΈΡ
ΠΈ Π²ΡΡ
ΠΎΠ΄ΡΡΠΈΡ
ΠΈΠ· ΡΠ»Π΅ΠΊΡΡΠΎΠ½ΠΎΠ² Π² ΡΡΠ΅Π΄Π΅, Π³Π΄Π΅ ΠΌΠ΅Π½ΡΡΠ΅ ΡΠ»Π΅ΠΊΡΡΠΎΠ½ΠΎΠ², Π² ΠΊΠΎΡΠΎΡΡΠ΅ ΠΌΠΎΠΆΠ½ΠΎ Π²Ρ
ΠΎΠ΄ΠΈΡΡ ΠΈ Π²ΡΡ
ΠΎΠ΄ΠΈΡΡ, β Π½ΠΎ ΠΊΠ°ΠΊΠΎΠ²ΠΎ Π±ΡΠ΄Π΅Ρ ΠΏΡΠΈΡΡΠΆΠ΅Π½ΠΈΠ΅ ΠΌΠ΅ΠΆΠ΄Ρ Π΄Π²ΡΠΌΡ ΠΌΠ°Π³Π½ΠΈΡΡ Π² ΡΡΠΎΠΉ ΡΡΠ΅Π΄Π΅? ΠΡΠΎ Π±ΡΠ»ΠΎ Π±Ρ ΡΠΎ ΠΆΠ΅ ΡΠ°ΠΌΠΎΠ΅, Π½Π΅ ΡΠ°ΠΊ Π»ΠΈ? ΠΠΎΠ½Π΅ΡΠ½ΠΎ, ΡΠ΄ΠΈΠ²ΠΈΡΠ΅Π»ΡΠ½ΠΎ, ΡΡΠΎ Π΅ΡΠ»ΠΈ Π΄Π²Π° ΡΠΈΠ»ΡΠ½ΡΡ
ΠΌΠ°Π³Π½ΠΈΡΠ° ΠΏΠΎΠΌΠ΅ΡΡΠΈΡΡ Π² ΡΠ°ΠΌΡΡ ΡΠ°Π·ΡΠ΅ΠΆΠ΅Π½Π½ΡΡ ΠΎΠ±Π»Π°ΡΡΡ ΠΏΡΠΎΡΡΡΠ°Π½ΡΡΠ²Π°, ΠΎΠ½ΠΈ Π²ΡΠ΅ ΡΠ°Π²Π½ΠΎ Π±ΡΠ΄ΡΡ ΠΎΠΊΠ°Π·ΡΠ²Π°ΡΡ ΠΌΠΎΡΠ½ΠΎΠ΅ Π²ΠΎΠ·Π΄Π΅ΠΉΡΡΠ²ΠΈΠ΅ Π΄ΡΡΠ³ Π½Π° Π΄ΡΡΠ³Π° (ΠΏΡΠΈΡΡΠ³ΠΈΠ²Π°ΡΡΠ΅Π΅ ΠΈΠ»ΠΈ ΠΎΡΡΠ°Π»ΠΊΠΈΠ²Π°ΡΡΠ΅Π΅), Π΄Π°ΠΆΠ΅ Π΅ΡΠ»ΠΈ ΠΎΠ½ΠΈ ΠΌΠΎΠ³ΡΡ Π±ΡΡΡ Π½Π΅ΠΏΠΎΠ΄Π²ΠΈΠΆΠ½Ρ ΠΎΡΠ½ΠΎΡΠΈΡΠ΅Π»ΡΠ½ΠΎ Π΄ΡΡΠ³ Π΄ΡΡΠ³Π°. ΠΠ΅ΠΆΠ΄Ρ Π½ΠΈΠΌΠΈ ΡΠ²Π½ΠΎ ΡΡΠΎ-ΡΠΎ Π΅ΡΡΡ. ΠΡΠΎ-ΡΠΎ Π²ΡΡΠ΅ ΡΠΊΠ°Π·Π°Π», ΡΡΠΎ ΠΌΠ°Π³Π½ΠΈΡΠ½ΠΎΠ΅ ΠΏΠΎΠ»Π΅ Π½Π΅ ΠΌΠΎΠΆΠ΅Ρ ΡΡΡΠ΅ΡΡΠ²ΠΎΠ²Π°ΡΡ ΡΠ°ΠΌΠΎ ΠΏΠΎ ΡΠ΅Π±Π΅ β ΡΡΠΎ ΡΠΎΡΡΠ°Π²Π»ΡΡΡΠ°Ρ ΡΠ»Π΅ΠΊΡΡΠΎΠΌΠ°Π³Π½ΠΈΡΠ½ΠΎΠ³ΠΎ ΠΏΠΎΠ»Ρ, ΠΈ ΠΌΠ΅Π½Ρ ΡΡΠΎ ΠΈΠ½ΡΠ΅ΡΠ΅ΡΡΠ΅Ρ, ΠΏΠΎΡΠΎΠΌΡ ΡΡΠΎ Ρ ΡΡΠΈΡΠ°Ρ, ΡΡΠΎ ΡΡΠΎ ΠΏΠΎΠ»Π΅ Π΄ΠΎΠ»ΠΆΠ½ΠΎ ΠΏΡΠΈΡΡΡΡΡΠ²ΠΎΠ²Π°ΡΡ, ΠΊΠΎΠ³Π΄Π° ΠΌΠ°Π³Π½ΠΈΡΡ Π½Π΅Ρ. Π― Π΄ΡΠΌΠ°Ρ, Π²ΠΏΠΎΠ»Π½Π΅ Π²Π΅ΡΠΎΡΡΠ½ΠΎ, ΡΡΠΎ ΡΠΎ, ΡΡΠΎ ΡΠΎΡΡΠ°Π²Π»ΡΠ΅Ρ ΡΡΠΎ ΠΏΠΎΠ»Π΅, ΡΠ°ΠΊΠΆΠ΅ ΡΠΎΡΡΠ°Π²Π»ΡΠ΅Ρ ΡΠΎ, Π² ΡΠ΅ΠΌ ΡΠ²Π΅Ρ ΡΠΎΠ·Π΄Π°Π΅Ρ Π²ΠΎΠ»Π½Ρ, Π° ΡΠ²Π΅Ρ, ΠΊΠ°ΠΊ Π½Π°ΠΌ Π³ΠΎΠ²ΠΎΡΡΡ, ΡΠ²Π»ΡΠ΅ΡΡΡ ΡΠ»Π΅ΠΊΡΡΠΎΠΌΠ°Π³Π½ΠΈΡΠ½ΠΎΠΉ Π²ΠΎΠ»Π½ΠΎΠΉ, ΡΠ°ΠΊΠΎΠΉ ΠΆΠ΅, ΠΊΠ°ΠΊ Π²ΡΡΠΎΠΊΠΎΡΠ°ΡΡΠΎΡΠ½Π°Ρ ΡΠ°Π΄ΠΈΠΎΠ²ΠΎΠ»Π½Π°. ΠΠ½Π΅ (ΠΏΠΎΠ»Π½ΠΎΠΌΡ Π΄ΠΈΠ»Π΅ΡΠ°Π½ΡΡ) ΠΊΠ°ΠΆΠ΅ΡΡΡ Π±Π΅ΡΡΠΌΡΡΠ»Π΅Π½Π½ΡΠΌ ΡΡΠ²Π΅ΡΠΆΠ΄Π΅Π½ΠΈΠ΅, ΡΡΠΎ ΡΠ²Π΅Ρ β ΡΡΠΎ Π²ΠΎΠ»Π½Π°, ΠΏΡΠΎΡ
ΠΎΠ΄ΡΡΠ°Ρ ΡΠΊΠ²ΠΎΠ·Ρ Π½ΠΈΡΡΠΎ. ΠΡΡΠ³ΠΈΠ΅ Π²ΠΎΠ»Π½Ρ β ΡΡΠΎ Π²ΡΠ΅ Π²ΠΎΠ»Π½Ρ Π² ΡΠ΅ΠΌ-ΡΠΎ β Π² Π²ΠΎΠ·Π΄ΡΡ
Π΅, Π² Π²ΠΎΠ΄Π΅, Π² Π²Π΅ΡΠ΅Π²ΠΊΠ΅ ΠΈ Ρ. Π΄. Π‘Π²Π΅Ρ ΡΠ°ΡΠΏΡΠΎΡΡΡΠ°Π½ΡΠ΅ΡΡΡ Π² ΠΏΡΠΎΡΡΡΠ°Π½ΡΡΠ²Π΅ Ρ ΠΎΡΠ΅Π½Ρ ΡΠΎΡΠ½ΠΎ ΠΎΠΏΡΠ΅Π΄Π΅Π»Π΅Π½Π½ΠΎΠΉ ΡΠΊΠΎΡΠΎΡΡΡΡ. ΠΠ΅ΡΠΎΠΌΠ½Π΅Π½Π½ΠΎ, Π΅ΡΠ»ΠΈ Π±Ρ ΡΡΡΡΠΊΡΡΡΠ° ΠΏΡΠΎΡΡΡΠ°Π½ΡΡΠ²Π° Π±ΡΠ»Π° Π½Π΅ΠΌΠ½ΠΎΠ³ΠΎ Π΄ΡΡΠ³ΠΎΠΉ, ΡΠ²Π΅Ρ Π΄Π²ΠΈΠ³Π°Π»ΡΡ Π±Ρ Π½Π΅ΠΌΠ½ΠΎΠ³ΠΎ ΠΌΠ΅Π΄Π»Π΅Π½Π½Π΅Π΅ ΠΈΠ»ΠΈ Π½Π΅ΠΌΠ½ΠΎΠ³ΠΎ Π±ΡΡΡΡΠ΅Π΅, ΡΠ°ΠΊ ΡΡΠΎ ΡΡΠΎ ΠΏΠΎΠ»Π΅ Π΄ΠΎΠ»ΠΆΠ½ΠΎ Π±ΡΡΡ ΡΠ΄ΠΈΠ²ΠΈΡΠ΅Π»ΡΠ½ΠΎ ΠΏΠΎΡΡΠΎΡΠ½Π½ΡΠΌ Π²ΠΎ Π²ΡΠ΅ΠΌ ΠΏΡΠΎΡΡΡΠ°Π½ΡΡΠ²Π΅ ΠΈ Π½Π΅ Π·Π°Π²ΠΈΡΠ΅ΡΡ ΠΎΡ ΠΏΠ»ΠΎΡΠ½ΠΎΡΡΠΈ Π°ΡΠΎΠΌΠΎΠ². ΠΡΠΎ, Π±Π΅Π·ΡΡΠ»ΠΎΠ²Π½ΠΎ, Π΄ΠΎΠ»ΠΆΠ½Π° Π±ΡΡΡ ΠΎΡΠ΅Π½Ρ ΡΡΡΠ΅ΠΊΡΠΈΠ²Π½Π°Ρ ΡΡΠ°Π½ΡΠΏΠΎΡΡΠ½Π°Ρ ΡΡΠ΅Π΄Π°, ΠΏΠΎΡΠΎΠΌΡ ΡΡΠΎ ΡΠ²Π΅Ρ ΠΌΠΎΠΆΠ΅Ρ ΠΏΡΡΠ΅ΡΠ΅ΡΡΠ²ΠΎΠ²Π°ΡΡ ΠΌΠΈΠ»Π»ΠΈΠ°ΡΠ΄Ρ Π»Π΅Ρ ΠΈ Π²ΡΠ΅ ΠΆΠ΅ Π΄ΠΎΡΡΠΈΠ³Π°ΡΡ Π½Π°Ρ, Π½Π΅ ΡΠ°ΡΡΠ΅ΠΈΠ²Π°ΡΡΡ Π±ΠΎΠ»ΡΡΠ΅, ΡΠ΅ΠΌ Π½Π° ΠΊΠ²Π°Π΄ΡΠ°Ρ ΡΠ°ΡΡΡΠΎΡΠ½ΠΈΡ. Π’Π°ΠΊ ΡΠ΅ΡΠ΅Π· ΡΡΠΎ ΠΎΠ½ ΠΏΡΡΠ΅ΡΠ΅ΡΡΠ²ΡΠ΅Ρ? ΠΡΡΡ Π»ΠΈ ΠΊΠ°ΠΊΠ°Ρ-Π½ΠΈΠ±ΡΠ΄Ρ ΠΏΠΎΠ΄ΡΠΊΠ°Π·ΠΊΠ° Π² ΡΠΎΠΌ, ΡΡΠΎ ΡΠ»Π΅ΠΊΡΡΠΈΡΠ΅ΡΠΊΠΈΠ΅ ΠΈ ΠΌΠ°Π³Π½ΠΈΡΠ½ΡΠ΅ ΠΊΠΎΠΌΠΏΠΎΠ½Π΅Π½ΡΡ ΡΠ°Π΄ΠΈΠΎΠΏΠ΅ΡΠ΅Π΄Π°ΡΠΈ Π½Π°Ρ
ΠΎΠ΄ΡΡΡΡ ΠΏΠΎΠ΄ ΠΏΡΡΠΌΡΠΌ ΡΠ³Π»ΠΎΠΌ? (Π― ΠΈΠΌΠ΅Ρ Π² Π²ΠΈΠ΄Ρ ΡΠ³ΠΎΠ» ΠΏΠΎΠ»ΡΡΠΈΠ·Π°ΡΠΈΠΈ). Π― Π²ΠΈΠ΄Π΅Π» ΠΎΠ±ΡΡΡΠ½Π΅Π½ΠΈΡ ΡΠΎΠ³ΠΎ, ΠΊΠ°ΠΊ ΡΠ²Π΅Ρ ΠΏΡΠΎΡ
ΠΎΠ΄ΠΈΡ ΡΠ΅ΡΠ΅Π· ΡΡΠ΅ΠΊΠ»ΠΎ ΠΈΠ»ΠΈ Π²ΠΎΠ΄Ρ (Π³Π΄Π΅ Π΅Π³ΠΎ ΡΠΊΠΎΡΠΎΡΡΡ ΠΎΡΠ»ΠΈΡΠ°Π΅ΡΡΡ ΠΎΡ ΡΠΊΠΎΡΠΎΡΡΠΈ Π² ΠΊΠΎΡΠΌΠΎΡΠ΅), Π½ΠΎ ΡΡΠΎ Π·Π°Π²ΠΈΡΠΈΡ ΠΎΡ ΡΠΎΠ³ΠΎ, ΡΡΠΎ Π°ΡΠΎΠΌΡ Π²ΠΎΠ·Π±ΡΠΆΠ΄Π°ΡΡΡΡ ΡΠ²Π΅ΡΠΎΠΌ ΠΈ ΠΈΡΠΏΡΡΠΊΠ°ΡΡ Π² ΠΎΡΠ²Π΅Ρ ΠΏΡΠΎΡΠΎΠ½ β ΡΠ΄ΠΈΠ²ΠΈΡΠ΅Π»ΡΠ½ΠΎ, Π²ΡΠ΅Π³Π΄Π° Π² ΠΎΠ΄Π½ΠΎΠΌ ΠΈ ΡΠΎΠΌ ΠΆΠ΅ Π½Π°ΠΏΡΠ°Π²Π»Π΅Π½ΠΈΠΈ. ΠΡΠΎ ΠΎΠ±ΡΡΡΠ½Π΅Π½ΠΈΠ΅ ΠΎΠ±ΡΡΡΠ½ΡΠ΅Ρ Π΄ΠΈΡΡΠ°ΠΊΡΠΈΡ, ΠΎΠ΄Π½Π°ΠΊΠΎ ΡΠΎΡ ΠΆΠ΅ ΠΌΠ΅Ρ
Π°Π½ΠΈΠ·ΠΌ Π½Π΅Π»ΡΠ·Ρ ΠΏΡΠΈΠΌΠ΅Π½ΠΈΡΡ ΠΊ ΠΏΡΡΡΠΎΠΌΡ ΠΏΡΠΎΡΡΡΠ°Π½ΡΡΠ²Ρ. ΠΡΡΡ Π»ΠΈ ΠΊΠ°ΠΊΠΎΠΉ-Π»ΠΈΠ±ΠΎ ΠΎΡΠ²Π΅Ρ, Π·Π° ΠΊΠΎΡΠΎΡΡΠΉ ΠΌΠΎΠΆΠ΅Ρ ΡΡ
Π²Π°ΡΠΈΡΡΡΡ Π½Π΅ΡΠΏΠ΅ΡΠΈΠ°Π»ΠΈΡΡ, ΠΈΠ»ΠΈ ΡΡΠΎ ΠΎΠ΄Π½Π° ΠΈΠ· ΡΠ΅Ρ
Π²Π΅ΡΠ΅ΠΉ, ΠΊΠ°ΠΊ sqrt(-1), ΠΊΠΎΡΠΎΡΡΠ΅ ΠΌΠΎΠ³ΡΡ Π±ΡΡΡ ΡΠΎΠ»ΡΠΊΠΎ ΠΏΠΎΠ½ΠΈΠΌΠ°Π΅ΡΠ΅ Π² ΠΎΠ±Π»Π°ΡΡΠΈ ΠΌΠ°ΡΠ΅ΠΌΠ°ΡΠΈΠΊΠΈ? ΠΠ°ΠΆΠ΅ ΡΠ°ΠΌ Π΅ΡΡΡ ΡΠ²Π΅ΡΠ΄ΠΎΠ΅ ΠΏΠΎΠ½ΠΈΠΌΠ°Π½ΠΈΠ΅ ΡΠΎΠ³ΠΎ, ΡΡΠΎ ΡΡΠΎ ΡΠ°ΠΊΠΎΠ΅, Π° Π½Π΅ ΠΊΠ°ΠΊΠΎΠ²Ρ Π΅Π³ΠΎ ΡΠ²ΠΎΠΉΡΡΠ²Π°?
— ΠΠ°ΠΉΠΊ ΠΠΎΠ»Π»ΠΈΠ½Π· (71 Π³ΠΎΠ΄)
ΠΡΠΈΠ½Π΅Ρ, Π£ΡΠ»ΡΡ
A:
ΠΡ ΠΏΡΠ°Π²Ρ, ΡΡΠΎ Π² ΡΠ΅Π»ΠΎΠΌ ΠΌΡ Π½Π΅ Π·Π½Π°Π΅ΠΌ Π²ΡΠ΅Ρ ΠΈΠ½Π³ΡΠ΅Π΄ΠΈΠ΅Π½ΡΠΎΠ² ΠΌΠΈΡΠ°. ΠΡ, Π²Π΅ΡΠΎΡΡΠ½ΠΎ, Π΄Π°ΠΆΠ΅ Π½Π΅ Π·Π½Π°Π΅ΠΌ ΠΎΡΠ½ΠΎΠ²Π½ΠΎΠΉ ΡΠΎΡΠΌΡ ΡΠ΅ΠΎΡΠΈΠΈ, ΡΠΎΠ³ΠΎ, ΠΊΠ°ΠΊ ΠΏΡΠΎΡΡΡΠ°Π½ΡΡΠ²ΠΎ-Π²ΡΠ΅ΠΌΡ Π²ΠΎΠ·Π½ΠΈΠΊΠ°Π΅Ρ ΠΈΠ· Π½Π΅ΠΊΠΎΡΠΎΡΡΡ Π±ΠΎΠ»Π΅Π΅ Π³Π»ΡΠ±ΠΎΠΊΠΈΡ ΡΠΎΡΠΌ ΠΈ Ρ. Π΄. Π’Π΅ΠΌ Π½Π΅ ΠΌΠ΅Π½Π΅Π΅, Π² ΠΊΠΎΠ½ΡΠ΅ΠΊΡΡΠ΅ ΡΠΎΠ³ΠΎ, ΡΡΠΎ ΠΌΡ Π·Π½Π°Π΅ΠΌ, Π² ΠΌΠ°Π³Π½Π΅ΡΠΈΠ·ΠΌΠ΅ Π½Π΅Ρ ΠΎΡΠΎΠ±ΠΎΠΉ ΡΠ°ΠΉΠ½Ρ. ΠΠ° ΡΠ°ΠΌΠΎΠΌ Π΄Π΅Π»Π΅ ΠΌΠ°Π³Π½Π΅ΡΠΈΠ·ΠΌ ΡΠ²Π»ΡΠ΅ΡΡΡ ΡΠ°ΡΡΡΡ ΡΠ»Π΅ΠΊΡΡΠΎΡΠ»Π°Π±ΠΎΠΉ ΡΠ΅ΠΎΡΠΈΠΈ, ΠΊΠΎΡΠΎΡΠ°Ρ ΡΠ²Π»ΡΠ΅ΡΡΡ ΡΠ°ΠΌΠΎΠΉ ΠΈΠ·Π²Π΅ΡΡΠ½ΠΎΠΉ ΠΈΠ· ΠΈΠΌΠ΅ΡΡΠΈΡ ΡΡ Ρ Π½Π°Ρ ΡΠ΅ΠΎΡΠΈΠΉ. ΠΠ½ ΠΏΡΠ΅Π΄ΡΠΊΠ°Π·ΡΠ²Π°Π΅Ρ, Π½Π°ΠΏΡΠΈΠΌΠ΅Ρ, ΠΌΠ°Π³Π½Π΅ΡΠΈΠ·ΠΌ ΡΠ»Π΅ΠΊΡΡΠΎΠ½Π° Ρ ΡΠΎΡΠ½ΠΎΡΡΡΡ Π±ΠΎΠ»Π΅Π΅ ΠΎΠ΄Π½ΠΎΠΉ ΡΡΠΎΠΌΠΈΠ»Π»ΠΈΠ°ΡΠ΄Π½ΠΎΠΉ.
ΠΠΎΠ»Π½Ρ, ΠΊΠΎΡΠΎΡΡΠ΅ Π²Ρ ΡΠΏΠΎΠΌΡΠ½ΡΠ»ΠΈ, ΠΌΠΎΠΆΠ½ΠΎ ΠΎΠΏΠΈΡΠ°ΡΡ ΠΊΠ°ΠΊ ΡΠΈΠΏ ΠΏΠΎΠ²Π΅Π΄Π΅Π½ΠΈΡ Π½ΠΈΠΆΠ΅Π»Π΅ΠΆΠ°ΡΠΈΡ ΡΡΠ΅Π΄ β Π²ΠΎΠ΄Ρ, Π²ΠΎΠ·Π΄ΡΡ Π° ΠΈ Ρ. Π΄. ΠΠ΄Π½Π°ΠΊΠΎ Π½Π° Π±ΠΎΠ»Π΅Π΅ Π³Π»ΡΠ±ΠΎΠΊΠΎΠΌ ΡΡΠΎΠ²Π½Π΅ ΠΈΠ½Π³ΡΠ΅Π΄ΠΈΠ΅Π½ΡΡ ΠΡΠ΅Π»Π΅Π½Π½ΠΎΠΉ (ΡΠΎΡΠΎΠ½Ρ, ΠΊΠ²Π°ΡΠΊΠΈ, Π½Π΅ΠΉΡΡΠΈΠ½ΠΎ, Π³Π»ΡΠΎΠ½Ρ…) Π² Π½Π°ΡΡΠΎΡΡΠ΅Π΅ Π²ΡΠ΅ΠΌΡ ΠΌΠΎΠ³ΡΡ Π±ΡΡΡ ΠΎΠΏΠΈΡΠ°Π½Ρ ΡΠΎΠ»ΡΠΊΠΎ ΠΊΠ°ΠΊ ΡΠΈΡΡΡΠ΅ ΠΊΠ²Π°Π½ΡΠΎΠ²ΡΠ΅ Π²ΠΎΠ»Π½Ρ ΡΠ°ΠΌΠΈ ΠΏΠΎ ΡΠ΅Π±Π΅. Π€ΠΎΡΠΎΠ½Ρ ΡΠ°ΠΊΠΈΠ΅ ΠΆΠ΅ ΠΎΡΠ½ΠΎΠ²Π½ΡΠ΅, ΠΊΠ°ΠΊ ΠΈ Π»ΡΠ±ΡΠ΅ Π΄ΡΡΠ³ΠΈΠ΅ ΠΈΠ½Π³ΡΠ΅Π΄ΠΈΠ΅Π½ΡΡ, ΠΊΠΎΡΠΎΡΡΠ΅ Ρ Π½Π°Ρ Π΅ΡΡΡ.
ΠΠΎΠ·ΠΌΠΎΠΆΠ½ΠΎ, ΠΊΠΎΠ³Π΄Π°-Π½ΠΈΠ±ΡΠ΄Ρ Π±ΡΠ΄ΡΡ Π½Π°ΠΉΠ΄Π΅Π½Ρ ΠΊΠ°ΠΊΠΈΠ΅-ΡΠΎ Π±ΠΎΠ»Π΅Π΅ Π³Π»ΡΠ±ΠΎΠΊΠΈΠ΅ ΠΈΠ½Π³ΡΠ΅Π΄ΠΈΠ΅Π½ΡΡ, ΠΈ Π²ΡΠ΅ Π½Π°ΡΠΈ Π½ΡΠ½Π΅ΡΠ½ΠΈΠ΅ ΡΡΠ½Π΄Π°ΠΌΠ΅Π½ΡΠ°Π»ΡΠ½ΡΠ΅ ΠΏΠΎΠ»Ρ ΡΠ°ΡΡΠΈΡ Π±ΡΠ΄ΡΡ ΡΠ°ΡΡΠΌΠ°ΡΡΠΈΠ²Π°ΡΡΡΡ ΠΊΠ°ΠΊ Π²ΠΎΠ·Π½ΠΈΠΊΠ°ΡΡΠΈΠ΅ ΠΈΠ· ΠΏΠΎΠ²Π΅Π΄Π΅Π½ΠΈΡ ΡΡΠΎΠΉ Π±ΠΎΠ»Π΅Π΅ Π³Π»ΡΠ±ΠΎΠΊΠΎΠΉ ΡΠ΅ΠΎΡΠΈΠΈ. ΠΠ΅ ΠΎΠΊΠ°ΠΆΠ΅ΡΡΡ Π»ΠΈ ΡΠΎΠ³Π΄Π°, ΡΡΠΎ Π±ΠΎΠ»Π΅Π΅ Π³Π»ΡΠ±ΠΎΠΊΠ°Ρ ΡΠ΅ΠΎΡΠΈΡ Π²ΠΎΠ·Π½ΠΈΠΊΠ½Π΅Ρ ΠΈΠ· Π΅ΡΠ΅ Π±ΠΎΠ»Π΅Π΅ Π³Π»ΡΠ±ΠΎΠΊΠΎΠΉ? ΠΡΠ΄Π΅Ρ Π»ΠΈ ΡΡΠΎΡ ΠΏΠ°ΡΡΠ΅ΡΠ½ ΠΏΡΠΎΠ΄ΠΎΠ»ΠΆΠ°ΡΡΡΡ Π²Π΅ΡΠ½ΠΎ ΠΈΠ»ΠΈ Π΄ΠΎΡΡΠΈΠ³Π½Π΅Ρ ΡΠ°ΠΌΠΎΠ³ΠΎ Π³Π»ΡΠ±ΠΎΠΊΠΎΠ³ΠΎ ΡΡΠΎΠ²Π½Ρ? ΠΡ Π½Π΅ Π·Π½Π°Π΅ΠΌ. ΠΠΎΠΊΠ° Π΅ΡΡΡ ΠΎΠ±ΠΎΡΠ²Π°Π½Π½ΡΠ΅ ΠΊΠΎΠ½ΡΡ (Π½Π΅ΡΠΎΠΎΡΠ²Π΅ΡΡΡΠ²ΠΈΠ΅ ΠΌΠ΅ΠΆΠ΄Ρ ΠΎΠ±ΡΠ΅ΠΉ ΡΠ΅ΠΎΡΠΈΠ΅ΠΉ ΠΎΡΠ½ΠΎΡΠΈΡΠ΅Π»ΡΠ½ΠΎΡΡΠΈ ΠΈ ΠΊΠ²Π°Π½ΡΠΎΠ²ΠΎΠΉ ΡΠ΅ΠΎΡΠΈΠ΅ΠΉ ΠΏΠΎΠ»Ρ, ΡΠ΅ΠΌΠ½ΠΎΠΉ ΡΠ½Π΅ΡΠ³ΠΈΠ΅ΠΉ ΠΈ ΡΠ΅ΠΌΠ½ΠΎΠΉ ΠΌΠ°ΡΠ΅ΡΠΈΠ΅ΠΉ…), ΠΌΡ Π·Π½Π°Π΅ΠΌ, ΡΡΠΎ ΠΌΡ Π½Π΅ Π² ΡΠ°ΠΌΠΎΠΌ Π½ΠΈΠ·Ρ ΡΡΠ΅ΠΊΠ°. ΠΡΠ»ΠΈ ΠΊΠ°ΠΊΠ°Ρ-ΡΠΎ ΡΠΎΡΠΊΠ° Π±ΡΠ΄Π΅Ρ Π΄ΠΎΡΡΠΈΠ³Π½ΡΡΠ° Π±Π΅Π· ΠΎΡΠ½ΠΎΠ²Π½ΡΡ ΠΎΠ±ΠΎΡΠ²Π°Π½Π½ΡΡ ΠΊΠΎΠ½ΡΠΎΠ², ΡΠΎ, Π²ΠΎΠ·ΠΌΠΎΠΆΠ½ΠΎ, ΠΌΡ ΠΎΠΊΠ°ΠΆΠ΅ΠΌΡΡ Π½Π° ΡΠ°ΠΌΠΎΠΌ Π³Π»ΡΠ±ΠΎΠΊΠΎΠΌ ΡΡΠΎΠ²Π½Π΅.
ΠΠ°ΠΉΠΊ Π.
(ΠΎΠΏΡΠ±Π»ΠΈΠΊΠΎΠ²Π°Π½ΠΎ 15.06.2017)
ΠΠΎΠΏΠΎΠ»Π½Π΅Π½ΠΈΠ΅ β 7: Π½Π΅ΠΎΠ±Ρ ΠΎΠ΄ΠΈΠΌΡ Π»ΠΈ ΠΌΠ°Π³Π½ΠΈΡΠ½ΡΠ΅ ΠΏΠΎΠ»Ρ?
Q:
Π― Ρ
ΠΎΡΠ΅Π» Π±Ρ Π·Π°Π΄Π°ΡΡ, ΠΊΠ°Π·Π°Π»ΠΎΡΡ Π±Ρ, Π³Π»ΡΠΏΡΠΉ Π²ΠΎΠΏΡΠΎΡ. ΠΠ°ΠΊΠΎΠ²Ρ ΡΠΊΡΠΏΠ΅ΡΠΈΠΌΠ΅Π½ΡΠ°Π»ΡΠ½ΡΠ΅ Π΄ΠΎΠΊΠ°Π·Π°ΡΠ΅Π»ΡΡΡΠ²Π° ΡΡΠ°Π΄ΠΈΡΠΈΠΎΠ½Π½ΠΎΠΉ ΠΈΠ΄Π΅ΠΈ ΠΌΠ°Π³Π½ΠΈΡΠ½ΡΡ
ΠΏΠΎΠ»Π΅ΠΉ? ΠΡ Π΄ΠΎΠ»ΠΆΠ½Ρ ΠΏΠΎΠΌΠ½ΠΈΡΡ, ΡΡΠΎ ΠΈΠ΄Π΅Ρ Π²ΡΠ°ΡΠ°ΡΡΠ΅Π³ΠΎΡΡ ΠΏΠ΅ΡΠΏΠ΅Π½Π΄ΠΈΠΊΡΠ»ΡΡΠ½ΠΎΠ³ΠΎ ΠΏΠΎΡΠΎΠΊΠ° Π±ΡΠ»Π° ΠΎΡΠ½ΠΎΠ²Π°Π½Π° Π½Π° ΠΈΠ³Π½ΠΎΡΠΈΡΠΎΠ²Π°Π½ΠΈΠΈ ΠΌΠ°Π³Π½Π΅ΡΠΈΠ·ΠΌΠ° β ΠΎΡΠ±ΠΈΡΠ°Π»ΡΠ½ΡΠ΅ ΠΈ Π²ΡΠ°ΡΠ°ΡΡΠΈΠ΅ΡΡ ΡΠ»Π΅ΠΊΡΡΠΎΠ½Ρ Π±ΡΠ»ΠΈ Π½Π΅ΠΈΠ·Π²Π΅ΡΡΠ½Ρ 150 Π»Π΅Ρ Π½Π°Π·Π°Π΄. Π― ΡΡΠ΅ΡΠ½ΠΎ ΠΏΡΠΎΡΠΈΠ» Π΄ΠΎΠΊΠ°Π·Π°ΡΠ΅Π»ΡΡΡΠ²Π° ΡΡΠΎΠ³ΠΎ ΠΏΠΎΡΠΎΠΊΠ°. ΠΠ°ΠΆΠ΅ΡΡΡ, ΡΡΠΎ Π²ΡΠ΅Π³ΠΎ Π»ΠΈΡΡ Π΄ΠΎΠ³Π°Π΄ΠΊΠ°, ΠΊΠΎΡΠΎΡΠ°Ρ ΠΏΡΠ΅Π²ΡΠ°ΡΠΈΠ»Π°ΡΡ Π² Π²Π΅ΡΡ. ΠΡΠ΅Π΄ΠΏΠΎΠ»ΠΎΠΆΠΈΠΌ, ΡΡΠΎ ΠΌΠ°Π³Π½ΠΈΡΡ Π±ΡΠ»ΠΈ Π½Π΅ΠΈΠ·Π²Π΅ΡΡΠ½Ρ Π² ΡΠΎ Π²ΡΠ΅ΠΌΡ. Π’ΠΎΠ³Π΄Π° ΡΠΊΡΠΏΠ΅ΡΠΈΠΌΠ΅Π½ΡΡ Ρ ΡΠ»Π΅ΠΊΡΡΠΈΡΠ΅ΡΡΠ²ΠΎΠΌ ΠΏΡΠΈΠ²Π΅Π»ΠΈ Π±Ρ ΠΊ ΠΏΡΠΎΡΡΠΎΠΌΡ Π·Π°ΠΊΠΎΠ½Ρ: ΠΎΠ΄ΠΈΠ½Π°ΠΊΠΎΠ²ΡΠ΅ ΡΠΎΠΊΠΈ ΠΏΡΠΈΡΡΠ³ΠΈΠ²Π°ΡΡΡΡ, Π° ΠΏΡΠΎΡΠΈΠ²ΠΎΠΏΠΎΠ»ΠΎΠΆΠ½ΡΠ΅ ΡΠΎΠΊΠΈ ΠΎΡΡΠ°Π»ΠΊΠΈΠ²Π°ΡΡΡΡ. ΠΠ°ΡΠ΅ΠΌ ΡΡΠΎΡ ΠΎΡΠ½ΠΎΠ²Π½ΠΎΠΉ Π·Π°ΠΊΠΎΠ½ ΠΎΠ±ΡΡΡΠ½ΡΠ΅Ρ ΠΌΠ°Π³Π½Π΅ΡΠΈΠ·ΠΌ, Π½Π°ΠΏΡΠΈΠΌΠ΅Ρ, Π²ΡΡΠ°Π²Π½ΠΈΠ²Π°Π½ΠΈΠ΅ ΠΆΠ΅Π»Π΅Π·Π½ΡΡ
ΠΎΠΏΠΈΠ»ΠΎΠΊ Π²ΠΎΠΊΡΡΠ³ ΠΌΠ°Π³Π½ΠΈΡΠ°. ΠΡΠΏΠΎΠ»ΡΠ·ΡΡ ΠΏΡΠΈΠ½ΡΠΈΠΏ ΠΠΊΠΊΠ°ΠΌΠ°, ΡΡΠ»ΠΎΠΆΠ½Π΅Π½ΠΈΠ΅ ΡΠΈΡΠΊΡΠ»ΠΈΡΡΡΡΠ΅Π³ΠΎ ΠΏΠ΅ΡΠΏΠ΅Π½Π΄ΠΈΠΊΡΠ»ΡΡΠ½ΠΎΠ³ΠΎ ΠΏΠΎΡΠΎΠΊΠ° ΠΊΠ°ΠΆΠ΅ΡΡΡ Π½Π΅ΠΎΠΏΡΠ°Π²Π΄Π°Π½Π½ΡΠΌ. (Π’Π°ΠΊΠΈΠΌ ΠΎΠ±ΡΠ°Π·ΠΎΠΌ, Π²ΠΌΠ΅ΡΡΠΎ ΡΠΎΠ³ΠΎ, ΡΡΠΎΠ±Ρ ΠΈΡΠΏΠΎΠ»ΡΠ·ΠΎΠ²Π°ΡΡ Π·Π°ΠΊΠΎΠ½ ΠΠΈΠΎ ΠΈ Π‘Π°Π²Π°ΡΠ° Π΄Π»Ρ ΠΏΡΠ΅Π΄ΡΠΊΠ°Π·Π°Π½ΠΈΡ ΠΏΠ»ΠΎΡΠ½ΠΎΡΡΠΈ ΠΏΠ΅ΡΠΏΠ΅Π½Π΄ΠΈΠΊΡΠ»ΡΡΠ½ΠΎΠ³ΠΎ ΠΏΠΎΡΠΎΠΊΠ°, ΠΌΠΎΠΆΠ½ΠΎ Π²ΠΊΠ»ΡΡΠΈΡΡ ΡΠΊΠ°Π»ΡΡΠ½ΠΎΠ΅ ΠΏΡΠΎΠΈΠ·Π²Π΅Π΄Π΅Π½ΠΈΠ΅ Π²Π΅ΠΊΡΠΎΡΠΎΠ² ΡΠΎΠΊΠ° Π΄Π»Ρ ΠΏΡΠ΅Π΄ΡΠΊΠ°Π·Π°Π½ΠΈΡ ΡΠΈΠ»Ρ). Π‘Π»Π΅Π΄ΠΎΠ²Π°ΡΠ΅Π»ΡΠ½ΠΎ, ΠΌΠ°Π³Π½ΠΈΡΠ½ΡΠ΅ ΡΠΈΠ»Ρ ΠΏΡΠΎΡΡΠΎ Π΄Π΅ΠΉΡΡΠ²ΡΡΡ Π²Π΄ΠΎΠ»Ρ ΠΏΡΡΠΌΡΡ
Π»ΠΈΠ½ΠΈΠΉ ΠΌΠ΅ΠΆΠ΄Ρ Π΄Π²ΠΈΠΆΡΡΠΈΠΌΠΈΡΡ Π·Π°ΡΡΠ΄Π°ΠΌΠΈ. ΠΡΠΎ ΡΠΎΡ ΠΆΠ΅ ΠΏΡΠΎΡΡΠΎΠΉ ΠΏΡΠΈΠ½ΡΠΈΠΏ, ΠΊΠΎΡΠΎΡΡΠΉ ΡΠ°Π±ΠΎΡΠ°Π΅Ρ Π΄Π»Ρ ΡΠ»Π΅ΠΊΡΡΠΎΡΡΠ°ΡΠΈΡΠ΅ΡΠΊΠΈΡ
ΡΠΈΠ» ΠΌΠ΅ΠΆΠ΄Ρ ΡΡΠ°ΡΠΈΠΎΠ½Π°ΡΠ½ΡΠΌΠΈ Π·Π°ΡΡΠ΄Π°ΠΌΠΈ. ΠΠ°ΠΌ Π½Π΅ Π½ΡΠΆΠ½ΠΎ ΠΏΡΠ΅Π΄ΠΏΠΎΠ»Π°Π³Π°ΡΡ, ΡΡΠΎ ΠΡΠ΅Π»Π΅Π½Π½Π°Ρ ΠΈΡΠΏΠΎΠ»ΡΠ·ΡΠ΅Ρ Π΄Π²Π° ΡΠΎΠ²Π΅ΡΡΠ΅Π½Π½ΠΎ ΡΠ°Π·Π½ΡΡ
ΡΠΈΠ»ΠΎΠ²ΡΡ
ΠΌΠ΅Ρ
Π°Π½ΠΈΠ·ΠΌΠ°. ΠΠ²ΠΈΠΆΠ΅Π½ΠΈΠ΅ ΠΏΡΠΎΡΡΠΎ ΠΈΠ·ΠΌΠ΅Π½ΡΠ΅Ρ ΡΠ»Π΅ΠΊΡΡΠΎΡΡΠ°ΡΠΈΡΠ΅ΡΠΊΠΈΠ΅ ΡΠΈΠ»Ρ. ΠΠ°Π³Π½ΠΈΡΠ½ΡΠ΅ ΠΏΠΎΠ»Ρ ΠΎΠΏΡΠ΅Π΄Π΅Π»ΡΡΡΡΡ ΠΊΠ°ΠΊ Π½Π΅ΠΏΡΠ΅ΡΡΠ²Π½ΡΠ΅. Π’Π°ΠΊΠΈΠΌ ΠΎΠ±ΡΠ°Π·ΠΎΠΌ, ΠΏΠΎΠ»Π΅, ΠΈΡΡ
ΠΎΠ΄ΡΡΠ΅Π΅ ΠΎΡ ΡΠ΅Π²Π΅ΡΠ½ΠΎΠ³ΠΎ ΠΊΠΎΠ½ΡΠ° ΡΡΠ΅ΡΠΆΠ½Π΅Π²ΠΎΠ³ΠΎ ΠΌΠ°Π³Π½ΠΈΡΠ°, ΠΎΠ³ΠΈΠ±Π°Π΅Ρ Π²Π½Π΅ΡΠ½ΡΡ ΡΡΠΎΡΠΎΠ½Ρ ΠΌΠ°Π³Π½ΠΈΡΠ° ΠΊ ΡΠΆΠ½ΠΎΠΌΡ ΠΏΠΎΠ»ΡΡΡ ΠΈ Π²ΠΎΠ·Π²ΡΠ°ΡΠ°Π΅ΡΡΡ ΡΠ΅ΡΠ΅Π· ΡΠ΅Π»ΠΎ ΠΌΠ°Π³Π½ΠΈΡΠ° ΠΎΠ±ΡΠ°ΡΠ½ΠΎ ΠΊ ΡΠ΅Π²Π΅ΡΠ½ΠΎΠΌΡ ΠΏΠΎΠ»ΡΡΡ. Π’Π΅ΠΏΠ΅ΡΡ ΠΏΡΠ΅Π΄ΡΡΠ°Π²ΡΡΠ΅ ΡΠ΅Π±Π΅ ΠΌΠ°Π³Π½ΠΈΡ, ΡΠ΄Π΅Π»Π°Π½Π½ΡΠΉ ΠΈΠ· ΠΎΡΠ΅Π½Ρ Π²ΡΠ·ΠΊΠΎΠ³ΠΎ ΠΌΠ°ΡΠ΅ΡΠΈΠ°Π»Π°, ΠΊΠΎΡΠΎΡΡΠΉ ΠΏΠΎΠ·Π²ΠΎΠ»ΡΠ΅Ρ ΡΠ²ΠΎΠ±ΠΎΠ΄Π½ΠΎ Π΄Π²ΠΈΠΆΡΡΠ΅ΠΌΡΡΡ ΡΠ΅Π²Π΅ΡΠ½ΠΎΠΌΡ ΠΏΠΎΠ»ΡΡΡ Π΄ΡΠ΅ΠΉΡΠΎΠ²Π°ΡΡ Π²Π½ΡΡΡΠΈ Π½Π΅Π³ΠΎ. ΠΡΠΎΡ Π²Π½ΡΡΡΠ΅Π½Π½ΠΈΠΉ ΡΠ΅Π²Π΅ΡΠ½ΡΠΉ ΠΏΠΎΠ»ΡΡ ΠΎΡΡΠ°Π»ΠΊΠΈΠ²Π°Π΅ΡΡΡ ΡΠΆΠ½ΡΠΌ ΠΏΠΎΠ»ΡΡΠΎΠΌ ΠΌΠ°Π³Π½ΠΈΡΠ° (?) ΠΈ ΡΠ½ΠΎΠ²Π° ΡΡ
ΠΎΠ΄ΠΈΡ ΡΠ΅ΡΠ΅Π· Π΅Π³ΠΎ ΡΠ΅Π²Π΅ΡΠ½ΡΠΉ ΠΏΠΎΠ»ΡΡ. ΠΠ°Ρ Π²ΡΠ΅Ρ
ΡΡΠΎΠΌΡ ΡΡΠ°Ρ, Π½ΠΎ Π΄Π»Ρ ΠΌΠ΅Π½Ρ ΡΡΠΎ Π½Π΅ ΠΈΠΌΠ΅Π΅Ρ ΡΠΌΡΡΠ»Π°. ΠΡ Π½Π΅ Π΄ΠΎΠ»ΠΆΠ½Ρ ΡΠ°ΡΡΠΌΠ°ΡΡΠΈΠ²Π°ΡΡ ΠΌΠ°Π³Π½ΠΈΡΡ ΠΊΠ°ΠΊ Π²Π΅ΡΠ½ΡΠ΅ Π΄Π²ΠΈΠ³Π°ΡΠ΅Π»ΠΈ. Π‘ΠΈΠ»Ρ Π½Π°ΡΠΈΠ½Π°ΡΡΡΡ ΠΈ Π·Π°ΠΊΠ°Π½ΡΠΈΠ²Π°ΡΡΡΡ Π² ΡΠΎΡΠΊΠ°Ρ
: ΠΎΠ½ΠΈ Π½Π΅ Π²ΡΠ°ΡΠ°ΡΡΡΡ ΠΏΠΎ ΠΊΡΡΠ³Ρ. ΠΠΈΠ½ΠΈΠΈ ΠΏΠΎΡΠΎΠΊΠ° ΠΏΠΎΠΊΠ°Π·ΡΠ²Π°ΡΡ, ΠΊΠ°ΠΊ ΠΎΡΠΊΠ»ΠΎΠ½ΡΡΡΡΡ ΠΌΠ°Π³Π½ΠΈΡΠ½ΡΠ΅ ΠΊΠΎΠΌΠΏΠ°ΡΡ, Π½ΠΎ Π½ΠΈΡΡΠΎ Π½Π΅ ΡΠΈΡΠΊΡΠ»ΠΈΡΡΠ΅Ρ, ΠΊΡΠΎΠΌΠ΅ Π·Π°ΡΡΠ΄ΠΎΠ². ΠΡΠ΅Π΄ΡΡΠ°Π²Π»Π΅Π½ΠΈΠ΅ ΠΎ ΠΊΡΡΠ³ΠΎΠ²ΡΡ
ΠΏΠΎΠ»ΡΡ
, Π²ΠΎΠ·ΠΌΠΎΠΆΠ½ΠΎ, Π²ΠΎΠ·Π½ΠΈΠΊΠ»ΠΎ, ΠΊΠΎΠ³Π΄Π° Π²ΠΎΠΊΡΡΠ³ ΠΏΡΠΎΠ²ΠΎΠ΄ΡΡΠ΅Π³ΠΎ ΠΏΡΠΎΠ²ΠΎΠ΄Π° Π±ΡΠ»ΠΈ Π·Π°ΠΌΠ΅ΡΠ΅Π½Ρ ΠΊΠΎΠ»ΡΡΠ° ΠΈΠ· ΠΆΠ΅Π»Π΅Π·Π½ΡΡ
ΠΎΠΏΠΈΠ»ΠΎΠΊ, Π½ΠΎ ΡΡΠΎ Π±ΡΠ»Π° ΠΎΡΠ΅Π½Ρ ΡΡΡΠ°Π½Π½Π°Ρ ΠΈΠ΄Π΅Ρ. ΠΡΡΠ³ΠΎΠ²ΠΎΠ΅ ΠΌΠ°Π³Π½ΠΈΡΠ½ΠΎΠ΅ ΠΏΠΎΠ»Π΅ Π² Π»ΡΠ±ΠΎΠΉ ΡΠΎΡΠΊΠ΅ ΠΎΠΏΡΠ΅Π΄Π΅Π»ΡΠ΅ΡΡΡ ΠΊΠ°ΠΊ Π²Π΅ΠΊΡΠΎΡ, ΠΏΠ΅ΡΠΏΠ΅Π½Π΄ΠΈΠΊΡΠ»ΡΡΠ½ΡΠΉ ΡΠΎΠ·Π΄Π°Π²Π°Π΅ΠΌΠΎΠΉ ΠΈΠΌ ΠΌΠ°Π³Π½ΠΈΡΠ½ΠΎΠΉ ΡΠΈΠ»Π΅. ΠΠ΄Π½Π°ΠΊΠΎ, Π΅ΡΠ»ΠΈ Π²Π΅ΠΊΡΠΎΡ ΠΏΡΠ΅Π΄ΡΡΠ°Π²Π»ΡΠ΅Ρ ΡΡΠΎ-ΡΠΎ, ΡΡΠΎ ΡΠ²Π½ΠΎ ΡΡΡΠ΅ΡΡΠ²ΡΠ΅Ρ, Π½Π°ΠΏΡΠΈΠΌΠ΅Ρ. ΡΠΈΠ·ΠΈΡΠ΅ΡΠΊΠΎΠΉ ΡΠΈΠ»Ρ ΠΈΠ»ΠΈ ΡΠΊΠΎΡΠΎΡΡΠΈ Π²Π΅ΡΡΠ°, Π΅Π΅ ΠΏΠ΅ΡΠΏΠ΅Π½Π΄ΠΈΠΊΡΠ»ΡΡΠ½ΡΠ΅ ΡΠΎΡΡΠ°Π²Π»ΡΡΡΠΈΠ΅ ΡΠ°Π²Π½Ρ Π½ΡΠ»Ρ, Ρ. Π΅. ΠΎΠ½Π° Π½Π΅ Π΄Π΅ΠΉΡΡΠ²ΡΠ΅Ρ Π² ΠΏΠ΅ΡΠΏΠ΅Π½Π΄ΠΈΠΊΡΠ»ΡΡΠ½ΠΎΠΌ Π½Π°ΠΏΡΠ°Π²Π»Π΅Π½ΠΈΠΈ. Π’Π°ΠΊΠΈΠΌ ΠΎΠ±ΡΠ°Π·ΠΎΠΌ, ΠΌΡ ΠΌΠΎΠΆΠ΅ΠΌ ΡΠΊΠ°Π·Π°ΡΡ, ΡΡΠΎ ΡΠΈΡΠΊΡΠ»ΠΈΡΡΡΡΠ΅Π³ΠΎ ΠΌΠ°Π³Π½ΠΈΡΠ½ΠΎΠ³ΠΎ ΠΏΠΎΠ»Ρ, ΠΎΠΊΠ°Π·ΡΠ²Π°ΡΡΠ΅Π³ΠΎ Π½Π°ΠΈΠ±ΠΎΠ»ΡΡΠ΅Π΅ Π²Π»ΠΈΡΠ½ΠΈΠ΅ Π² ΠΏΠ΅ΡΠΏΠ΅Π½Π΄ΠΈΠΊΡΠ»ΡΡΠ½ΠΎΠΌ Π½Π°ΠΏΡΠ°Π²Π»Π΅Π½ΠΈΠΈ, Π½Π΅ ΡΡΡΠ΅ΡΡΠ²ΡΠ΅Ρ. ΠΠ°Π³Π½ΠΈΡΠ½ΠΎΠ΅ ΠΏΠΎΠ»Π΅ Π½Π΅ Π±ΠΎΠ»Π΅Π΅ Π·Π°Π³Π°Π΄ΠΎΡΠ½ΠΎ, ΡΠ΅ΠΌ ΡΠ»Π΅ΠΊΡΡΠΈΡΠ΅ΡΠΊΠΎΠ΅.
— ΠΠ½Π΄ΡΡ (67 Π»Π΅Ρ)
Π¨ΡΠΎΠΏΡΠΈΡ, ΠΠ½Π³Π»ΠΈΡ
A:
ΠΡ ΠΏΡΠ°Π²Ρ Π² ΡΠΎΠΌ, ΡΡΠΎ ΡΠΈΠ»Ρ ΠΌΠ΅ΠΆΠ΄Ρ ΠΊΠ»Π°ΡΡΠΈΡΠ΅ΡΠΊΠΈΠΌΠΈ ΡΠ΅ΡΠ΅Π½ΠΈΡΠΌΠΈ Ρ Π΄ΠΎΡΡΠ°ΡΠΎΡΠ½ΠΎ ΡΡΠ°ΡΠΈΡΠ½ΡΠΌ ΡΠ°ΡΠΏΠΎΠ»ΠΎΠΆΠ΅Π½ΠΈΠ΅ΠΌ ΠΌΠΎΠ³ΡΡ Π±ΡΡΡ Π²ΡΡΠ°ΠΆΠ΅Π½Ρ Π½Π΅ΠΏΠΎΡΡΠ΅Π΄ΡΡΠ²Π΅Π½Π½ΠΎ ΡΠ΅ΡΠ΅Π· ΡΠ΅ΡΠ΅Π½ΠΈΡ ΠΈ ΡΠΌΠ΅ΡΠ΅Π½ΠΈΡ ΠΌΠ΅ΠΆΠ΄Ρ Π½ΠΈΠΌΠΈ. ΠΡΠΎ Π½Π΅ ΡΠ°ΠΊ ΠΏΡΠΎΡΡΠΎ, ΠΊΠ°ΠΊ ΡΠ»Π΅ΠΊΡΡΠΎΡΡΠ°ΡΠΈΠΊΠ°, ΠΏΠΎΡΠΎΠΌΡ ΡΡΠΎ Π½Π°ΠΏΡΠ°Π²Π»Π΅Π½ΠΈΠ΅ ΡΠΌΠ΅ΡΠ΅Π½ΠΈΡ ΠΈ Π½Π°ΠΏΡΠ°Π²Π»Π΅Π½ΠΈΡ ΡΠΎΠΊΠΎΠ² Π²Ρ ΠΎΠ΄ΡΡ Π² Π½Π΅ΡΠΊΠΎΠ»ΡΠΊΠΎ ΡΠ»ΠΎΠΆΠ½Π΅Π΅. ΠΏΡΡΡ. ΠΠΎ Π²ΡΠ΅ ΠΆΠ΅ Π² ΡΡΠΎΠΌ ΡΠ»ΡΡΠ°Π΅ ΠΈΡΠΏΠΎΠ»ΡΠ·ΠΎΠ²Π°Π½ΠΈΠ΅ ΠΎΠΏΠΈΡΠ°Π½ΠΈΠΉ ΠΌΠ°Π³Π½ΠΈΡΠ½ΠΎΠ³ΠΎ ΠΏΠΎΠ»Ρ ΡΠ²Π»ΡΠ΅ΡΡΡ Π»ΠΈΡΡ Π΄ΠΎΠΏΠΎΠ»Π½ΠΈΡΠ΅Π»ΡΠ½ΡΠΌ ΠΌΠ°ΡΠ΅ΠΌΠ°ΡΠΈΡΠ΅ΡΠΊΠΈΠΌ ΡΠ΄ΠΎΠ±ΡΡΠ²ΠΎΠΌ. ΠΠ°ΠΊ ΡΠΎΠ»ΡΠΊΠΎ Π²Ρ Π½Π°ΡΠ½Π΅ΡΠ΅ Π³ΠΎΠ²ΠΎΡΠΈΡΡ ΠΎ ΡΡΠΊΠΎΡΡΠ΅Ρ ΠΊΠ»Π°ΡΡΠΈΡΠ΅ΡΠΊΠΈΡ Π·Π°ΡΡΠ΄ΠΎΠ², ΠΎΠ΄Π½Π°ΠΊΠΎ Π² Π΄Π΅Π»ΠΎ Π²ΡΡΡΠΏΠ°ΡΡ ΠΏΠΎΠ»Π½ΡΠ΅ ΡΡΠ°Π²Π½Π΅Π½ΠΈΡ ΡΠ»Π΅ΠΊΡΡΠΎΠΌΠ°Π³Π½ΠΈΡΠ½ΠΎΠ³ΠΎ ΠΏΠΎΠ»Ρ. ΠΡΠ»ΠΎ Π±Ρ ΠΊΡΠ°ΠΉΠ½Π΅ Π½Π΅ΡΠ΄ΠΎΠ±Π½ΠΎ ΠΎΠΏΠΈΡΡΠ²Π°ΡΡ ΡΠ»Π΅ΠΊΡΡΠΎΠΌΠ°Π³Π½ΠΈΡΠ½ΡΠ΅ Π²ΠΎΠ»Π½Ρ (Π½Π°ΠΏΡΠΈΠΌΠ΅Ρ, ΡΠ°Π΄ΠΈΠΎΠ²ΠΎΠ»Π½Ρ ΠΈ ΡΠ²Π΅Ρ) Π² ΠΎΠ±ΠΎΠ·Π½Π°ΡΠ΅Π½ΠΈΡΡ , ΠΎΡΠ½ΠΎΠ²Π°Π½Π½ΡΡ Π½Π° Π·Π°ΡΡΠ΄Π°Ρ ΠΈ ΡΠΎΠΊΠ°Ρ , ΠΏΠΎΡΠΎΠΌΡ ΡΡΠΎ ΡΡΠΈ Π²ΠΎΠ»Π½Ρ ΡΠ²ΠΎΠ±ΠΎΠ΄Π½ΠΎ ΡΠ°ΡΠΏΡΠΎΡΡΡΠ°Π½ΡΡΡΡΡ Π²Π΄Π°Π»ΠΈ ΠΎΡ Π»ΡΠ±ΡΡ Π·Π°ΡΡΠ΄ΠΎΠ² ΠΈ ΡΠΎΠΊΠΎΠ².
ΠΠ°ΠΊ ΡΠΎΠ»ΡΠΊΠΎ Π²Ρ Π·Π°Ρ ΠΎΡΠΈΡΠ΅ Π²ΠΊΠ»ΡΡΠΈΡΡ ΠΊΠ²Π°Π½ΡΠΎΠ²ΡΠ΅ ΡΠΏΠΈΠ½Ρ, ΠΎΠΏΠΈΡΠ°Π½ΠΈΡ, ΠΊΠΎΡΠΎΡΡΠ΅ Π½Π΅ ΡΡΠΈΡΡΠ²Π°ΡΡ ΠΌΠ°Π³Π½ΠΈΡΠ½ΡΠ΅ ΠΏΠΎΠ»Ρ ΠΈΠ»ΠΈ Π΄Π°ΠΆΠ΅ Π±ΠΎΠ»Π΅Π΅ Π°Π±ΡΡΡΠ°ΠΊΡΠ½ΡΠ΅ ΡΡΡΠ½ΠΎΡΡΠΈ (Π²Π΅ΠΊΡΠΎΡΠ½ΡΠ΅ ΠΏΠΎΡΠ΅Π½ΡΠΈΠ°Π»Ρ,. ..), ΡΡΠ°Π½ΠΎΠ²ΡΡΡΡ Π±Π΅ΡΠΏΠΎΠ»Π΅Π·Π½ΡΠΌΠΈ. ΠΡΠ΅ ΠΏΡΠ°ΠΊΡΠΈΡΠ΅ΡΠΊΠΈΠ΅ ΠΌΠ°Π³Π½ΠΈΡΠ½ΡΠ΅ ΠΌΠ°ΡΠ΅ΡΠΈΠ°Π»Ρ Π²ΠΊΠ»ΡΡΠ°ΡΡ ΡΠ°ΠΊΠΈΠ΅ ΡΠΏΠΈΠ½Ρ.
ΠΠ°ΠΉΠΊ Π.
(ΠΎΠΏΡΠ±Π»ΠΈΠΊΠΎΠ²Π°Π½ΠΎ 26.07.2017)
ΠΠΎΠΏΠΎΠ»Π½Π΅Π½ΠΈΠ΅ β8: ΠΊΠ²Π°Π½ΡΠΎΠ²ΡΠ΅ ΡΠ»ΡΠΊΡΡΠ°ΡΠΈΠΈ ΡΠΎΡΠΎΠ½Ρ … ΠΏΠΎΡΡΠΎΡΠ½Π½ΠΎ ΠΈΡΠΏΡΡΠΊΠ°ΡΡΡΡ ΠΈ ΠΏΠΎΠ²ΡΠΎΡΠ½ΠΎ ΠΏΠΎΠ³Π»ΠΎΡΠ°ΡΡΡΡ ΡΠ»Π΅ΠΊΡΡΠΎΠ½ΠΎΠΌΒ», Π½ΠΎ Π½Π° ΡΠ°ΠΌΠΎΠΌ Π΄Π΅Π»Π΅ ΠΌΡ ΠΈΠΌΠ΅Π΅ΠΌ Π² Π²ΠΈΠ΄Ρ Π½Π΅ ΡΡΠΎ. ΠΠ²Π΅ ΡΠ»Π΅ΠΊΡΡΠΎΠΌΠ°Π³Π½ΠΈΡΠ½ΠΎ Π²Π·Π°ΠΈΠΌΠΎΠ΄Π΅ΠΉΡΡΠ²ΡΡΡΠΈΠ΅ ΡΠ°ΡΡΠΈΡΡ Π΄Π΅ΠΉΡΡΠ²ΠΈΡΠ΅Π»ΡΠ½ΠΎ ΠΎΠΊΡΡΠΆΠ΅Π½Ρ Π²ΠΈΡΡΡΠ°Π»ΡΠ½ΡΠΌ ΡΠΎΡΠΎΠ½Π½ΡΠΌ ΠΎΠ±Π»Π°ΠΊΠΎΠΌ. Π°ΡΠΎΠΌ) ΡΡΠΎ ΠΎΠ±Π»Π°ΠΊΠΎ ΡΠΎΠ²Π΅ΡΡΠ΅Π½Π½ΠΎ Π½Π΅ΠΈΠ·ΠΌΠ΅Π½Π½ΠΎ Π²ΠΎ Π²ΡΠ΅ΠΌΠ΅Π½ΠΈ. ΠΠΎΠΎΠ±ΡΠ΅ Π½ΠΈΡΠ΅Π³ΠΎ Π½Π΅ ΠΏΡΠΎΠΈΡΡ ΠΎΠ΄ΠΈΡ. Π‘Π»ΠΎΠ²Π° ΠΎ Π²Π΅ΡΠ°Ρ , ΡΠ»ΡΠΊΡΡΠΈΡΡΡΡΠΈΡ Π²ΠΎΠΊΡΡΠ³, β Π³ΡΡΠ±ΡΠΉ ΡΠΏΠΎΡΠΎΠ± ΠΏΠ΅ΡΠ΅Π΄Π°ΡΡ ΠΎΠ΄Π½ΠΎ ΠΈΠ· ΡΠΏΠ΅ΡΠΈΡΠΈΡΠ΅ΡΠΊΠΈΡ ΡΠ²ΠΎΠΉΡΡΠ² ΠΊΠ²Π°Π½ΡΠΎΠ²ΡΡ ΠΏΠΎΠ»Π΅ΠΉΒ». ΠΠ°ΠΊ Π²Π·Π°ΠΈΠΌΠΎΠ΄Π΅ΠΉΡΡΠ²ΠΈΠ΅ ΠΌΠ΅ΠΆΠ΄Ρ Π΄Π²ΡΠΌΡ ΡΠ°ΡΡΠΈΡΠ°ΠΌΠΈ ΠΌΠΎΠΆΠ΅Ρ ΠΏΡΠΈΠ²Π΅ΡΡΠΈ ΠΊ ΡΠΎΡΡΠΎΡΠ½ΠΈΡ, Π² ΠΊΠΎΡΠΎΡΠΎΠΌ Β«Π²ΠΎΠΎΠ±ΡΠ΅ Π½ΠΈΡΠ΅Π³ΠΎ Π½Π΅ ΠΏΡΠΎΠΈΡΡ ΠΎΠ΄ΠΈΡΒ»? Π Π°Π·Π²Π΅ ΡΠΈΠ·ΠΈΡΠ΅ΡΠΊΠΎΠ΅ Π²Π·Π°ΠΈΠΌΠΎΠ΄Π΅ΠΉΡΡΠ²ΠΈΠ΅ Π½Π΅ ΠΎΠ±ΡΠ·Π°ΡΠ΅Π»ΡΠ½ΠΎ ΠΏΡΠΈΠ²ΠΎΠ΄ΠΈΡ ΠΊ ΠΊΠ°ΠΊΠΈΠΌ-ΡΠΎ ΠΈΠ·ΠΌΠ΅Π½Π΅Π½ΠΈΡΠΌ? ΠΠ°ΠΆΠ΅ Π² ΡΠΈΡΡΠΎΠΌ Π²Π°ΠΊΡΡΠΌΠ΅ ΠΌΠ½ΠΎΠ³ΠΎΠ΅ ΠΏΡΠΎΠΈΡΡ ΠΎΠ΄ΠΈΡ, ΡΠΎ Π΅ΡΡΡ ΡΡΡΠ΅ΡΡΠ²ΡΡΡ ΡΠ»ΡΠΊΡΡΠ°ΡΠΈΠΈ Π½ΡΠ»Π΅Π²ΠΎΠΉ ΡΠ½Π΅ΡΠ³ΠΈΠΈ. ΠΠ΅ ΠΌΠΎΠ³Π»ΠΈ Π±Ρ Π²Ρ ΠΏΠΎΠ΄ΡΠΎΠ±Π½Π΅Π΅ ΡΠ°ΡΡΠΊΠ°Π·Π°ΡΡ ΠΎ ΡΠΎΠΌ, ΡΡΠΎ Π²Ρ ΠΈΠΌΠ΅Π΅ΡΠ΅ Π² Π²ΠΈΠ΄Ρ ΠΎΡΠ½ΠΎΡΠΈΡΠ΅Π»ΡΠ½ΠΎ Π²Π·Π°ΠΈΠΌΠΎΠ΄Π΅ΠΉΡΡΠ²ΠΈΠΉ ΠΌΠ΅ΠΆΠ΄Ρ Π°ΡΠΎΠΌΠ½ΡΠΌΠΈ ΡΠ°ΡΡΠΈΡΠ°ΠΌΠΈ ΠΈ Π²ΠΈΡΡΡΠ°Π»ΡΠ½ΡΠΌΠΈ ΡΠΎΡΠΎΠ½Π°ΠΌΠΈ Π² ΡΡΠΎΠΌ ΡΠΌΡΡΠ»Π΅? ΠΡΠΎΠΌΠ΅ ΡΠΎΠ³ΠΎ, ΠΏΠΎΡΠ΅ΠΌΡ ΠΈΡΠΏΠΎΠ»ΡΠ·ΡΠ΅ΡΡΡ ΡΠ·ΡΠΊ, ΡΠΊΠ°Π·ΡΠ²Π°ΡΡΠΈΠΉ Π½Π° ΠΈΠ½ΡΠ΅ΡΠ°ΠΊΡΠΈΠ²Π½ΠΎΠ΅ Π΄Π²ΠΈΠΆΠ΅Π½ΠΈΠ΅ ΠΌΠ΅ΠΆΠ΄Ρ ΡΠ°ΡΡΠΈΡΠ°ΠΌΠΈ (Β«ΡΠ»ΡΠΊΡΡΠ°ΡΠΈΡ Π²ΠΎΠΊΡΡΠ³Β»), Π΅ΡΠ»ΠΈ Π² ΡΡΠ±Π°ΡΠΎΠΌΠ½ΡΡ ΠΎΠ±Π»Π°ΠΊΠ°Ρ , ΡΠ°ΠΊΠΈΡ ΠΊΠ°ΠΊ Π²ΠΈΡΡΡΠ°Π»ΡΠ½ΡΠ΅ ΡΠΎΡΠΎΠ½Ρ, Π½Π΅Ρ Π΄Π²ΠΈΠΆΠ΅Π½ΠΈΡ, ΠΈ Π½Π΅ ΠΌΠΎΠ³Π»ΠΈ Π±Ρ Π²Ρ Π±ΠΎΠ»Π΅Π΅ ΡΠ΅ΡΠΊΠΎ ΠΎΠ±ΡΡΡΠ½ΠΈΡΡ, ΠΊΠ°ΠΊ ΡΠΈΠ·ΠΈΠΊΠ° Π΄Π²ΠΈΠΆΠ΅Π½ΠΈΡ Π½Π°ΡΡΡΠ°Π΅ΡΡΡ Π½Π° ΡΡΠ±Π°ΡΠΎΠΌΠ½ΠΎΠΌ ΡΡΠΎΠ²Π½Π΅? -Π°ΡΠΎΠΌΠ½ΡΡ ΠΌΠ°ΡΡΡΠ°Π±Π°Ρ , Π΅ΡΠ»ΠΈ ΡΡΠΎ Π΄Π΅ΠΉΡΡΠ²ΠΈΡΠ΅Π»ΡΠ½ΠΎ ΡΠΎ, ΡΡΠΎ ΠΏΡΠΎΠΈΡΡ ΠΎΠ΄ΠΈΡ?
— Π ΠΎΠ΄ΡΠΈ ΠΡΠ΄Π΅Π½Ρ (33 Π³ΠΎΠ΄Π°)
ΠΡΠ³Π»Π°Ρ, ΠΎΡΡΡΠΎΠ² ΠΡΠ½, ΠΡΠΈΡΠ°Π½ΡΠΊΠΈΠ΅ ΠΎΡΡΡΠΎΠ²Π°
A:
Π€ΠΈΠ·ΠΈΡΠ΅ΡΠΊΠΎΠ΅ Π²Π·Π°ΠΈΠΌΠΎΠ΄Π΅ΠΉΡΡΠ²ΠΈΠ΅ Π½Π΅ ΠΎΠ±ΡΠ·Π°ΡΠ΅Π»ΡΠ½ΠΎ ΠΎΠ·Π½Π°ΡΠ°Π΅Ρ, ΡΡΠΎ ΡΡΠΎ-ΡΠΎ ΠΏΡΠΎΠΈΡΡ ΠΎΠ΄ΠΈΡ, ΠΏΠΎ ΠΊΡΠ°ΠΉΠ½Π΅ΠΉ ΠΌΠ΅ΡΠ΅, Π² ΠΎΠ±ΡΡΠ½ΠΎΠΌ ΡΠΌΡΡΠ»Π΅ ΡΡΠΎΠ³ΠΎ ΡΠ»ΠΎΠ²Π°. ΠΠ°ΠΏΡΠΈΠΌΠ΅Ρ, ΠΏΡΠ΅Π΄ΡΡΠ°Π²ΡΡΠ΅ ΡΠ΅Π±Π΅ ΠΊΠΎΡΠΎΠ±ΠΊΡ, ΡΡΠΎΡΡΡΡ Π½Π° ΠΏΠΎΠ»Ρ. ΠΠΎΡΠΎΠ±ΠΊΠ° ΠΈ ΠΏΠΎΠ», Π±Π΅Π·ΡΡΠ»ΠΎΠ²Π½ΠΎ, Π²Π·Π°ΠΈΠΌΠΎΠ΄Π΅ΠΉΡΡΠ²ΡΡΡ. ΠΠΎ ΠΌΠ°Π»ΠΎ ΡΡΠΎ ΠΏΡΠΎΠΈΡΡ ΠΎΠ΄ΠΈΡ. ΠΠΈΡΠ΅Π³ΠΎ Π½Π΅ ΠΌΠ΅Π½ΡΠ΅ΡΡΡ.
Β«ΠΠ°ΠΆΠ΅ Π² ΡΠΈΡΡΠΎΠΌ Π²Π°ΠΊΡΡΠΌΠ΅ ΠΌΠ½ΠΎΠ³ΠΎΠ΅ ΠΏΡΠΎΠΈΡΡ ΠΎΠ΄ΠΈΡ, Ρ. Π΅. ΡΡΡΠ΅ΡΡΠ²ΡΡΡ ΡΠ»ΡΠΊΡΡΠ°ΡΠΈΠΈ Π½ΡΠ»Π΅Π²ΠΎΠΉ ΡΠ½Π΅ΡΠ³ΠΈΠΈΒ». ΠΡΠΎ Π²Π΅ΡΠ½ΠΎ Π² ΡΠ»Π΅Π΄ΡΡΡΠ΅ΠΌ ΡΠΌΡΡΠ»Π΅: Π·Π½Π°ΡΠ΅Π½ΠΈΡ ΡΠ°Π·Π»ΠΈΡΠ½ΡΡ ΠΏΠΎΠ»Π΅ΠΉ Π½Π΅ ΡΠΈΠΊΡΠΈΡΡΡΡΡΡ ΡΠΎΡΠ½ΠΎ Π½Π° Π½ΡΠ»Π΅, Π° ΠΈΠΌΠ΅ΡΡ ΡΠ°ΡΠΏΡΠΎΡΡΡΠ°Π½ΡΠ΅ΡΡΡ Π²ΠΎΠΊΡΡΠ³ ΡΡΠΎΠ³ΠΎ, ΡΠΎΡΠ½ΠΎ ΡΠ°ΠΊ ΠΆΠ΅, ΠΊΠ°ΠΊ ΡΠ°ΡΠΏΡΠΎΡΡΡΠ°Π½ΡΠ΅ΡΡΡ ΠΏΠΎΠ»ΠΎΠΆΠ΅Π½ΠΈΠ΅ Π²ΠΎΠ»Π½Ρ. Π’Π΅ΠΌ Π½Π΅ ΠΌΠ΅Π½Π΅Π΅, ΡΠ°Π·Π±ΡΠΎΡ ΠΏΠΎΠ»Ρ Π½Π΅ ΠΌΠ΅Π½ΡΠ΅ΡΡΡ Π²ΠΎ Π²ΡΠ΅ΠΌΠ΅Π½ΠΈ.
Π― Π΄ΡΠΌΠ°Ρ, ΡΡΠΎ Π½Π΅Π±ΡΠ΅ΠΆΠ½ΡΠΉ ΡΠ·ΡΠΊ ΠΈΡΠΏΠΎΠ»ΡΠ·ΡΠ΅ΡΡΡ, ΠΏΠΎΡΠΎΠΌΡ ΡΡΠΎ ΠΌΡ ΠΈΠ½ΡΡΠΈΠ½ΠΊΡΠΈΠ²Π½ΠΎ ΠΏΡΡΠ°Π΅ΠΌΡΡ Π²ΡΠΈΡΠ½ΡΡΡ ΠΊΠ²Π°Π½ΡΠΎΠ²ΡΡ ΡΠ΅Π°Π»ΡΠ½ΠΎΡΡΡ Π² ΠΊΠ»Π°ΡΡΠΈΡΠ΅ΡΠΊΠΈΠ΅ ΠΊΠ°ΡΡΠΈΠ½ΠΊΠΈ.
Mike W.
(ΠΎΠΏΡΠ±Π»ΠΈΠΊΠΎΠ²Π°Π½ΠΎ 30.09.2017)
ΠΠΎΠΏΠΎΠ»Π½Π΅Π½ΠΈΠ΅ β9: ΠΏΡΠ΅Π΄ΠΏΠΎΠ»ΠΎΠΆΠ΅Π½ΠΈΡ ΠΎΠ± ΡΠ»Π΅ΠΊΡΡΠΎΠΌΠ°Π³Π½ΠΈΡΠ½ΠΎΠΌ ΠΏΠΎΠ»Π΅
Q:
ΠΡΠ΅ΠΆΠ΄Π΅ Π²ΡΠ΅Π³ΠΎ, ΡΡΡΠ΅ΡΡΠ²ΡΠ΅Ρ Π±ΠΎΠ»ΡΡΠ°Ρ ΡΠ°Π·Π½ΠΈΡΠ° ΠΌΠ΅ΠΆΠ΄Ρ ΡΡΠ°ΡΠΈΡΠ΅ΡΠΊΠΈΠΌΠΈ ΠΏΠΎΠ»ΡΠΌΠΈ ΠΈ ΡΠ»Π΅ΠΊΡΡΠΎΠΌΠ°Π³Π½ΠΈΡΠ½ΠΎΠΉ ΠΈΠ½Π΄ΡΠΊΡΠΈΠ΅ΠΉ . Π‘ΠΎΠ³Π»Π°ΡΠ½ΠΎ ΠΎΡΠΈΡΠΈΠ°Π»ΡΠ½ΠΎΠΉ Π½Π°ΡΠΊΠ΅, ΠΌΠ°Π³Π½Π΅ΡΠΈΠ·ΠΌ ΡΠ²Π»ΡΠ΅ΡΡΡ Β«ΠΏΠΎΠ±ΠΎΡΠ½ΡΠΌ ΡΡΡΠ΅ΠΊΡΠΎΠΌΒ» ΡΠ»Π΅ΠΊΡΡΠΈΡΠ΅ΡΠΊΠΎΠ³ΠΎ ΡΠΎΠΊΠ°, ΠΈ ΠΎΠ±Π΅ ΡΠΈΠ»Ρ (ΠΌΠ°Π³Π½ΠΈΡΠ½Π°Ρ ΠΈ ΡΠ»Π΅ΠΊΡΡΠΈΡΠ΅ΡΠΊΠ°Ρ) ΡΠ²Π»ΡΡΡΡΡ Π»ΠΈΡΡ Π΄Π²ΡΠΌΡ ΡΡΠΎΡΠΎΠ½Π°ΠΌΠΈ ΠΎΠ΄Π½ΠΎΠΉ ΠΌΠ΅Π΄Π°Π»ΠΈ (ΡΠ»Π΅ΠΊΡΡΠΎΠΌΠ°Π³Π½Π΅ΡΠΈΠ·ΠΌ). ΠΠ»Π΅ΠΊΡΡΠΎΡΡΠ°ΡΠΈΡΠ΅ΡΠΊΠΎΠ΅ ΠΈ ΠΌΠ°Π³Π½ΠΈΡΠΎΡΡΠ°ΡΠΈΡΠ΅ΡΠΊΠΎΠ΅ ΠΏΠΎΠ»Ρ β ΡΡΠΎ Π΄Π²Π° ΡΠ°Π·Π½ΡΡ
Π°ΡΠΏΠ΅ΠΊΡΠ° ΡΠΈΠ·ΠΈΡΠ΅ΡΠΊΠΎΠΉ ΠΌΠ°ΡΠ΅ΡΠΈΠΈ. ΠΠ»Π΅ΠΊΡΡΠΎΡΡΠ°ΡΠΈΡΠ΅ΡΠΊΠΎΠ΅ ΠΏΠΎΠ»Π΅ Π½Π΅ Π²Π»ΠΈΡΠ΅Ρ Π½Π° ΡΡΡΠ΅Π»ΠΊΡ ΠΊΠΎΠΌΠΏΠ°ΡΠ°, Π½ΠΎ ΠΎΠ½ΠΎ Π²Π»ΠΈΡΠ΅Ρ Π½Π° ΠΌΠ΅ΡΠ°Π»Π»Ρ, ΡΠ°ΠΊΠΈΠ΅ ΠΊΠ°ΠΊ Π°Π»ΡΠΌΠΈΠ½ΠΈΠΉ, Π½Π° ΠΊΠΎΡΠΎΡΡΠ΅ ΠΌΠ°Π³Π½ΠΈΡΠΎΡΡΠ°ΡΠΈΡΠ΅ΡΠΊΠΎΠ΅ ΠΏΠΎΠ»Π΅ Π½Π΅ ΠΎΠΊΠ°Π·ΡΠ²Π°Π΅Ρ Π²ΠΈΠ΄ΠΈΠΌΠΎΠ³ΠΎ Π΄Π΅ΠΉΡΡΠ²ΠΈΡ, Π½ΠΎ ΡΠ²Π½ΠΎ Π²Π»ΠΈΡΠ΅Ρ Π½Π° Π΄ΡΡΠ³ΠΈΠ΅ ΠΌΠ°Π³Π½ΠΈΡΠ½ΡΠ΅ ΠΏΠΎΠ»Ρ. ΠΠ»Π΅ΠΊΡΡΠΎΡΡΠ°ΡΠΈΡΠ΅ΡΠΊΠΎΠ΅ ΠΏΠΎΠ»Π΅ ΡΠΎΠ·Π΄Π°Π΅ΡΡΡ Π½Π°ΠΏΡΡΠΆΠ΅Π½ΠΈΠ΅ΠΌ ΠΌΠ΅ΠΆΠ΄Ρ ΠΏΡΠΎΡΠΈΠ²ΠΎΠΏΠΎΠ»ΠΎΠΆΠ½ΡΠΌΠΈ ΡΠ»Π΅ΠΊΡΡΠΈΡΠ΅ΡΠΊΠΈΠΌΠΈ Π·Π°ΡΡΠ΄Π°ΠΌΠΈ. ΠΠ»Π΅ΠΊΡΡΠΈΡΠ΅ΡΠΊΠ°Ρ ΡΠΈΠ»Π° Β«ΠΏΠΈΡΠ°Π΅ΡΡΡΒ» ΡΠ°Π·Π½ΠΎΡΡΡΡ Π²Π΅Π»ΠΈΡΠΈΠ½, ΠΊΠΎΡΠΎΡΡΠ΅ ΡΡΡΠ΅ΠΌΡΡΡΡ ΠΎΠ±Π½ΡΠ»ΠΈΡΡΡΡ, Π΄ΠΎΡΡΠΈΠ³Π½ΡΠ² Π½Π΅ΠΉΡΡΠ°Π»ΡΠ½ΠΎΠ³ΠΎ Π·Π½Π°ΡΠ΅Π½ΠΈΡ, ΠΏΠΎΠ΄ΠΎΠ±Π½ΠΎ ΠΏΡΠΎΡΠΈΠ²ΠΎΠΏΠΎΠ»ΠΎΠΆΠ½ΡΠΌ ΡΠΈΡΡΠ΅ΠΌΠ°ΠΌ Π΄Π°Π²Π»Π΅Π½ΠΈΡ Π²ΠΎΠ·Π΄ΡΡ
Π° ΠΈΠ»ΠΈ ΡΡΠΎΠ²Π½Ρ Π²ΠΎΠ΄Ρ Π² ΡΠΎΠ΅Π΄ΠΈΠ½Π΅Π½Π½ΡΡ
Π΅ΠΌΠΊΠΎΡΡΡΡ
. ΠΠ°ΠΌ Π½ΡΠΆΠ΅Π½ ΠΊΠ°ΠΊΠΎΠΉ-ΡΠΎ Π΄ΠΈΡΠ±Π°Π»Π°Π½Ρ Π½Π΅ΠΉΡΡΠ°Π»ΡΠ½ΠΎΠΉ ΠΌΠ°ΡΠ΅ΡΠΈΠΈ, ΡΡΠΎΠ±Ρ ΡΠ΄Π΅Π»Π°ΡΡ Π΅Π΅ ΡΠ»Π΅ΠΊΡΡΠΈΡΠ΅ΡΠΊΠΈ Π·Π°ΡΡΠΆΠ΅Π½Π½ΠΎΠΉ, ΡΡΠΎ ΡΠ°ΡΡΠΎ ΡΠ²ΡΠ·Π°Π½ΠΎ Ρ Π΄ΠΎΠΏΠΎΠ»Π½ΠΈΡΠ΅Π»ΡΠ½ΠΎΠΉ ΡΠ°Π±ΠΎΡΠΎΠΉ. ΠΠ°Π³Π½ΠΈΡΠ½ΠΎΠ΅ ΠΏΠΎΠ»Π΅ ΡΠΏΡΠ°Π²Π»ΡΠ΅ΡΡΡ ΠΏΡΠΎΡΠΈΠ²ΠΎΠΏΠΎΠ»ΠΎΠΆΠ½ΡΠΌΠΈ ΠΎΡΠΈΠ΅Π½ΡΠ°ΡΠΈΡΠΌΠΈ ΠΈ ΠΌΠΎΠΆΠ΅Ρ ΡΠΎΠ·Π΄Π°Π²Π°ΡΡΡΡ ΡΠ»Π΅ΠΊΡΡΠΈΡΠ΅ΡΠΊΠΈ Π½Π΅ΠΉΡΡΠ°Π»ΡΠ½ΠΎΠΉ ΠΌΠ°ΡΠ΅ΡΠΈΠ΅ΠΉ. ΠΠ΅ΡΠ½Π°ΠΌΠ΅Π½ΡΠ½ΡΠ΅ ΠΌΠ°Π³Π½ΠΈΡΡ Π½Π΅ ΡΡΠ΅Π±ΡΡΡ Π΄ΠΎΠΏΠΎΠ»Π½ΠΈΡΠ΅Π»ΡΠ½ΠΎΠΉ ΡΠ°Π±ΠΎΡΡ Π΄Π»Ρ ΡΠΎΠ·Π΄Π°Π½ΠΈΡ ΠΌΠ°Π³Π½ΠΈΡΠ½ΡΡ
ΠΏΠΎΠ»Π΅ΠΉ. ΠΡΠΎΡΠΈΠ²ΠΎΠΏΠΎΠ»ΠΎΠΆΠ½ΡΠ΅ ΠΏΠΎΠ»ΡΡΠ½ΠΎΡΡΠΈ ΠΏΡΠΈΡΡΠ³ΠΈΠ²Π°ΡΡΡΡ Π΄ΡΡΠ³ ΠΊ Π΄ΡΡΠ³Ρ, Π½ΠΎ ΠΎΠ½ΠΈ Π½Π΅ Π½Π΅ΠΉΡΡΠ°Π»ΠΈΠ·ΡΡΡ Π΄ΡΡΠ³ Π΄ΡΡΠ³Π° β Π΅ΡΠ»ΠΈ Π²Ρ ΡΠΎΠ΅Π΄ΠΈΠ½ΠΈΡΠ΅ 2 ΠΌΠ°Π³Π½ΠΈΡΠ°, ΠΎΠ½ΠΈ Π±ΡΠ΄ΡΡ ΡΠ°Π±ΠΎΡΠ°ΡΡ ΠΊΠ°ΠΊ ΠΎΠ΄ΠΈΠ½ ΠΌΠ°Π³Π½ΠΈΡ, ΠΈ ΠΌΠ°Π³Π½ΠΈΡΠ½ΠΎΠ΅ ΠΏΠΎΠ»Π΅ Π±ΡΠ΄Π΅Ρ ΡΠΈΠ»ΡΠ½Π΅Π΅. ΠΡΡΠΎΡΠ½ΠΈΠΊΠΎΠΌ ΡΠ»Π΅ΠΊΡΡΠΈΡΠ΅ΡΠΊΠΎΠ³ΠΎ ΠΏΠΎΠ»Ρ ΡΠ²Π»ΡΠ΅ΡΡΡ ΡΠ»Π΅ΠΊΡΡΠΈΡΠ΅ΡΠΊΠΈΠΉ Π·Π°ΡΡΠ΄ ΡΡΠ±Π°ΡΠΎΠΌΠ½ΠΎΠΉ ΡΠ°ΡΡΠΈΡΡ, Π° ΠΈΡΡΠΎΡΠ½ΠΈΠΊΠΎΠΌ ΠΌΠ°Π³Π½ΠΈΡΠ½ΠΎΠ³ΠΎ ΠΏΠΎΠ»Ρ — Π΅Π΅ ΠΊΠ²Π°Π½ΡΠΎΠ²ΡΠΉ ΡΠΏΠΈΠ½. ΡΡΠΎ Π΄Π²Π° ΡΠ°Π·Π½ΡΡ
Π²Π½ΡΡΡΠ΅Π½Π½ΠΈΡ
ΡΠ²ΠΎΠΉΡΡΠ²Π° ΡΠ°ΡΡΠΈΡΡ, ΠΈ ΠΎΠ±Π° ΠΎΠ΄ΠΈΠ½Π°ΠΊΠΎΠ²ΠΎ Π²Π°ΠΆΠ½Ρ … Π‘ΠΎΠ³Π»Π°ΡΠ½ΠΎ ΠΎΡΠΈΡΠΈΠ°Π»ΡΠ½ΠΎΠΉ Π½Π°ΡΠΊΠ΅, Π°ΡΠΎΠΌΡ ΠΌΠΎΠ³ΡΡ Π³Π΅Π½Π΅ΡΠΈΡΠΎΠ²Π°ΡΡ ΠΌΠ°Π³Π½ΠΈΡΠ½ΡΠ΅ ΠΏΠΎΠ»Ρ ΠΈΠ·-Π·Π° ΡΠ»Π΅ΠΊΡΡΠΎΠ½ΠΎΠ², ΠΊΠΎΡΠΎΡΡΠ΅ Π΄Π²ΠΈΠΆΡΡΡΡ Π²ΠΎΠΊΡΡΠ³ ΡΠ΄ΡΠ°, ΡΠΎΠ·Π΄Π°Π²Π°Ρ ΠΌΠ°Π³Π½ΠΈΡΠ½ΠΎΠ΅ ΠΏΠΎΠ»Π΅ Π·Π° ΡΡΠ΅Ρ ΠΈΠ½Π΄ΡΠΊΡΠΈΠΈ. ΠΡΠΎΠ±Π»Π΅ΠΌΠ° Π² ΡΠΎΠΌ, ΡΡΠΎ ΠΊΠΎΠ½ΡΠ΅ΠΏΡΠΈΡ ΡΠ»Π΅ΠΊΡΡΠΎΠ½Π°, Π΄Π²ΠΈΠΆΡΡΠ΅Π³ΠΎΡΡ ΠΏΠΎ ΠΎΡΠ±ΠΈΡΠ΅, ΠΊΠ°ΠΊ ΠΏΠ»Π°Π½Π΅ΡΠ° Π²ΠΎΠΊΡΡΠ³ Π·Π²Π΅Π·Π΄Ρ, ΡΠΎΠ²Π΅ΡΡΠ΅Π½Π½ΠΎ Π½Π΅Π²Π΅ΡΠ½Π°. ΠΠ»Π΅ΠΊΡΡΠΎΠ½Ρ ΡΠΎΠ·Π΄Π°ΡΡ ΠΎΠ±Π»Π°ΠΊΠ°, Π² ΠΊΠΎΡΠΎΡΡΡ
ΠΈΡ
ΠΏΠΎΠ»ΠΎΠΆΠ΅Π½ΠΈΠ΅, ΡΠΊΠΎΡΠΎΡΡΡ ΠΈ ΠΎΡΠΈΠ΅Π½ΡΠ°ΡΠΈΡ ΠΊΠ²Π°Π½ΡΠΎΠ²ΠΎΠ³ΠΎ ΡΠΏΠΈΠ½Π° ΠΎΡΡΠ°ΡΡΡΡ Π² ΠΏΠΎΡΡΠΎΡΠ½Π½ΠΎΠΉ ΡΡΠΏΠ΅ΡΠΏΠΎΠ·ΠΈΡΠΈΠΈ ΠΈ Π½Π΅ ΠΎΠΏΡΠ΅Π΄Π΅Π»ΡΡΡΡΡ Π΄ΠΎ ΡΠ΅Ρ
ΠΏΠΎΡ, ΠΏΠΎΠΊΠ° Π½Π΅ Π±ΡΠ΄ΡΡ ΠΈΠ·ΠΌΠ΅ΡΠ΅Π½Ρ, ΠΏΠΎΡΡΠΎΠΌΡ Ρ ΡΠ»Π΅ΠΊΡΡΠΎΠ½Π½ΠΎΠ³ΠΎ ΠΎΠ±Π»Π°ΠΊΠ° Π½Π΅Ρ Π°Π±ΡΠΎΠ»ΡΡΠ½ΠΎ Π½ΠΈΠΊΠ°ΠΊΠΎΠΉ Π²ΠΎΠ·ΠΌΠΎΠΆΠ½ΠΎΡΡΠΈ ΠΈΠ½Π΄ΡΡΠΈΡΠΎΠ²Π°ΡΡ ΠΎΠΏΡΠ΅Π΄Π΅Π»Π΅Π½Π½ΠΎΠ΅ ΠΈ ΠΎΠ΄Π½ΠΎΡΠΎΠ΄Π½ΠΎΠ΅ ΠΌΠ°Π³Π½ΠΈΡΠ½ΠΎΠ΅ ΠΏΠΎΠ»Π΅. Π, Π½Π°ΠΊΠΎΠ½Π΅Ρ… ΠΠ΅Ρ. Π½Π° ΡΠ°ΠΌΠΎΠΌ Π΄Π΅Π»Π΅ ΠΈΠ·Π²Π΅ΡΡΠ½ΠΎ, ΡΡΠΎ ΠΌΠΎΠΆΠ΅Ρ Π±ΡΡΡ ΡΠΈΠ·ΠΈΡΠ΅ΡΠΊΠΈΠΌ Π½ΠΎΡΠΈΡΠ΅Π»Π΅ΠΌ Π΄Π»Ρ «Π²ΠΈΡΡΡΠ°Π»ΡΠ½ΡΡ
» ΡΠΈΠ»ΠΎΠ²ΡΡ
Π»ΠΈΠ½ΠΈΠΉ Π² ΠΏΡΠΎΡΡΡΠ°Π½ΡΡΠ²Π΅ — Π° ΡΡΠ°ΡΠΈΡΠ΅ΡΠΊΠΈΠ΅ ΠΏΠΎΠ»Ρ ΠΌΠΎΠ³ΡΡ Π²Π·Π°ΠΈΠΌΠΎΠ΄Π΅ΠΉΡΡΠ²ΠΎΠ²Π°ΡΡ Π΄ΡΡΠ³ Ρ Π΄ΡΡΠ³ΠΎΠΌ Π½Π° ΠΠΠ ΠΠΠΠΠ ΡΠ°ΡΡΡΠΎΡΠ½ΠΈΠΈ (ΡΡΡΡΡΠΈ ΡΠ²Π΅ΡΠΎΠ²ΡΡ
Π»Π΅Ρ) — ΡΠ°ΠΊ ΡΡΠΎ Π²ΠΈΡΡΡΠ°Π»ΡΠ½ΡΠ΅ ΡΠΎΡΠΎΠ½Ρ ΡΠ°Π±ΠΎΡΠ°ΡΡ Π½Π΅ Π±ΡΠ΄ΡΡ. ΠΠ΄Π½Π°ΠΊΠΎ Π½Π° «ΠΎΠ±ΡΡΠ½ΡΠ΅» ΡΠΎΡΠΎΠ½Ρ Π½ΠΈΠΊΠ°ΠΊΠΈΠΌ Π²ΠΈΠ΄ΠΈΠΌΡΠΌ ΠΎΠ±ΡΠ°Π·ΠΎΠΌ Π½Π΅ Π²Π»ΠΈΡΡΡ ΡΠ»Π΅ΠΊΡΡΠΎΡΡΠ°ΡΠΈΡΠ΅ΡΠΊΠΈΠ΅ ΠΈ ΠΌΠ°Π³Π½ΠΈΡΠ½ΡΠ΅ ΠΏΠΎΠ»Ρ — ΠΏΠΎΡΡΠΎΠΌΡ ΠΏΠΎ Π»ΠΎΠ³ΠΈΠΊΠ΅ ΠΎΠ½ΠΈ Π½Π΅ ΠΌΠΎΠ³ΡΡ Π½Π΅ΡΡΠΈ ΠΈΡ
ΠΊΠ°ΠΊ ΠΎΡΠ΄Π΅Π»ΡΠ½ΡΠ΅ ΡΠΈΠ»Ρ. ΠΠ Π²ΡΠ΅ ΡΠ»Π΅ΠΊΡΡΠΎΠΌΠ°Π³Π½ΠΈΡΠ½ΡΠ΅ Π²ΠΎΠ»Π½Ρ Π²ΠΊΠ»ΡΡΠ°ΡΡ Π² ΡΠ΅Π±Ρ ΠΌΠ°Π³Π½ΠΈΡΠ½ΡΠ΅ ΠΈ ΡΠ»Π΅ΠΊΡΡΠΈΡΠ΅ΡΠΊΠΈΠ΅ ΠΊΠΎΠΌΠΏΠΎΠ½Π΅Π½ΡΡ. ΠΡΠΎ Π½Π΅ Π±ΠΎΠ»Π΅Π΅, ΡΠ΅ΠΌ ΠΏΡΠ΅Π΄ΠΏΠΎΠ»ΠΎΠΆΠ΅Π½ΠΈΠ΅, Π½ΠΎ ΠΠΠΠΠ’ Π±ΡΡΡ, ΡΡΠ°ΡΠΈΡΠ΅ΡΠΊΠΈΠ΅ ΠΏΠΎΠ»Ρ Π΄Π΅ΠΉΡΡΠ²ΠΈΡΠ΅Π»ΡΠ½ΠΎ ΠΌΠΎΠ³ΡΡ ΠΈΠ·ΠΌΠ΅Π½ΠΈΡΡ ΡΡΠΈ Π·Π½Π°ΡΠ΅Π½ΠΈΡ ΠΈ ΠΏΠΎΠ»ΡΡΠΈΠ·ΠΎΠ²Π°ΡΡ ΡΠ²Π΅Ρ, ΡΡΠΎ Π±Ρ ΠΏΡΠ΅Π²ΡΠ°ΡΠΈΡΡ Π΅Π³ΠΎ Π² Π½ΠΎΡΠΈΡΠ΅Π»Ρ ΡΠΈΠ»ΠΎΠ²ΡΡ
Π»ΠΈΠ½ΠΈΠΉ???
— ΠΡΡΡΠ°Π» (Π²ΠΎΠ·ΡΠ°ΡΡ 33 Π³ΠΎΠ΄Π°)
ΠΠΎΠ»ΡΡΠ°
A:
ΠΠ΄Π΅ΡΡ Π΅ΡΡΡ ΠΈΠ·ΡΡΠ½ Π² Π²Π°ΡΠΈΡ ΡΠ°ΡΡΡΠΆΠ΄Π΅Π½ΠΈΡΡ : «»Π½ΠΎΡΠΌΠ°Π»ΡΠ½ΡΠ΅» ΡΠΎΡΠΎΠ½Ρ Π½ΠΈΠΊΠ°ΠΊΠΈΠΌ Π²ΠΈΠ΄ΠΈΠΌΡΠΌ ΠΎΠ±ΡΠ°Π·ΠΎΠΌ Π½Π΅ ΠΏΠΎΠ΄Π²Π΅ΡΠΆΠ΅Π½Ρ Π²Π»ΠΈΡΠ½ΠΈΡ ΡΠ»Π΅ΠΊΡΡΠΎΡΡΠ°ΡΠΈΡΠ΅ΡΠΊΠΈΡ ΠΈ ΠΌΠ°Π³Π½ΠΈΡΠ½ΡΡ ΠΏΠΎΠ»Π΅ΠΉ — ΠΏΠΎΡΡΠΎΠΌΡ ΠΏΠΎ Π»ΠΎΠ³ΠΈΠΊΠ΅ ΠΎΠ½ΠΈ Π½Π΅ ΠΌΠΎΠ³ΡΡ ΠΈΡ Π½Π΅ΡΡΠΈ , ΠΊΠ°ΠΊ ΠΎΡΠ΄Π΅Π»ΡΠ½ΡΠ΅ ΡΠΈΠ»ΡΒ». ΠΠΎΠΊΠ° ΡΡΠ°Π²Π½Π΅Π½ΠΈΡ Π΄Π»Ρ ΠΏΠΎΠ»Ρ Π»ΠΈΠ½Π΅ΠΉΠ½Ρ, ΠΊΠ°ΠΊ ΠΈ ΠΊΠ»Π°ΡΡΠΈΡΠ΅ΡΠΊΠΈΠ΅ ΡΠ»Π΅ΠΊΡΡΠΎΠΌΠ°Π³Π½ΠΈΡΠ½ΡΠ΅ ΡΡΠ°Π²Π½Π΅Π½ΠΈΡ, Π½ΠΈ ΠΎΠ΄ΠΈΠ½ ΠΊΠΎΠΌΠΏΠΎΠ½Π΅Π½Ρ ΠΏΠΎΠ»Ρ Π½Π΅ Π²Π»ΠΈΡΠ΅Ρ Π½Π° ΠΏΠΎΠ²Π΅Π΄Π΅Π½ΠΈΠ΅ Π»ΡΠ±ΠΎΠ³ΠΎ Π΄ΡΡΠ³ΠΎΠ³ΠΎ ΠΊΠΎΠΌΠΏΠΎΠ½Π΅Π½ΡΠ°. Π’Π°ΠΊΠΈΠΌ ΠΎΠ±ΡΠ°Π·ΠΎΠΌ, ΠΎΡΡΡΡΡΡΠ²ΠΈΠ΅ Π²Π·Π°ΠΈΠΌΠΎΠ΄Π΅ΠΉΡΡΠ²ΠΈΡ ΠΌΠ΅ΠΆΠ΄Ρ ΡΡΠ°ΡΠΈΡΠ΅ΡΠΊΠΈΠΌΠΈ ΠΏΠΎΠ»ΡΠΌΠΈ ΠΈ ΡΠ°ΡΠΏΡΠΎΡΡΡΠ°Π½ΡΡΡΠΈΠΌΠΈΡΡ ΠΏΡΠΎΡΡΠΎ Π³ΠΎΠ²ΠΎΡΠΈΡ ΠΎ ΡΠΎΠΌ, ΡΡΠΎ ΡΡΠ°Π²Π½Π΅Π½ΠΈΡ Π»ΠΈΠ½Π΅ΠΉΠ½Ρ, Π° Π½Π΅ ΠΎ ΡΠΎΠΌ, ΠΌΠΎΠ³ΡΡ Π»ΠΈ ΠΎΠ΄Π½ΠΈ ΠΈ ΡΠ΅ ΠΆΠ΅ ΡΡΠ½Π΄Π°ΠΌΠ΅Π½ΡΠ°Π»ΡΠ½ΡΠ΅ ΡΠΈΠΏΡ ΠΏΠΎΠ»Π΅ΠΉ Π΄Π΅ΠΌΠΎΠ½ΡΡΡΠΈΡΠΎΠ²Π°ΡΡ ΡΠ°ΠΊΠΎΠ΅ ΡΠ°Π·Π»ΠΈΡΠ½ΠΎΠ΅ ΠΏΠΎΠ²Π΅Π΄Π΅Π½ΠΈΠ΅.
ΠΠ°Ρ Π°ΡΠ³ΡΠΌΠ΅Π½Ρ ΠΎ ΡΠΎΠΌ, ΡΡΠΎ Π²ΠΈΡΡΡΠ°Π»ΡΠ½ΡΠ΅ ΡΠΎΡΠΎΠ½Ρ Π½Π΅ ΠΌΠΎΠ³ΡΡ Π²ΡΠ·ΡΠ²Π°ΡΡ Π΄Π°Π»ΡΠ½ΠΎΠ΄Π΅ΠΉΡΡΠ²ΡΡΡΠΈΡ (1/ΠΊΠ²Π°Π΄ΡΠ°Ρ ΡΠ°ΡΡΡΠΎΡΠ½ΠΈΡ) ΡΡΠ°ΡΠΈΡΠ΅ΡΠΊΠΈΡ Π²Π·Π°ΠΈΠΌΠΎΠ΄Π΅ΠΉΡΡΠ²ΠΈΠΉ, Π½Π΅Π²Π΅ΡΠ΅Π½. ΠΠΌΠ΅Π½Π½ΠΎ ΡΠ°ΠΊΠΎΠ΅ ΠΏΠΎΠ²Π΅Π΄Π΅Π½ΠΈΠ΅ ΠΎΠΆΠΈΠ΄Π°Π΅ΡΡΡ Π΄Π»Ρ Π²ΠΈΡΡΡΠ°Π»ΡΠ½ΠΎΠΉ ΡΠΎΡΠΎΠ½Π½ΠΎΠΉ ΠΊΠ°ΡΡΠΈΠ½Ρ.
Mike W.
(ΠΎΠΏΡΠ±Π»ΠΈΠΊΠΎΠ²Π°Π½ΠΎ 01.11.2017)
ΠΠΎΠΏΠΎΠ»Π½Π΅Π½ΠΈΠ΅ β10: ΠΏΠΎΠ»ΡΡΠΈΠ·ΠΎΠ²Π°Π½Π½ΡΠΉ ΡΠ²Π΅Ρ ΠΈ ΠΊΠΎΠΌΠΏΠ°ΡΡ Π΄Π»Ρ ΠΏΡΠΈΡ
Q:
ΠΌΠ°Π³Π½ΠΈΡΠ½ΡΠ΅ ΠΈΠ»ΠΈ ΡΠ»Π΅ΠΊΡΡΠΎΡΡΠ°ΡΠΈΡΠ΅ΡΠΊΠΈΠ΅ ΠΏΠΎΠ»Ρ. ΠΠ° ΡΠ°ΠΌΠΎΠΌ Π΄Π΅Π»Π΅, Π΅ΡΡΡ Π½Π΅Π΄Π°Π²Π½Π΅Π΅ ΠΈΡΡΠ»Π΅Π΄ΠΎΠ²Π°Π½ΠΈΠ΅, ΠΊΠΎΡΠΎΡΠΎΠ΅ ΠΏΠΎΠΊΠ°Π·ΡΠ²Π°Π΅Ρ, ΡΡΠΎ Β«Π²Π½ΡΡΡΠ΅Π½Π½ΠΈΠΉ ΠΊΠΎΠΌΠΏΠ°ΡΒ» ΠΏΡΠΈΡ ΡΡ
ΠΎΠ΄ΠΈΡ Ρ ΡΠΌΠ°, ΠΊΠΎΠ³Π΄Π° ΠΎΠ½ΠΈ ΠΏΠΎΠ΄Π²Π΅ΡΠ³Π°ΡΡΡΡ Π²ΠΎΠ·Π΄Π΅ΠΉΡΡΠ²ΠΈΡ ΠΏΠΎΠ»ΡΡΠΈΠ·ΠΎΠ²Π°Π½Π½ΠΎΠ³ΠΎ ΡΠ²Π΅ΡΠ°: http://physicsworld.com/cws/article/news/2016/feb/03/polarized -ΡΠ²Π΅Ρ-ΡΠ±ΡΠ°ΡΡΠ²Π°Π΅Ρ-ΠΏΡΠΈΡ-ΠΌΠ°Π³Π½ΠΈΡΠ½ΡΠΉ-ΠΊΠΎΠΌΠΏΠ°Ρ-Ρ ΠΊΡΡΡΠ°ΠΠΎΠ²ΠΎΡΡΡ, ΡΡΠΎ ΠΏΠΎΠ»ΡΡΠΈΠ·ΠΎΠ²Π°Π½Π½ΡΠΉ ΡΠ²Π΅Ρ ΠΏΠΎΠ·Π²ΠΎΠ»ΡΠ΅Ρ ΠΏΡΠΈΡΠ°ΠΌ Β«Π²ΠΎΡΠΏΡΠΈΠ½ΠΈΠΌΠ°ΡΡΒ» ΠΌΠ΅ΡΡΠ½ΠΎΠ΅ ΠΌΠ°Π³Π½ΠΈΡΠ½ΠΎΠ΅ ΠΏΠΎΠ»Π΅. ΠΡΠΎ, Π²Π΅ΡΠΎΡΡΠ½ΠΎ, Π΄ΠΎΠΊΠ°Π·ΡΠ²Π°Π΅Ρ, ΡΡΠΎ ΠΈΠ½ΡΠΎΡΠΌΠ°ΡΠΈΡ ΠΎ ΠΏΠΎΠ»Π΅ Β«Π·Π°ΠΏΠΈΡΡΠ²Π°Π΅ΡΡΡΒ» Π² ΠΏΠΎΠ»ΡΡΠΈΠ·Π°ΡΠΈΠΈ ΡΠ²Π΅ΡΠΎΠ²ΠΎΠΉ Π²ΠΎΠ»Π½Ρ…
— ΠΡΡΡΠ°Π» (Π²ΠΎΠ·ΡΠ°ΡΡ 33 Π³ΠΎΠ΄Π°)
ΠΠΎΠ»ΡΡΠ°
A:
ΠΡΠΎ ΠΎΡΠ΅Π½Ρ ΠΊΠ»Π°ΡΡΠ½ΡΠΉ ΡΠ΅Π·ΡΠ»ΡΡΠ°Ρ, ΡΠ±Π΅Π΄ΠΈΡΠ΅Π»ΡΠ½ΠΎ ΠΏΠΎΠ΄ΡΠ²Π΅ΡΠΆΠ΄Π°ΡΡΠΈΠΉ ΡΠ΅ΠΎΡΠΈΡ ΠΌΠΎΠ΅Π³ΠΎ ΠΏΠΎΠΊΠΎΠΉΠ½ΠΎΠ³ΠΎ ΠΊΠΎΠ»Π»Π΅Π³ΠΈ ΠΠ»Π°ΡΡΠ° Π¨ΡΠ»ΡΡΠ΅Π½Π° ΠΎΠ± ΠΎΠ΄Π½ΠΎΠΌ ΡΠΏΠΎΡΠΎΠ±Π΅ Π²ΠΎΡΠΏΡΠΈΡΡΠΈΡ ΠΏΡΠΈΡΠ°ΠΌΠΈ ΠΌΠ°Π³Π½ΠΈΡΠ½ΡΡ ΠΏΠΎΠ»Π΅ΠΉ. ΠΠ΅Ρ Π°Π½ΠΈΠ·ΠΌ Π²ΠΊΠ»ΡΡΠ°Π΅Ρ Π½Π΅ΠΊΠΎΡΠΎΡΡΠ΅ Π½Π΅ΡΠ°Π²Π½ΠΎΠ²Π΅ΡΠ½ΡΠ΅ ΡΠΊΠΎΡΠΎΡΡΠΈ Ρ ΠΈΠΌΠΈΡΠ΅ΡΠΊΠΈΡ ΡΠ΅Π°ΠΊΡΠΈΠΉ, ΠΊΠΎΡΠΎΡΡΠ΅ Π² Π½Π΅ΠΎΠ±ΡΡΠ°ΠΉΠ½ΠΎΠΉ ΡΡΠ΅ΠΏΠ΅Π½ΠΈ Π·Π°Π²ΠΈΡΡΡ ΠΎΡ ΠΌΠ°Π³Π½ΠΈΡΠ½ΡΡ ΠΏΠΎΠ»Π΅ΠΉ. ΠΠΎΠ»ΡΡΠΈΠ·Π°ΡΠΈΡ ΡΠ²Π΅ΡΠ° ΡΠ»ΡΠΆΠΈΡ ΡΠΎΠ»ΡΠΊΠΎ Π΄Π»Ρ ΡΠΎΠ³ΠΎ, ΡΡΠΎΠ±Ρ Π²ΡΠ·Π²Π°ΡΡ ΠΏΡΠ°Π²ΠΈΠ»ΡΠ½ΡΡ Ρ ΠΈΠΌΠΈΡ Π² Π³Π»Π°Π·Π°Ρ ΠΏΡΠΈΡ. ΠΠ½ΡΠΎΡΠΌΠ°ΡΠΈΡ ΠΎ ΠΏΠΎΠ»Π΅ Π²ΠΎΠΎΠ±ΡΠ΅ ΠΎΡΡΡΡΡΡΠ²ΡΠ΅Ρ Π² ΡΠ²Π΅ΡΠΎΠ²ΠΎΠΉ Π²ΠΎΠ»Π½Π΅.
ΠΠ°ΠΉΠΊ Π.
(ΠΎΠΏΡΠ±Π»ΠΈΠΊΠΎΠ²Π°Π½ΠΎ 10.11.2017)
ΠΠΎΠΏΠΎΠ»Π½Π΅Π½ΠΈΠ΅ β11: ΠΌΠ°Π³Π½ΠΈΡΠΎΠΎΠΏΡΠΈΠΊΠ°
ΠΠΎΠΏΡΠΎΡ:
Π ΠΊΠ°ΠΊ Π½Π°ΡΡΠ΅Ρ ΠΌΠ°Π³Π½ΠΈΡΠΎΠΎΠΏΡΠΈΡΠ΅ΡΠΊΠΈΡ
Π΄Π°ΡΡΠΈΠΊΠΎΠ², ΡΠΏΠΎΡΠΎΠ±Π½ΡΡ
ΡΠ΅Π°Π»ΡΠ½ΠΎ Π²ΠΈΠ·ΡΠ°Π»ΠΈΠ·ΠΈΡΠΎΠ²Π°ΡΡ ΠΌΠ°Π³Π½ΠΈΡΠ½ΡΠ΅ ΠΏΠΎΠ»Ρ Ρ ΠΏΠΎΠΌΠΎΡΡΡ ΠΏΠΎΠ»ΡΡΠΈΠ·ΠΎΠ²Π°Π½Π½ΡΡ
Π»Π΅Π³ΠΊΠΈΠΉ: https://www.rdmag.com/content/new-sensors-optically-visualize-magnet-fields ΠΠ° Π»ΠΈΠ½Π΅ΠΉΠ½ΠΎ ΠΏΠΎΠ»ΡΡΠΈΠ·ΠΎΠ²Π°Π½Π½ΡΠΉ ΡΠ²Π΅Ρ Π²ΠΎΠ·Π΄Π΅ΠΉΡΡΠ²ΡΡΡ ΠΌΠ°Π³Π½ΠΈΡΠ½ΡΠ΅ ΠΏΠΎΠ»Ρ, ΠΈ ΡΡΠΎΡ ΡΡΡΠ΅ΠΊΡ ΠΌΠΎΠΆΠ½ΠΎ Π½Π°Π±Π»ΡΠ΄Π°ΡΡ Π² ΡΠ΅ΠΆΠΈΠΌΠ΅ ΡΠ΅Π°Π»ΡΠ½ΠΎΠ³ΠΎ Π²ΡΠ΅ΠΌΠ΅Π½ΠΈ. Π’Π°ΠΊ ΡΡΠΎ ΠΆΠ΅ ΡΠ²Π»ΡΠ΅ΡΡΡ Π² Π΄Π°Π½Π½ΠΎΠΌ ΡΠ»ΡΡΠ°Π΅ Π½Π°ΠΈΠ±ΠΎΠ»Π΅Π΅ ΠΎΡΠ΅Π²ΠΈΠ΄Π½ΡΠΌ Π½ΠΎΡΠΈΡΠ΅Π»Π΅ΠΌ ΠΈΠ½ΡΠΎΡΠΌΠ°ΡΠΈΠΈ ΠΎ ΠΌΠ°Π³Π½ΠΈΡΠ½ΡΡ
ΠΏΠΎΠ»ΡΡ
? Π€ΠΎΡΠΎΠ½Ρ — Π° ΡΠΎΡΠ½Π΅Π΅ ΠΌΠ°Π³Π½ΠΈΡΠ½Π°Ρ ΡΠΎΡΡΠ°Π²Π»ΡΡΡΠ°Ρ ΠΠ Π²ΠΎΠ»Π½. Π‘Π²ΠΎΠΉΡΡΠ²Π° ΡΠΎΡΠΎΠ½ΠΎΠ² ΠΎΠΏΡΠ΅Π΄Π΅Π»ΡΡΡΡΡ ΠΈΡΡΠΎΡΠ½ΠΈΠΊΠΎΠΌ ΠΈΠ·Π»ΡΡΠ΅Π½ΠΈΡ, ΠΊΠ°ΠΊ ΡΡΠΏΠ΅ΡΠΏΠΎΠ·ΠΈΡΠΈΠ΅ΠΉ 3-Ρ
ΡΠΎΡΡΠ°Π²Π»ΡΡΡΠΈΡ
(ΠΠ ΠΈ ΡΠ°ΡΠΏΡΠΎΡΡΡΠ°Π½Π΅Π½ΠΈΡ). ΠΠΈΠ½Π΅ΠΉΠ½Π°Ρ ΠΏΠΎΠ»ΡΡΠΈΠ·Π°ΡΠΈΡ ΡΠ²Π΅ΡΠ° ΠΎΠΏΡΠ΅Π΄Π΅Π»ΡΠ΅Ρ Π΅Π³ΠΎ ΡΠ°ΡΠΏΡΠΎΡΡΡΠ°Π½Π΅Π½ΠΈΠ΅ β ΠΏΡΠΈ ΡΡΠΎΠΌ Π²ΠΎΠ·Π΄Π΅ΠΉΡΡΠ²ΠΈΠ΅ Π²Π½Π΅ΡΠ½Π΅Π³ΠΎ ΠΏΠΎΠ»Ρ Π²Π»ΠΈΡΠ΅Ρ Π½Π° ΡΠ»Π΅ΠΊΡΡΠΎΠΌΠ°Π³Π½ΠΈΡΠ½ΡΠ΅ ΡΠΎΡΡΠ°Π²Π»ΡΡΡΠΈΠ΅ β ΠΈ ΡΡΠΎΡ ΡΡΡΠ΅ΠΊΡ ΠΌΠΎΠΆΠ½ΠΎ ΠΈΠ·ΠΌΠ΅ΡΠΈΡΡ.
— ΠΡΡΡΠ°Π» (Π²ΠΎΠ·ΡΠ°ΡΡ 33 Π³ΠΎΠ΄Π°)
ΠΠΎΠ»ΡΡΠ°
A:
ΠΠ½ΠΎΠ³ΠΈΠ΅ ΠΌΠ°ΡΠ΅ΡΠΈΠ°Π»Ρ Π΄Π΅ΠΌΠΎΠ½ΡΡΡΠΈΡΡΡΡ ΡΡΡΠ΅ΠΊΡΡ Π€Π°ΡΠ°Π΄Π΅Ρ ΠΈ ΡΡΡΠ΅ΠΊΡΡ ΠΠ΅ΡΡΠ° — ΡΠΏΠΎΡΠΎΠ±Ρ, Π² ΠΊΠΎΡΠΎΡΡΡ ΡΠ°ΡΠΏΡΠΎΡΡΡΠ°Π½Π΅Π½ΠΈΠ΅ ΠΈΠ»ΠΈ ΠΎΡΡΠ°ΠΆΠ΅Π½ΠΈΠ΅ ΠΏΠΎΠ»ΡΡΠΈΠ·ΠΎΠ²Π°Π½Π½ΠΎΠ³ΠΎ ΡΠ²Π΅ΡΠ° Π·Π°Π²ΠΈΡΠΈΡ ΠΎΡ ΠΌΠ°Π³Π½ΠΈΡΠ½ΠΎΠ³ΠΎ ΠΏΠΎΠ»Ρ Π½Π° ΠΌΠ°ΡΠ΅ΡΠΈΠ°Π»Π΅. Π£ΡΠ²Π΅ΡΠΆΠ΄Π΅Π½ΠΈΠ΅, ΡΡΠΎ ΠΌΠ°Π³Π½ΠΈΡΠ½ΠΎΠ΅ ΠΏΠΎΠ»Π΅ ΠΈΠ·ΠΌΠ΅Π½ΡΠ΅Ρ ΠΎΠΏΡΠΈΡΠ΅ΡΠΊΠΈΠ΅ ΡΠ²ΠΎΠΉΡΡΠ²Π° ΠΌΠ°ΡΠ΅ΡΠΈΠ°Π»ΠΎΠ², ΡΠΈΠ»ΡΠ½ΠΎ ΠΎΡΠ»ΠΈΡΠ°Π΅ΡΡΡ ΠΎΡ ΡΡΠ²Π΅ΡΠΆΠ΄Π΅Π½ΠΈΡ, ΡΡΠΎ ΡΠ°ΠΌΠΈ ΡΠ²Π΅ΡΠΎΠ²ΡΠ΅ Π²ΠΎΠ»Π½Ρ ΡΠΆΠ΅ Π½Π΅ΡΡΡ ΠΈΠ½ΡΠΎΡΠΌΠ°ΡΠΈΡ ΠΎΠ± ΡΡΠΈΡ ΡΡΠ°ΡΠΈΡΠ΅ΡΠΊΠΈΡ ΠΏΠΎΠ»ΡΡ Π΄Π°ΠΆΠ΅ Π² Π²Π°ΠΊΡΡΠΌΠ΅.
ΠΠ°ΠΉΠΊ Π.
(ΠΎΠΏΡΠ±Π»ΠΈΠΊΠΎΠ²Π°Π½ΠΎ 16.11.2017)
Follow-up on this answer
Related Questions
re-magnetizing magnets
magnets with signatures
heated ferromagnet re-magnetized
magnet questions
magnetic poles attract
ΠΠ»ΠΈΡΡΡ Π»ΠΈ ΡΠ΅ΠΏΠ»ΠΎ ΠΈ Ρ ΠΎΠ»ΠΎΠ΄ Π½Π° ΠΌΠ°Π³Π½ΠΈΡ?0047
ΠΌΠ°Π³Π½ΠΈΡΡ ΠΈ Π³ΡΠ°Π²ΠΈΡΠ°ΡΠΈΡ
ΠΠ΅ΠΌΠ½ΠΎΠΉ ΠΌΠ°Π³Π½Π΅ΡΠΈΠ·ΠΌ ΠΈ Π³ΡΠ°Π²ΠΈΡΠ°ΡΠΈΡ
ΠΡΠ΅ Π΅ΡΠ΅ Π»ΡΠ±ΠΎΠΏΡΡΠ½ΠΎ?
ΠΠΎΠΏΡΠΎΡΡ ΠΈ ΠΎΡΠ²Π΅ΡΡ ΠΏΠΎ Expore Π² ΡΠ²ΡΠ·Π°Π½Π½ΡΡ ΠΊΠ°ΡΠ΅Π³ΠΎΡΠΈΡΡ
- ΠΠ°Π³Π½ΠΈΡΡ
Π§Π°ΡΡΠΈΡΡ Π² ΠΌΠ°Π³Π½ΠΈΡΠ½ΡΡ ΠΏΠΎΠ»ΡΡ : ΡΠΈΠΏΡ
ΠΠ½Π°Π΅ΡΠ΅ Π»ΠΈ Π²Ρ, ΡΡΠΎ ΠΌΠ°Π³Π½Π΅ΡΠΈΠ·ΠΌ ΠΎΠ±Π΅ΡΠΏΠ΅ΡΠΈΠ²Π°Π΅Ρ ΡΠ°Π±ΠΎΡΡ ΠΌΠ°Π³Π½Π΅ΡΡΠΎΠ½Π°, Π° Π·Π½Π°ΡΠΈΡ, ΠΈ ΠΌΠΈΠΊΡΠΎΠ²ΠΎΠ»Π½ΠΎΠ²ΠΎΠΉ ΠΏΠ΅ΡΠΈ? ΠΠ΄Π½Π°ΠΊΠΎ Π΄ΠΎ ΠΈΠ·ΠΎΠ±ΡΠ΅ΡΠ΅Π½ΠΈΡ ΠΌΠΈΠΊΡΠΎΠ²ΠΎΠ»Π½ΠΎΠ²ΠΎΠΉ ΠΏΠ΅ΡΠΈ ΠΌΠ°Π³Π½Π΅ΡΡΠΎΠ½Ρ ΡΠ°ΠΊΡΠΈΡΠ΅ΡΠΊΠΈ ΠΈΡΠΏΠΎΠ»ΡΠ·ΠΎΠ²Π°Π»ΠΈΡΡ Π² ΡΠ°Π΄ΠΈΠΎΠ»ΠΎΠΊΠ°ΡΠΈΠΎΠ½Π½ΡΡ ΡΠΈΡΡΠ΅ΠΌΠ°Ρ Π²ΠΎ Π²ΡΠ΅ΠΌΡ ΠΡΠΎΡΠΎΠΉ ΠΌΠΈΡΠΎΠ²ΠΎΠΉ Π²ΠΎΠΉΠ½Ρ! ΠΠΈΠΊΡΠΎΠ²ΠΎΠ»Π½Ρ β ΡΡΠΎ ΡΠ»Π΅ΠΊΡΡΠΎΠΌΠ°Π³Π½ΠΈΡΠ½ΡΠ΅ Π²ΠΎΠ»Π½Ρ Ρ Π΄Π»ΠΈΠ½ΠΎΠΉ Π²ΠΎΠ»Π½Ρ ΠΎΡ ΠΌΠΈΠ»Π»ΠΈΠΌΠ΅ΡΡΠ° Π΄ΠΎ ΠΌΠ΅ΡΡΠ°. ΠΡ ΠΌΠΎΠΆΠ½ΠΎ ΠΈΡΠΊΡΡΡΡΠ²Π΅Π½Π½ΠΎ ΡΠ³Π΅Π½Π΅ΡΠΈΡΠΎΠ²Π°ΡΡ Ρ ΠΏΠΎΠΌΠΎΡΡΡ ΡΡΡΡΠΎΠΉΡΡΠ²Π° ΠΏΠΎΠ΄ Π½Π°Π·Π²Π°Π½ΠΈΠ΅ΠΌ 9.0059 ΠΌΠ°Π³Π½Π΅ΡΡΠΎΠ½ , ΠΊΠΎΡΠΎΡΡΠΉ Π·Π°ΡΡΠ°Π²Π»ΡΠ΅Ρ Π±ΡΡΡΡΠΎ Π΄Π²ΠΈΠΆΡΡΠΈΠ΅ΡΡ ΡΠ»Π΅ΠΊΡΡΠΎΠ½Ρ Π΄Π²ΠΈΠ³Π°ΡΡΡΡ ΠΏΠΎ ΠΊΡΡΠ³ΠΎΠ²ΡΠΌ ΡΡΠ°Π΅ΠΊΡΠΎΡΠΈΡΠΌ. ΠΠ΅Π»Π΅Π³ΠΊΠΎ Π·Π°ΡΡΠ°Π²ΠΈΡΡ ΡΡΠΎ-ΡΠΎ ΡΡΠΎΠ»Ρ ΠΌΠ°Π»Π΅Π½ΡΠΊΠΎΠ΅, ΠΊΠ°ΠΊ ΡΠ»Π΅ΠΊΡΡΠΎΠ½Ρ, Π΄Π²ΠΈΠ³Π°ΡΡΡΡ ΠΏΠΎ ΠΊΡΡΠ³Ρ, Π½ΠΎ ΡΡΠΎ Π²ΠΎΠ·ΠΌΠΎΠΆΠ½ΠΎ, Π΅ΡΠ»ΠΈ ΠΌΡ ΠΏΡΠΈΠ»ΠΎΠΆΠΈΠΌ ΠΊ Π½ΠΈΠΌ ΠΌΠ°Π³Π½ΠΈΡΠ½ΠΎΠ΅ ΠΏΠΎΠ»Π΅. ΠΠΎΠ³Π΄Π° Π·Π°ΡΡΠΆΠ΅Π½Π½ΡΠ΅ ΡΠ°ΡΡΠΈΡΡ Π΄Π²ΠΈΠΆΡΡΡΡ ΡΠ΅ΡΠ΅Π· ΠΌΠ°Π³Π½ΠΈΡΠ½ΠΎΠ΅ ΠΏΠΎΠ»Π΅, Π²Π·Π°ΠΈΠΌΠΎΠ΄Π΅ΠΉΡΡΠ²ΠΈΠ΅ ΡΠΎΠ·Π΄Π°Π΅Ρ ΠΌΠ°Π³Π½ΠΈΡΠ½ΡΡ ΡΠΈΠ»Ρ Π½Π° ΡΠ°ΡΡΠΈΡΠ΅, ΠΊΠΎΡΠΎΡΠ°Ρ Π·Π°ΡΡΠ°Π²Π»ΡΠ΅Ρ Π΅Π΅ ΠΎΡΠΊΠ»ΠΎΠ½ΡΡΡΡΡ. Π ΡΡΠΎΠΉ ΡΡΠ°ΡΡΠ΅ ΠΌΡ ΠΈΡΡΠ»Π΅Π΄ΡΠ΅ΠΌ ΡΠΈΠ·ΠΈΠΊΡ Π² ΡΠ°Π±ΠΎΡΠ΅, ΡΡΠΎΠ±Ρ ΡΠΎΠ·Π΄Π°ΡΡ ΡΠΈΠ»Ρ Π½Π° Π·Π°ΡΡΠΆΠ΅Π½Π½ΡΡ ΡΠ°ΡΡΠΈΡΡ, ΠΏΡΠΎΠ°Π½Π°Π»ΠΈΠ·ΠΈΡΡΠ΅ΠΌ, ΠΊΠ°ΠΊ ΡΡΠΎ Π²Π»ΠΈΡΠ΅Ρ Π½Π° Π΄Π²ΠΈΠΆΠ΅Π½ΠΈΠ΅ ΡΠ°ΡΡΠΈΡ Π² ΠΌΠ°Π³Π½ΠΈΡΠ½ΠΎΠΌ ΠΏΠΎΠ»Π΅, ΠΈ, Π½Π°ΠΊΠΎΠ½Π΅Ρ, ΡΠ°ΡΡΠΌΠΎΡΡΠΈΠΌ ΠΏΠΎΠ²Π΅Π΄Π΅Π½ΠΈΠ΅ Π½Π΅ΠΊΠΎΡΠΎΡΡΡ ΡΠ°Π·Π»ΠΈΡΠ½ΡΡ ΡΠΈΠΏΠΎΠ² ΡΠ°ΡΡΠΈΡ.
ΠΠ°ΠΊ ΠΌΡ Π·Π½Π°Π΅ΠΌ, ΡΠ»Π΅ΠΊΡΡΠΈΡΠ΅ΡΡΠ²ΠΎ ΠΈ ΠΌΠ°Π³Π½Π΅ΡΠΈΠ·ΠΌ β ΡΡΠΎ ΡΠ°Π·Π΄Π΅Π»Ρ ΡΠΈΠ·ΠΈΠΊΠΈ, ΠΊΠΎΡΠΎΡΡΠ΅ ΠΊΠ°ΠΆΡΡΡΡ ΡΠ°Π·Π½ΡΠΌΠΈ, Π½ΠΎ ΡΠ²ΡΠ·Π°Π½Ρ ΡΠ΅ΡΠ΅Π· ΠΏΠΎΠ»Π΅ ΡΠ»Π΅ΠΊΡΡΠΎΠΌΠ°Π³Π½Π΅ΡΠΈΠ·ΠΌΠ° . ΠΡΠΎ ΡΠ°Π·Π΄Π΅Π» ΡΠΈΠ·ΠΈΠΊΠΈ, ΠΈΠ·ΡΡΠ°ΡΡΠΈΠΉ ΡΠ»Π΅ΠΊΡΡΠΎΠΌΠ°Π³Π½ΠΈΡΠ½ΡΡ ΡΠΈΠ»Ρ , ΠΊΠΎΡΠΎΡΠ°Ρ ΡΠ²Π»ΡΠ΅ΡΡΡ ΠΎΠ΄Π½ΠΎΠΉ ΠΈΠ· ΡΠ΅ΡΡΡΠ΅Ρ ΡΡΠ½Π΄Π°ΠΌΠ΅Π½ΡΠ°Π»ΡΠ½ΡΡ ΡΠΈΠ» ΠΏΡΠΈΡΠΎΠ΄Ρ ΠΈ Π²Π»ΠΈΡΠ΅Ρ Π½Π° Π²Π·Π°ΠΈΠΌΠΎΠ΄Π΅ΠΉΡΡΠ²ΠΈΠ΅ ΠΌΠ΅ΠΆΠ΄Ρ ΡΠ»Π΅ΠΊΡΡΠΈΡΠ΅ΡΠΊΠΈ Π·Π°ΡΡΠΆΠ΅Π½Π½ΡΠΌΠΈ ΡΠ°ΡΡΠΈΡΠ°ΠΌΠΈ.
Π£ΡΠ°Π²Π½Π΅Π½ΠΈΡ ΠΠ°ΠΊΡΠ²Π΅Π»Π»Π° ΠΎΠΏΠΈΡΡΠ²Π°ΡΡ, ΠΊΠ°ΠΊ ΡΠ»Π΅ΠΊΡΡΠΈΡΠ΅ΡΠΊΠΈΠ΅ ΠΈ ΠΌΠ°Π³Π½ΠΈΡΠ½ΡΠ΅ ΠΏΠΎΠ»Ρ Π³Π΅Π½Π΅ΡΠΈΡΡΡΡΡΡ Π²ΠΌΠ΅ΡΡΠ΅ ΠΈ ΠΏΠΎΡΡΠΎΡΠ½Π½ΠΎ Π²Π·Π°ΠΈΠΌΠΎΠ΄Π΅ΠΉΡΡΠ²ΡΡΡ ΠΈ Π²Π»ΠΈΡΡΡ Π΄ΡΡΠ³ Π½Π° Π΄ΡΡΠ³Π°. ΠΠ΄Π½Π°ΠΊΠΎ ΡΡΠΎ Π²ΡΡ ΠΎΠ΄ΠΈΡ Π·Π° ΡΠ°ΠΌΠΊΠΈ ΡΠΎΠ³ΠΎ, ΡΡΠΎ Π²Π°ΠΌ Π½ΡΠΆΠ½ΠΎ ΠΏΠΎΠ½ΠΈΠΌΠ°ΡΡ Π΄Π»Ρ ΡΠΊΠ·Π°ΠΌΠ΅Π½ΠΎΠ² GCSE!
ΠΠΏΡΠ΅Π΄Π΅Π»Π΅Π½ΠΈΡ ΠΌΠ°Π³Π½ΠΈΡΠ½ΠΎΠΉ ΡΠΈΠ»Ρ ΠΈ ΠΌΠ°Π³Π½ΠΈΡΠ½ΠΎΠ³ΠΎ ΠΏΠΎΠ»Ρ
ΠΠΊΡΠΏΠ΅ΡΠΈΠΌΠ΅Π½ΡΠ°Π»ΡΠ½ΠΎ Π±ΡΠ»ΠΎ ΠΎΠ±Π½Π°ΡΡΠΆΠ΅Π½ΠΎ, ΡΡΠΎ ΠΊΠΎΠ³Π΄Π° ΡΠ»Π΅ΠΊΡΡΠΈΡΠ΅ΡΠΊΠΈΠ΅ Π·Π°ΡΡΠ΄Ρ Π΄Π²ΠΈΠΆΡΡΡΡ (Ρ. Π΅. ΠΈΠΌΠ΅ΡΡ Π½Π΅Π½ΡΠ»Π΅Π²ΡΡ ΡΠΊΠΎΡΠΎΡΡΡ), Π² Π΄ΠΎΠΏΠΎΠ»Π½Π΅Π½ΠΈΠ΅ ΠΊ ΡΠ»Π΅ΠΊΡΡΠΈΡΠ΅ΡΠΊΠΎΠΉ ΡΠΈΠ»Π΅ ΠΈΠ·ΠΌΠ΅ΡΡΠ΅ΡΡΡ Π΅ΡΠ΅ ΠΎΠ΄ΠΈΠ½ Π²ΠΊΠ»Π°Π΄ Π² ΡΠΈΠ»Ρ, Ρ ΠΊΠΎΡΠΎΡΠΎΠΉ ΡΡΠ°Π»ΠΊΠΈΠ²Π°ΡΡΡΡ Π·Π°ΡΡΠΆΠ΅Π½Π½ΡΠ΅ ΡΠ°ΡΡΠΈΡΡ. , ΡΡΠΎ Π±ΡΠ»ΠΎ ΡΠΆΠ΅ ΠΈΠ·Π²Π΅ΡΡΠ½ΠΎ. ΠΡΠ° Π·Π°Π³Π°Π΄ΠΎΡΠ½Π°Ρ ΡΠΈΠ»Π° Π±ΡΠ»Π° ΡΠ»Π΅ΠΊΡΡΠΎΠΌΠ°Π³Π½ΠΈΡΠ½ΠΎΠΉ ΡΠΈΠ»ΠΎΠΉ (ΡΠ°ΠΊΠΆΠ΅ Π½Π°Π·ΡΠ²Π°Π΅ΠΌΠΎΠΉ ΠΌΠ°Π³Π½ΠΈΡΠ½ΠΎΠΉ ΡΠΈΠ»ΠΎΠΉ), Π²ΡΠ·Π²Π°Π½Π½ΠΎΠΉ Π΄Π²ΠΈΠΆΠ΅Π½ΠΈΠ΅ΠΌ Π½ΠΎΡΠΈΡΠ΅Π»Π΅ΠΉ Π·Π°ΡΡΠ΄Π° ΡΠ΅ΡΠ΅Π· ΠΊΠ°ΠΊΠΎΠ΅-ΡΠΎ ΠΏΡΠΈΠ»ΠΎΠΆΠ΅Π½Π½ΠΎΠ΅ ΠΈΠ·Π²Π½Π΅ ΠΌΠ°Π³Π½ΠΈΡΠ½ΠΎΠ΅ ΠΏΠΎΠ»Π΅.
ΠΡΠΎΡΡΠΎΠΉ ΡΠΊΡΠΏΠ΅ΡΠΈΠΌΠ΅Π½Ρ, ΠΊΠΎΡΠΎΡΡΠΉ Π²Ρ ΠΌΠΎΠΆΠ΅ΡΠ΅ ΠΏΡΠΎΠ²Π΅ΡΡΠΈ Π΄ΠΎΠΌΠ°, ΡΡΠΎΠ±Ρ Π½Π°Π±Π»ΡΠ΄Π°ΡΡ Π·Π° Π΄Π΅ΠΉΡΡΠ²ΠΈΠ΅ΠΌ ΠΌΠ°Π³Π½ΠΈΡΠ½ΠΎΠΉ ΡΠΈΠ»Ρ, ΠΈΡΠΏΠΎΠ»ΡΠ·ΡΠ΅Ρ ΠΏΠΎΠ»ΠΎΡΠΊΡ Π°Π»ΡΠΌΠΈΠ½ΠΈΠ΅Π²ΠΎΠΉ ΡΠΎΠ»ΡΠ³ΠΈ, Π±Π°ΡΠ°ΡΠ΅Ρ ΠΈ ΠΏΠΎΠ΄ΠΊΠΎΠ²ΠΎΠΎΠ±ΡΠ°Π·Π½ΡΠΉ ΠΌΠ°Π³Π½ΠΈΡ. ΠΠ·Π½Π°ΡΠ°Π»ΡΠ½ΠΎ ΠΌΠ°Π³Π½ΠΈΡ Π½Π΅ Π΄ΠΎΠ»ΠΆΠ΅Π½ Π²ΠΎΠ·Π΄Π΅ΠΉΡΡΠ²ΠΎΠ²Π°ΡΡ Π½Π° ΠΏΠΎΠ»ΠΎΡΠΊΡ ΡΠΎΠ»ΡΠ³ΠΈ, ΡΠ°ΠΊ ΠΊΠ°ΠΊ ΠΎΠ½Π° ΡΠ΄Π΅Π»Π°Π½Π° ΠΈΠ· Π°Π»ΡΠΌΠΈΠ½ΠΈΡ. ΠΠ΄Π½Π°ΠΊΠΎ, Π΅ΡΠ»ΠΈ ΠΌΡ ΡΠΎΠ·Π΄Π°Π΄ΠΈΠΌ ΡΠ΅ΠΏΡ, ΠΏΡΠΈΠΊΡΠ΅ΠΏΠΈΠ² ΠΊΠ°ΠΆΠ΄ΡΠΉ ΠΊΠΎΠ½Π΅Ρ ΠΏΠΎΠ»ΠΎΡΠΊΠΈ ΡΠΎΠ»ΡΠ³ΠΈ ΠΊ ΠΊΠ»Π΅ΠΌΠΌΠ°ΠΌ Π±Π°ΡΠ°ΡΠ΅ΠΈ, ΡΠ΅ΠΏΠ΅ΡΡ ΡΠ΅ΡΠ΅Π· ΡΠΎΠ»ΡΠ³Ρ Π±ΡΠ΄ΡΡ ΡΠ΅ΡΡ ΡΠ»Π΅ΠΊΡΡΠΎΠ½Ρ (Π·Π°ΡΡΠΆΠ΅Π½Π½ΡΠ΅ ΡΠ°ΡΡΠΈΡΡ). ΠΡΠ»ΠΈ ΠΌΡ ΡΠ΅ΠΏΠ΅ΡΡ ΠΏΡΠΈΠ±Π»ΠΈΠ·ΠΈΠΌ ΠΏΠΎΠ»ΠΎΡΠΊΡ ΠΊ ΠΌΠ°Π³Π½ΠΈΡΠ½ΠΎΠΌΡ ΠΏΠΎΠ»Ρ, ΠΎΠ½Π° ΠΎΡΠΊΠ»ΠΎΠ½ΠΈΡΡΡ! ΠΡΠΎ Π΄Π΅ΠΌΠΎΠ½ΡΡΡΠΈΡΡΠ΅Ρ, ΡΡΠΎ ΠΌΠ°Π³Π½ΠΈΡΠ½Π°Ρ ΡΠΈΠ»Π° ΡΠΎΠ·Π΄Π°Π΅ΡΡΡ Π΄Π²ΠΈΠΆΠ΅Π½ΠΈΠ΅ΠΌ Π·Π°ΡΡΠΆΠ΅Π½Π½ΡΡ ΡΠ°ΡΡΠΈΡ ΡΠ΅ΡΠ΅Π· ΠΌΠ°Π³Π½ΠΈΡΠ½ΠΎΠ΅ ΠΏΠΎΠ»Π΅, ΠΏΠΎΡΠΊΠΎΠ»ΡΠΊΡ ΡΠΈΠ»Π° ΠΈΡΡΠ΅Π·Π°Π΅Ρ, ΠΊΠΎΠ³Π΄Π° ΠΌΡ ΡΠ°Π·ΡΠ΅Π΄ΠΈΠ½ΡΠ΅ΠΌ ΡΠ΅ΠΏΡ ΠΈΠ· ΡΠΎΠ»ΡΠ³ΠΈ ΠΈ ΡΠ°ΡΡΠΈΡΡ ΠΏΠ΅ΡΠ΅ΡΡΠ°ΡΡ Π΄Π²ΠΈΠ³Π°ΡΡΡΡ.
A ΠΌΠ°Π³Π½ΠΈΡΠ½Π°Ρ ΡΠΈΠ»Π° β ΡΡΠΎ ΡΠΈΠ»Π°, ΠΊΠΎΡΠΎΡΡΡ ΠΈΡΠΏΡΡΡΠ²Π°Π΅Ρ Π·Π°ΡΡΠΆΠ΅Π½Π½Π°Ρ ΡΠ°ΡΡΠΈΡΠ° (ΡΠ»Π΅ΠΊΡΡΠΎΠ½, ΠΏΡΠΎΡΠΎΠ½, ΠΈΠΎΠ½ ΠΈ Ρ. Π΄.), ΠΊΠΎΠ³Π΄Π° ΠΎΠ½Π° Π΄Π²ΠΈΠΆΠ΅ΡΡΡ ΡΠ΅ΡΠ΅Π· ΠΌΠ°Π³Π½ΠΈΡΠ½ΠΎΠ΅ ΠΏΠΎΠ»Π΅.
ΠΠ°Π³Π½ΠΈΡΠ½Π°Ρ ΡΠΈΠ»Π° ΠΈΠ·ΠΌΠ΅ΡΡΠ΅ΡΡΡ Π² Π½ΡΡΡΠΎΠ½Π°Ρ , ΠΊΠ°ΠΊ ΠΈ Π»ΡΠ±Π°Ρ Π΄ΡΡΠ³Π°Ρ ΡΠΈΠ»Π°. Π’Π°ΠΊΠΆΠ΅ Π²Π°ΠΆΠ½ΠΎ ΠΎΡΠΌΠ΅ΡΠΈΡΡ, ΡΡΠΎ Π·Π°ΡΡΠΆΠ΅Π½Π½Π°Ρ ΡΠ°ΡΡΠΈΡΠ° Π΄ΠΎΠ»ΠΆΠ½Π° Π΄Π²ΠΈΠ³Π°ΡΡΡΡ Π½Π° ΠΎΡΠ½ΠΎΡΠΈΡΠ΅Π»ΡΠ½ΠΎ ΠΌΠ°Π³Π½ΠΈΡΠ½ΠΎΠ³ΠΎ ΠΏΠΎΠ»Ρ Π½Π° , ΡΡΠΎΠ±Ρ ΠΈΡΠΏΡΡΠ°ΡΡ Π½Π° ΡΠ΅Π±Π΅ ΠΌΠ°Π³Π½ΠΈΡΠ½ΡΡ ΡΠΈΠ»Ρ.
Π’Π΅ΠΏΠ΅ΡΡ ΠΌΡ Π΄ΠΎΠ»ΠΆΠ½Ρ Π²ΡΡΡΠ½ΠΈΡΡ, ΠΊΠ°ΠΊ ΠΌΠ°Π³Π½ΠΈΡΡ ΡΠΎΠ·Π΄Π°ΡΡ ΡΡΡ ΡΠΈΠ»Ρ, ΠΈ Π΄Π»Ρ ΡΡΠΎΠ³ΠΎ Π½Π°ΠΌ Π½ΡΠΆΠ½ΠΎ ΠΎΠ±ΡΡΠ΄ΠΈΡΡ ΠΌΠ°Π³Π½ΠΈΡΠ½ΠΎΠ΅ ΠΏΠΎΠ»Π΅. ΠΠΏΡΠ΅Π΄Π΅Π»Π΅Π½ΠΈΠ΅ ΠΌΠ°Π³Π½ΠΈΡΠ½ΠΎΠ΅ ΠΏΠΎΠ»Π΅ Π²ΡΠ³Π»ΡΠ΄ΠΈΡ ΡΠ»Π΅Π΄ΡΡΡΠΈΠΌ ΠΎΠ±ΡΠ°Π·ΠΎΠΌ.
ΠΠ°Π³Π½ΠΈΡΠ½ΠΎΠ΅ ΠΏΠΎΠ»Π΅ ΠΏΡΠ΅Π΄ΡΡΠ°Π²Π»ΡΠ΅Ρ ΡΠΎΠ±ΠΎΠΉ ΠΎΠ±Π»Π°ΡΡΡ Π² ΠΏΡΠΎΡΡΡΠ°Π½ΡΡΠ²Π΅, Π³Π΄Π΅ Π΄Π²ΠΈΠΆΡΡΠΈΠΉΡΡ Π·Π°ΡΡΠ΄ ΠΈΠ»ΠΈ ΠΏΠΎΡΡΠΎΡΠ½Π½ΡΠΉ ΠΌΠ°Π³Π½ΠΈΡ ΠΎΡΡΡΠ°ΡΡ ΡΠΈΠ»Ρ .
ΠΠ°Π³Π½ΠΈΡΠ½ΠΎΠ΅ ΠΏΠΎΠ»Π΅ ΠΏΡΠΈΡΡΡΡΡΠ²ΡΠ΅Ρ Π² Π»ΡΠ±ΠΎΠΉ ΡΠΎΡΠΊΠ΅ ΠΏΡΠΎΡΡΡΠ°Π½ΡΡΠ²Π°, Π³Π΄Π΅ Π½Π° Π΄Π²ΠΈΠΆΡΡΡΡΡΡ Π·Π°ΡΡΠΆΠ΅Π½Π½ΡΡ ΡΠ°ΡΡΠΈΡΡ Π΄Π΅ΠΉΡΡΠ²ΡΠ΅Ρ ΡΠΈΠ»Π°. Π‘ΠΈΠ»Π° ΠΌΠ°Π³Π½ΠΈΡΠ½ΠΎΠ³ΠΎ ΠΏΠΎΠ»Ρ ΠΎΠ±ΡΡΠ½ΠΎ Π½Π°Π·ΡΠ²Π°Π΅ΡΡΡ ΠΏΠ»ΠΎΡΠ½ΠΎΡΡΡΡ ΠΌΠ°Π³Π½ΠΈΡΠ½ΠΎΠ³ΠΎ ΠΏΠΎΡΠΎΠΊΠ° ΠΈΠ»ΠΈ Π½Π°ΠΏΡΡΠΆΠ΅Π½Π½ΠΎΡΡΡΡ ΠΌΠ°Π³Π½ΠΈΡΠ½ΠΎΠ³ΠΎ ΠΏΠΎΠ»Ρ ΠΈ ΠΎΠ±ΠΎΠ·Π½Π°ΡΠ°Π΅ΡΡΡ ΡΠΈΠΌΠ²ΠΎΠ»ΠΎΠΌ ΠΠ΄ΠΈΠ½ΠΈΡΠ΅ΠΉ ΠΈΠ·ΠΌΠ΅ΡΠ΅Π½ΠΈΡ ΠΌΠ°Π³Π½ΠΈΡΠ½ΠΎΠ³ΠΎ ΠΏΠΎΠ»Ρ ΡΠ²Π»ΡΠ΅ΡΡΡ Π’Π΅ΡΠ»Π° , ΡΡΠΎ ΡΠΊΠ²ΠΈΠ²Π°Π»Π΅Π½ΡΠ½ΠΎ Π½ΡΡΡΠΎΠ½Π°ΠΌ Π½Π° Π°ΠΌΠΏΠ΅Ρ-ΠΌΠ΅ΡΡ ,.
ΠΠ°Π³Π½ΠΈΡΠ½ΠΎΠ΅ ΠΏΠΎΠ»Π΅ Π½Π΅ Π²ΡΠ΅Π³Π΄Π° ΠΈΠΌΠ΅Π΅Ρ ΠΏΠΎΡΡΠΎΡΠ½Π½ΡΡ ΡΠΈΠ»Ρ ΠΈΠ»ΠΈ ΠΏΠ»ΠΎΡΠ½ΠΎΡΡΡ ΠΏΠΎΡΠΎΠΊΠ° . ΠΠΎΠ»Ρ ΠΎΠ±ΡΡΠ½ΠΎ ΠΏΡΠ΅Π΄ΡΡΠ°Π²Π»ΡΡΡΡΡ Π»ΠΈΠ½ΠΈΡΠΌΠΈ ΠΌΠ°Π³Π½ΠΈΡΠ½ΠΎΠ³ΠΎ ΠΏΠΎΠ»Ρ, ΠΊΠΎΡΠΎΡΡΠ΅ ΠΏΡΠΎΡ ΠΎΠ΄ΡΡ ΠΎΡ ΡΠ΅Π²Π΅ΡΠ½ΠΎΠ³ΠΎ ΠΊ ΡΠΆΠ½ΠΎΠΌΡ ΠΏΠΎΠ»ΡΡΡ ΠΌΠ°Π³Π½ΠΈΡΠ°, ΠΏΡΠΈΡΠ΅ΠΌ Π½Π°ΠΏΡΡΠΆΠ΅Π½Π½ΠΎΡΡΡ ΠΏΠΎΠ»Ρ ΠΌΠ°ΠΊΡΠΈΠΌΠ°Π»ΡΠ½Π° ΡΠ°ΠΌ, Π³Π΄Π΅ Π»ΠΈΠ½ΠΈΠΈ Π±Π»ΠΈΠΆΠ΅ Π²ΡΠ΅Π³ΠΎ Π΄ΡΡΠ³ ΠΊ Π΄ΡΡΠ³Ρ. ΠΠ°ΠΊ ΠΏΡΠ°Π²ΠΈΠ»ΠΎ, ΡΡΠΎ ΠΏΡΠΈΠ²ΠΎΠ΄ΠΈΡ ΠΊ ΡΠΎΠΌΡ, ΡΡΠΎ ΠΏΠΎΠ»Ρ ΡΡΠ°Π½ΠΎΠ²ΡΡΡΡ ΡΠΈΠ»ΡΠ½Π΅Π΅ Π±Π»ΠΈΠΆΠ΅ ΠΊ ΠΏΠΎΠ»ΡΡΠ°ΠΌ ΠΌΠ°Π³Π½ΠΈΡΠ° ΠΈ ΠΎΡΠ»Π°Π±Π΅Π²Π°ΡΡ Ρ ΡΠ²Π΅Π»ΠΈΡΠ΅Π½ΠΈΠ΅ΠΌ ΡΠ°ΡΡΡΠΎΡΠ½ΠΈΡ.
ΠΠ°ΡΡΠΆΠ΅Π½Π½ΡΠ΅ ΡΠ°ΡΡΠΈΡΡ Π² ΠΌΠ°Π³Π½ΠΈΡΠ½ΠΎΠΌ ΠΏΠΎΠ»Π΅
ΠΠ°ΠΊ ΠΌΡ ΡΠ·Π½Π°Π»ΠΈ ΠΈΠ· ΡΠ΅ΠΌΡ ΠΎΠ± ΡΠ»Π΅ΠΊΡΡΠΈΡΠ΅ΡΡΠ²Π΅, ΠΏΠΎΡΠΎΠΊ ΠΏΠΎΠ»ΠΎΠΆΠΈΡΠ΅Π»ΡΠ½ΡΡ ΡΠ»Π΅ΠΊΡΡΠΈΡΠ΅ΡΠΊΠΈΡ Π·Π°ΡΡΠ΄ΠΎΠ² ΠΏΡΠ΅Π΄ΡΡΠ°Π²Π»ΡΠ΅Ρ ΡΠΎΠ±ΠΎΠΉ ΠΎΠ±ΡΡΠ½ΡΠΉ ΡΠ»Π΅ΠΊΡΡΠΈΡΠ΅ΡΠΊΠΈΠΉ ΡΠΎΠΊ. Π‘Π»Π΅Π΄ΠΎΠ²Π°ΡΠ΅Π»ΡΠ½ΠΎ, ΡΠ»Π΅ΠΊΡΡΠΈΡΠ΅ΡΠΊΠΈΠ΅ ΡΠΎΠΊΠΈ Π±ΡΠ΄ΡΡ ΠΈΡΠΏΡΡΡΠ²Π°ΡΡ ΡΠΈΠ»Ρ Π² ΠΌΠ°Π³Π½ΠΈΡΠ½ΠΎΠΌ ΠΏΠΎΠ»Π΅. ΠΡΠ»ΠΈ ΠΌΡ ΠΏΠΎΠ΄ΡΠΌΠ°Π΅ΠΌ ΠΎΠ± ΡΠ»Π΅ΠΊΡΡΠΎΠ½Π°Ρ , Π΄Π²ΠΈΠΆΡΡΠΈΡ ΡΡ ΠΏΠΎ ΠΊΡΡΠ³Ρ Π² ΠΌΠ°Π³Π½Π΅ΡΡΠΎΠ½Π΅, ΠΊΠ°ΠΊ ΠΌΡ ΡΠΏΠΎΠΌΠΈΠ½Π°Π»ΠΈ Π² Π½Π°ΡΠ°Π»Π΅ ΡΡΠ°ΡΡΠΈ, ΠΊΠ°ΠΊ ΠΌΡ ΠΌΠΎΠΆΠ΅ΠΌ Π±ΡΡΡ ΡΠ²Π΅ΡΠ΅Π½Ρ, ΡΡΠΎ ΡΠ»Π΅ΠΊΡΡΠΎΠ½Ρ Π±ΡΠ΄ΡΡ Π΄Π²ΠΈΠ³Π°ΡΡΡΡ ΠΈΠΌΠ΅Π½Π½ΠΎ ΡΠ°ΠΊΠΈΠΌ ΠΎΠ±ΡΠ°Π·ΠΎΠΌ, ΠΏΠΎΡΠΊΠΎΠ»ΡΠΊΡ ΠΎΠ½ΠΈ Π½Π΅ ΡΠΎΠ΄Π΅ΡΠΆΠ°ΡΡΡ Π²Π½ΡΡΡΠΈ ΠΏΡΠΎΠ²ΠΎΠ΄Π°? ΠΠΊΠ°Π·ΡΠ²Π°Π΅ΡΡΡ, ΡΡΡΠ΅ΡΡΠ²ΡΠ΅Ρ ΡΠ²ΡΠ·Ρ ΠΌΠ΅ΠΆΠ΄Ρ Π½Π°ΠΏΡΠ°Π²Π»Π΅Π½ΠΈΡΠΌΠΈ ΠΌΠ°Π³Π½ΠΈΡΠ½ΠΎΠ΅ ΠΏΠΎΠ»Π΅ , ΠΌΠ°Π³Π½ΠΈΡΠ½Π°Ρ ΡΠΈΠ»Π° ΠΈ ΡΠΎΠΊ . ΠΡΠ»ΠΈ ΠΌΡ Π·Π½Π°Π΅ΠΌ Π½Π°ΠΏΡΠ°Π²Π»Π΅Π½ΠΈΠ΅ Π»ΡΠ±ΡΡ Π΄Π²ΡΡ ΠΈΠ· ΡΡΠΈΡ Π²Π΅Π»ΠΈΡΠΈΠ½, ΠΌΡ ΠΌΠΎΠΆΠ΅ΠΌ Π½Π°ΠΉΡΠΈ Π½Π°ΠΏΡΠ°Π²Π»Π΅Π½ΠΈΠ΅ ΡΡΠ΅ΡΡΠ΅ΠΉ.
ΠΡΠ°Π²ΠΈΠ»ΠΎ Π»Π΅Π²ΠΎΠΉ ΡΡΠΊΠΈ
Π§ΡΠΎΠ±Ρ ΠΎΠΏΡΠ΅Π΄Π΅Π»ΠΈΡΡ Π½Π°ΠΏΡΠ°Π²Π»Π΅Π½ΠΈΠ΅ ΡΠΈΠ»Ρ, ΠΊΠΎΡΠΎΡΡΡ Π±ΡΠ΄Π΅Ρ ΠΎΡΡΡΠ°ΡΡ Π΄Π²ΠΈΠΆΡΡΠΈΠΉΡΡ Π·Π°ΡΡΠ΄, ΠΊΠΎΠ³Π΄Π° ΠΎΠ½ Π²Ρ ΠΎΠ΄ΠΈΡ Π² ΠΌΠ°Π³Π½ΠΈΡΠ½ΠΎΠ΅ ΠΏΠΎΠ»Π΅, ΠΈΡΠΏΠΎΠ»ΡΠ·ΡΠ΅ΡΡΡ ΠΏΡΠ°Π²ΠΈΠ»ΠΎ Π»Π΅Π²ΠΎΠΉ ΡΡΠΊΠΈ Π€Π»Π΅ΠΌΠΈΠ½Π³Π° . ΠΡΠ°Π²ΠΈΠ»ΠΎ Π»Π΅Π²ΠΎΠΉ ΡΡΠΊΠΈ Π³Π»Π°ΡΠΈΡ, ΡΡΠΎ Π²Ρ Π΄ΠΎΠ»ΠΆΠ½Ρ ΡΠ°ΡΡΡΠ°Π²ΠΈΡΡ Π±ΠΎΠ»ΡΡΠΎΠΉ, ΡΠΊΠ°Π·Π°ΡΠ΅Π»ΡΠ½ΡΠΉ ΠΈ ΡΡΠ΅Π΄Π½ΠΈΠΉ ΠΏΠ°Π»ΡΡΡ ΡΠ°ΠΊ, ΡΡΠΎΠ±Ρ ΠΎΠ½ΠΈ Π½Π°Ρ ΠΎΠ΄ΠΈΠ»ΠΈΡΡ ΠΏΠΎΠ΄ ΠΏΡΡΠΌΡΠΌ ΡΠ³Π»ΠΎΠΌ Π΄ΡΡΠ³ ΠΊ Π΄ΡΡΠ³Ρ (ΠΊΠ°ΠΊ ΠΏΠΎΠΊΠ°Π·Π°Π½ΠΎ Π½Π° Π΄ΠΈΠ°Π³ΡΠ°ΠΌΠΌΠ΅ Π½ΠΈΠΆΠ΅).
- ΠΠ°ΠΏΡΠ°Π²ΡΡΠ΅ ΡΠΊΠ°Π·Π°ΡΠ΅Π»ΡΠ½ΡΠΉ ΠΏΠ°Π»Π΅Ρ Π² Π½Π°ΠΏΡΠ°Π²Π»Π΅Π½ΠΈΠΈ ΠΌΠ°Π³Π½ΠΈΡΠ½ΠΎΠ³ΠΎ ΠΏΠΎΠ»Ρ, ΡΠΎ Π΅ΡΡΡ Ρ ΡΠ΅Π²Π΅ΡΠ½ΠΎΠ³ΠΎ ΠΏΠΎΠ»ΡΡΠ° Π½Π° ΡΠΆΠ½ΡΠΉ ΠΏΠΎΠ»ΡΡ.
- ΠΠ°Ρ ΡΡΠ΅Π΄Π½ΠΈΠΉ ΠΏΠ°Π»Π΅Ρ Π΄Π°Π΅Ρ Π½Π°ΠΏΡΠ°Π²Π»Π΅Π½ΠΈΠ΅ ΡΡΠ»ΠΎΠ²Π½ΠΎΠ³ΠΎ ΡΠΎΠΊΠ°(Π΄Π²ΠΈΠΆΠ΅Π½ΠΈΠ΅ ΠΏΠΎΠ»ΠΎΠΆΠΈΡΠ΅Π»ΡΠ½ΠΎΠ³ΠΎ Π·Π°ΡΡΠ΄Π°).
- ΠΠ°Ρ Π±ΠΎΠ»ΡΡΠΎΠΉ ΠΏΠ°Π»Π΅Ρ ΡΠΊΠ°ΠΆΠ΅Ρ Π½Π°ΠΏΡΠ°Π²Π»Π΅Π½ΠΈΠ΅ ΡΠΈΠ»Ρ, Π΄Π΅ΠΉΡΡΠ²ΡΡΡΠ΅ΠΉ Π½Π° ΡΠ°ΡΡΠΈΡΡ.
ΠΡΠ°Π²ΠΈΠ»ΠΎ Π»Π΅Π²ΠΎΠΉ ΡΡΠΊΠΈ ΠΈΡΠΏΠΎΠ»ΡΠ·ΡΠ΅ΡΡΡ Π΄Π»Ρ ΠΎΠΏΡΠ΅Π΄Π΅Π»Π΅Π½ΠΈΡ Π½Π°ΠΏΡΠ°Π²Π»Π΅Π½ΠΈΡ ΡΠΈΠ»Ρ, Π΄Π΅ΠΉΡΡΠ²ΡΡΡΠ΅ΠΉ Π½Π° Π·Π°ΡΡΠΆΠ΅Π½Π½ΡΡ ΡΠ°ΡΡΠΈΡΡ, ΠΊΠΎΠ³Π΄Π° ΠΎΠ½Π° Π΄Π²ΠΈΠΆΠ΅ΡΡΡ ΡΠ΅ΡΠ΅Π· ΠΌΠ°Π³Π½ΠΈΡΠ½ΠΎΠ΅ ΠΏΠΎΠ»Π΅ ΠΏΠΎΠ΄ ΠΏΡΡΠΌΡΠΌ ΡΠ³Π»ΠΎΠΌ ΠΊ ββΠ½Π΅ΠΌΡ. ΠΠΈΠΊΠΈΡΠΊΠ»Π°Π΄ CC BY-SA 4.0
Π‘ΠΈΠ»Π°, Π΄Π΅ΠΉΡΡΠ²ΡΡΡΠ°Ρ Π½Π° Π·Π°ΡΡΠΆΠ΅Π½Π½ΡΠ΅ ΡΠ°ΡΡΠΈΡΡ Π² ΠΌΠ°Π³Π½ΠΈΡΠ½ΠΎΠΌ ΠΏΠΎΠ»Π΅
ΠΠ±ΡΠΈΠΉ Π·Π°ΠΊΠΎΠ½, ΠΎΠΏΡΠ΅Π΄Π΅Π»ΡΡΡΠΈΠΉ ΠΏΠΎΠ²Π΅Π΄Π΅Π½ΠΈΠ΅ ΡΠ»Π΅ΠΊΡΡΠΈΡΠ΅ΡΠΊΠΎΠ³ΠΎ Π·Π°ΡΡΠ΄Π° Π² ΠΏΡΠΈΡΡΡΡΡΠ²ΠΈΠΈ ΡΠ»Π΅ΠΊΡΡΠΎΠΌΠ°Π³Π½ΠΈΡΠ½ΠΎΠ³ΠΎ ΠΏΠΎΠ»Ρ, ΠΈΠ·Π²Π΅ΡΡΠ΅Π½ ΠΊΠ°ΠΊ ΡΠΈΠ»Π° ΠΠΎΡΠ΅Π½ΡΠ° . ΠΠ±ΡΠ΅Π΅ Π²ΡΡΠ°ΠΆΠ΅Π½ΠΈΠ΅ ΡΠ°ΠΊΠΆΠ΅ Π²ΠΊΠ»ΡΡΠ°Π΅Ρ Π²Π»ΠΈΡΠ½ΠΈΠ΅ Π²Π½Π΅ΡΠ½Π΅Π³ΠΎ ΡΠ»Π΅ΠΊΡΡΠΈΡΠ΅ΡΠΊΠΎΠ³ΠΎ ΠΏΠΎΠ»Ρ, Π½ΠΎ Π·Π΄Π΅ΡΡ ΠΌΡ ΠΎΠ³ΡΠ°Π½ΠΈΡΠΈΠΌΡΡ ΡΠΈΡΡΠ°ΡΠΈΡΠΌΠΈ, ΠΊΠΎΠ³Π΄Π° ΠΏΡΠΈΡΡΡΡΡΠ²ΡΠ΅Ρ ΡΠΎΠ»ΡΠΊΠΎ ΠΌΠ°Π³Π½ΠΈΡΠ½ΠΎΠ΅ ΠΏΠΎΠ»Π΅.
ΠΡΡΠ°ΠΆΠ΅Π½ΠΈΠ΅ Π΄Π»Ρ ΡΠΈΠ»Ρ, Π΄Π΅ΠΉΡΡΠ²ΡΡΡΠ΅ΠΉ Π½Π° Π·Π°ΡΡΠΆΠ΅Π½Π½ΡΡ ΡΠ°ΡΡΠΈΡΡ, Π΄Π²ΠΈΠΆΡΡΡΡΡΡ ΠΏΠ΅ΡΠΏΠ΅Π½Π΄ΠΈΠΊΡΠ»ΡΡΠ½ΠΎ ΠΌΠ°Π³Π½ΠΈΡΠ½ΠΎΠΌΡ ΠΏΠΎΠ»Ρ, ΠΈΠΌΠ΅Π΅Ρ Π²ΠΈΠ΄:
Π³Π΄Π΅ β Π·Π°ΡΡΠ΄ ΡΠ°ΡΡΠΈΡΡ, β Π²Π΅Π»ΠΈΡΠΈΠ½Π° Π΅Π΅ ΡΠΊΠΎΡΠΎΡΡΠΈ (ΡΠΊΠΎΡΠΎΡΡΠΈ), Π° β Π½Π°ΠΏΡΡΠΆΠ΅Π½Π½ΠΎΡΡΡ ΠΌΠ°Π³Π½ΠΈΡΠ½ΠΎΠ³ΠΎ ΠΏΠΎΠ»Ρ.
ΠΡΠΎ ΡΡΠ°Π²Π½Π΅Π½ΠΈΠ΅ ΡΡΠ½ΠΎ ΡΠΊΠ°Π·ΡΠ²Π°Π΅Ρ Π½Π° ΡΠΎ, ΡΡΠΎ ΡΠ°ΡΡΠΈΡΠ° Π΄ΠΎΠ»ΠΆΠ½Π° Π΄Π²ΠΈΠ³Π°ΡΡΡΡ, ΡΡΠΎΠ±Ρ ΠΌΠ°Π³Π½ΠΈΡΠ½Π°Ρ ΡΠΈΠ»Π° Π²ΠΎΠΎΠ±ΡΠ΅ ΠΎΡΡΡΠ°Π»Π°ΡΡ. Π£ Π½Π°Ρ Π΅ΡΡΡ ΠΎΡΠ½ΠΎΡΠ΅Π½ΠΈΠ΅, Π² ΠΊΠΎΡΠΎΡΠΎΠΌ Π½Π°ΠΏΡΠ°Π²Π»Π΅Π½ΠΈΠ΅ Π΄Π²ΠΈΠΆΠ΅Π½ΠΈΡ, ΡΠΈΠ»Ρ ΠΈ ΠΏΠΎΠ»Ρ Π½Π°Ρ ΠΎΠ΄ΡΡΡΡ ΠΏΠΎΠ΄ ΠΏΡΡΠΌΡΠΌ ΡΠ³Π»ΠΎΠΌ Π΄ΡΡΠ³ ΠΊ Π΄ΡΡΠ³Ρ ΠΈ ΠΌΠΎΠ³ΡΡ Π±ΡΡΡ ΠΎΠΏΡΠ΅Π΄Π΅Π»Π΅Π½Ρ Ρ ΠΏΠΎΠΌΠΎΡΡΡ ΠΏΡΠ°Π²ΠΈΠ»Π° Π»Π΅Π²ΠΎΠΉ ΡΡΠΊΠΈ.
ΠΡΠ»ΠΈ Π·Π°ΡΡΠΆΠ΅Π½Π½Π°Ρ ΡΠ°ΡΡΠΈΡΠ° Π΄Π²ΠΈΠΆΠ΅ΡΡΡ ΡΠ΅ΡΠ΅Π· ΠΌΠ°Π³Π½ΠΈΡΠ½ΠΎΠ΅ ΠΏΠΎΠ»Π΅ Π½Π΅ΠΏΠ΅ΡΠΏΠ΅Π½Π΄ΠΈΠΊΡΠ»ΡΡΠ½ΠΎ, Π½Π° Π½Π΅Π΅ Π²ΡΠ΅ ΡΠ°Π²Π½ΠΎ Π΄Π΅ΠΉΡΡΠ²ΡΠ΅Ρ ΡΠΈΠ»Π°, ΠΎΠ΄Π½Π°ΠΊΠΎ ΠΎΠ½Π° Π±ΡΠ΄Π΅Ρ ΠΌΠ΅Π½ΡΡΠ΅ ΠΌΠ°ΠΊΡΠΈΠΌΠ°Π»ΡΠ½ΠΎΠΉ ΡΠΈΠ»Ρ, Π΄Π΅ΠΉΡΡΠ²ΡΡΡΠ΅ΠΉ, ΠΊΠΎΠ³Π΄Π° Π΄Π²Π° Π²Π΅ΠΊΡΠΎΡΠ° Π½Π°Ρ ΠΎΠ΄ΡΡΡΡ ΠΏΠΎΠ΄ ΠΏΡΡΠΌΡΠΌ ΡΠ³Π»ΠΎΠΌ. ΠΡΠΈ Π΄ΠΎΠ±Π°Π²Π»Π΅Π½ΠΈΠΈ ΡΠ»Π΅Π½Π° ΠΊ ΡΡΠ°Π²Π½Π΅Π½ΠΈΡ, Π³Π΄Π΅ ΡΠ³ΠΎΠ» ΠΌΠ΅ΠΆΠ΄Ρ ΠΌΠ°Π³Π½ΠΈΡΠ½ΡΠΌ ΠΏΠΎΠ»Π΅ΠΌ ΠΈ Π²Π΅ΠΊΡΠΎΡΠΎΠΌ ΡΠΊΠΎΡΠΎΡΡΠΈ ΡΠ°ΡΡΠΈΡΡ, ΡΡΠ°Π²Π½Π΅Π½ΠΈΠ΅ ΠΌΠ°Π³Π½ΠΈΡΠ½ΠΎΠΉ ΡΠΈΠ»Ρ ΠΏΡΠΈΠ½ΠΈΠΌΠ°Π΅Ρ Π²ΠΈΠ΄:
ΠΡΡΠ³ΠΎΠ²ΠΎΠ΅ Π΄Π²ΠΈΠΆΠ΅Π½ΠΈΠ΅ Π·Π°ΡΡΠΆΠ΅Π½Π½ΡΡ ΡΠ°ΡΡΠΈΡ Π² ΠΌΠ°Π³Π½ΠΈΡΠ½ΠΎΠΌ ΠΏΠΎΠ»Π΅
ΠΡ Π²ΠΈΠ΄Π΅Π»ΠΈ, ΡΡΠΎ Π½Π° Π·Π°ΡΡΠΆΠ΅Π½Π½ΡΡ ΡΠ°ΡΡΠΈΡΡ Π΄Π΅ΠΉΡΡΠ²ΡΠ΅Ρ ΡΠΈΠ»Π°, ΠΏΠ΅ΡΠΏΠ΅Π½Π΄ΠΈΠΊΡΠ»ΡΡΠ½Π°Ρ Π½Π°ΠΏΡΠ°Π²Π»Π΅Π½ΠΈΡ Π΅Π΅ Π΄Π²ΠΈΠΆΠ΅Π½ΠΈΡ, ΠΊΠΎΠ³Π΄Π° ΠΎΠ½Π° Π²Ρ ΠΎΠ΄ΠΈΡ Π² ΠΌΠ°Π³Π½ΠΈΡΠ½ΠΎΠ΅ ΠΏΠΎΠ»Π΅ ΠΏΠΎΠ΄ ΠΏΡΡΠΌΡΠΌ ΡΠ³Π»ΠΎΠΌ ΠΊ ββΠΏΠΎΠ»Ρ. ΠΡΠΎ ΠΎΠ·Π½Π°ΡΠ°Π΅Ρ, ΡΡΠΎ Π½Π°ΠΏΡΠ°Π²Π»Π΅Π½ΠΈΠ΅ Π΄Π²ΠΈΠΆΠ΅Π½ΠΈΡ ΡΠ°ΡΡΠΈΡΡ Π±ΡΠ΄Π΅Ρ ΠΌΠ΅Π½ΡΡΡΡΡ ΠΏΠΎ ΠΌΠ΅ΡΠ΅ ΡΠΎΠ³ΠΎ, ΠΊΠ°ΠΊ ΠΌΠ°Π³Π½ΠΈΡΠ½Π°Ρ ΡΠΈΠ»Π° ΠΎΡΠΊΠ»ΠΎΠ½ΡΠ΅Ρ ΡΠ°ΡΡΠΈΡΡ. ΠΠΎΡΠΊΠΎΠ»ΡΠΊΡ Π·Π°ΡΡΠΆΠ΅Π½Π½Π°Ρ ΡΠ°ΡΡΠΈΡΠ° ΡΠ΅ΠΏΠ΅ΡΡ ΠΈΠ·ΠΌΠ΅Π½ΠΈΠ»Π° Π½Π°ΠΏΡΠ°Π²Π»Π΅Π½ΠΈΠ΅, ΡΡΠΎ ΠΏΡΠ΅Π΄ΡΡΠ°Π²Π»ΡΠ΅Ρ ΡΠΎΠ±ΠΎΠΉ ΠΈΠ·ΠΌΠ΅Π½Π΅Π½ΠΈΠ΅ Π½Π°ΠΏΡΠ°Π²Π»Π΅Π½ΠΈΡ ΡΠΎΠΊΠ°. ΠΡΠ»ΠΈ ΠΏΠΎΠ»Π΅ ΠΎΡΡΠ°Π΅ΡΡΡ ΠΏΠΎΡΡΠΎΡΠ½Π½ΡΠΌ, Π° ΡΠΎΠΊ ΠΌΠ΅Π½ΡΠ΅Ρ Π½Π°ΠΏΡΠ°Π²Π»Π΅Π½ΠΈΠ΅, ΡΠΎ ΡΠΎΠ·Π΄Π°Π²Π°Π΅ΠΌΠ°Ρ ΠΌΠ°Π³Π½ΠΈΡΠ½Π°Ρ ΡΠΈΠ»Π° ΡΠ°ΠΊΠΆΠ΅ Π΄ΠΎΠ»ΠΆΠ½Π° ΠΏΠΎΡΡΠΎΡΠ½Π½ΠΎ ΠΌΠ΅Π½ΡΡΡ Π½Π°ΠΏΡΠ°Π²Π»Π΅Π½ΠΈΠ΅! ΠΡΠΎ ΠΏΡΠΈΠ²ΠΎΠ΄ΠΈΡ ΠΊ Π·Π°ΠΏΡΡΠ°Π½Π½ΠΎΠΌΡ ΡΡΠ΅Π½Π°ΡΠΈΡ, Π½ΠΎ ΠΎΠ±ΡΡΡΠ½ΡΠ΅Ρ, ΠΊΠ°ΠΊ Π·Π°ΡΡΠΆΠ΅Π½Π½Π°Ρ ΡΠ°ΡΡΠΈΡΠ° Π² ΠΌΠ°Π³Π½ΠΈΡΠ½ΠΎΠΌ ΠΏΠΎΠ»Π΅ ΠΌΠΎΠΆΠ΅Ρ Π΄Π²ΠΈΠ³Π°ΡΡΡΡ ΠΏΠΎ ΠΊΡΡΠ³ΠΎΠ²ΠΎΠΉ ΠΏΡΡΡ .
ΠΠ΅ Π±ΡΠ΄Π΅ΠΌ ΡΠ»ΠΈΡΠΊΠΎΠΌ Π·Π°ΠΏΡΡΡΠ²Π°ΡΡΡΡ ΠΈ ΠΏΠΎΠΏΡΡΠ°Π΅ΠΌΡΡ ΠΏΠΎΠ½ΡΡΡ, ΡΡΠΎ ΠΏΡΠΎΠΈΡΡ ΠΎΠ΄ΠΈΡ, Π½Π° ΠΏΡΠΈΠΌΠ΅ΡΠ΅ ΡΠΈΡΡΠ½ΠΊΠ° Π½ΠΈΠΆΠ΅.
- ΠΡΠ΅Π΄ΡΡΠ°Π²ΡΡΠ΅ ΡΠ΅Π±Π΅ ΡΠ»Π΅ΠΊΡΡΠΎΠ½, Π΄Π²ΠΈΠΆΡΡΠΈΠΉΡΡ Ρ ΠΏΠΎΡΡΠΎΡΠ½Π½ΠΎΠΉ ΡΠΊΠΎΡΠΎΡΡΡΡ Π² ΠΎΠ΄Π½ΠΎΡΠΎΠ΄Π½ΠΎΠΌ ΠΌΠ°Π³Π½ΠΈΡΠ½ΠΎΠΌ ΠΏΠΎΠ»Π΅ ΠΈ ΡΠΊΠ°Π·ΡΠ²Π°ΡΡΠΈΠΉ Π½Π° ΡΡΡΠ°Π½ΠΈΡΡ (ΠΎΠ±ΠΎΠ·Π½Π°ΡΠ°Π΅ΡΡΡ ΡΡΠΈΠΌ ΡΠΈΠΌΠ²ΠΎΠ»ΠΎΠΌ β) .
- ΠΠΎΠ³Π΄Π° ΡΠ»Π΅ΠΊΡΡΠΎΠ½ Π΄Π²ΠΈΠΆΠ΅ΡΡΡ Π² ΠΏΠΎΠ»Π΅, Π½Π° Π½Π΅Π³ΠΎ Π΄Π΅ΠΉΡΡΠ²ΡΠ΅Ρ ΡΠΈΠ»Π°, Π΄Π΅ΠΉΡΡΠ²ΡΡΡΠ°Ρ ΠΏΠΎΠ΄ ΠΏΡΡΠΌΡΠΌ ΡΠ³Π»ΠΎΠΌ ΠΊ ββΡΠΊΠΎΡΠΎΡΡΠΈ. Π’ΡΠ°Π΅ΠΊΡΠΎΡΠΈΡ ΡΠ»Π΅ΠΊΡΡΠΎΠ½Π° ΡΠ»Π΅Π³ΠΊΠ° ΠΈΡΠΊΡΠΈΠ²Π»ΡΠ΅ΡΡΡ, ΠΈ ΡΠ΅ΠΏΠ΅ΡΡ Π΅Π³ΠΎ ΡΠΊΠΎΡΠΎΡΡΡ ΠΈΠΌΠ΅Π΅Ρ Π΄ΡΡΠ³ΠΎΠ΅ Π½Π°ΠΏΡΠ°Π²Π»Π΅Π½ΠΈΠ΅. ΠΠ΄Π½Π°ΠΊΠΎ ΡΠΊΠΎΡΠΎΡΡΡ ΠΏΠΎ-ΠΏΡΠ΅ΠΆΠ½Π΅ΠΌΡ Π½Π°Ρ ΠΎΠ΄ΠΈΡΡΡ ΠΏΠΎΠ΄ ΠΏΡΡΠΌΡΠΌ ΡΠ³Π»ΠΎΠΌ ΠΊ ββΠ½Π°ΠΏΡΠ°Π²Π»Π΅Π½ΠΈΡ ΠΌΠ°Π³Π½ΠΈΡΠ½ΠΎΠ³ΠΎ ΠΏΠΎΠ»Ρ.
- Π‘ΠΈΠ»Π° ΠΈ ΡΠΊΠΎΡΠΎΡΡΡ ΠΏΠΎ-ΠΏΡΠ΅ΠΆΠ½Π΅ΠΌΡ Π½Π°Ρ ΠΎΠ΄ΡΡΡΡ ΠΏΠΎΠ΄ ΠΏΡΡΠΌΡΠΌ ΡΠ³Π»ΠΎΠΌ Π΄ΡΡΠ³ ΠΊ Π΄ΡΡΠ³Ρ ΠΈ ΠΎΡΡΠ°ΡΡΡΡ Π² ΠΎΠ΄Π½ΠΎΠΉ ΠΏΠ»ΠΎΡΠΊΠΎΡΡΠΈ. ΠΠ° ΡΠ°ΠΌΠΎΠΌ Π΄Π΅Π»Π΅ ΠΌΠ°Π³Π½ΠΈΡΠ½Π°Ρ ΡΠΈΠ»Π° Π½Π°ΠΏΡΠ°Π²Π»Π΅Π½Π° ββΠΊ ΡΠ΅Π½ΡΡΡ ΠΊΡΡΠ³ΠΎΠ²ΠΎΠ³ΠΎ ΠΏΡΡΠΈ, ΠΏΠΎ ΠΊΠΎΡΠΎΡΠΎΠΌΡ Π΄Π²ΠΈΠΆΠ΅ΡΡΡ ΡΠ»Π΅ΠΊΡΡΠΎΠ½.
- ΠΡΠΎ ΠΏΡΠΎΠ΄ΠΎΠ»ΠΆΠ°Π΅ΡΡΡ, ΡΠΈΠ»Π° ΠΈ ΡΠΊΠΎΡΠΎΡΡΡ Π²ΡΠ΅Π³Π΄Π° ΠΏΠ΅ΡΠΏΠ΅Π½Π΄ΠΈΠΊΡΠ»ΡΡΠ½Ρ. ΠΡΠ»ΠΈ ΠΌΡ ΠΏΠΎΠΌΠ½ΠΈΠΌ ΠΈΠ· Π΄Π²ΠΈΠΆΠ΅Π½ΠΈΡ ΠΏΠΎ ΠΊΡΡΠ³Ρ, Ρ Π½Π°Ρ Π΅ΡΡΡ ΠΊΡΡΠ³ΠΎΠ²ΡΡ Π΄Π²ΠΈΠΆΠ΅Π½ΠΈΠΉ , Π΅ΡΠ»ΠΈ ΡΡΠΈ Π΄Π²Π΅ Π²Π΅Π»ΠΈΡΠΈΠ½Ρ ΠΏΠ΅ΡΠΏΠ΅Π½Π΄ΠΈΠΊΡΠ»ΡΡΠ½Ρ Ρ ΠΏΠΎΡΡΠΎΡΠ½Π½ΡΠΌΠΈ Π²Π΅Π»ΠΈΡΠΈΠ½Π°ΠΌΠΈ.
ΠΠ° ΡΡΠΎΠΌ ΠΈΠ·ΠΎΠ±ΡΠ°ΠΆΠ΅Π½ΠΈΠΈ ΠΏΠΎΠΊΠ°Π·Π°Π½Π° ΠΎΡΡΠΈΡΠ°ΡΠ΅Π»ΡΠ½ΠΎ Π·Π°ΡΡΠΆΠ΅Π½Π½Π°Ρ ΡΠ°ΡΡΠΈΡΠ°, ΡΠ°ΠΊΠ°Ρ ΠΊΠ°ΠΊ ΡΠ»Π΅ΠΊΡΡΠΎΠ½, Π΄Π²ΠΈΠΆΡΡΠ°ΡΡΡ Π² ΠΌΠ°Π³Π½ΠΈΡΠ½ΠΎΠΌ ΠΏΠΎΠ»Π΅. Π‘ΠΈΠ»Π°, Π΄Π΅ΠΉΡΡΠ²ΡΡΡΠ°Ρ Π½Π° ΡΠ°ΡΡΠΈΡΡ, ΠΏΠΎΡΡΠΎΡΠ½Π½ΠΎ ΠΌΠ΅Π½ΡΠ΅Ρ Π½Π°ΠΏΡΠ°Π²Π»Π΅Π½ΠΈΠ΅, Π½ΠΎ ΠΏΠΎΡΡΠΎΡΠ½Π½Π° ΠΏΠΎ Π²Π΅Π»ΠΈΡΠΈΠ½Π΅, ΠΊΠ°ΠΊ ΠΈ ΡΠΊΠΎΡΠΎΡΡΡ. ΠΠΎΡΠΊΠΎΠ»ΡΠΊΡ Π΄Π²Π΅ Π²Π΅Π»ΠΈΡΠΈΠ½Ρ Π²ΡΠ΅Π³Π΄Π° ΠΏΠ΅ΡΠΏΠ΅Π½Π΄ΠΈΠΊΡΠ»ΡΡΠ½Ρ, ΡΠ»Π΅ΠΊΡΡΠΎΠ½ Π΄Π²ΠΈΠΆΠ΅ΡΡΡ ΠΏΠΎ ΠΎΠΊΡΡΠΆΠ½ΠΎΡΡΠΈ, Wikimedia Commons CC 4.0
ΠΠΏΡΠ΅Π΄Π΅Π»ΡΠ΅ΠΌ Π½Π°ΠΏΡΠ°Π²Π»Π΅Π½ΠΈΠ΅ ΡΠΈΠ»Ρ ΠΏΠΎ ΠΏΡΠ°Π²ΠΈΠ»Ρ Π»Π΅Π²ΠΎΠΉ ΡΡΠΊΠΈ. ΠΠ»Π΅ΠΊΡΡΠΎΠ½Ρ Π·Π°ΡΡΠΆΠ΅Π½Ρ ΠΎΡΡΠΈΡΠ°ΡΠ΅Π»ΡΠ½ΠΎ, Π° ΡΡΠΎ ΠΎΠ·Π½Π°ΡΠ°Π΅Ρ, ΡΡΠΎ ΡΠΎΠΊ ΠΏΡΠΎΡΠΈΠ²ΠΎΠΏΠΎΠ»ΠΎΠΆΠ΅Π½ Π½Π°ΠΏΡΠ°Π²Π»Π΅Π½ΠΈΡ Π΄Π²ΠΈΠΆΠ΅Π½ΠΈΡ ΡΠ»Π΅ΠΊΡΡΠΎΠ½Π°.
Π Π°ΡΡΠΌΠΎΡΡΠ΅Π½ΠΈΠ΅ ΠΏΠΎΠ»ΠΎΠΆΠ΅Π½ΠΈΡ ΡΠ»Π΅ΠΊΡΡΠΎΠ½Π° Π² ΠΌΠ°Π³Π½ΠΈΡΠ½ΠΎΠΌ ΠΏΠΎΠ»Π΅, ΠΏΠΎΠ΄ΠΎΠ±Π½ΠΎΠ³ΠΎ ΠΈΠ·ΠΎΠ±ΡΠ°ΠΆΠ΅Π½Π½ΠΎΠΌΡ Π½Π° Π΄ΠΈΠ°Π³ΡΠ°ΠΌΠΌΠ΅ Π½ΠΈΠΆΠ΅, Π½Π΅ ΠΏΠΎΠ·Π²ΠΎΠ»ΡΠ΅Ρ ΡΠ°ΡΡΠΈΡΠ°ΠΌ Π΄Π²ΠΈΠ³Π°ΡΡΡΡ ΠΏΠΎ ΠΊΡΡΠ³ΠΎΠ²ΠΎΠΉ ΡΡΠ°Π΅ΠΊΡΠΎΡΠΈΠΈ. ΠΡΠΎ ΠΏΡΠΎΠΈΡΡ ΠΎΠ΄ΠΈΡ ΠΏΠΎ Π΄Π²ΡΠΌ ΠΏΡΠΈΡΠΈΠ½Π°ΠΌ:
- Π§Π°ΡΡΠΈΡΠ° ΡΠ½Π°ΡΠ°Π»Π° Π½Π°ΡΠΈΠ½Π°Π΅Ρ Π΄Π²ΠΈΠ³Π°ΡΡΡΡ ΠΏΠΎ ΠΊΡΡΠ³ΠΎΠ²ΠΎΠΉ ΡΡΠ°Π΅ΠΊΡΠΎΡΠΈΠΈ, Π½ΠΎ ΠΏΠΎΠΊΠΈΠ΄Π°Π΅Ρ Π½ΠΈΠΆΠ½ΡΡ ΡΠ°ΡΡΡ ΠΏΠ»ΠΎΡΠΊΠΎΡΡΠΈ, ΠΏΠΎΡΠΎΠΌΡ ΡΡΠΎ ΠΌΠ°Π³Π½ΠΈΡΠ½ΠΎΠ΅ ΠΏΠΎΠ»Π΅ Π·Π°ΠΊΠ°Π½ΡΠΈΠ²Π°Π΅ΡΡΡ. ΠΡΠΎ ΠΌΠΎΠΆΠ½ΠΎ ΡΠ΅ΡΠΈΡΡ, ΡΠ°ΡΡΠΈΡΠΈΠ² ΠΌΠ°Π³Π½ΠΈΡΠ½ΠΎΠ΅ ΠΏΠΎΠ»Π΅ Π² ΡΡΠΎΠΌ Π½Π°ΠΏΡΠ°Π²Π»Π΅Π½ΠΈΠΈ.
- ΠΡΠ»ΠΈ ΠΌΠ°Π³Π½ΠΈΡΠ½ΠΎΠ΅ ΠΏΠΎΠ»Π΅ ΡΠ°ΡΡΡΠ½ΡΡΠΎ Π² Π½Π°ΠΏΡΠ°Π²Π»Π΅Π½ΠΈΠΈ, ΡΠΊΠ°Π·Π°Π½Π½ΠΎΠΌ Π² ΠΏΡΠ½ΠΊΡΠ΅ 1, ΡΠ°ΡΡΠΈΡΠ° Π²ΡΠΉΠ΄Π΅Ρ ΠΈΠ· ΠΎΠ±Π»Π°ΡΡΠΈ Π²Π»Π΅Π²ΠΎ, Π·Π°Π²Π΅ΡΡΠΈΠ² ΠΎΠ΄ΠΈΠ½ ΠΏΠΎΠ»ΡΠΊΡΡΠ³. ΠΡΠΎ ΠΏΡΠΎΠΈΠ·ΠΎΡΠ»ΠΎ Π±Ρ Π² Π»ΡΠ±ΠΎΠΌ ΡΠ»ΡΡΠ°Π΅, ΠΊΠΎΠ³Π΄Π° ΡΠ°ΡΡΠΈΡΠ° ΠΏΠΎΠΏΠ°Π΄Π°Π΅Ρ Π² ΠΎΠ±Π»Π°ΡΡΡ ΠΌΠ°Π³Π½ΠΈΡΠ½ΠΎΠ³ΠΎ ΠΏΠΎΠ»Ρ: ΡΠ°ΡΡΠΈΡΠ° Π²ΡΡΠ²Π΅ΡΡΡ, ΠΏΡΠΎΠΉΠ΄Ρ ΠΏΠΎΠ»ΠΎΠ²ΠΈΠ½Ρ ΠΎΠΊΡΡΠΆΠ½ΠΎΡΡΠΈ.
- Π§ΡΠΎΠ±Ρ Π·Π°ΡΡΠ°Π²ΠΈΡΡ ΡΠ°ΡΡΠΈΡΡ Π΄Π²ΠΈΠ³Π°ΡΡΡΡ ΠΏΠΎ ΠΊΡΡΠ³Ρ, ΠΌΠ°Π³Π½ΠΈΡΠ½ΠΎΠ΅ ΠΏΠΎΠ»Π΅ Π΄ΠΎΠ»ΠΆΠ½ΠΎ Π±ΡΡΡ ΠΏΡΠΈΠ»ΠΎΠΆΠ΅Π½ΠΎ ΠΈΠ·Π²Π½Π΅, ΠΊΠ°ΠΊ ΡΠΎΠ»ΡΠΊΠΎ ΡΠ°ΡΡΠΈΡΠ° ΡΠΆΠ΅ Π½Π°Ρ ΠΎΠ΄ΠΈΡΡΡ Π² Π΅Π³ΠΎ ΠΎΠ±Π»Π°ΡΡΠΈ. ΠΠ»ΠΎΡΠ°Π΄Ρ ΠΌΠ°Π³Π½ΠΈΡΠ½ΠΎΠ³ΠΎ ΠΏΠΎΠ»Ρ ΡΠ°ΠΊΠΆΠ΅ Π΄ΠΎΠ»ΠΆΠ½Π° Π±ΡΡΡ Π΄ΠΎΡΡΠ°ΡΠΎΡΠ½ΠΎ Π±ΠΎΠ»ΡΡΠΎΠΉ, ΡΡΠΎΠ±Ρ ΠΎΡ Π²Π°ΡΡΠ²Π°ΡΡ Π²Π΅ΡΡ ΠΊΡΡΠ³ΠΎΠ²ΠΎΠΉ ΠΏΡΡΡ, ΠΏΠΎ ΠΊΠΎΡΠΎΡΠΎΠΌΡ Π±ΡΠ΄Π΅Ρ Π΄Π²ΠΈΠ³Π°ΡΡΡΡ Π·Π°ΡΡΠΆΠ΅Π½Π½Π°Ρ ΡΠ°ΡΡΠΈΡΠ°.
ΠΠ»Π΅ΠΊΡΡΠΎΠ½ Π±ΡΠ΄Π΅Ρ Π΄Π²ΠΈΠ³Π°ΡΡΡΡ ΠΏΠΎ ΠΎΠΊΡΡΠΆΠ½ΠΎΡΡΠΈ, Π΅ΡΠ»ΠΈ ΠΎΠ½ Π²Ρ ΠΎΠ΄ΠΈΡ Π² ΠΎΠ΄Π½ΠΎΡΠΎΠ΄Π½ΠΎΠ΅ ΠΌΠ°Π³Π½ΠΈΡΠ½ΠΎΠ΅ ΠΏΠΎΠ»Π΅ Ρ ΠΏΠΎΡΡΠΎΡΠ½Π½ΠΎΠΉ ΡΠΊΠΎΡΠΎΡΡΡΡ. ΠΠ½ ΠΏΡΠ΅ΠΊΡΠ°ΡΠΈΡ ΡΠ²ΠΎΠ΅ ΠΊΡΡΠ³ΠΎΠ²ΠΎΠ΅ Π΄Π²ΠΈΠΆΠ΅Π½ΠΈΠ΅, Π΅ΡΠ»ΠΈ Π²ΡΠΉΠ΄Π΅Ρ ΠΈΠ· ΠΎΠ±Π»Π°ΡΡΠΈ, ΡΠΎΠ΄Π΅ΡΠΆΠ°ΡΠ΅ΠΉ ΠΌΠ°Π³Π½ΠΈΡΠ½ΠΎΠ΅ ΠΏΠΎΠ»Π΅, ΡΡΠΎ ΠΏΡΠΎΠΈΡΡ ΠΎΠ΄ΠΈΡ Π² Π½ΠΈΠΆΠ½Π΅ΠΉ ΡΠ°ΡΡΠΈ ΡΡΠΎΠΉ Π΄ΠΈΠ°Π³ΡΠ°ΠΌΠΌΡ. ΠΠ·ΡΡΠ°ΠΉΡΠ΅ ΡΠΌΠ½ΡΠ΅ ΠΎΡΠΈΠ³ΠΈΠ½Π°Π»Ρ.
Π’ΠΈΠΏΡ ΡΠ°ΡΡΠΈΡ Π² ΠΌΠ°Π³Π½ΠΈΡΠ½ΡΡ ΠΏΠΎΠ»ΡΡ
ΠΡ Π²ΠΈΠ΄Π΅Π»ΠΈ, ΠΊΠ°ΠΊ ΡΠ»Π΅ΠΊΡΡΠΎΠ½Ρ ΠΎΡΠΊΠ»ΠΎΠ½ΡΡΡΡΡ ΠΌΠ°Π³Π½ΠΈΡΠ½ΡΠΌΠΈ ΠΏΠΎΠ»ΡΠΌΠΈ, Π½ΠΎ ΠΌΡ ΠΌΠΎΠΆΠ΅ΠΌ Π½Π°Π±Π»ΡΠ΄Π°ΡΡ ΠΏΠΎΠ΄ΠΎΠ±Π½ΡΠ΅ ΠΎΡΠΊΠ»ΠΎΠ½Π΅Π½ΠΈΡ ΠΈ Π΄Π»Ρ Π΄ΡΡΠ³ΠΈΡ ΡΠ°ΡΡΠΈΡ. ΠΠ΄Π΅ΡΡ ΠΌΡ ΡΠ°ΡΡΠΌΠΎΡΡΠΈΠΌ ΡΡΠΈ ΠΈΠ· ΡΡΠΈΡ ΡΠ°ΡΡΠΈΡ; Π°Π»ΡΡΠ°-ΡΠ°ΡΡΠΈΡΠ°, Π±Π΅ΡΠ°-ΡΠ°ΡΡΠΈΡΠ° ΠΈ Π³Π°ΠΌΠΌΠ°-ΡΠ°ΡΡΠΈΡΠ° .
ΠΠ»ΡΡΠ°-, Π±Π΅ΡΠ°- ΠΈ Π³Π°ΠΌΠΌΠ°-ΡΠ°ΡΡΠΈΡΡ Π² ΠΌΠ°Π³Π½ΠΈΡΠ½ΠΎΠΌ ΠΏΠΎΠ»Π΅
ΠΠ· Π°ΡΠΎΠΌΠΎΠ² ΠΈ ΠΈΠ·Π»ΡΡΠ΅Π½ΠΈΡ ΠΌΡ Π·Π½Π°Π΅ΠΌ, ΡΡΠΎ Π°Π»ΡΡΠ°-ΡΠ°ΡΡΠΈΡΡ ΠΏΡΠ΅Π΄ΡΡΠ°Π²Π»ΡΡΡ ΡΠΎΠ±ΠΎΠΉ ΡΠ΄ΡΠ° Π³Π΅Π»ΠΈΡ (ΠΎΠ½ΠΈ ΡΠΎΠ΄Π΅ΡΠΆΠ°Ρ Π΄Π²Π° Π½Π΅ΠΉΡΡΠΎΠ½Π° ΠΈ Π΄Π²Π° ΠΏΡΠΎΡΠΎΠ½Π°, ΡΡΠΎ Π΄Π΅Π»Π°Π΅Ρ ΠΈΡ ΠΏΠΎΠ»ΠΎΠΆΠΈΡΠ΅Π»ΡΠ½ΠΎ Π·Π°ΡΡΠΆΠ΅Π½Π½ΡΠΌΠΈ). ΠΠ½ΠΈ ΡΠ°ΠΊΠΆΠ΅ Π΄ΠΎΠ²ΠΎΠ»ΡΠ½ΠΎ ΡΡΠΆΠ΅Π»ΡΠ΅, ΠΏΠΎ ΠΊΡΠ°ΠΉΠ½Π΅ΠΉ ΠΌΠ΅ΡΠ΅, ΠΏΠΎ ΡΡΠ°Π²Π½Π΅Π½ΠΈΡ Ρ ΡΠ»Π΅ΠΊΡΡΠΎΠ½ΠΎΠΌ, Π° ΡΡΠΎ ΠΎΠ·Π½Π°ΡΠ°Π΅Ρ, ΡΡΠΎ ΡΡΠ΅Π±ΡΠ΅ΡΡΡ Π±ΠΎΠ»ΡΡΠ°Ρ ΠΌΠ°Π³Π½ΠΈΡΠ½Π°Ρ ΡΠΈΠ»Π°, ΡΡΠΎΠ±Ρ ΠΎΡΠΊΠ»ΠΎΠ½ΠΈΡΡ ΠΈΡ Π½Π° ΡΡ ΠΆΠ΅ Π²Π΅Π»ΠΈΡΠΈΠ½Ρ.
ΠΠ΅ΡΠ°-ΡΠ°ΡΡΠΈΡΡ β ΡΡΠΎ Π±ΡΡΡΡΠΎ Π΄Π²ΠΈΠΆΡΡΠΈΠ΅ΡΡ ΡΠ»Π΅ΠΊΡΡΠΎΠ½Ρ, ΠΏΠΎΡΡΠΎΠΌΡ ΠΎΠ½ΠΈ Π΄Π΅ΠΌΠΎΠ½ΡΡΡΠΈΡΡΡΡ ΡΠΎ ΠΆΠ΅ ΠΏΠΎΠ²Π΅Π΄Π΅Π½ΠΈΠ΅ ΠΎΡΠΊΠ»ΠΎΠ½Π΅Π½ΠΈΡ, ΠΊΠΎΡΠΎΡΠΎΠ΅ ΠΌΡ Π½Π°Π±Π»ΡΠ΄Π°Π»ΠΈ ΡΠ°Π½Π΅Π΅. ΠΠ° ΡΠΈΡΡΠ½ΠΊΠ΅ Π½ΠΈΠΆΠ΅ ΠΏΠΎΠΊΠ°Π·Π°Π½Π° ΡΠ°Π·Π½ΠΈΡΠ° ΠΌΠ΅ΠΆΠ΄Ρ ΡΠ΅ΠΌ, ΠΊΠ°ΠΊ Π°Π»ΡΡΠ°- ΠΈ Π±Π΅ΡΠ°-ΡΠ°ΡΡΠΈΡΡ ΠΎΡΠΊΠ»ΠΎΠ½ΡΡΡΡΡ ΠΎΠ΄Π½ΠΈΠΌ ΠΈ ΡΠ΅ΠΌ ΠΆΠ΅ ΠΎΠ΄Π½ΠΎΡΠΎΠ΄Π½ΡΠΌ ΠΌΠ°Π³Π½ΠΈΡΠ½ΡΠΌ ΠΏΠΎΠ»Π΅ΠΌ ΠΏΡΠΈ Π΄Π²ΠΈΠΆΠ΅Π½ΠΈΠΈ ΡΠ΅ΡΠ΅Π· Π½Π΅Π³ΠΎ.
ΠΠ°ΠΌΠΌΠ°-ΡΠ°ΡΡΠΈΡΡ ΠΏΡΠ΅Π΄ΡΡΠ°Π²Π»ΡΡΡ ΡΠΎΠ±ΠΎΠΉ ΡΠΎΡΠΎΠ½Ρ Π²ΡΡΠΎΠΊΠΎΠΉ ΡΠ½Π΅ΡΠ³ΠΈΠΈ, ΠΈΡΠΏΡΡΠΊΠ°Π΅ΠΌΡΠ΅ ΠΏΡΠΈ ΡΠ°Π΄ΠΈΠΎΠ°ΠΊΡΠΈΠ²Π½ΠΎΠΌ ΡΠ°ΡΠΏΠ°Π΄Π΅. ΠΠ½ΠΈ ΡΠ°ΡΡΠΎ ΠΈΡΠΏΡΡΠΊΠ°ΡΡΡΡ Π²ΠΌΠ΅ΡΡΠ΅ Ρ Π°Π»ΡΡΠ°- ΠΈ Π±Π΅ΡΠ°-ΡΠ°ΡΡΠΈΡΠ°ΠΌΠΈ. ΠΠ΄Π½Π°ΠΊΠΎ Π±ΠΎΠ»ΡΡΠ°Ρ ΡΠ°Π·Π½ΠΈΡΠ° Π² ΡΠΎΠΌ, ΡΡΠΎ ΡΡΠΎ ΡΠΎΡΠΎΠ½Ρ ΠΈ, ΡΠ»Π΅Π΄ΠΎΠ²Π°ΡΠ΅Π»ΡΠ½ΠΎ, Π½Π΅Π·Π°ΡΡΠΆΠ΅Π½Π½ΡΡ . ΠΠ½ΠΈ Π½Π΅ Π±ΡΠ΄ΡΡ ΠΎΡΠΊΠ»ΠΎΠ½ΡΡΡΡΡ ΠΌΠ°Π³Π½ΠΈΡΠ½ΡΠΌ ΠΏΠΎΠ»Π΅ΠΌ. ΠΠ° ΠΏΡΠΈΠ²Π΅Π΄Π΅Π½Π½ΠΎΠΉ Π½ΠΈΠΆΠ΅ Π΄ΠΈΠ°Π³ΡΠ°ΠΌΠΌΠ΅ ΠΏΠΎΠΊΠ°Π·Π°Π½Ρ ΠΏΡΡΠΈ Π°Π»ΡΡΠ°-, Π±Π΅ΡΠ°- ΠΈ Π³Π°ΠΌΠΌΠ°-ΡΠ°ΡΡΠΈΡ, ΠΊΠΎΠ³Π΄Π° ΠΎΠ½ΠΈ Π΄Π²ΠΈΠΆΡΡΡΡ ΠΏΠ΅ΡΠ²ΠΎΠ½Π°ΡΠ°Π»ΡΠ½ΠΎ ΠΏΠΎ ΠΏΡΡΠΌΠΎΠΉ Π»ΠΈΠ½ΠΈΠΈ Π² ΠΎΠ±Π»Π°ΡΡΡ, ΡΠΎΠ΄Π΅ΡΠΆΠ°ΡΡΡ ΠΌΠ°Π³Π½ΠΈΡΠ½ΠΎΠ΅ ΠΏΠΎΠ»Π΅.
ΠΠ»ΡΡΠ°- ΠΈ Π±Π΅ΡΠ°-ΡΠ°ΡΡΠΈΡΡ Π±ΡΠ΄ΡΡ ΠΎΡΠΊΠ»ΠΎΠ½ΡΡΡΡΡ, Π΅ΡΠ»ΠΈ ΠΏΠΎΠΏΠ°Π΄ΡΡ Π² ΠΎΠ±Π»Π°ΡΡΡ Ρ ΠΌΠ°Π³Π½ΠΈΡΠ½ΡΠΌ ΠΏΠΎΠ»Π΅ΠΌ. Π§Π°ΡΡΠΈΡΡ ΠΎΡΠΊΠ»ΠΎΠ½ΡΡΡΡΡ Π² ΠΏΡΠΎΡΠΈΠ²ΠΎΠΏΠΎΠ»ΠΎΠΆΠ½ΡΡ Π½Π°ΠΏΡΠ°Π²Π»Π΅Π½ΠΈΡΡ , ΡΠ°ΠΊ ΠΊΠ°ΠΊ ΠΎΠ½ΠΈ Π·Π°ΡΡΠΆΠ΅Π½Ρ ΠΏΡΠΎΡΠΈΠ²ΠΎΠΏΠΎΠ»ΠΎΠΆΠ½ΠΎ. ΠΠ»ΡΡΠ°-ΡΠ°ΡΡΠΈΡΠ° ΠΎΡΠΊΠ»ΠΎΠ½ΡΠ΅ΡΡΡ ΠΌΠ΅Π½ΡΡΠ΅, ΠΏΠΎΡΠΎΠΌΡ ΡΡΠΎ ΠΎΠ½Π° Π½Π°ΠΌΠ½ΠΎΠ³ΠΎ ΡΡΠΆΠ΅Π»Π΅Π΅ Π±Π΅ΡΠ°-ΡΠ°ΡΡΠΈΡΡ. ΠΠ°ΠΌΠΌΠ°-ΡΠ°ΡΡΠΈΡΠ° Π½Π΅ ΠΎΡΠΊΠ»ΠΎΠ½ΡΠ΅ΡΡΡ Π² ΠΌΠ°Π³Π½ΠΈΡΠ½ΠΎΠΌ ΠΏΠΎΠ»Π΅, ΡΠ°ΠΊ ΠΊΠ°ΠΊ ΠΎΠ½Π° Π½Π΅ Π·Π°ΡΡΠΆΠ΅Π½Π°. StudySmarter Originals
ΠΠ»ΡΡΠ°-ΡΠ°ΡΡΠΈΡΠ° ΠΎΡΠΊΠ»ΠΎΠ½ΡΠ΅ΡΡΡ Π² ΠΌΠ΅Π½ΡΡΠ΅ΠΉ ΡΡΠ΅ΠΏΠ΅Π½ΠΈ, Π΄Π°ΠΆΠ΅ Π΅ΡΠ»ΠΈ ΠΎΠ½Π° ΠΈΠΌΠ΅Π΅Ρ Π±ΠΎΠ»ΡΡΠΈΠΉ Π·Π°ΡΡΠ΄, ΡΠ΅ΠΌ Π±Π΅ΡΠ°-ΡΠ°ΡΡΠΈΡΠ°. ΠΡΠΎ ΠΏΠΎΡΠΎΠΌΡ, ΡΡΠΎ ΠΎΠ½ ΡΡΠΆΠ΅Π»Π΅Π΅ (ΠΈΠΌΠ΅Π΅Ρ Π±ΠΎΠ»ΡΡΡΡ ΠΌΠ°ΡΡΡ) ΠΈ ΡΠΈΠ»Π΅ ΡΡΡΠ΄Π½Π΅Π΅ Π΅Π³ΠΎ ΠΎΡΠΊΠ»ΠΎΠ½ΠΈΡΡ. ΠΠ²Π΅ ΡΠ°ΡΡΠΈΡΡ ΠΎΡΠΊΠ»ΠΎΠ½ΡΡΡΡΡ Π² ΠΏΡΠΎΡΠΈΠ²ΠΎΠΏΠΎΠ»ΠΎΠΆΠ½ΡΡ Π½Π°ΠΏΡΠ°Π²Π»Π΅Π½ΠΈΡΡ ΠΈΠ·-Π·Π° ΡΠΎΠ³ΠΎ, ΡΡΠΎ ΠΈΡ Π·Π°ΡΡΠ΄Ρ ΠΈΠΌΠ΅ΡΡ ΠΏΡΠΎΡΠΈΠ²ΠΎΠΏΠΎΠ»ΠΎΠΆΠ½ΡΠ΅ Π·Π½Π°ΠΊΠΈ (Π°Π»ΡΡΠ°-ΡΠ°ΡΡΠΈΡΡ ΠΏΠΎΠ»ΠΎΠΆΠΈΡΠ΅Π»ΡΠ½ΡΠ΅, Π° Π±Π΅ΡΠ°-ΡΠ°ΡΡΠΈΡΡ ΠΎΡΡΠΈΡΠ°ΡΠ΅Π»ΡΠ½ΡΠ΅), ΡΠΎΠ·Π΄Π°Π²Π°Ρ ΠΌΠ°Π³Π½ΠΈΡΠ½ΡΠ΅ ΡΠΈΠ»Ρ Π² ΠΏΡΠΎΡΠΈΠ²ΠΎΠΏΠΎΠ»ΠΎΠΆΠ½ΡΡ Π½Π°ΠΏΡΠ°Π²Π»Π΅Π½ΠΈΡΡ . ΠΠ°ΠΌΠΌΠ°-ΡΠ°ΡΡΠΈΡΠ° Π½Π΅ Π²Π·Π°ΠΈΠΌΠΎΠ΄Π΅ΠΉΡΡΠ²ΡΠ΅Ρ Ρ ΠΌΠ°Π³Π½ΠΈΡΠ½ΡΠΌ ΠΏΠΎΠ»Π΅ΠΌ ΠΈ ΠΏΡΠΎΡ ΠΎΠ΄ΠΈΡ ΡΠ΅ΡΠ΅Π· ΠΎΠ±Π»Π°ΡΡΡ Π±Π΅Π· ΠΎΡΠΊΠ»ΠΎΠ½Π΅Π½ΠΈΡ.
Π§Π°ΡΡΠΈΡΡ Π² ΠΌΠ°Π³Π½ΠΈΡΠ½ΡΡ ΠΏΠΎΠ»ΡΡ β ΠΎΡΠ½ΠΎΠ²Π½ΡΠ΅ Π²ΡΠ²ΠΎΠ΄Ρ
ΠΠ°Π³Π½ΠΈΡΠ½Π°Ρ ΡΠΈΠ»Π° β ΡΡΠΎ ΡΠΈΠ»Π°, ΠΊΠΎΡΠΎΡΡΡ ΠΈΡΠΏΡΡΡΠ²Π°Π΅Ρ Π·Π°ΡΡΠΆΠ΅Π½Π½Π°Ρ ΡΠ°ΡΡΠΈΡΠ° (ΡΠ»Π΅ΠΊΡΡΠΎΠ½, ΠΏΡΠΎΡΠΎΠ½, ΠΈΠΎΠ½ ΠΈ Ρ. Π΄.), ΠΊΠΎΠ³Π΄Π° ΠΎΠ½Π° Π΄Π²ΠΈΠΆΠ΅ΡΡΡ ΡΠ΅ΡΠ΅Π· ΠΌΠ°Π³Π½ΠΈΡΠ½ΠΎΠ΅ ΠΏΠΎΠ»Π΅.
ΠΠ°Π³Π½ΠΈΡΠ½ΠΎΠ΅ ΠΏΠΎΠ»Π΅ β ΡΡΠΎ ΠΎΠ±Π»Π°ΡΡΡ Π² ΠΏΡΠΎΡΡΡΠ°Π½ΡΡΠ²Π΅, Π² ΠΊΠΎΡΠΎΡΠΎΠΉ Π΄Π²ΠΈΠΆΡΡΠΈΠΉΡΡ Π·Π°ΡΡΠ΄ ΠΈΠ»ΠΈ ΠΏΠΎΡΡΠΎΡΠ½Π½ΡΠΉ ΠΌΠ°Π³Π½ΠΈΡ ΠΈΡΠΏΡΡΡΠ²Π°Π΅Ρ ΡΠΈΠ»Ρ.
Π Π·Π°ΡΡΠ΄, ΠΈ Π΄Π²ΠΈΠΆΠ΅Π½ΠΈΠ΅ ΡΠ°ΡΡΠΈΡΡ Π½Π΅ΠΎΠ±Ρ ΠΎΠ΄ΠΈΠΌΡ Π΄Π»Ρ ΠΏΡΠΎΡΠ²Π»Π΅Π½ΠΈΡ ΡΠΈΠ»Ρ ΠΏΠΎΠ»Π΅ΠΌ.
Π‘ΠΈΠ»Π°, Π΄Π΅ΠΉΡΡΠ²ΡΡΡΠ°Ρ ΠΌΠ°Π³Π½ΠΈΡΠ½ΡΠΌ ΠΏΠΎΠ»Π΅ΠΌ ΡΠΈΠ»Ρ Π½Π° Π·Π°ΡΡΠΆΠ΅Π½Π½ΡΡ ΡΠ°ΡΡΠΈΡΡ, Π΄Π²ΠΈΠΆΡΡΡΡΡΡ ΡΠΎ ΡΠΊΠΎΡΠΎΡΡΡΡ, ΠΎΠΏΡΠ΅Π΄Π΅Π»ΡΠ΅ΡΡΡ ΠΏΠΎ ΠΏΡΠΈΠ²Π΅Π΄Π΅Π½Π½ΠΎΠΉ Π½ΠΈΠΆΠ΅ ΡΠΎΡΠΌΡΠ»Π΅. ΠΠ°ΠΏΡΠ°Π²Π»Π΅Π½ΠΈΠ΅ ΡΠΈΠ»Ρ ΠΏΠ΅ΡΠΏΠ΅Π½Π΄ΠΈΠΊΡΠ»ΡΡΠ½ΠΎ Π½Π°ΠΏΡΠ°Π²Π»Π΅Π½ΠΈΡ Π΄Π²ΠΈΠΆΠ΅Π½ΠΈΡ ΡΠ°ΡΡΠΈΡΡ ΠΈ ΠΌΠ°Π³Π½ΠΈΡΠ½ΠΎΠ³ΠΎ ΠΏΠΎΠ»Ρ ΠΈ ΠΎΠΏΡΠ΅Π΄Π΅Π»ΡΠ΅ΡΡΡ Π²ΡΡΠ°ΠΆΠ΅Π½ΠΈΠ΅ΠΌ:
ΠΠ²ΠΈΠΆΡΡΠ°ΡΡΡ Π·Π°ΡΡΠΆΠ΅Π½Π½Π°Ρ ΡΠ°ΡΡΠΈΡΠ° Π² ΠΎΠ±Π»Π°ΡΡΠΈ Ρ ΠΎΠ΄Π½ΠΎΡΠΎΠ΄Π½ΡΠΌ ΠΌΠ°Π³Π½ΠΈΡΠ½ΡΠΌ ΠΏΠΎΠ»Π΅ΠΌ Π΄Π²ΠΈΠΆΠ΅ΡΡΡ ΠΏΠΎ ΠΊΡΡΠ³ΠΎΠ²ΠΎΠΉ ΡΡΠ°Π΅ΠΊΡΠΎΡΠΈΠΈ.